Apuntes MAT-023 USM

544
Departamento Matem´ atica UTFSM Santiago MAT023 A PUNTES DE CLASES

description

Apuntes correspondientes al ramo de Mat-023 de la U. Técnica Federico Santa María

Transcript of Apuntes MAT-023 USM

Page 1: Apuntes MAT-023 USM

1

Departamento Matematica UTFSM Santiago

MAT023APUNTES DE CLASES

Page 2: Apuntes MAT-023 USM

Parte I

Ecuaciones diferenciales ordinarias

5

sysadmin
Rectángulo
sysadmin
Rectángulo
sysadmin
Rectángulo
Page 3: Apuntes MAT-023 USM

Indice general

Indice general 1

I Ecuaciones diferenciales ordinarias 5

1. Ecuaciones diferenciales ordinarias de primer orden 6

1.1. Definiciones . . . . . . . . . . . . . . . . . . . . . . . . . . . . . . . . . . . 6

1.2. Modelos simples . . . . . . . . . . . . . . . . . . . . . . . . . . . . . . . . . 11

1.3. Metodos Elementales de Resolucion . . . . . . . . . . . . . . . . . . . . . . 17

1.3.1. Integracion directa . . . . . . . . . . . . . . . . . . . . . . . . . . . 18

1.3.2. Ecuaciones de variable separable . . . . . . . . . . . . . . . . . . . . 19

1.3.3. Ecuacion lineal de primer orden . . . . . . . . . . . . . . . . . . . . 23

1.3.4. Ecuacion de Bernoulli . . . . . . . . . . . . . . . . . . . . . . . . . 29

1.3.5. Ecuacion de Ricatti . . . . . . . . . . . . . . . . . . . . . . . . . . . 32

1.3.6. Ecuaciones homogeneas . . . . . . . . . . . . . . . . . . . . . . . . 36

1.3.7. Otros cambios de variables . . . . . . . . . . . . . . . . . . . . . . . 38

1.4. Modelos simples: Segunda parte . . . . . . . . . . . . . . . . . . . . . . . . 44

1.5. Analisis cualitativo . . . . . . . . . . . . . . . . . . . . . . . . . . . . . . . 56

1.5.1. Metodos cualitativos . . . . . . . . . . . . . . . . . . . . . . . . . . 57

1.5.2. Ecuaciones diferenciales autonomas . . . . . . . . . . . . . . . . . . 59

1.5.3. Equilibrio y estabilidad . . . . . . . . . . . . . . . . . . . . . . . . . 63

1.6. Ejercicios del capıtulo . . . . . . . . . . . . . . . . . . . . . . . . . . . . . . 72

2. Ecuaciones diferenciales lineales de orden superior 80

2.1. Elementos de transformaciones lineales . . . . . . . . . . . . . . . . . . . . 80

2.1.1. Definiciones . . . . . . . . . . . . . . . . . . . . . . . . . . . . . . . 80

2.1.2. Nucleo e imagen . . . . . . . . . . . . . . . . . . . . . . . . . . . . 88

2.1.3. Isomorfismo . . . . . . . . . . . . . . . . . . . . . . . . . . . . . . . 94

2.1.4. Matriz asociada a una transformacion lineal . . . . . . . . . . . . . 94

2.1.5. Calculo con coordenadas . . . . . . . . . . . . . . . . . . . . . . . . 101

2.1.6. Ejercicios resueltos de Transformaciones lineales . . . . . . . . . . . 104

2.2. Ecuaciones diferenciales lineales . . . . . . . . . . . . . . . . . . . . . . . . 140

2.2.1. Definiciones . . . . . . . . . . . . . . . . . . . . . . . . . . . . . . . 140

2.3. Teorema de Existencia y Unicidad . . . . . . . . . . . . . . . . . . . . . . . 145

2.4. El wronskiano . . . . . . . . . . . . . . . . . . . . . . . . . . . . . . . . . . 148

2.5. Ecuaciones diferenciales a coeficientes constantes . . . . . . . . . . . . . . . 153

2.5.1. Definiciones . . . . . . . . . . . . . . . . . . . . . . . . . . . . . . . 153

2.5.2. La ecuacion de orden 2 . . . . . . . . . . . . . . . . . . . . . . . . . 154

1

Page 4: Apuntes MAT-023 USM

Apuntes Mat023 (Segundo semestre 2014)

2.5.3. La ecuacion de orden superior . . . . . . . . . . . . . . . . . . . . . 158

2.6. Metodo de variacion de parametros . . . . . . . . . . . . . . . . . . . . . . 162

2.7. Metodo del anulador . . . . . . . . . . . . . . . . . . . . . . . . . . . . . . 168

2.8. Movimiento vibratorio . . . . . . . . . . . . . . . . . . . . . . . . . . . . . 181

2.9. Ejercicios del capıtulo . . . . . . . . . . . . . . . . . . . . . . . . . . . . . . 187

3. Sistemas de ecuaciones diferenciales 192

3.1. Definiciones . . . . . . . . . . . . . . . . . . . . . . . . . . . . . . . . . . . 192

3.2. Ecuacion con coeficientes constantes . . . . . . . . . . . . . . . . . . . . . . 196

3.2.1. Matriz A diagonalizable . . . . . . . . . . . . . . . . . . . . . . . . 196

3.3. Variacion de parametros en sistemas . . . . . . . . . . . . . . . . . . . . . 201

3.4. Analisis cualitativo de sistemas . . . . . . . . . . . . . . . . . . . . . . . . 207

3.4.1. Valores propios reales y distintos (no nulos) . . . . . . . . . . . . . 207

3.4.2. Valores propios complejos . . . . . . . . . . . . . . . . . . . . . . . 211

3.4.3. Valores propios repetidos (no nulos) . . . . . . . . . . . . . . . . . . 214

3.5. Ejercicios del capıtulo . . . . . . . . . . . . . . . . . . . . . . . . . . . . . . 225

4. Transformacion integral de Laplace 229

4.1. Definiciones y teoremas fundamentales . . . . . . . . . . . . . . . . . . . . 229

4.2. Calculo de transformadas . . . . . . . . . . . . . . . . . . . . . . . . . . . . 233

4.3. Primer Teorema de la Traslacion . . . . . . . . . . . . . . . . . . . . . . . 236

4.4. Transformada de la derivada . . . . . . . . . . . . . . . . . . . . . . . . . . 237

4.5. Funciones escalonadas y Segundo Teorema de la Traslacion . . . . . . . . . 243

4.6. La Transformada de integrales de convolucion . . . . . . . . . . . . . . . . 247

4.7. Ejercicios del capıtulo . . . . . . . . . . . . . . . . . . . . . . . . . . . . . . 255

5. Series de Fourier 262

5.1. Definiciones . . . . . . . . . . . . . . . . . . . . . . . . . . . . . . . . . . . 262

5.1.1. El espacio SC [a, b] . . . . . . . . . . . . . . . . . . . . . . . . . . . 262

5.1.2. Teorema de la mejor aproximacion . . . . . . . . . . . . . . . . . . 267

5.2. Convergencia Puntual de series de Fourier . . . . . . . . . . . . . . . . . . 278

5.3. Series de Fourier de senos y cosenos . . . . . . . . . . . . . . . . . . . . . . 282

5.4. Derivacion e integracion de Series de Fourier . . . . . . . . . . . . . . . . . 292

5.5. Ejercicios del capıtulo . . . . . . . . . . . . . . . . . . . . . . . . . . . . . . 293

II Calculo diferencial en varias variables 298

6. Elementos de topologıa de Rn 299

6.1. El espacio euclidiano Rn . . . . . . . . . . . . . . . . . . . . . . . . . . . . 299

6.2. Producto interno y norma . . . . . . . . . . . . . . . . . . . . . . . . . . . 299

2

Page 5: Apuntes MAT-023 USM

Apuntes Mat023 (Segundo semestre 2014)

6.3. Elementos de topologıa de Rn . . . . . . . . . . . . . . . . . . . . . . . . . 302

6.4. Ejercicios del capıtulo . . . . . . . . . . . . . . . . . . . . . . . . . . . . . . 316

7. Funciones de varias variables 318

7.1. Definiciones basicas . . . . . . . . . . . . . . . . . . . . . . . . . . . . . . . 318

7.2. Graficos, conjuntos de nivel y trazas . . . . . . . . . . . . . . . . . . . . . . 321

7.3. Ejercicios del capıtulo . . . . . . . . . . . . . . . . . . . . . . . . . . . . . . 327

8. Lımites y continuidad 329

8.1. Definiciones . . . . . . . . . . . . . . . . . . . . . . . . . . . . . . . . . . . 329

8.2. Calculo de lımites . . . . . . . . . . . . . . . . . . . . . . . . . . . . . . . . 336

8.2.1. Algebra de lımites . . . . . . . . . . . . . . . . . . . . . . . . . . . . 336

8.2.2. Desigualdades y Teorema del Sandwich o teorema de acotamiento . 337

8.3. Continuidad . . . . . . . . . . . . . . . . . . . . . . . . . . . . . . . . . . . 339

8.4. Algebra de funciones continuas . . . . . . . . . . . . . . . . . . . . . . . . 341

8.5. Continuidad de funciones vectoriales . . . . . . . . . . . . . . . . . . . . . 342

8.6. Ejercicios del capıtulo . . . . . . . . . . . . . . . . . . . . . . . . . . . . . . 347

9. Diferenciacion en varias variables 349

9.1. Derivadas parciales . . . . . . . . . . . . . . . . . . . . . . . . . . . . . . . 349

9.2. Interpretacion de la derivada parcial . . . . . . . . . . . . . . . . . . . . . 359

9.3. Diferenciabilidad . . . . . . . . . . . . . . . . . . . . . . . . . . . . . . . . 365

9.4. Derivadas de orden superior y funciones de clase Cn . . . . . . . . . . . . . 383

9.5. Gradiente y matriz jacobiana . . . . . . . . . . . . . . . . . . . . . . . . . 387

9.6. La regla de la cadena . . . . . . . . . . . . . . . . . . . . . . . . . . . . . . 390

9.7. Gradiente y planos tangentes . . . . . . . . . . . . . . . . . . . . . . . . . 406

9.8. Derivada direccional y direcciones de crecimiento maximo . . . . . . . . . . 410

9.9. Ejercicios del capıtulo . . . . . . . . . . . . . . . . . . . . . . . . . . . . . . 415

10.Maximos y mınimos 427

10.1. Extremos locales . . . . . . . . . . . . . . . . . . . . . . . . . . . . . . . . 427

10.2. Maximos y mınimos en compactos y/o con restricciones . . . . . . . . . . . 443

10.3. Extremos restringidos Multiplicadores de Lagrange . . . . . . . . . . . . . 445

10.3.1. Criterio de la segunda derivada para extremos condicionados . . . . 458

10.4. Ejercicios del capıtulo . . . . . . . . . . . . . . . . . . . . . . . . . . . . . . 463

11.Funciones implıcitas e inversas 468

11.1. El teorema de la funcion implıcita . . . . . . . . . . . . . . . . . . . . . . . 468

11.2. El teorema de la funcion inversa . . . . . . . . . . . . . . . . . . . . . . . . 483

11.3. Ejercicios del capıtulo . . . . . . . . . . . . . . . . . . . . . . . . . . . . . . 492

3

Page 6: Apuntes MAT-023 USM

Apuntes Mat023 (Segundo semestre 2014)

III Evaluaciones de anos anteriores 495

12.Controles 496

13.Certamenes 509

Bibliografıa 541

4

Page 7: Apuntes MAT-023 USM

Parte I

Ecuaciones diferenciales ordinarias

5

Page 8: Apuntes MAT-023 USM

Capıtulo 1 : Ecuaciones diferenciales ordinarias de primer orden

Definiciones

En general, entenderemos el modelamiento matematico como el proceso de establecer

un modelo matematico (es decir, un sistema expresado en terminos de variables, funciones,

ecuaciones, etc.) que represente una situacion —principalmente de naturaleza fısica—, su

resolucion matematica, y finalmente la interpretacion de los resultados en los terminos

fısicos originales.

Como muchos conceptos de la naturaleza, tales como velocidad, aceleracion, las reaccio-

nes quımicas, los cambios de temperatura observados en un cuerpo, etc. se expresan como

razones de cambio tiene pleno sentido el uso de derivadas de funciones adecuadas. En este

tipo de situaciones, un modelo matematico es frecuentemente una ecuacion que contiene

una o mas derivadas de una funcion desconocida. Tal modelo matematico es llamado una

ecuacion diferencial [1].

Ejemplo 1.1.1. Son ejemplos de ecuaciones diferenciales las siguientes expresiones ma-

tematicas:

1. ddx

((1− x2) dy

dx

)+ n (n+ 1) y = 0 (Ecuacion de Legendre)

2. x2 d2ydx2

+ xdydx

+ (x2 − α2) y = 0 (Ecuacion de Bessel)

3. x3 d3ydx3

+ 4x2 d2ydx2

+ xdydx

+ 5y = 0 (Ecuacion de Euler)

4. y′′ + ky = A sin(ωox), A, ωo ∈ R (Problemas de resortes)

5. y′′ − xy = 0 (Ecuacion de Airy)

Definicion 1.1.1. Una ecuacion diferencial se dice de orden n si n corresponde al mayor

orden de derivada de la variable dependiente y presente en la ecuacion.

Ejemplo 1.1.2. La ecuacion d2ydx2

+ (5x) y3 = y6 es una ecuacion diferencial de segundo

grado. La ecuacion(

d3ydx3

)4

= dydx

+ 5y es una ecuacion diferencial de grado tres.

Comenzaremos el estudio de las ecuaciones diferenciales con las ecuaciones diferenciales

de primer orden.

6

Page 9: Apuntes MAT-023 USM

Apuntes Mat023 (Segundo semestre 2014)

Definicion 1.1.2. Sea f : U ⊆ R2 → R una funcion de dos variables. Una ecuacion

diferencial de primer orden es una ecuacion de la forma:

y′ = f (x, y) (1.1)

La variable x en este caso se conoce como variable independiente. Si la variable inde-

pendiente es el tiempo t, frecuentemente una ecuacion diferencial se anota como:

y′ = f (t, y) o bien y = f (t, y)

Observacion 1.1.1. Una ecuacion diferencial (ordinaria) de orden n es una ecuacion de

la forma:

f(x, y, y′, y′′, . . . , y(n)

)= 0

para una cierta funcion f .

Ejemplo 1.1.3. Son ecuaciones diferenciales de primer orden:

1. y′ = f (x) , f funcion integrable.

2. y′ + y = cosx

3. x−3y′ − y2 = 0, x > 0

Definicion 1.1.3. Sea I ⊆ R un intervalo abierto del tipo ]a, b[, o posiblemente intervalos

abiertos infinitos del tipo ]−∞, b[ , ]a,+∞[, o bien ]−∞,+∞[. Una funcion ϕ : I ⊆ R→ Rse dice solucion de la ecuacion diferencial (1.1) en el intervalo I si:

1.(t, ϕ (t)

)∈ U, ∀t ∈ I

2. ϕ′ (t) = f(t, ϕ (t)

), ∀t ∈ I

Ejemplo 1.1.4. La funcion ϕ : ]1,+∞[→ R, t→ ϕ (t) = 1t−1

es solucion de la EDO

dy

dt= −y2

pues, para t ∈ ]1,+∞[

d

dt

(1

t− 1

)= − 1

(t− 1)2

= −(

1

t− 1

)2

Ejemplo 1.1.5. Verifique que la funcion ϕ (x) = sinx definida en R es solucion de la

ecuacion diferencial de segundo orden:

y′′ + y = 0

7

Page 10: Apuntes MAT-023 USM

Apuntes Mat023 (Segundo semestre 2014)

Solucion. Derivando la funcion ϕ obtenemos: dϕdx

= cosx y d2ϕdx2

= − sinx se sigue

d2ϕ (x)

dx2+ ϕ (x) = 0

para todo x ∈ R.

Observacion 1.1.2. Note que al escribir:

ϕC (x) = sin x+ C

con C una constante cualquiera, ϕC tambien es solucion de la ecuacion diferencial.

Ejemplo 1.1.6. Consideremos la ecuacion:

xy′ − x2 − y = 0, x > 0

Se sabe que toda funcion de la forma:

y = x2 + Cx, C ∈ R (1.2)

es solucion de la ecuacion diferencial. En efecto, derivando la ecuacion (1.2), tenemos:

y′ = 2x+ C

Reemplazando en la ecuacion diferencial, se tiene que:

x (2x+ C)− x2 −(x2 + Cx

)= 0

para todo x > 0.

Ejemplo 1.1.7. Hallar una ecuacion diferencial para la familia de parabolas:

y = C1 (x− C2)2

donde C1 y C2 constantes arbitrarias.

Solucion. Despejamos C1y

(x− C2)2 = C1

luegod

dx

(y

(x− C2)2

)= 0

esto esy′ (x− C2)2 − y2 (x− C2)

(x− C2)4 = 0

8

Page 11: Apuntes MAT-023 USM

Apuntes Mat023 (Segundo semestre 2014)

se sigue

y′ (x− C2)2 − y2 (x− C2) = 0

luego

y′ (x− C2) = 2y

de donde obtenemos

x− C2 =2y

y′

derivando

1 =

(2y

y′

)′se sigue

1

2=

(y′)2 − yy′′

(y′)2

luego(y′)2

2= yy′′

Ejercicios de la seccion

1. Establezca el orden de la ecuacion diferencial dada:

(a) (1− x) y′′ − 4xy′ + 3y = tanx (b) y(4) − 2xy(6)y′′ =

(dy

dx

)7

(c) xd3y

dx3−(

dy

dx

)4

+ exy = 0 (d)d2y

dx2=

√1 +

(dy

dx

)2

2. Comprobar que las siguientes funciones satisfacen las ecuaciones dadas y dar un

intervalo en el cual esto se cumpla:

(a) y = A sin (x+B) ; y′′ + y = 0 (b) y = ex − e−x; y′′ − y = 0

(c) y = tan (x) ; y′ = 1 + y2 (d) y′ = 25 + y2; y = 5 tan 5x

3. Demostrar que la ecuacion dada define implıcitamente una solucion de la ecuacion

dada

(a) x2y − y3

3= 1 dy

dx= −2xy

x2−y2 para x 6= y

(b) x3 + 3xy2 = 1 2xyy′ + x2 + y2 = 0 para x ∈ ]0, 1[

4. Muestre que la familia de funciones

y = e−x2

∫ x

0

et2

dt+ Ce−x2

9

Page 12: Apuntes MAT-023 USM

Apuntes Mat023 (Segundo semestre 2014)

son soluciones de la ecuaciondy

dx+ 2xy = 1

5. Muestre que la funcion definida por tramos

ϕ (x) =

−x2 si x < 0

x2 si x ≥ 0

es una solucion de la ecuacion diferencial xy′ − 2y = 0 en R.

6. Determine α ∈ R para que la funcion y = xα sea solucion de 2x2y′′ − y = 0. Si

encuentra mas de un valor, muestre que cualquier combinacion lineal de esas dos

funciones resulta ser una solucion del problema.

7. Encontrar valores de m para los cuales la funcion es solucion de la ecuacion dada:

a) y (x) = emx donde y′′′ − 3y′′ − 4y′ + 12y = 0

b) y (x) = xm donde x2y′′ + 2xy′ − 6y = 0

8. Encontrar una E.D.O. de la forma y′′ + A (x) y′ + B (x) y = 0 que tenga entre sus

soluciones las funciones y1 (x) = ex y y2 (x) = xex.

9. Juan, Leo y Roberto estan tomando cafe y un joven del paralelo 19 les pregunta por

la solucion de la ecuacion diferencial

dy

dt=y + 1

t+ 1

despues de un rato, Juan dice y (t) = t, Leo y (t) = 2t+ 1 y Roberto y (t) = t2 − 2

¿Quien esta en lo correcto?

10. Construir una ecuacion diferencial de la forma

dy

dt= 2y − t+ g (y)

que tenga la funcion y (t) = e2t como solucion.

11. Construir una ecuacion diferencial de la formady

dt= f (t, y) que tenga por solucion

y (t) = et2

donde f (t, y) dependa explıcitamente de t y y.

12. Hallar una ecuacion diferencial para la familia de curvas:

y = C1 + (x− C2)2

donde C1 y C2 constantes arbitrarias.

13. Construya una ecuacion diferencial que no tenga ninguna solucion real.

10

Page 13: Apuntes MAT-023 USM

Apuntes Mat023 (Segundo semestre 2014)

Modelos simples

Estudiaremos algunos ejemplos elementales de modelamiento matematico:

Problema 1.2.1 ([2]). Desde una cierta altura se ha arrojado un cuerpo de masa m.

Determinar la ley segun la cual varıa la velocidad de caıda v, si sobre el cuerpo, ademas

de la fuerza de gravedad, actua la fuerza de resistencia del aire que es proporcional a la

velocidad v.

Solucion. Sea m la masa del cuerpo en caıda libre. En virtud de la Segunda Ley de

Newton: ∑i

Fi = ma

donde∑

i Fi representa la suma (vectorial) de fuerzas aplicadas al cuerpo y a representa la

aceleracion del cuerpo, se obtiene:

mdv

dt= mg − kv (1.3)

donde g es la aceleracion de gravedad y k es la constante (positiva) de proporcionalidad.

Resolver esta ecuacion diferencial significa encontrar una funcion v = f (t) que satisfaga

identicamente la ecuacion diferencial dada. Existen una infinidad de funciones de este tipo

(esto sera probado mas adelante). Es facil comprobar que toda funcion del tipo:

v (t) = Ce−kmt +

mg

k(1.4)

11

Page 14: Apuntes MAT-023 USM

Apuntes Mat023 (Segundo semestre 2014)

satisface la ecuacion (1.3), cualquiera que sea la constante C. Pero, ¿cual de estas funciones

dara la dependencia buscada entre v y t? Para encontrar dicha relacion, se debe utilizar un

condicion adicional. Esta condicion adicional se llama condicion inicial. Supongamos que

en el momento inicial del experimento arrojamos el cuerpo con una velocidad inicial v0

conocida. Ası, la funcion v = f (t) que deseamos encontrar debe satisfacer la condicion:

f (0) = v0

Es decir, v = v0 en t = 0. Ası, reemplazando en (1.4), se obtiene:

C = v0 −mg

k

Por lo tanto, la ley segun la cual varıa la velocidad de caıda esta dada por la ecuacion:

v (t) =(v0 −

mg

k

)e−

kmt +

mg

k

Observacion 1.2.1. Una cuestion de interes es el comportamiento asintotico de la solucion.

Es decir, el comportamiento de la solucion para t suficientemente grande. Mas precisamente,

nos interesa:

lımt→+∞

v (t)

En particular, para el ejemplo:

lımt→+∞

v (t) = lımt→+∞

(v0 −

mg

k

)e−

kmt +

mg

k

=mg

k

La interpretacion del resultado anterior, es que cuando el tiempo t es suficientemente

grande, la velocidad final del cuerpo no depende de la velocidad inicial.

Observacion 1.2.2. Observar que desde la ecuacion (1.3), si suponemos k = 0, es decir,

suponemos que la resistencia del aire es tan pequena que puede ser despreciada, se obtiene:

v (t) = v0 + gt

Definicion 1.2.1. Se llamara solucion general de la ecuacion diferencial de primer

orden:

y′ = f (x, y)

a la funcion:

y = ϕ (x,C)

que depende de una constante arbitraria C y satisface las condiciones siguientes:

1. satisface la ecuacion diferencial para cualquier valor de la constante C.

12

Page 15: Apuntes MAT-023 USM

Apuntes Mat023 (Segundo semestre 2014)

2. cualquiera que sea la condicion inicial y = y0 para x = x0, es decir, y (x0) = y0

se puede encontrar un valor C = C0 tal que la funcion y = ϕ (x,C0) satisfaga la

condicion inicial dada.

Problema 1.2.2 (Espejo parabolico). Hallar la forma que debe tener un espejo convexo,

simetrico respecto de un eje, llamado eje focal, de tal modo que si un haz de luz es apuntado

hacia el espejo, paralelo al eje focal, se refleje directamente en un punto F fijo (llamado

foco) del eje focal.

Solucion. Obtendremos tal espejo mediante la rotacion de una curva en el plano. Sea

C : y = f (x) tal curva. Consideremos un sistema de coordenadas de tal modo que el

origen del sistema coincida con el foco F de la curva. En particular, el eje focal de la curva

coincide con el eje de las abscisas de tal sistema de referencia. Considere un punto P (x, y)

en la curva C y sea T la recta tangente a C en el punto P (x, y). Denotemos por Q el punto

de interseccion de T con el eje focal (o de las abscisas del sistema de referencia). Si L es

una recta que representa el haz de luz paralelo al eje focal y que incide en P , por la ley de

Snell, el angulo de incidencia ]TPL y angulo de reflexion ]QPF coinciden. Es decir, se

tiene que:

]TPL = ]QPF = α

Entonces, del triangulo 4QPF se obtiene:

tan 2α =y

x(1.5)

por ser F el origen del sistema de coordenadas. Como T es tangente a la curva C, se tiene

que ]PQF es tambien α. Luego:

tanα = y′ (1.6)

13

Page 16: Apuntes MAT-023 USM

Apuntes Mat023 (Segundo semestre 2014)

Reemplazando las ecuaciones (1.5) y (1.6), en la identidad trigonometrica siguiente:

tan 2α =2 tanα

1− tan2 α

se obtiene la ecuacion diferencial de primer orden:

y

x=

2y′

1− (y′)2 (1.7)

Problema 1.2.3 (Braquistocrona). [3] Hallar la forma que debe tener un alambre de

modo que una argolla que se desliza por el, sin roce, bajo la accion de la gravedad de un

punto A a un punto B de menor altura en el mismo plano y no exactamente bajo el punto

A, lo haga en el menor tiempo posible.

Solucion. Este problema fue planteado en 1696 por Jean Bernoulli a la comunidad cientıfi-

ca de su epoca. La solucion que el mismo encontro (independientemente tambien lo hicieron

Leibnitz, L’Hopital, Newton) usa una version generalizada de la Ley de Snell de la optica.

Comenzaremos modelando primeramente esta situacion: la version generalizada de la Ley

de Snell. A modo de ejercicio lo haremos utilizando las herramientas del calculo diferencial,

en particular minimizacion. Considere, entonces, el siguiente problema:

Supongamos que un atleta, situado en la orilla oriental de un rıo de ancho a debe atravesarlo

nadando a velocidad constante v1 hasta un cierto punto C en la orilla occidental. Luego de

esto, debe correr a velocidad constante v2 por sobre la arena de la ribera de ancho b del rıo

hasta la meta en el punto B. Se supone que v1 < v2. Se desea encontrar el punto C en la orilla

occidental de tal modo que el tiempo empleado por el atleta desde el punto A hasta la meta

en B sea el menor posible.

14

Page 17: Apuntes MAT-023 USM

Apuntes Mat023 (Segundo semestre 2014)

Consideremos las rectas paralelas (y verticales) l1, l2 y l3. Supongamos que la recta l1representa la orilla oriental del rıo, l2 representa la ribera del rıo y l3 la lınea de meta. Por

la condiciones del problema tenemos:

1. A ∈ l1

2. C ∈ l2

3. B ∈ l3

4. a es la distancia entre l1 y l2

5. b es la distancia entre l2 y l3

Supongamos, ademas, que c y x son las distancias verticales de A a B, y de A a C,

respectivamente. Entonces, el tiempo total de la carrera esta dado por:

T (x) =

√x2 + a2

v1

+

√(c− x)2 + b2

v2

Derivando e igualando a 0, se obtiene la coordenada x0 de tiempo mınimo. Es decir, x0

debe cumplir con:x0

v1

√x2

0 + a2=

c− x0

v2

√(c− x0)2 + b2

Si introducimos los angulos de incidencia α y β respecto de la recta normal a l2 para este

valor de x0 se obtiene:sinα

v1

=sin β

v2

que es la conocida Ley de Snell.

Ahora bien, si consideramos otro segmento (es decir, otra recta vertical l4) por el cual

el atleta se deba desplazar a velocidad constante v3, se obtendra:

sinα1

v1

=sinα2

v2

=sinα3

v3

= constante

De manera analoga, la argolla de masa m que cae bajo la accion de la gravedad g tiene

una velocidad v que va en aumento de acuerdo a la distancia vertical recorrida y. Ası,

igualando energıas potencial y cineticas se obtiene la ecuacion:

1

2mv2 = mgy

de donde:

v =√

2gy (1.8)

15

Page 18: Apuntes MAT-023 USM

Apuntes Mat023 (Segundo semestre 2014)

Por esta razon, Bernoulli conjeturo la Ley de Snell generalizada siguiente:

sinα

v= λ, λ constante (1.9)

donde α es el angulo que instantaneamente forma la tangente a la curva en la posicion de

la argolla y la vertical. Finalmente, notando que:

sinα = cos(π

2− α

)=

1√1 + tan2

(π2− α

)=

1√1 + (y′)2

Utilizando, entonces, las formulas (1.8) y (1.9) se obtiene la ecuacion:√2gy =

1

λ√

1 + (y′)2

O bien:

y(

1 + (y′)2)

= k2

donde k = 12gλ2

.

Ejercicios de la seccion

1. Determine la ecuacion que debe cumplir la familia de curvas que forman un angulo

de 45 grados en la interseccion con la familia de curvas y (x+ c) = 1.

2. Determine la ecuacion de la familia de curvas ortogonales a la familia y2 = cx3.

3. En este problema se analiza la caıda de una gota de agua. Supongamos que al

caer esta se evapora y mantiene su forma esferica, la rapidez con que se evapora es

proporcional al area con una constante de proporcionalidad ζ < 0 y no se considera

la resistencia del aire. Designemos por ρ la densidad del agua, r0 el radio de la gota

cuando t = 0 y la direccion positiva es hacia abajo.

a) Muestre que el radio de la gota r (t) disminuye de acuerdo a la ley

r (t) =

ρ

)t+ r0

b) Obtener la ecuacion diferencial que satisface la velocidad v (t) de la gota en su

caıda libre. Determine la velocidad si la gota cae del reposo.

16

Page 19: Apuntes MAT-023 USM

Apuntes Mat023 (Segundo semestre 2014)

4. En la figura suponga que el eje y y la recta x = 1 corresponden a las orillas oeste

y este de un rıo de 1 [km] de ancho. El rıo fluye hacia el norte con una velocidad

vr donde ‖vr‖ = vr [km/h]. Un hombre entra al rıo en el punto (1, 0) en la costa

este y nada en direccion a la ribera contraria a una velocidad constante de ‖vs‖ = vs[km/h]. El hombre quiere llegar al punto (0, 0) de modo que nada de forma tal que

su vector velocidad vs siempre apunta a (0, 0). Muestre que la trayectoria que sigue

el nadador satisface la ecuacion

dy

dx=vsy − vr

√x2 + y2

vsx

Metodos Elementales de Resolucion

Recordaremos primeramente un teorema esencial para resolver ecuaciones diferenciales

elementales: el Teorema Fundamental del Calculo.

Teorema 1.3.1. (Teorema Fundamental del Calculo)

Sea f : [a, b]→ R funcion integrable en [a, b]. Dado x0 ∈ [a, b] e y0 ∈ R se tiene que la

funcion F : [a, b]→ R dada por

F (x) =

x∫x0

f(t)dt+ y0 con x ∈ [a, b],

es continua en [a, b] y F (x0) = y0. Ademas, si f es continua en [a, b], entonces la funcion

F es derivable en [a, b] y satisface la ecuacion:

F ′(x) = f(x)

Observacion 1.3.1. Recordemos que en este contexto la funcion F es conocida como una

primitiva de f .

17

Page 20: Apuntes MAT-023 USM

Apuntes Mat023 (Segundo semestre 2014)

Integracion directa

Definicion 1.3.1. Una ecuacion diferencial ordinaria de primer orden, digamos y′ = f(x, y),

se dice que es resoluble por integracion directa si existe una funcion g integrable sobre

un intervalo abierto I ⊆ R tal que f(x, y) = g(x)

Observacion 1.3.2. En particular, una ecuacion diferencial de integracion directa es de

la forma:

y′ = g(x) (1.10)

para todo x ∈ I.

Ası, si g : D ⊆ R→ R es una funcion integrable, entonces integrando en ambos lados de

la igualdad 1.3.2 y usando el Teorema Fundamental del Calculo, obtenemos que la solucion

general de la ecuacion diferencial es de la forma:

φ(x) = F (x) + C,

donde F es una primitiva de g y C ∈ R es una constante.

Ejemplo 1.3.1. Una partıcula se mueve a lo largo de una lınea recta de manera que su

velocidad en el instante t es 2 sin t. Si f(t) indica su posicion en el tiempo t medio a partir

del punto de partida, se tiene que f ′(t) = 2 sin t. Por la observacion anterior, se concluye

que:

f(t) = −2 cos t+ C

Note que para fijar la funcion posicion se necesita algun otro dato. En particular, si se

conoce el valor de f en algun instante en particular, entonces se puede determinar C. Por

ejemplo, si f(0) = 0, entonces C = 2 y la funcion posicion es f(t) = −2 cos t+ 2.

Ejemplo 1.3.2. Determine la solucion del problema

dy

dx= e−x

2

y (0) = 5

Solucion. Se puede demostrar que∫e−x

2dx no es una funcion elemental, sin embargo,

podemos expresar la solucion por

y (x) =

∫ x

0

e−t2

dt+ 5

18

Page 21: Apuntes MAT-023 USM

Apuntes Mat023 (Segundo semestre 2014)

Ecuaciones de variable separable

Definicion 1.3.2. Una ecuacion diferencial de primer orden y′ = f (x, y) se dice una

ecuacion de variable separable si:

f (x, y) = Q (x)R (y) (1.11)

para Q y R funciones continuas sobre algun intervalo abierto I ⊆ R.

Observacion 1.3.3. [5] Si R (y) 6= 0 se puede dividir por R (y) y escribir la ecuacion

(1.11) en la forma:

A (y) y′ = Q (x) (1.12)

donde A (y) = 1/R (y). Recordemos, ademas, que y representa una cierta funcion descono-

cida y = Y (x), luego la ecuacion (1.12), se expresa como:

A (Y (x))Y ′ (x) = Q (x)

Integrando la ecuacion anterior, se sigue que:∫A (Y (x))Y ′ (x) dx =

∫Q (x) dx+ C

Haciendo la sustitucion y = Y (x) en la integral de la izquierda, se tiene que dy = Y ′ (x) dx.

Por consiguiente, se obtiene: ∫A (y) dy =

∫Q (x) dx+ C

Ahora bien, si G es una primitiva de A y H es una primitiva de Q, la ecuacion anterior se

puede escribir como:

G (y) = H (x) + C (1.13)

Recıprocamente, si y es una funcion que satisface la ecuacion (1.13), entonces:

G′ (y) y′ = H ′ (x)

es decir, tenemos A (y) y′ = Q (x). Por lo tanto, la ecuacion (1.13) da una representacion

implıcita de una familia de soluciones a un parametro.

Observacion 1.3.4. En particular, el formalismo anterior se reduce considerablemente si

se considera la notacion de Leibnitz para las derivadas. Es decir, escribimos y′ = dy/dx y

separamos las variables, con lo cual la ecuacion (1.12) se escribe directamente como:

A (y) dy = Q (x) dx

y la ecuacion anterior tiene sentido al ser dy la diferencial de la funcion desconocida y.

Formalmente, podemos integrar la ecuacion anterior, obteniendo:∫A (y) dy =

∫Q (x) dx+ C

pero esto solo es formal, pues las variables de integracion son distintas.

19

Page 22: Apuntes MAT-023 USM

Apuntes Mat023 (Segundo semestre 2014)

Ejemplo 1.3.3. Resolver la ecuacion

dy

dx= ey−x

Solucion. Se trata de una E.D.O. de variables separadas pues

dy

dx= ey−x = e−xey

se sigue

e−y(x)y′ (x) = e−x

integrando respecto a x ∫e−y(x)y′ (x) dx =

∫e−xdx

se sigue

−e−y(x) = −e−x + C

despejando

e−y(x) = e−x +K

luego

y (x) = − ln(e−x +K

)es una familia de soluciones.

Ejemplo 1.3.4. Resolver la ecuacion

dy

dx= x2y + x2

Solucion. Note que x2y+x2 = x2 (y + 1) luego se trata de una EDO de variables separables

dy

dx= x2 (y + 1)

se sigue ∫dy

y + 1=

∫x2dx

integrando

ln |y + 1| =x3

3+ C

⇒y + 1 = Ke

x3

3

ası

y (x) = Kex3

3 − 1

20

Page 23: Apuntes MAT-023 USM

Apuntes Mat023 (Segundo semestre 2014)

Ejemplo 1.3.5. Hallar la solucion general de la ecuacion diferencial:

x cos y +√x+ 1 sen y y′ = 0

indicando claramente el dominio en el cual esta definida.

Solucion. Notamos primeramente que la ecuacion esta definida para x > −1 y es una

ecuacion de variable separable. Luego, anotamos:

x cos y +√x+ 1 sen y

dy

dx= 0

Es decir:

x cos y = −√x+ 1 sen y

dy

dx

Ahora bien, si y 6= π2

+ kπ, con k ∈ Z, escribimos la ecuacion anterior como:

x√x+ 1

= − tan ydy

dx

Separando variables e integrando, obtenemos:∫x√x+ 1

dx = −∫

tan y dy + C

con C una constante de integracion. Entonces, la ecuacion anterior queda:∫ √x+ 1 dx−

∫1√x+ 1

dx = − ln | cos y|+ C

Es decir, obtenemos:

2

3

√(x+ 1

)3 − 2√x+ 1 = − ln | cos y|+ C

Por tanto, obtenemos:2

3

√x+ 1 (x− 2) + ln | cos y| = C

Finalmente, notamos que las funciones constantes:

y =π

2+ 2kπ, k ∈ Z

son soluciones de la ecuacion diferencial. Por lo tanto, la solucion general de la ecuacion

esta dada por:2

3

√x+ 1 (x− 2) + ln | cos y| = C , si y 6= ±π

2, ±3π

2, . . .

y =π

2+ 2kπ, k ∈ Z

21

Page 24: Apuntes MAT-023 USM

Apuntes Mat023 (Segundo semestre 2014)

Ejemplo 1.3.6. Diremos que una ecuacion diferencial es una ecuacion diferencial lineal

de primer orden homogenea si es de la forma:

y′ + P (x) y = 0 (1.14)

si P es una funcion continua sobre su dominio abierto.

Solucion. Separando las variables e integrando:∫dy

y= −

∫P (x) dx+K

con K una constante de integracion. Luego, la ecuacion anterior implica que:

ln |y| = −∫P (x) dx+K

Ası:

y = ±eK e−∫P (x)dx

Por consiguiente:

y = C e−∫P (x)dx

Observacion 1.3.5. Note que en el procedimiento anterior, se ha dividido por y, por

tanto, debe explicarse que toda solucion de (1.14) se puede expresar mediante la formula

y = C e−∫P (x)dx. Sea y una solucion de (1.14) y considere la funcion g definida por:

g (x) = y e∫P (x)dx

Luego:

g′ (x) = y′e∫P (x)dx + P (x) ye

∫P (x)dx

= e∫P (x)dx (y′ + P (x) y)

= 0

como x pertenece a un intervalo abierto, se obtiene que g (x) = C, por el Teorema del Valor

Medio. Por tanto:

y = C e−∫P (x)dx

Observacion 1.3.6. El razonamiento anterior da origen a un metodo de resolucion de

ecuaciones diferenciales del tipo:

y′ + P (x) y = Q (x)

donde P y Q son funciones continuas en un intervalo abierto I ⊆ R. La ecuacion diferencial

anterior se llama ecuacion diferencial lineal de primer orden.

22

Page 25: Apuntes MAT-023 USM

Apuntes Mat023 (Segundo semestre 2014)

Ecuacion lineal de primer orden

Definicion 1.3.3. Una ecuacion diferencial de primer orden se dice lineal si es de la forma:

y′ + P (x) y = Q (x) (1.15)

para dos funciones P y Q continuas en un intervalo abierto I ⊆ R.

Definicion 1.3.4. Un factor integrante o un factor de integracion para la ecuacion

(1.15) es una expresion de la forma:

µ (x) = e∫P (x)dx (1.16)

Observacion 1.3.7. Multipliquemos la ecuacion diferencial (1.15) por el factor integrante

en (1.16), luego:

e∫P (x)dx y′ + P (x) y = e

∫P (x)dxQ (x)

Note que la ecuacion anterior, puede escribirse como:

d

dx

ye∫P (x)dx

= e

∫P (x)dxQ (x)

Integrando respecto de x, obtenemos:

ye∫P (x)dx =

∫e∫P (x)dxQ (x) dx+ C

Finalmente:

y = e−∫P (x)dx

∫e∫P (x)dxQ (x) dx+ C

Teorema 1.3.2 (Formula de Leibnitz). Sean P y Q dos funciones continuas sobre un

intervalo abierto I ⊆ R. La solucion general de la ecuacion diferencial lineal:

y′ + P (x) y = Q (x)

esta dada por la formula:

y (x) = e−∫P (x)dx

∫e∫P (x)dxQ (x) dx+ C

Observacion 1.3.8. Si µ (x) = e

∫P (x)dx es el factor integrante, la formula anterior queda:

y (x) =1

µ (x)

∫µ (x)Q (x) dx+ C

Ejemplo 1.3.7. Resolver la ecuacion lineal de primer orden:

xy′ + (1− x) y = e2x

23

Page 26: Apuntes MAT-023 USM

Apuntes Mat023 (Segundo semestre 2014)

Solucion. Supongamos que x 6= 0. La ecuacion anterior queda como:

y′ +

(1

x− 1

)y =

e2x

x

Utilizando las notaciones del teorema anterior, tenemos que P (x) = 1/x−1 y Q (x) = e2x/x.

Calculamos, primeramente, el factor integrante:

µ (x) = e∫( 1x−1)dx

= |x| e−x

Luego:

y (x) =1

µ (x)

∫µ (x)Q (x) dx+ C

=

ex

|x|

∫|x| e−x e

2x

xdx+ C

Si x > 0, se tiene que |x| = x y la solucion es:

y =ex

x

∫exdx+ C

=

e2x

x+ C

ex

x

Por otro lado, si x < 0, se tiene que |x| = −x y la solucion es:

y =ex

−x

−∫exdx+ C

=

e2x

x− Ce

x

x

Ahora bien, como C es una constante arbitraria, si x 6= 0 se puede escribir:

y =e2x

x+ C

ex

x

Ejemplo 1.3.8. Deternine una funcion f : D ⊆ R→ R tal que

f (t) = t+ et∫ t

0

e−uf (u) du+ tf (t)

Solucion. Notemos que f (0) = 0 y

(f (t)− t− tf (t)) e−t =

∫ t

0

e−uf (u) du

24

Page 27: Apuntes MAT-023 USM

Apuntes Mat023 (Segundo semestre 2014)

usando el teorema fundamental del calculo

(f ′ (t)− 1− f (t)− tf ′ (t)) e−t − (f (t)− t− tf (t)) e−t = e−tf (t)

se sigue

f ′ (t)− 1− f (t)− tf ′ (t)− f (t) + t+ tf (t) = f (t)

ası

(1− t) f ′ (t)− (3− t) f (t) = 1− t

es una EDO de primer orden lineal

df

dt− 3− t

1− tf = 1

el factor de integracion es

µ (t) = e∫− 3−t

1−tdt = e2 ln(t−1)−t

= (t− 1)2 e−t

se sigued

dt

((t− 1)2 e−tf

)= (t− 1)2 e−t

integrando

(t− 1)2 e−tf =

∫(t− 1)2 e−tdt

es decir

(t− 1)2 e−tf = −e−t(t2 + 1

)+ C

de donde

f (t) =−e−t (t2 + 1) + C

(t− 1)2 e−t

como f (0) = 0 se sigue

0 = −1 + C ⇒ C = 1

finalmente

f (t) =−e−t (t2 + 1) + 1

(t− 1)2 e−t

Ejemplo 1.3.9. Resolver la ecuacion

dy

dx− 2xy = x

25

Page 28: Apuntes MAT-023 USM

Apuntes Mat023 (Segundo semestre 2014)

Solucion. Es una ecuacion lineal, el factor integrante es µ (x) = e∫−2xdx = e−x

2, multipli-

cando la ecuacion por µ (x) se tiene

e−x2 dy

dx− 2xe−x

2

y = xe−x2

luegod

dx

(e−x

2

y)

= xe−x2

integrando

e−x2

y =

∫xe−x

2

dx

= −1

2e−x

2

+ C

ası

y (x) = −1

2+ Cex

2

Ejemplo 1.3.10. Considere el problema de valor inicial:

(1 + x2)y′ + 2xy = f(x), y(0) = 0.

en donde:

f(x) =

x, si x ∈ [0, 1),

−x, si x ≥ 1.

Hallar, si acaso existe, una solucion continua para este problema.

Solucion. Por definicion de f , notamos primeramente que el dominio de la ecuacion

diferencial es el intervalo [0,+∞). Entonces, si x ∈ [0, 1) tenemos el problema de valor

inicial siguiente:

(1 + x2)y′ + 2xy = x, y(0) = 0 (1.17)

Entonces:

y′ +2x

1 + x2y =

x

1 + x2

Ası:

y =1

1 + x2

(∫(1 + x2)

x

1 + x2dx+ C

)=

1

1 + x2

(x2

2+ C

)Ahora bien, como y(0) = 0, obtenemos que C = 0. Por tanto, la unica solucion del problema

de valor inicial (1.17) esta dada por:

y =x2

2(1 + x2)

26

Page 29: Apuntes MAT-023 USM

Apuntes Mat023 (Segundo semestre 2014)

Por otro lado, si x ∈ [1,+∞) tenemos la ecuacion diferencial:

(1 + x2)y′ + 2xy = −x

O bien:

y′ +2x

1 + x2y = − x

1 + x2(1.18)

cuya solucion general esta dada por:

y =1

1 + x2

(−x

2

2+Q

)(1.19)

Ası, para que exista una solucion continua, se debe cumplir que:

lımx→1−

y[0,1)(x) = y[1,+∞)(1)

donde y[0,1) denota la solucion para 0 ≤ x < 1 e y[1,+∞) denota la solucion para x ≥ 1.

Entonces, como:

lımx→1−

y[0,1)(x) = lımx→1−

x2

2(1 + x2)=

1

4

tenemos que elegir el valor de Q en la familia de funciones (1.19) tal que y[1,+∞)(1) = 14.

Entonces, Q = 1. Por tanto, el problema:

(1 + x2)y′ + 2xy = f(x), y(0) = 0.

en donde:

f(x) =

x, si x ∈ [0, 1),

−x, si x ≥ 1.

tiene una solucion continua y = y(x) y esta dada por:

y(x) =

x2

2(1 + x2), 0 ≤ x < 1

1

1 + x2

(−x

2

2+ 1)

, x ≥ 1.

Ejemplo 1.3.11. Una curva continua y = f(x) en el primer cuadrante parte desde el

origen y tiene la propiedad de que el area debajo del cuadrado de dicha curva, desde el

origen hasta un punto (x, y) en ella, es igual al cubo de la abscisa de (x, y) mas el cuadrado

del area del rectangulo que tiene a los puntos (0, 0) y (x, y) como vertices opuestos. Hallar

una ecuacion, como solucion de una ecuacion diferencial de primer orden, que defina

implıcitamente la curva y = f(x).

Indicacion: No intente calcular la integral

∫x2e

1x dx.

27

Page 30: Apuntes MAT-023 USM

Apuntes Mat023 (Segundo semestre 2014)

Solucion. Sean C : y = f(x) una curva continua en el primer cuadrante y (x, y) ∈ C.

Sabemos que el area bajo el cuadrado de la curva C : y = f(x) desde el origen (0, 0) hasta

el punto (x, y), esta dada por la integral:∫ x

0y(t)2 dt. Ası, la ecuacion que define la curva C

es la ecuacion: ∫ x

0

y(t)2 dt = x3 + (xy)2 (1.20)

con x > 0, en donde la expresion (xy)2 representa el cuadrado del area del rectangulo

que tiene a los puntos (0, 0) y (x, y) como vertices opuestos. Como y = f(x) es continua,

entonces, por el Teorema Fundamental del Calculo, la integral indefinida del lado izquierdo

es derivable. Luego, derivando la ecuacion (1.20) respecto de x, obtenemos:

y2 = 3x2 + 2(xy)(y + xy′

), x > 0

Esto es:

y2 = 3x2 + 2xy2 + 2x2yy′, x > 0 (1.21)

Ahora, sea u = y2, entonces u′ = 2yy′. Reemplazando en (1.21), tenemos:

u = 3x2 + 2xu+ x2u′, x > 0

Reordenando la ecuacion, obtenemos la ecuacion diferencial lineal de primer orden siguiente:

u′ +

(2

x− 1

x2

)u = −3, x > 0 (1.22)

Ahora bien, note que: ∫ (2

x− 1

x2

)dx = 2 ln x+

1

x+Q

Luego, el factor integrante µ(x) de (1.22), esta dado por:

e∫( 2x− 1x2

) dx = x2e1x

Entonces, por la formula de Leibniz:

u(x) =1

µ(x)

(−3

∫µ(x) dx+ C

)en donde C ∈ R, se obtiene que:

u(x) =e−

1x

x2

(C − 3

∫x2e

1x dx

)Sin embargo, u = y2, por lo tanto, la ecuacion que define implıcitamente la curva C : y =

f(x) esta dada por:

y2 = Ce−

1x

x2− 3

e−1x

x2

∫x2e

1x dx

28

Page 31: Apuntes MAT-023 USM

Apuntes Mat023 (Segundo semestre 2014)

Ecuacion de Bernoulli

Observacion 1.3.9. Numerosas aplicaciones pueden ser modeladas con ecuaciones di-

ferenciales ordinarias que no son lineales, sin embargo, mediante cambios de variables

adecuados y algo de manipulacion algebraica, estas ecuaciones pueden ser transformadas

en ecuaciones diferenciales lineales. Un caso importante es la llamada ecuacion de Bernoulli.

Definicion 1.3.5. Sean P (x) y Q (x) funciones continuas sobre un intervalo abierto I ⊆ R.

Una ecuacion diferencial de la forma:

y′ + P (x) y = Q (x) yα, α 6= 1 (1.23)

se llama ecuacion de Bernoulli.

Observacion 1.3.10. Multipliquemos la ecuacion de Bernoulli en (1.23) por y−α, de donde

obtenemos:

y−αy′ + P (x) y1−α = Q (x) (1.24)

Sea z = y1−α. Luego, por la regla de la cadena, obtenemos:

z′ = (1− α) y−αy′

Reemplazando en la ecuacion (1.24), se obtiene:

1

1− αz′ + P (x) z = Q (x)

O bien:

z′ + (1− a)P (x) z = (1− α)Q (x) (1.25)

que es una ecuacion diferencial lineal de primer orden. Es importante notar que una vez

resuelta la ecuacion (1.25) se debe volver a la variable original y = y (x).

Ejemplo 1.3.12. Hallar la solucion de la ecuacion diferencial:

dy

dx+ xy = x3y3

Solucion. Dividiendo todos los terminos por y3, tenemos:

y−3y′ + xy−2 = x3 (1.26)

Consideremos, tal como en la observacion anterior, el cambio de variables z = y−2. Luego:

dz

dx= −2y−3 dy

dx

29

Page 32: Apuntes MAT-023 USM

Apuntes Mat023 (Segundo semestre 2014)

Reemplazando la ecuacion anterior en (1.26), se obtiene la ecuacion diferencial lineal:

dz

dx− 2xz = −2x3

cuya solucion es:

z = 1 + x2 + Cex2

Por consiguiente, la solucion general de la ecuacion diferencial dada es:

y =1√

1 + x2 + Cex2

con C una constante cualquiera.

Ejemplo 1.3.13 (Ecuacion logıstica). Resuelva la siguiente ecuacion de Bernoulli, conocida

como ecuacion logıstica:dN

dt= αN (β −N) (1.27)

con α, β > 0.

Solucion. Note que podemos escribir la ecuacion (1.27) como:

dN

dt= αβN − αN2

Sean A = αβ y B = α, entonces:

dN

dt− AN = −BN2

Dividiendo la ecuacion anterior por N2, obtenemos:

N−2 dN

dt− AN−1 = −B (1.28)

Sea z = N−1, luego:dz

dt= −N−2 dN

dt

Reemplazando en la ecuacion (1.28), obtenemos:

−dz

dt− Az = −B

Es decir:dz

dt+ Az = B

cuya solucion exacta es:

z = Ce−At +B

A

30

Page 33: Apuntes MAT-023 USM

Apuntes Mat023 (Segundo semestre 2014)

Pero z = 1N

, luego:

N =1

z=

1

Ce−At +B/A

Finalmente, como A = αβ y B = α, obtenemos:

N (t) =β

1 + Ce−αβt

Ejemplo 1.3.14. Se ha determinado experimentalmente que la variacion de peso de un

tipo de pez sigue la leydp

dt= αe−

β3tp2/3 − βp

donde p = p (t) representa el peso del pez, α y β son constantes positivas. Si p (0) = p0 > 0

determine el peso maximo del pez.

Solucion. Se trata de una ecuacion de Bernoulli

dp

dt+ βp = αe−

β3tp2/3

multiplicando por p−2/3 se sigue dpdtp−2/3 + βp1/3 = αe−

β3t hacemos el cambio u = p1/3

entonces dudt

= 13p−2/3 dp

dtası

3du

dt+ βu = αe−

β3t

du

dt+β

3u =

α

3e−

β3t

resolvemos esta ecuacion lineal al multiplicar por el factor µ (t) = eβ3t se sigue

d

dt

(ue

β3t)

3

u =(α

3t+ C

)e−

β3t

volvemos a la variable original

p (t) =(α

3t+ C

)3

e−βt

como p (0) = p0 se tiene p0 = C3 ası

p (t) =(α

3t+ 3√p0

)3

e−βt

para buscar el maximo derivamos

p′ (t) = α(α

3t+ 3√p0

)2

e−βt − β(α

3t+ 3√p0

)3

e−βt

=(α

3t+ 3√p0

)2

e−βt(α− β

(α3t+ 3√p0

))=

(α3t+ 3√p0

)2

e−βt(α− β 3

√p0 −

αβ

3t

)31

Page 34: Apuntes MAT-023 USM

Apuntes Mat023 (Segundo semestre 2014)

ası

t =α− β 3

√p0

αβ3

note que antes de ese numero la derivada es positiva y despues negativa, el punto es punto

de maximo global, si este numero es negativo, el maximo se alcanza en t = 0 y corresponde

a p0, en otro caso, el peso maximo es

pmax = p

(α− β 3

√p0

αβ3

)

=

3

(α− β 3

√p0

αβ3

)+ 3√p0

)3

e−β(α−β 3√p0

αβ3

)

=

β

)3

e−3(1− βα

3√p0)

Ecuacion de Ricatti

Definicion 1.3.6. Una ecuacion de Ricatti es una ecuacion diferencial de la forma:

y′ + P (x) y +Q (x) y2 = R (x) (1.29)

donde P,Q y R son funciones continuas sobre un intervalo I ⊆ R.

Observacion 1.3.11. Existe un metodo para obtener una familia de soluciones de una

ecuacion de Ricatti si se conoce una solucion particular u = u (x). Supongamos esto y

consideremos el cambio de variables:

y = u+1

v(1.30)

con v = v (x) la nueva variable. Derivando la ecuacion anterior, obtenemos:

y′ = u′ − 1

v2v′ (1.31)

Reemplazando las ecuaciones (1.30) y (1.31) en la ecuacion de Ricatti (1.29), se sigue que:(u′ − 1

v2v′)

+ P (x)

(u+

1

v

)+Q (x)

(u+

1

v

)2

= R (x)

Reordenando los terminos de la ecuacion anterior, tenemos que:

u′ + P (x)u+Q (x)u2

− v′

v2+ P (x)

1

v+Q (x)

2u

v+

1

v2

= R (x)

32

Page 35: Apuntes MAT-023 USM

Apuntes Mat023 (Segundo semestre 2014)

Ahora bien, como u es una solucion particular de la ecuacion (1.29), obtenemos:

− v′

v2+ P (x)

1

v+Q (x)

2u

v+

1

v2

= 0

Amplificando la ecuacion anterior por −v2, finalmente se obtiene la ecuacion:

v′ − P (x) v −Q (x)u (x) v −Q (x) = 0

Es decir:

v′ − P (x) + u (x)Q (x) v = Q (x)

que es una ecuacion lineal de primer orden.

Ejemplo 1.3.15. Resuelva la ecuacion de Ricatti:

y′ = y2 − 2

x2(1.32)

Solucion. Notamos primeramente que u = 1x

es una solucion particular de la ecuacion.

En efecto:

u2 − 2

x2=

1

x2− 2

x2

= − 1

x2

= u′

Ahora bien, sea y = 1x

+ 1v, luego:

dy

dx= − 1

x2− 1

v2

dv

dx

Reemplazando la ecuacion anterior y el cambio de variables y = 1x

+ 1v

en la ecuacion (1.32),

obtenemos:

− 1

x2− 1

v2

dv

dx=

(1

x+

1

v

)2

− 2

x2

Simplificando y agrupando terminos semejantes, se tiene:

− 1

v2

dv

dx=

2

vx+

1

v2

Amplificando por −v2, finalmente se obtiene:

v′ +2

xv = −1

Como se puede observar, la ecuacion anterior es una ecuacion lineal de primer orden cuya

solucion exacta es:

v =C

x2− x

3

33

Page 36: Apuntes MAT-023 USM

Apuntes Mat023 (Segundo semestre 2014)

Sin embargo, recordemos que y = 1x

+ 1v. Luego, la solucion obtenida es:

y =1

x+

1Cx2− x

3

O bien:

y =1

x+

3x2

C − x3

Ejemplo 1.3.16. Obtenga una solucion del problema

dy

dx=

1

x2y2 − 1

xy + 1

y (1) = 3

Indicacion:Primero buscar una solucion dela ecuacion de la forma y = ax+ b.

Solucion. Usamos la indicacion para buscar una solucion (la ecuacion es de Ricatti)

entonces

x2a = (ax+ b)2 − x (ax+ b) + x2

se sigue

ax2 =(a2 − a+ 1

)x2 + (2ab− b)x+ b2

ası

b = 0

y

a = a2 + 1− a

que tiene solucion a = 1, se sigue que una solucion es

y = x

(la cual no cumple y (1) = 3) hacemos el cambio de variables

y =1

u+ x

entonces

− 1

u2

du

dx+ 1 =

1

x2

(1

u+ x

)2

− 1

x

(1

u+ x

)+ 1

ası

− 1

u2

du

dx+ 1 =

1

ux+

1

u2x2+ 1

eliminando

− 1

u2

du

dx=

1

ux+

1

u2x2

34

Page 37: Apuntes MAT-023 USM

Apuntes Mat023 (Segundo semestre 2014)

luegodu

dx= −u

x− 1

x2

es lineal y tiene solucion

u =C

x− 1

xlnx

ası, como y = 1u

+ x se sigue

y =1

Cx− 1

xlnx

+ x

pero y (1) = 3 ası

3 =1

C+ 1

luego C = 12

reemplazando

y =1

12x− 1

xlnx

+ x

es la solucion del problema.

Ejemplo 1.3.17. Resolver la ecuacion

y′ − xy2 + (2x− 1) y = x− 1

si se sabe que tiene una solucion constante.

Solucion. La solucion constante es y = 1 (verificar) entonces podemos hacer el cambio

y =1

z+ 1

dy

dx= −z−2 dz

dx

reemplazamos

−z−2 dz

dx− x

(1

z+ 1

)2

+ (2x− 1)

(1

z+ 1

)− x+ 1 = 0

−z−2 dz

dx− 1

z2(x+ z) = 0

asıdz

dx+ (x+ z) = 0

la cual es lineal, la solucion es z = Ae−x − x+ 1 ası

y =1

Ae−x − x+ 1+ 1

35

Page 38: Apuntes MAT-023 USM

Apuntes Mat023 (Segundo semestre 2014)

Ecuaciones homogeneas

Definicion 1.3.7. Una funcion f : U ⊆ R2 → R se dice homogenea de grado n si:

f (tx, ty) = tn f (x, y)

para todo t ∈ R tal que (tx, ty) ∈ U .

Observacion 1.3.12. Si M y N son funciones homogeneas de grado n, entonces la

ecuacion:

M (x, y) dx+N (x, y) dy = 0 (1.33)

se escribe como:dy

dx= −M (x, y)

N (x, y)= −

M(1, y

x

)N(1, y

x

) = F(yx

)donde F : R→ R. Luego, la ecuacion (1.33) puede escribirse como:

y′ = F(yx

)Definicion 1.3.8. La ecuacion de primer orden:

y′ = f (x, y)

se llama homogenea, si la funcion f (x, y) es homogenea de grado 0.

Observacion 1.3.13. En vista de la observacion anterior, toda ecuacion homogenea puede

escribirse en la forma:

y′ = F(yx

)(1.34)

haciendo t = 1x. Para resolver esta ecuacion se considera el cambio de variables z = y

x; o

bien, y = zx. Luego, al derivar respecto de x tenemos que:

dy

dx= z + x

dz

dx

Reemplazando la ecuacion anterior en la ecuacion (1.34), se obtiene:

z + xdz

dx= F (z)

la cual es una ecuacion de variable separable. Finalmente, podemos escribir:∫dz

F (z)− z=

∫dx

x+ C

con C una constante arbitraria.

36

Page 39: Apuntes MAT-023 USM

Apuntes Mat023 (Segundo semestre 2014)

Ejemplo 1.3.18. Resuelva la ecuacion diferencial:

y′ =x+ y

x− y

Solucion. Se trata de una ecuacion con funcion homogenea de grado cero pues

y′ =x+ y

x− y=

1 + yx

1− yx

= F(yx

)ponemos u = y

xluego u+ xu′ = y′ reemplazando

u+ xu′ =1 + u

1− u

luego

du

dx=

1

x

(1 + u

1− u− u)

=1

x

(u2 + 1

1− u

)esta ecuacion es de variables separadas, se sigue∫

1− u1 + u2

du =

∫dx

x

luego

arctanu− 1

2ln(1 + u2

)= ln |x|+ C

volvemos a la variable

arctan(yx

)− 1

2ln

(1 +

(yx

)2)

= ln |x|+ C

Ejemplo 1.3.19. Resuelva el P.V.I.

xdy

dx= y +

(x2 − y2

)1/2

y (1) = 0

Solucion. Note que

dy

dx=y

x+

(1−

(yx

)2)1/2

es homogenea, hacemos el cambio

u =y

x⇒ u+ x

du

dx=

dy

dx

37

Page 40: Apuntes MAT-023 USM

Apuntes Mat023 (Segundo semestre 2014)

luego

u+ xdu

dx= u+

√1− u2

se siguedu

dx=

√1− u2

x

resolvemos esta ecuacion de variables separables∫du√

1− u2=

∫dx

x

arcsinu = ln |x|+ C

ası

u = sin (ln |x|+ C)

luego

y = x sin (ln |x|+ C)

evaluando

0 = sin (ln |1|+ C) = sinC

se sigue C = kπ con k ∈ Z, ası

y = x sin (ln |x|+ kπ) con k ∈ Z

Otros cambios de variables

Observacion 1.3.14. La regla de la cadena nos permite cambiar variables en las ecuaciones

diferenciales para llevarlas a ecuaciones diferenciales que se resuelven mediante los metodos

elementales. Consideremos los siguientes ejemplos:

Ejemplo 1.3.20. Recordemos la ecuacion (1.7) obtenida en el problema del espejo pa-

rabolico. Es decir, la ecuacion:y

x=

2y′

1− (y′)2

Utilizando un cambio de variables adecuado, resolveremos esta ecuacion diferencial. En

primer lugar, despejando y′ de la ecuacion, se obtiene:

y (y′)2

+ 2xy′ − y = 0

Por la formula de la ecuacion de segundo grado, encontramos que:

y′ =−x±

√x2 + y2

y

38

Page 41: Apuntes MAT-023 USM

Apuntes Mat023 (Segundo semestre 2014)

Por la simetrıa de la curva y = f (x) y el hecho que |x| <√x2 + y2, se obtiene que y′ > 0.

Luego:

y′ =−x+

√x2 + y2

y(1.35)

Consideremos, ahora, el cambio de variables z = x2 + y2. Entonces:

dz

dx= 2x+ 2y

dy

dx

Reemplazando la ecuacion anterior en la ecuacion (1.35), se obtiene:

dz

dx= 2x+ 2y

(−x+

√x2 + y2

y

)= 2x− 2x+ 2

√x2 + y2

Es decir, obtenemos la ecuacion de variable separable:

dz

dx= 2√z

Ası: ∫dz

2√z

=

∫dx+ C

implica que: √z = x+ C

Por tanto, elevando al cuadrado y como z = x2 + y2, se obtiene finalmente:

x2 + y2 = x2 + 2Cx+ C2

O bien:

y2 = 2Cx+ C2

con C ∈ R.

Ejemplo 1.3.21. Resuelva la ecuacion diferencial:

dy

dx= (x+ y + 1)2 − 2

utilizando para ello un cambio de variables adecuado.

Solucion. Haciendo z = x+ y + 1, tenemos z′ = 1 + y′, de modo que:

z′ − 1 = z2 − 2

o bien:

z′ = z2 − 1

39

Page 42: Apuntes MAT-023 USM

Apuntes Mat023 (Segundo semestre 2014)

Separando variables, obtenemos: ∫dz

z2 − 1=

∫dx+ C

Integrando se tiene que:

1

2ln (z − 1)− ln (z + 1) = x+ C

o:

lnz − 1

z + 1= 2x+ C

Luego:z − 1

z + 1= e2x+C

Es decir, obtenemos:

z − 1 = Ce2x (z + 1)

Despejando z y recordando que z = x+ y + 1, se obtiene finalmente:

y =1 + Ce2x

1− Ce2x− x− 1

Ejemplo 1.3.22. Para un valor de n ∈ N adecuado, el cambio de variables u = xny

transforma la ecuacion diferencial

x2 dy

dx+ 2xy =

√1− y2x4x2

en una ecuacion de variables separables. Determine tal valor de n y resolver la ecuacion.

Solucion. Note que

y = x−nu

luegody

dx= −nx−n−1u+ x−n

du

dx

reemplazando

x2

(−nx−n−1u+ x−n

du

dx

)+ 2x

(x−nu

)=

√1− (x−nu)2 x4x2

ordenando

−nx−n+1u+ x2−ndu

dx+ 2

(x−n+1u

)=√

1− (x4−2nu2)x2

se sigue

x2−ndu

dx+ (2− n)

(x−n+1u

)=√

1− (x4−2nu2)x2

40

Page 43: Apuntes MAT-023 USM

Apuntes Mat023 (Segundo semestre 2014)

tomando n = 2 obtenemosdu

dx=√

1− u2x2

que es de variables separables, se sigue∫du√

1− u2=

∫x2dx

ası

arcsinu =x3

3+ C

luego u = sin(x3

3+ C

)y finalmente

y =sin(x3

3+ C

)x2

Ejemplo 1.3.23. Resolver la ecuacion

yy′′ = y′y2 + (y′)2

haciendo el cambio u = y′.

Solucion. Si u = y′ entonces

y′′ =du

dy

dy

dx

= udu

dy

reemplazando

yudu

dy= uy2 + (u)2

ydu

dy= y2 + u

esta ecuacion es linealdu

dy− 1

yu = y

que tiene solucion

u = y2 + cy

luegody

dx= y2 + cy

es de variables separadas, tiene solucion

y (x) =c

Ae−cx − 1

Obs.: Esta tecnica reduce el orden de la E.D.O. si es de la forma F (y, y′, y′′) es decir,

no depende de la variable x.

41

Page 44: Apuntes MAT-023 USM

Apuntes Mat023 (Segundo semestre 2014)

Ejemplo 1.3.24. Resuelva la ecuacion diferencial:

3xy2y′ + xex2

+ y3 = 0

Solucion. Considere el cambio de variables u = y3. Luego, u′ = 3y2y′, entonces la ecuacion:

3xy2y′ + xex2

+ y3 = 0

queda como:

xu′ + xex2

+ u = 0

la cual es lineal. Ası, si x 6= 0, la ecuacion queda como:

u′ +1

xu = −ex2

Denotando por µ = µ(x) el factor de integracion de la ecuacion anterior, tenemos que:

µ(x) = e∫

1xdx = eln |x| = |x|

Por tanto, si x > 0 la solucion u esta dada por:

u =1

µ(x)

(−∫µ(x)ex

2

dx+ C)

=1

|x|

(−∫|x|ex2 dx+ C

)=

1

x

(−∫xex

2

dx+ C)

=C

x− ex

2

2x

Por otro lado, si x < 0, entonces:

u =1

|x|

(−∫|x|ex2 dx+ C

)= −1

x

(∫xex

2

dx+ C)

= −1

x

(ex22

+ C)

=C

x− ex

2

2x

Por tanto, si x 6= 0 tenemos que:

u =C

x− ex

2

2x

42

Page 45: Apuntes MAT-023 USM

Apuntes Mat023 (Segundo semestre 2014)

con C ∈ R. Finalmente, como u = y3, obtenemos que la solucion de la ecuacion diferencial

esta dada por:

y3 =C

x− ex

2

2x⇔

y =3

√C

x− ex2

2x

con C una constante arbitraria.

Observacion 1.3.15. Como muestran los ejemplos anteriores, en general, se puede efectuar

cualquier cambio de variables o sustitucion que se desee al intentar resolver una ecuacion

diferencial. Sin embargo, cualquier no asegura el exito en facilitar la resolucion de la

ecuacion diferencial. Por tanto, debe buscarse el cambio de variables adecuado, [4].

Ejercicios de la seccion

1. Resuelva las siguientes ecuaciones mediante integracion directa

(a)(x2 + 4

) dy

dx= 4 (b)

dy

dx= x lnx

(c)1

arctanxy′ =

1

(1 + x2)(d) e−x

dy

dx= sinx

2. Resuelva las siguientes ecuaciones de variables separables:

(a) yy′ − x = xy2 (b) y′ = ex+y

(c) y ln y + xy′ = 0 (d) (1 + ex) y′ = ey

(e) y′ = 1 + x+ y + xy (f) y′ = x2y2 + x2

3. Verifique que las siguientes ecuaciones son homogeneas y resuelvalas:

(a) 3x− y +dy

dx(2y − x) = 0 (b) x

dy

dx= y +

√y2 − x2

(c) 4x2 + xy − 3y2 +dy

dx

(−5x2 + 2xy + y2

)= 0 (d)

(3x2 − y2

) dy

dx= 2xy

(e)dy

dx=x+ y

x− y(f) x2y

dy

dx= x3 + y3

4. Resuelva las siguientes ecuaciones lineales de primer orden:

(a) xy′ + y = xex (b) y′ + y cos (x) = sin (x) cos (x)

(c) y′ + 2y = x2 + 2x (d) y′ + 2y = sinx

(e)(1 + x2

)y′ + xy = 1 (f) y′ +

1

xy =

1

x2

43

Page 46: Apuntes MAT-023 USM

Apuntes Mat023 (Segundo semestre 2014)

5. Resolver las siguientes ecuaciones de Bernoulli:

1.- y′ − 4y = 2exy1/2 3.- xy′ − 2y = 4x3y1/2

2.- y′ − y + y2 (x2 + x+ 1) = 0 4.- xy′ + y = y2x2 lnx

6. Las ecuaciones en este ejercicios pueden ser transformadas en lineales mediante un

cambio de variables adecuado, descubra tal cambio y resuelva la ecuacion:

a) y′ + x tan y = x2 sec y

b) 2xyy′ + (1 + x) y2 = ex

7. Sean A,B,C,D,E, F constantes. Muestre que utilizando un cambio de coordenadas

conveniente podemos transformar una ecuacion del tipo

dx

dt= f

(Ax+Bt+ C

Dx+ Et+ F

)en una ecuacion de variables separables o en una homogenea. Ind.: x = X+h, y = Y +k

donde h, k son constantes por determinar.

8. Resolver el P.V.I.

dx

dt=

2x+ t− 1

x+ 2t+ 1x (0) = 1

9. Resuelva la ecuacion de Ricatti:

y′ −(2x3 + 1

)y = −x2y2 − x4 − x+ 1

sabiendo que u (x) = x es una solucion particular de la ecuacion.

10. Si u (x) = x2 es una solucion particular de la ecuacion:

y′ +(3− 2x2 sinx

)y = −y2 sinx+ 2x+ 3x2 − x4 sinx

Hallar una familia infinita de soluciones.

Modelos simples: Segunda parte

Observacion 1.4.1. Ahora consideraremos algunos ejemplos mas: el problema de mezclas,

o en terminos mas generales, el problema de analisis de compartimientos, [4], el problema

de las curvas de persecucion y un problema geometrico.

44

Page 47: Apuntes MAT-023 USM

Apuntes Mat023 (Segundo semestre 2014)

Observacion 1.4.2 (Analisis de compartimientos, [4]). Un proceso fısico o biologico com-

plejo puede ser dividido algunas veces en varios estados distintos. El proceso total puede

describirse por la interaccion entre los estados individuales. Cada estado se llama comparti-

miento (lo podemos considerar como un tanque) y se supone ademas que el contenido de

cada compartimiento esta mezclado homogeneamente. En cada compartimiento se transfiere

material que es inmediatamente incorporado al siguiente en el sistema.

Considere un sistema formado por un solo compartimiento, suponga que un material es

introducido en tal compartimiento a una razon e (t), el cual se incorpora a una cantidad

x (t) de material existente al interior del compartimiento, y luego se extrae material (que

puede pasar a otro compartimiento) a una razon de s (t). Por consiguiente, la variacion de

material al interior del compartimiento esta dada entonces por la ecuacion diferencial:

dx

dt= e (t)− s (t)

Consideremos algunos ejemplos:

Ejemplo 1.4.1. Considere un tanque que contiene 100 litros de agua, en el cual se han

disuelto 50 kilogramos de sal. Suponga que 2 litros de salmuera cada uno con 1 kilogramo

de sal disuelta, entran por minuto al tanque, y la mezcla que se mantiene homogenea

revolviendola a gran velocidad, sale del tanque a razon de 2 litros por minuto. Hallar la

cantidad de sal al interior del tanque en el tiempo t.

Solucion. Sea x (t) el numero de kilogramos de sal disueltos en el tanque en t minutos.

Notamos que las unidades ayudan bastante en la extraccion de informacion. En efecto,

dx/dt esta en [kg/mın] y entonces, e (t) y s (t) deben esta en las mismas unidades. Ası:

e (t) = 2

[kg

lt

] [lt

mın

]= 2

[kg

mın

]45

Page 48: Apuntes MAT-023 USM

Apuntes Mat023 (Segundo semestre 2014)

y

s (t) = 2

[lt

mın

]· x (t)

100

[kg

lt

]=x (t)

50

[kg

mın

]Por tanto, la ecuacion diferencial queda:

dx

dt= 2− x (t)

50

la cual es una ecuacion lineal de primer orden. Ası, por la formula de Leibnitz, tenemos

que:

x (t) = e−t/50

2

∫et/50dt+ C

= 100 + Ce−t/50

pero sabemos que en t = 0, x (0) = 50. Ası, 50 = 100 + C. Por tanto:

x (t) = 100− 50e−t/50

Observacion 1.4.3 (Curvas de persecucion, [4]). Utilizando la propiedad geometrica de

que la pendiente de la recta tangente a una curva y en un punto dado de la curva es y′, se

pueden construir ecuaciones diferenciales que permiten estudiar la trayectoria que describe

un depredador tras su presa.

Consideremos algunos ejemplos:

Ejemplo 1.4.2. Un esquiador acuatico P localizado en el punto (a, 0), con a > 0, es

halado por un bote de motor Q localizado en el origen y que viaja hacia arriba a lo largo

del eje Y . Hallar la trayectoria del esquiador si este se dirige en todo momento hacıa el

bote. La trayectoria se denomina tractriz.

Solucion. Observemos que la recta que une P y Q, digamos←→PQ es tangente al camino

recorrido por P . Por tanto, su pendiente esta dada por:

dy

dx= −√a2 − x2

x(1.36)

46

Page 49: Apuntes MAT-023 USM

Apuntes Mat023 (Segundo semestre 2014)

puesto que la longitud del segmento PQ es a. La ecuacion diferencial (1.36) es de integracion

directa, luego:

y = −∫ √

a2 − x2

xdx+ C

Ası1:

y = a ln

(a+√a2 − x2

x

)−√a2 − x2 + C

Como y = 0 cuando x = a, vemos que C = 0, de modo que la ecuacion de la trayectoria es:

y = a ln

(a+√a2 − x2

x

)−√a2 − x2

Ejemplo 1.4.3. Suponga que un halcon P situado en el punto (a, 0) descubre una paloma

Q en el origen, la cual vuela a lo largo del eje Y a una velocidad v. El halcon emprende el

vuelo inmediatamente hacia la paloma a una velocidad w. ¿Cual sera el camino seguido

por el halcon en su vuelo?

Solucion. Sea t = 0 el instante en que el halcon comienza a volar hacia la paloma. Despues

de t segundos la paloma estara en el punto Q = (0, vt) y el halcon en P (x, y). Como la

recta T =←→PQ es otra vez tangente a la trayectoria, encontramos que su pendiente mT = y′.

Luego, se obtiene la ecuacion diferencial:

y′ =y − vtx

(1.37)

Debemos ahora eliminar t de la ecuacion anterior, pues y′ = dy/dx. Para ello debemos

calcular la longitud del camino recorrido por el halcon. Si ds representa un elemento

diferencial de longitud del arco formado por la trayectoria, tenemos que:

wt =

∫ a

x

ds

pero ds =√

1 + (y′)2dx. Ası, la formula anterior queda como:

t =1

w

∫ a

x

√1 + (y′)2dx

Despejando t de la ecuacion (1.37) e igualando con la ecuacion anterior, obtenemos:

y − xy′

v=

1

w

∫ a

x

√1 + (y′)2dx

1

∫ √a2 ± x2

xdx =

√a2 ± x2 − a ln

∣∣∣∣∣a +√a2 ± x2

x

∣∣∣∣∣+ C

47

Page 50: Apuntes MAT-023 USM

Apuntes Mat023 (Segundo semestre 2014)

Derivando:y′ − (y′ + xy′′)

v=

1

w

√1 + (y′)2

Ordenando la ecuacion anterior, se tiene que:

xy′′ =v

w

√1 + (y′)2 (1.38)

Note que la ecuacion anterior es una ecuacion diferencial de segundo orden. Sin embargo,

mediante el cambio de variables:

u = y′

la ecuacion (1.38) queda como:

xu′ =v

w

√1 + u2

que es una ecuacion de primer orden de variable separable. Entonces, separando variables

e integrando, obtenemos: ∫du√

1 + u2=v

w

∫dx

x+ C

Luego:

ln(u+√

1 + u2)

=v

wlnx+K

pero u = y′ = 0 cuando x = a, se sigue que K = (v/w) ln a. Tomando exponenciales a

ambos lados de la ecuacion anterior se tiene:

u+√

1 + u2 =(xa

)v/wque, despues de algunas operaciones algebraicas, nos da:

dy

dx=

1

2

(xa

)v/w−(xa

)−v/wSuponiendo que w > v, se obtiene finalmente que:

y =a

2

(x/a)1+v/w

1 + v/w− (x/a)1−v/w

1− v/w+ C

Ejemplo 1.4.4. Determine la curva que pasa por(

12, 3

2

)y corta a cada miembro de la

familia x2 + y2 = c2 con c ∈ R+ formando un angulo de 45o.

Solucion. El angulo entre dos curvas es dado por el angulo entre sus rectas tangentes

luego

tan θ =m1 −m2

1 +m1m2

48

Page 51: Apuntes MAT-023 USM

Apuntes Mat023 (Segundo semestre 2014)

note que la pendiente de las rectas tangentes a la familia de curvas es

2x+ 2yy′ = 0⇒ m1 =−xy

ası

tan(π

4

)=

−xy− y′

1− xyy′

esto es

1 =

−xy− y′

1− xyy′

se sigue

1− x

yy′ =

−xy− y′

⇔1 +

x

y=

x

yy′ − y′

luegody

dx=

1 + xy

xy− 1

=yx

+ 1

1− yx

que es una ecuacion homogenea , hacemos el cambio u = yx

de donde

u+ xdu

dx=u+ 1

1− u

luego

xdu

dx=

u+ 1

1− u− u

⇔du

dx=

(u2 + 1

1− u

)1

x

resolvemos ∫1− u1 + u2

du =

∫1

xdx

se sigue

arctanu− 1

2ln(1 + u2

)= ln |x|+ C

volvemos a la variable y,

arctan(yx

)− 1

2ln(x2 + y2

)= C

49

Page 52: Apuntes MAT-023 USM

Apuntes Mat023 (Segundo semestre 2014)

y determinamos la constante con el punto(

12, 3

2

)arctan (3)− 1

2ln

(5

2

)= C

ası la curva es

arctan(yx

)− 1

2ln(x2 + y2

)= arctan 3− 1

2ln

(5

2

)Ejemplo 1.4.5. Hallar una curva C : y = f(x), con x > 0, que pase por el punto

(x0, y0) = (1, 1) y que tenga la propiedad de que el segmento sobre la recta tangente a dicha

curva, trazado entre un punto de tangencia P (x, y) y un punto en el eje y, es bisectado

por el eje x.

Solucion. Considere el diagrama siguiente, asociado al problema geometrico:

x

yyyyy

y = f(x)

P (x, y)

x

y

x2

Q

αPM B

De la figura e hipotesis del problema, notamos que el punto PM es el punto medio del

segmento PQ, donde P = P (x, y) es el punto de tangencia a la curva incognita y = f(x).

Por tanto, del triangulo 4PMBP rectangulo en B, se obtiene que:

tanα =yx2

Por otro lado, si denotamos por T la recta tangente a la curva y = f(x), entonces PQ ⊆ T .

Ası:

y′ = mT = tanα

Ahora bien, la curva C : y = f(x) debe pasar por (x0, y0) = (1, 1). Por tanto, obtenemos

el problema de valor inicial:

y′ =2y

x, y(1) = 1

Separando variables e integrando obtenemos:∫dy

y= 2

∫dx

x+ C

50

Page 53: Apuntes MAT-023 USM

Apuntes Mat023 (Segundo semestre 2014)

Esto es:

ln |y| = 2 ln x+ C

O bien:

y = Cx2

pero y(1) = 1, luego C = 1. Por tanto, la curva con la propiedad buscada en el problema

es:

C : y = x2

Ejemplo 1.4.6. Un paracaidista se deja caer desde un avion y despues de 5,5 sg. abre

el paracaıdas provocando una fuerza de resistencia del aire igual wv2

256[lbs.] , donde w es

el peso total del hombre y del paracaıdas y v la velocidad con que va cayendo. Hallar la

velocidad en cualquier momento despues de abierto el paracaıdas. Asumir que antes de

abierto el paracaıdas no hay resistencia del aire.

Solucion. Primero modelamos la situacion antes de abrir el paracaıdas:

mdv

dt= mg ⇒ v = gt⇒ v (5,5) = (32) (5,5) = 176

ahora cuando se abre el paracaıdas ponemos t = 0 nuevamente pero con una velocidad

inicial

mdv

dt= mg − mgv2

256v (0) = 176

asıdv

dt= 32− 32v2

256

la cual tiene solucion

v (t) = −16C + 16e4t

C − e4t

reemplazando la C.I.

176 = −16C + 16

C − 1

se obtiene C = 56

ası

v (t) = −16(

56

)+ 16e4t(

56

)− e4t

es decir

v (t) = 16

(6e4t + 5

6e4t − 5

)

51

Page 54: Apuntes MAT-023 USM

Apuntes Mat023 (Segundo semestre 2014)

Ejemplo 1.4.7. Suponga que un tanque cilındrico recto con radio de la base 12

metro

y altura 4 metros tiene inicialmente 2 litros de agua pura. Una solucion de salmuera se

bombea hacia el tanque a una rapidez de(1 + 1

1+t

)litros por minuto, la concentracion de

sal en el flujo de entrada es de 12

kilogramo por litro. La solucion en el tanque es homogenea

y se extrae a 11+t

litros por minuto. Determinar la cantidad de sal en el tanque cuando este

se llena.

Solucion. Sea x (t) la cantidad de sal en el tanque en minuto t entonces

dx

dt= (Sal que entra por minuto) - (Sal que sale por minuto)

notemos que

Sal que entra por minuto =

(1 +

1

1 + t

)[lt

min

]1

2

[kg

lt

]=

1

2

(1 +

1

1 + t

) [kg

min

]y

Sal que sale por minuto =

(1

1 + t

)[lt

min

](x (t)

2 + t

)[kg

lt

]=

(1

(1 + t) (2 + t)

)x (t)

[kg

min

]se sigue

dx

dt=

1

2

(1 +

1

1 + t

)−(

1

(1 + t) (2 + t)

)x

x (0) = 0

se trata de un problema de valores iniciales con E.D.O lineal de primer orden.

Resolvemos aplicando la tecnica del factor integrante

µ (t) = e∫

dt(1+t)(2+t) =

t+ 1

t+ 2

asıd

dt

(t+ 1

t+ 2x

)=

1

2

(1 +

1

1 + t

)(t+ 1

t+ 2

)=

1

2

se siguet+ 1

t+ 2x =

t

2+K

ası

x (t) =t

2

(t+ 2

t+ 1

)+K

(t+ 2

t+ 1

)52

Page 55: Apuntes MAT-023 USM

Apuntes Mat023 (Segundo semestre 2014)

como x (0) = 0 se sigue

K = 0

y ası

x (t) =

(t

2

)(t+ 2

t+ 1

)es la cantidad de sal en el tanque en el tiempo t. La cantidad de litros de agua en el tanque

es

l (t) = 2 + t

la cantidad de litros de capacidad del tanque es

V = π

(1

2

)2

4 metros3

= π m3

= 103π litros

los cuales se logran a

t = 103π − 2 minutos

y en la cantidad de sal es

x(103π − 2

)=

(103π − 2

2

)(103π

103π − 1

)≈ 1570. 3 kg

Ejercicios de la seccion

1. Determinar la curva que pasa por(

12, 3

2

)y corta a cada miembro de la familia de

curvas x2 + y2 = c2 formando un angulo de 30o.

2. Encuentre la curva que pertenece a la familia de trayectorias ortogonales de la familia

de curvas x+ y = cey que pasa por (0, 5).

3. Suponga que un halcon situado en (a, 0) descubre una paloma en el origen, la

cual vuela a lo largo del eje Y a una velocidad v; El halcon emprende el vuelo

inmediatamente hacia la paloma con una velocidad de w. ¿Cual es el camino seguido

por el halcon en su vuelo persecutorio?.

4. Un destructor esta en medio de una niebla muy densa que se levanta por un momento

y deja ver un submarino enemigo en la superficie a cuatro kilometros de distancia.

Suponga:

a) que el submarino se sumerge inmediatamente y avanza a toda maquina en una

direccion desconocida.

53

Page 56: Apuntes MAT-023 USM

Apuntes Mat023 (Segundo semestre 2014)

b) que el destructor viaja tres kilometros en lınea recta hacia el submarino.

¿Que trayectoria deberıa seguir el destructor para estar seguro que pasara directamente

sobre el submarino, si su velocidad v es tres veces la del submarino?

5. Suponga que el eje Y y la recta x = b forman las orillas de un rıo cuya corriente tiene

una velocidad v en la direccion negativa del eje Y . Un hombre esta en el origen y

su perro esta en el punto (b, 0). Cuando el hombre llama al perro, este se lanza al

rıo y nada hacia el hombre a una velocidad constante w (con w > v). ¿Cual es la

trayectoria seguida por el perro?.

6. Cuatro caracoles situados en las esquinas de un cuadrado [0, a]× [0, a] comienzan a

moverse con la misma velocidad, dirigiendose cada uno hacia el caracol situado a su

derecha. ¿Que distancia recorreran los caracoles al encontrarse?

7. Hallar la ecuacion de todas las curvas que tienen la propiedad de que el punto de

tangencia es punto medio del segmento tangente entre los ejes coordenados.

8. Un cuerpo se calienta a 110o C y se expone al aire libre a una temperatura de 100 C.

Si al cabo de una hora su temperatura es de 60o C. ¿Cuanto tiempo adicional debe

transcurrir para que se enfrıe a 30o C?

9. Una persona de un pueblo de 1000 habitantes regreso con gripe. Si se supone que la

gripe se propaga con una rapidez directamente proporcional al numero de agripados

como tambien al numero de no agripados. Determinar el numero de agripados cinco

dıas despues, si se observa que el numero de agripados el primer dıa es 100.

10. Un colorante solido disuelto en un lıquido no volatil, entra a un tanque a una velocidad

v1 galones de solucion/minuto y con una concentracion de c1 libras de colorante/galon

de solucion. La solucion bien homogeneizada sale del tanque a una velocidad de v2

galones de solucion/min. y entra a un segundo tanque del cual sale posteriormente a

una velocidad de v3 galones de solucion/min.

54

Page 57: Apuntes MAT-023 USM

Apuntes Mat023 (Segundo semestre 2014)

Inicialmente el primer tanque tenıa P1 libras de colorante disueltas en Q1 galones

de solucion y el segundo tanque P2 libras de colorante disueltas en Q2 galones de

solucion. Encontrar dos ecuaciones que determinen las libras de colorante presentes

en cada tanque en cualquier tiempo t.

11. Un teatro de dimensiones 10 × 30 × 50m3, contiene al salir el publico 0,1 % por

volumen de CO2. Se sopla aire fresco a razon de 500 m3 por minuto y el sistema

de aire acondicionado lo extrae a la misma velocidad. Si el aire atmosferico tiene

un contenido de CO2 del 0,04 % por volumen y el lımite saludable es de 0,05 % por

volumen. ¿ En que tiempo podra entrar el publico?.

12. Un tanque contiene inicialmente agua pura. Salmuera que contiene 2 libras de sal/gal.

entra al tanque a una velocidad de 4 gal./min. Asumiendo la mezcla uniforme, la

salmuera sale a una velocidad de 3 gal./min. Si la concentracion alcanza el 90 % de

su valor maximo en 30 minutos, calcular los galones de agua que habıan inicialmente

en el tanque.

13. Un tanque de una cierta forma geometrica esta inicialmente lleno de agua hasta una

altura H. El tanque tiene un orificio en el fondo cuya area es A pie2. Se abre el orificio

y el lıquido cae libremente. La razon volumetrica de salida dQdt

es proporcional a la

velocidad de salida y al area del orificio, es decir,

dQ

dt= −kAv

aplicando la ecuacion de energıa 12mv2 = mgh se obtiene v =

√2gh donde g = 32

pie/seg2.

La constante k depende de la forma del orificio:

a) Si el orificio es de forma rectangular, la constante k = 0, 8.

b) Si el orificio es de forma triangular, la constante 0, 65 ≤ k ≤ 0, 75.

c) Si el orificio es de forma circular, la constante k = 0, 6.

Con estos datos:

a) Un tanque semiesferico tiene un radio de 1 pie; el tanque esta inicialmente lleno

de agua y en el fondo tiene un orificio de 1 pulg. de diametro. Calcular el tiempo

de vaciado.

b) Modelar el caso: Cilindro circular de altura H0 pies y radio r pies, dispuesto en

forma vertical y con un orificio circular de diametro ρ (pulgadas), suponga que

esta lleno de agua y calcule el tiempo de vaciado.

55

Page 58: Apuntes MAT-023 USM

Apuntes Mat023 (Segundo semestre 2014)

14. Un torpedo se desplaza a una velocidad de 60 millas/hora en el momento de agotarse

el combustible; si el agua se opone al movimiento con una fuerza proporcional a su

velocidad y si en una milla de recorrido reduce su velocidad a 30 millas/hora. ¿A que

distancia se detendra?

15. Una bala se introduce en una tabla de h = 10 cm. de espesor con una velocidad v0 =

200 m/seg, traspasandola con v1 = 80 m/seg. Suponiendo que la resistencia de la

tabla al movimiento de la bala es proporcional al cuadrado de la velocidad. Hallar el

tiempo que demora la bala en atravesar la tabla.

16. Una cadena de 4 pies de longitud tiene 1 pie de longitud colgando del borde de una

mesa. Despreciando el rozamiento, hallar el tiempo que tarda la cadena en deslizarse

fuera de la mesa.

Analisis cualitativo

Es equivocado pensar que el objetivo principal del estudio de las ecuaciones diferenciales

consiste en encontrar artificios de calculo que permitan resolverlas. Anteriormente presenta-

mos una seleccion de tecnicas que permiten resolver algunas ecuaciones diferenciales. Como

en toda seleccion la lista no es completa. Existen tratados en donde se elaboran tablas de

soluciones de manera analoga a las tablas de antiderivadas.

La pericia para resolver ecuaciones diferenciales va perdiendo poco a poco importancia

con la llegada de los computadores y el diseno de software especializado para computacion

simbolica. La tendencia actual es dejar al computador este tipo de tareas de calculo. Un

programa como Mathematica puede resolver mediante instrucciones sencillas casi todas las

ecuaciones diferenciales tratadas en este curso.

Sin quitarle importancia a este tipo de programas debe quedar claro que ni el mas

refinado de los software ni el mas ingenioso de los matematicos puede resolver en terminos

de funciones elementales todas las ecuaciones diferenciales, ni siquiera las mas importantes

de ellas. El problema mas que de habilidad es de principio. En casos tan simples como

dx

dt= −1

x+ t

se desconocen soluciones clasicas. La busqueda de recetas para resolver todas las ecuaciones

diferenciales en terminos de funciones elementales es una busqueda sin esperanzas. Ante este

hecho se presentan algunas alternativas: Los metodos cualitativos, los metodos numericos,

y los metodos de aproximacion. No es parte de los objetivos de estas notas un estudio

detallado al respecto. Se quiere sin embargo ilustrar los metodos cualitativos.

56

Page 59: Apuntes MAT-023 USM

Apuntes Mat023 (Segundo semestre 2014)

Metodos cualitativos

En muchos problemas, mas que calculos cuantitativos puntuales, lo que interesa es el

comportamiento cualitativo de las soluciones en terminos de las condiciones iniciales o de

valores de los parametros. Saber que una solucion es creciente, que es concava o que tiene

un lımite en el infinito puede ser de ayuda en el entendimiento de un modelo. Ocurre, que

bajo ciertas circunstancias, podemos obtener tal informacion sin resolver explıcitamente la

ecuacion diferencial. Analizaremos primero el siguiente modelo:

El modelo de Verhulst

Resumiremos los principales resultados concernientes al modelo de Verhulst para la

dinamica de poblacionesdx

dt= x (a− bx) (1.39)

donde a, b > 0, esta ecuacion tiene dos soluciones constantes x1 (t) = 0 y x2 (t) = ab. Estas

soluciones dividen al plano xt en 3 regiones de poblaciones

R1 =

(t, x) :a

b< x

, R2 =

(t, x) : 0 < x <

a

b

, R3 = (t, x) : x < 0

tales que el grafico de cualquier solucion no constante x = x(t) de (1.39) permanece

confinado en una y solo una de estas regiones. Mas aun, podemos determinar cuando es

creciente, y cuando es decreciente la solucion x = x(t) de (1.39) a partir de la condicion

inicial x(t0) = x0.

∫dx

x (a− bx)=

∫dt∫ (

1

ax+

1

a

b

a− bx

)= t+ C(

1

aln |x| − 1

aln |a− bx|

)= t+ C

ln

∣∣∣∣ x

a− bx

∣∣∣∣ = at+ C

se sigue xa−bx = Keat,

x (t) =Kaeat

Kbeat + 1=

a

b+ Ce−at

1. Si x0 < 0 entonces x0 = ab+Ce−at0

, entonces C = − 1e−at0

(b− a

x0

), se sigue

x (t) =ax0

bx0 + (a− bx0) e−a(t−t0)

57

Page 60: Apuntes MAT-023 USM

Apuntes Mat023 (Segundo semestre 2014)

note que ax0 < 0 y

bx0 + (a− bx0) e−a(t−t0) = 0

⇔(a− bx0) e−a(t−t0) = −bx0

⇔e−a(t−t0) =

−bx0

(a− bx0)> 0

ası

T = −1

aln−bx0

(a− bx0)+ t0 > t0

el intervalo de definicion de esta solucion es]−∞,− 1

aln −bx0

(a−bx0)+ t0

[≡ ]−∞, T [ en

este intervalo

d

dt

ax0

bx0 + (a− bx0) e−a(t−t0)=

a2x0e−a(t−t0) (a− bx0)

(bx0 + aeat0−at − bx0eat0−at)2 < 0

la funcion es estrictamente decreciente y

lımt→T−

ax0

bx0 + (a− bx0) e−a(t−t0)= −∞

Diremos que esta solucion explota en un tiempo finito. Note que esta solucion no

representara una solucion asociada al problema de poblaciones pues siempre es

negativa.

2. Si 0 < x0 <ab

entonces x (t) = ax0bx0+(a−bx0)e−a(t−t0)

esta bien definida en todo R pues el

denominador no se anula

0 < bx0 + (a− bx0) e−a(t−t0)

ademas

0 < x (t) =ax0

bx0 + (a− bx0) e−a(t−t0)<a

b

yd

dt

ax0

bx0 + (a− bx0) e−a(t−t0)=

a2x0e−a(t−t0) (a− bx0)

(bx0 + aeat0−at − bx0eat0−at)2 > 0

la funcion es estrictamente creciente y

lımt→+∞

ax0

bx0 + (a− bx0) e−a(t−t0)=a

b

ademas

lımt→−∞

ax0

bx0 + (a− bx0) e−a(t−t0)= 0

58

Page 61: Apuntes MAT-023 USM

Apuntes Mat023 (Segundo semestre 2014)

3. Si x0 >ab

entonces x (t) = ax0bx0+(a−bx0)e−a(t−t0)

esta bien definida en]− 1a

ln −bx0(a−bx0)

+ t0,+∞[≡

]T,+∞[ donde T < t0 ademas

lımt→+∞

ax0

bx0 + (a− bx0) e−a(t−t0)=a

b

yd

dt

ax0

bx0 + (a− bx0) e−a(t−t0)=

a2x0e−a(t−t0) (a− bx0)

(bx0 + aeat0−at − bx0eat0−at)2 < 0

la funcion es estrictamente decreciente, x (t) > ab.

En el grafico se muestra el comportamiento de las soluciones

Veremos ahora que es posible analizar los comportamientos de las soluciones sin la

necesidad de resolver la ecuacion.

Ecuaciones diferenciales autonomas

Definicion 1.5.1. Diremos que una ecuacion diferencial de primer orden es autonoma si

se puede expresar en la formadx

dt= f (x) (1.40)

donde f : Ω→ R es una funcion definida en el intervalo abierto Ω.

Las ecuaciones dxdt

= x (1− x2); dxdt

= x (1− x); dxdt

= e−x2

sinx son ejemplos de

ecuaciones autonomas, mientras la ecuacion dxdt

= etx+ t no lo es.

59

Page 62: Apuntes MAT-023 USM

Apuntes Mat023 (Segundo semestre 2014)

Teorema 1.5.1. Sean f : Ω→ R una funcion de clase C1 (Ω), x0 ∈ Ω y t0 ∈ R entonces

existe una unica solucion del P.V.I.

dx

dt= f (x)

x (t0) = x0

x : I → R de clase C1 (I) definida en un intervalo abierto que contiene a t0.

Definicion 1.5.2. Llamaremos intervalo maximal de definicion al mayor intervalo abierto

donde esta definida la solucion del P.V.I. anterior.

Ejemplo 1.5.1. Determine el intervalo maximal de la solucion del P.V.I.

dx

dt= x2

x (0) = 2

Solucion. Aplicando la tecnica de separacion de variables∫x′ (t)

x (t)2dt =

∫1dt

⇔−1

x (t)= t+ C

⇒x (t) =

−1

t+ C

usando la condicion inicial 2 = −1C

entonces c = −1/2 se sigue

x (t) =−1

t− 12

el mayor intervalo que contiene a t = 0 en el cual esta funcion esta definida es I =]−∞, 1

2

[el cual corresponde al intervalo maximal.

Observacion 1.5.1. Note que las ecuaciones autonomas son de variables separadas.

Definicion 1.5.3. Las soluciones constantes x (t) = c, t ∈ R de la ecuacion (1.40) son

llamadas soluciones de equilibrio.

60

Page 63: Apuntes MAT-023 USM

Apuntes Mat023 (Segundo semestre 2014)

Note que si x (t) = c es una solucion de equilibrio

0 =dx (t)

dt= f (x (t)) = f (c)

en otras palabras las soluciones de equilibrio corresponden a las raıces de la ecuacion

f (x) = 0.

Ejemplo 1.5.2. Determine las soluciones de equilibrio de la ecuacion

dx

dt= sinx+ sin2 x

Solucion. Las soluciones de equilibrio corresponden a las raıces de la ecuacion

sinx+ sin2 x = 0

esto es

(sinx) (1 + sinx) = 0

luego x = kπ con k ∈ Z o bien x = 3π2

+ 2lπ con l ∈ Z.

Teorema 1.5.2. Las soluciones de la ecuacion (1.40) son soluciones de equilibrio o

funciones monotonas estrictas.

Si x (t) es una solucion definida en su intervalo maximal I, mostraremos que la derivada

no se puede anular a menos que la solucion sea de equilibrio. Supongamos que x′ (tc) = 0

para algun tc ∈ I y definamos φ (t) = x (tc) = α una funcion constante, como

0 = φ′ (t)

y

x′ (tc) = f (x (tc))

se sigue

0 = φ′ (t) = f (φ (t))

luego φ (t) es solucion de equilibrio pero

dx

dt= f (x)

x (tc) = α

tiene 2 soluciones, por el teorema de existencia y unicidad la solucion debe ser la misma,

es decir x (t) = α para todo t.

Esta demostracion tiene otra consecuencia

61

Page 64: Apuntes MAT-023 USM

Apuntes Mat023 (Segundo semestre 2014)

Teorema 1.5.3. Las graficas de dos soluciones distintas de (1.40) no se intersectan.

Teorema 1.5.4. Sean Ω = ]α, β[, x0 ∈ Ω, t0 ∈ R. Si x = x (t) es solucion de (1.40) con

x (t0) = x0 y su intervalo maximal de definicion es ]a, b[ entonces los lımites

lımt→a+

x (t) = A y lımt→b−

x (t) = B

existen o son ±∞, mas aun, A debe tomar el valor α o β si a 6= −∞, y B debe tomar uno

de los valores α o β si b 6=∞.

Ejemplo 1.5.3. Consideremos la ecuacion

dx

dt= x (1− x)

en este caso Ω = R ası α = −∞ y β = +∞. Para 0 < x0 < 1 sean x (t) definida en ]a, b[ la

solucion que cumple x (t0) = x0, note que x1 (t) = 0 y x2 (t) = 1 son soluciones de equilibrio

luego

0 < x (t) < 1

se sigue que 0 ≤ A,B ≤ 1 entonces a = −∞ y b = +∞. La solucion estara definida en

todo R.

Teorema 1.5.5. Si c ∈ Ω y x (t) es una solucion de (1.40) tal que lımt→+∞ x (t) = c o

lımt→−∞ x (t) = c entonces φ (t) = c, t ∈ R es una solucion de equilibrio.

Demostracion. Tenemos que probar que f (c) = 0. Supongamos que f (c) > 0 entonces

por la continuidad de f existirıa un intervalo ]c− δ, c+ δ[ tal que x ∈ ]c− δ, c+ δ[ implica

f (x) > f(c)2> 0, como

lımt→+∞

x (t) = c

se sigue que existe un t0 tal que t ≥ t0 implica |x (t)− c| < δ se sigue

x (t) = x (t0) +

∫ t

t0

x′ (u) du

= x (t0) +

∫ t

t0

f (x (u)) du

> x (t0) +

∫ t

t0

f (c)

2du

= x (t0) +f (c)

2(t− t0)

para t ≥ t0 pero esto contradice lımt→+∞ x (t) = c (se puede llegar a una contradiccion

similar si f (c) < 0).

62

Page 65: Apuntes MAT-023 USM

Apuntes Mat023 (Segundo semestre 2014)

Equilibrio y estabilidad

Notemos que en la ecuacion autonoma

dx

dt= f (x)

se nos indica la pendiente de la recta tangente

a la grafica de la funcion solucion x = x (t) es-

ta viene dada por f (x), esto es, dado un valor

de x las pendientes siempre son las mismas

(independiente de t), las curvas isoclinas co-

rresponde x = c, ademas en los intervalos en

los cuales f (x) > 0 la solucion es estrictamen-

te creciente y en los intervalos en los cuales

f (x) < 0 la funcion solucion es estrictamente

decreciente.

Si x (t) es una solucion dedx

dt= f (x)

entonces la funcion ϕ (t) = x (t+ c) tambien es solucion, en efecto

ϕ′ (t) = x′ (t+ c) = f (x (t+ c)) = f (ϕ (t))

esto significa que las traslaciones de una solucion de la ecuacion autonoma tambien es

solucion, note que si se conoce la solucion x (t) de

dx

dt= f (x)

x (t0) = x0

entonces la solucion de

dx

dt= f (x)

x (µ0) = x0

es ϕ (t) = x (t+ (t0 − µ0)), esto nos dice que en cada banda limitada por las las soluciones

de equilibrio las soluciones son traslaciones de una solucion dada.

63

Page 66: Apuntes MAT-023 USM

Apuntes Mat023 (Segundo semestre 2014)

Ejemplo 1.5.4. Considere la ecuacion

dx

dt= x

en este caso la solucion de equilibrio es

x (t) = 0. Por teorema las demas soluciones

son monotonas estrictas. Para x > 0 las so-

luciones son estrictamente crecientes y para

x < 0 estrictamente decrecientes. En este caso

es facil resolver explıcitamente

x1 (t) = et

es una solucion en la region x > 0 las demas

soluciones son xc (t) = et+k = eket = Cet

donde C > 0 son traslaciones de la solucion

anterior. Note tambien que x (t) = −et es

una solucion en la region x < 0 y las demas

soluciones en esa region son xK (t) = −et+k =

−Ket

En el ejemplo anterior el comportamiento de las soluciones en el entorno de la solucion

de equilibrio es como indica el diagrama

diremos en este caso que el punto de equilibrio es un repulsor, las soluciones se alejan de

esta solucion de equilibrio.

Llamaremos diagrama de fases o lıneas de fases al grafico del comportamiento de las

soluciones en el plano xt.

Definicion 1.5.4. Dependiendo del comportamiento local de las soluciones de la ecuacion

alrededor de un punto de equilibrio aislado x0 en las lıneas de fases, se distinguen los

siguientes tipos:

64

Page 67: Apuntes MAT-023 USM

Apuntes Mat023 (Segundo semestre 2014)

1. x0 es llamado Repulsor si existe δ > 0 tal que

x0 − δ < x < x0 x = x0 x0 < x < x0 + δ

signo f(x) −−− 0 + + +

2. x0 es llamado Atractor si existe δ > 0 tal que

x0 − δ < x < x0 x = x0 x0 < x < x0 + δ

signo f (x) + + + 0 −−−

3. x0 es llamado Atractor-repulsor si existe δ > 0 tal que

x0 − δ < x < x0 x = x0 x0 < x < x0 + δ

signo f (x) + + + 0 + + +

4. x0 es llamado Repulsor-atractor si existe δ > 0 tal que

x0 − δ < x < x0 x = x0 x0 < x < x0 + δ

signo f (x) −−− 0 −−−

Para analizar entonces el tipo de solucion de equilibrio tenemos que analizar el signo

de la funcion en el entorno de la solucion de equilibrio, adicionalmente se cuenta con el

siguiente teorema:

Teorema 1.5.6. Si x (t) = c es una solucion de equilibrio de

dx

dt= f (x)

entonces:

1. Si f ′ (c) < 0, c es un atractor

2. Si f ′ (c) > 0, c es un repulsor.

65

Page 68: Apuntes MAT-023 USM

Apuntes Mat023 (Segundo semestre 2014)

Ejemplo 1.5.5. Bosquejar el diagrama de

fases de la ecuacion

dx

dt= x2 (2− x) (x− 3)

Desarrollo: La funcion x2 (2− x) (x− 3) esta

bien definida y es de clase C∞ (R) existe so-

lucion unica en cada punto (t0, x0) del plano

xt. Las soluciones de equilibrio corresponden

a x1 (t) = 0, x2 (t) = 2 y x2 (t) = 3, en el

siguiente diagrama se analiza el signo de f

0 2 3

x2 + + + 0 + + + + + + + + + + +

x− 2 −−− − −−− 0 + + + + + + +

x− 3 −−− − −−− − −−− 0 + + +

f (x) = −x2 (x− 2) (x− 3) −−− 0 −−− 0 + + + 0 −−−

se sigue que x1 (t) = 0 es un repulsor-atractor, x2 (t) = 2 un repulsor y x2 (t) = 3 un

atractor.

Ejemplo 1.5.6. Sean a, b ∈ R, a 6= 0. Considere la ecuacion diferencial autonoma

dx

dt= a ((x− 1) (x− 4)− b)

66

Page 69: Apuntes MAT-023 USM

Apuntes Mat023 (Segundo semestre 2014)

1. Determine los valores y/o condiciones sobre a y b de modo que la funcion x (t) ≡ 5

sea una solucion de equilibrio y ademas un atractor.

Si f (x) = a ((x− 1) (x− 4)− b) entonces x (t) ≡ 5 sera una solucion de equilibrio si

y solo si

0 = f (5) = a (4− b)⇒ b = 4

(pues a 6= 0), luego

dx

dt= a ((x− 1) (x− 4)− 4)

= a(x2 − 5x

)notemos que

f ′ (x) = a (2x− 5)

f ′ (5) = 5a 6= 0

si a < 0 entonces x (t) ≡ 5 es un atractor.

2. Para los valores y/o condiciones obtenidos en la parte anterior, bosquejar el diagrama

de fases.

De la parte anteriordx

dt= α

(5x− x2

)donde α > 0 entonces las soluciones de equilibrio corresponden a x (t) = 0 y x (t) = 5,

el analisis de signo es

factor \ numero 0 5

x −−− 0 + + + + + + +

5− x + + + + + + + 0 −−−α (5x− x2) −−− 0 + + + 0 −−−

el diagrama de fases es como en la figura:

67

Page 70: Apuntes MAT-023 USM

Apuntes Mat023 (Segundo semestre 2014)

3. Para los valores y/o condiciones obtenidos en la parte I), analizar el comportamiento

de la solucion definida por el P.V.I.

dx

dt= a ((x− 1) (x− 4)− b)

x (0) = 1

esto es: Intervalos de crecimiento, concavidad, limites a ±∞ si estos tienen sentido

(examinar el intervalo maximal de definicion).

De las partes anteriores, el problema es:

dx

dt= α

(5x− x2

)x (0) = 1

Note que la condicion inicial implica que la solucion del P.V.I queda entre las soluciones

de equilibrio. Si el intervalo maximal de definicion es ]a, b[ entonces x : ]a, b[ → Rdebe cumplir

0 < x (t) < 5 para t ∈ ]a, b[ , 0 ∈ ]a, b[ y x (0) = 1

De teoremas sabemos que las soluciones de una ecuacion autonoma o son de equilibrio

o son monotonas estrictas, en este caso, por el intervalo al cual pertenece la solucion,

esta debe ser estrictamente creciente (ver parte anterior). Por el teorema del intervalo

maximal sabemos

lımt→a+

x (t) = A y lımt→b−

x (t) = B

entonces A y B debe existir o ser ±∞, si a 6= −∞ entonces A debe ser ±∞ esto no

se puede cumplir pues 0 < x (t) < 5 implica

0 ≤ lımt→a+

x (t) , lımt→b−

x (t) ≤ 5

se sigue a = −∞ de manera similar b = +∞ y la solucion esta definida en todo R.

Como x es estrictamente creciente entonces

lımt→+∞

x (t)

existe y debe ser una solucion de equilibrio, se sigue

lımt→+∞

x (t) = 5

de manera similar

lımt→−∞

x (t) = 0

68

Page 71: Apuntes MAT-023 USM

Apuntes Mat023 (Segundo semestre 2014)

finalmente para la concavidad (recuerde que 0 < x < 5)

d2x

dt2=

d

dtα(5x− x2

)= α (5x′ − 2xx′)

= α (5− 2x)dx

dt= α (5− 2x)α

(5x− x2

)= α2 (5− 2x)

(5x− x2

)luego, si x ∈

]52, 5[

es concava y si x ∈]0, 5

2

[es convexa.

Obs.: No se entregan puntos por resolver explıcitamente y luego realizar el analisis.

Ejercicios de la seccion

1. En una red de computadores sea x (t) la proporcion de maquinas infectadas con el

virus MTA320 en un instante t dado (note que 0 ≤ x (t) ≤ 1). x′ (t) mide la rapidez

de propagacion sobre la red. Los estudios de expertos de internet determinan que el

virus satisface el problema con valores iniciales

x′ = αx

(1

3− x)(

2

3− x)

(1− x)

x (0) = x0

donde x0 es la proporcion de computadores en que el virus se implanta inicialmente

y α > 0 es una constante.

a) Encuentre las soluciones de equilibrio, clasificarlas y dibujar el diagrama de fase

del sistema.

b) Demostrar que si 0 < x0 < 2/3 entonces el virus alcanzara finalmente a un

tercio del total.

c) ¿Cual debe ser la cantidad de maquinas infectada inicialmente para que todas

se infecten?

2. Sea x : I → R la solucion maximal del P.V.I.

dx

dt= x2 (1− x) (2− x) ex

2

x (0) =3

2

muestre que I = R y determine lımt→+∞ x (t), lımt→−∞ x (t).

69

Page 72: Apuntes MAT-023 USM

Apuntes Mat023 (Segundo semestre 2014)

3. Considere la ecuacion diferencial

dy

dt= y2/3

a) Demuestre que y1 (t) = 0 para todo t ∈ R es solucion.

b) Comprobar que y2 (t) = t3/27 es tambien solucion.

c) Notar que y1 (0) = y2 (0) pero ambas funciones no son iguales para toda t ¿Por

que este ejemplo no contradice el teorema de unicidad?

4. Esboce las lıneas de fase para la ecuacion diferencial dada. Identifique los puntos de

equilibrio:

(a) dydt

= 3y (1− y) (b) dydt

= y2 − 6y − 16 (c) dydt

= cos y

(d) dwdt

= w cosw (e) dwdt

= (w − 2) sinw (f) dydt

= 1y−2

(g) dwdt

= w2 + 2w + 10 (h) dydt

= tan y

5. Considere la ecuacion diferencial

dy

dt= y2 − 4y + 2

con las siguientes condiciones iniciales:

a) y (0) = 0

b) y (0) = 1

c) y (0) = −1

d) y (0) = −10

e) y (0) = 10

f ) y (3) = 1

describir el comportamiento a largo plazo de la solucion.

6. Describir el diagrama de fases de la ecuacion

dx

dt= (x+ λ)

(x2 + λ

)(cosx+ 2)

para los distintos valores del parametro λ.

7. Determine los valores de α ∈ R para los cuales la solucion de equilibrio x (t) = 13

de

dx

dt= (x− α)

(1

3− x)(

x2 − α)

es:

70

Page 73: Apuntes MAT-023 USM

Apuntes Mat023 (Segundo semestre 2014)

a) Un atractor.

b) Un repulsor.

o justifique la no existencia de tales valores.

8. Sea x (t) la solucion del P.V.I.

dx

dt= ex−x

2

sin (x) arctanx

x (3) = 4

a) Determine su intervalo maximal de definicion.

b) ¿Es x (t) estrictamente creciente?

c) Si existen, calcular el valor de lımt→+∞ x (t) y lımt→−∞ x (t)

9. Sean α, β > 0. La ecuaciondP

dt= αP 2/3 − βP

modela el peso de un pez en el tiempo t. Sin resolver la ecuacion, determine el peso

maximo del pez si P (0) = p0 > 0 (justificar sus calculos).

10. Muestre que la solucion de equilibrio y (t) ≡ 0 de la ecuacion

dy

dt= y

(cos(y5 + 2y

)− 27πy4

)corresponde a un repulsor.

71

Page 74: Apuntes MAT-023 USM

Apuntes Mat023 (Segundo semestre 2014)

Ejercicios del capıtulo

1. Resuelva las siguientes ecuaciones diferenciales:

a) dydx

= 1−x2y2

b) dydx

= y (2 + sin x) c) dydx

= sec2 x1+x2

d) dydx

= 1xy3

e) dydx

= 3xy2 f) xdvdx

= 1−4v2

3v

g) dxdt

+ x2 = x h) dydx

= 3x2 (1 + y2) i) (x+ xy2) + ex2y dy

dx= 0

2. Resuelva los siguientes problemas de valor inicial:

a) y′ = x3 (1− y) , y (0) = 3 b) dydx

= (1 + y2) tanx, y (0) =√

3

c) dydx

= 3x2+4x+22y+1

, y (0) = −1 d) dydx

= 2√y + 1 cosx, y (0) = 2

e) dydx

= y sinx, y (π) = −3 f) x2 + 2y dydx

= 0, y (1) = 2

3. Resuelva las siguientes ecuaciones lineales:

a) dydx− y = e3x b) dy

dx= y

x+ 2x+ 1 c) dr

dθ+ r tan θ = sec θ

d) xdydx

+ 3y + 2x2 = x3 + 4x e) (x2 + 1) dydx

+ xy = x f) dydx

= x2e−4x − 4y

4. Considere el problema con valor inicial:

dy

dx+ y√

1 + sin2 x = x, y (0) = 2

Utilice la integral indefinida para mostrar que el factor integrante para la ecuacion

diferencial se puede escribir como:

µ (x) = exp

(∫ x

0

√1 + sin2 t dt

)5. Resuelva las siguientes ecuaciones diferenciales:

a) (x− y) + xdydx

= 0 b) dydx

= (3x+ 2y − 1)2 + 2 c) dydx

= sin (x− y)

d) y′ + 1 =√x+ y e) dx

dt= x2+t

√t2+x2

txf) dy

dx= y(ln y−lnx+1)

x

6. Hallar la solucion general de la ecuacion de Bernoulli:

2y′ + y tanx = (−2x secx) y3

72

Page 75: Apuntes MAT-023 USM

Apuntes Mat023 (Segundo semestre 2014)

7. Resuelva la ecuacion:

3y′ = (1− 2t) y4 − y

sabiendo que y (0) = 1.

8. Resolver:dy

dx= y − x− 1 +

1

x− y − 2

9. Considere la ecuacion diferencial:

y − xy′ = a(1 + x2y′

), a > 1

a) Hallar la solucion general de la ecuacion.

b) Encuentre una solucion particular de la ecuacion, tal que y (0) = 1.

c) Hallar el intervalo mas grande donde la solucion particular anterior este definida.

10. Demuestre que la ecuacion diferencial:

2x4y y′ + y4 = 4x6

se reduce a una ecuacion homogenea mediante el cambio de variables:

y = zn

para cierto n ∈ Z. Determine el valor de n y resuelva la ecuacion.

11. Muestre que la ecuacion:

2(x3y − y3

)dy = 3

(x5 + x4y4

)dx

se reduce a una ecuacion homogenea, realizando el cambio de coordenadas:

x = up ∧ y = vq

para ciertas constantes adecuadas p, q ∈ R. Hallar tales constantes y resolver la

ecuacion diferencial.

12. Hallar la solucion general de la ecuacion:

(x− 2y − 4) + (2x− y + 2)dy

dx= 0

13. Considere la ecuacion de Ricatti:

y′ + 2 (1− x) y − y2 = x (x− 2)

73

Page 76: Apuntes MAT-023 USM

Apuntes Mat023 (Segundo semestre 2014)

a) Hallar constantes A,B ∈ R de modo que:

y = Ax+B

sea una solucion particular de la ecuacion.

b) Determine la solucion general de la ecuacion diferencial.

14. Resuelva la ecuacion:dy

dx= x3 (y − x)2 +

y

x

15. Sea x > 0. Considere la ecuacion:

y′ + e−2xy2 − 1

x

(1 + 4x+ 2x2

)y = −e

2x

x

(1 + x+ 2x2 + x3

)a) Hallar una solucion particular de la forma:

y = e2x (Ax+B)

b) Resuelva la ecuacion diferencial.

16. Un esquiador acuatico ubicado en el punto (a, 0) es tirado por un bote que se

encuentra en el origen O y viaja hacia el norte en direccion OY . Hallar la trayectoria

que sigue el bote si este se dirige en todo momento hacia el bote.

17. Considere un tanque que contiene inicialmente 1000 litros de agua pura, dentro del

cual empieza a fluir una solucion salina a razon de 6 litros por minuto. La solucion

dentro del estanque se mantiene bien agitada y fluye hacia el exterior del tanque

a una velocidad de 5 litros por minuto. Si la concentracion de sal en la salmuera

que entra en el estanque es de 1 kilogramo por litro, determine el instante en que la

concentracion de sal dentro del tanque sea de 6364

kilogramo por litro.

18. El modelo de Malthus para el crecimiento de poblaciones esta dado por la ecuacion:

dN

dt= rN, N (0) = N0

donde r es una constante positiva intrınseca a la poblacion y N0 es la poblacion

inicial.

a) Resuelva la ecuacion diferencial.

b) En 1790, la poblacion de EE.UU. era de 3, 93 millones de habitantes, y en 1890

era de 62, 98 millones de habitantes. Estime la poblacion para EE.UU para el

ano 2000.

74

Page 77: Apuntes MAT-023 USM

Apuntes Mat023 (Segundo semestre 2014)

c) El censo realizado en el ano 2000 en EE.UU. estimo en 281, 42 millones de

habitantes

d) El modelo de Malthus solo considera muertes por causas naturales, sin embargo,

un analisis mas detallado indica que hay muertes debida a enfermedades, a

desnutricion, a crımenes. En resumen, por la competencia entre los miembros

de la poblacion. El modelo logıstico implementa dichas consideraciones:

dN

dt= rN (N −K) , N (0) = N0

para constantes positivas adecuadas r y K. Resuelva la ecuacion diferencial.

e) Para la poblacion de EE.UU. se sabe, ademas, que esta alcanzo los 17, 07

millones de habitantes en 1840. Calcule, usando el modelo logıstico, la poblacion

de EE.UU. en el ano 2000. Compare respecto al censo del ano 2000.

f ) Considerando el modelo logıstico, calcule la poblacion lımite de EE.UU.

19. La sangre conduce un medicamento a un organo a razon de 3[

cm3

seg

]y sale de el a la

misma razon. El organo tiene un volumen lıquido de 125 [cm3]. Si la concentracion

del medicamento en la sangre que entra al organo es de 0, 2[

grcm3

]:

a) ¿Cual es la concentracion del medicamento en el organo en el instante t, si

inicialmente no habıa rastros de dicho medicamento?

b) ¿En que momento llegara la concentracion del medicamento en el organo a

0, 1[

grcm3

]20. Una taza de cafe caliente, inicialmente a 95C, se enfrıa hasta 80C en 5 minutos,

al estar en un cuarto con temperatura de 21C. Use solo la ley de enfriamiento de

Newton (ver MAT022) y determine el momento en que la temperatura del cafe estara a

unos agradables 50C.

21. Era el mediodıa en un frıo dıa de julio en Vina del Mar: 16C. Un inspector de la

Policıa de Investigaciones (PDI) llega a la escena de un crimen, encontrando un

detective sobre un cadaver. El detective dijo que habıa varios sospechosos detenidos.

Dadas las coartadas se podrıa hallar al culpable sabiendo el momento exacto de

la muerte. El inspector saco un termometro y midio la temperatura del cadaver:

34C. Luego de realizar peritos en la escena que duraron exactamente una hora,

midio nuevamente la temperatura del cuerpo: 33, 7C. Bajo que la hipotesis de que

la temperatura normal de un cuerpo humano es de 37C. Determine el momento en

que ocurrio el crimen.

75

Page 78: Apuntes MAT-023 USM

Apuntes Mat023 (Segundo semestre 2014)

22. El economista ingles Thomas Malthus fue unos de los primeros en intentar modelar el

crecimiento poblacional humano por medio de matematicas en 1978. Basicamente, el

fundamento del modelo maltusiano es la suposicion de que la rapidez a la que crece

la poblacion de un pais en cierto tiempo es proporcional a la poblacion del paıs en

ese momento.

a) Proponer una ecuacion para este modelo.

b) Determine la ecuacion diferencial si se permite inmigracion a tasa constante

r > 0.

23. La ley de enfriamiento (o calentamiento) de Newton establece que la rapidez a la

que cambia la temperatura de un cuerpo es proporcional a la diferencia entre la

temperatura del cuerpo y la temperatura del medio circundante. Obtener una E.D.O.

para esta ley. Si la temperatura del medio cambia en el tiempo, escribir la ecuacion

e identificarla.

24. Una enfermedad contagiosa se disemina en una comunidad por medio de la gente que

entra en contacto con otras personas. Suponer que la rapidez con que se disemina la

enfermedad es proporcional al numero de interacciones entre las personas contagiadas

y no contagiadas (puede ser considerado como el producto de los contagiados y no

contagiados). Suponga que en una poblacion pequena de n personas se introduce un

enfermo obtener una EDO cuya solucion permita obtener el numero de contagiados

en un tiempo t.

25. Resolver las siguientes ecuaciones diferenciales ordinarias:

76

Page 79: Apuntes MAT-023 USM

Apuntes Mat023 (Segundo semestre 2014)

1)dy

dx=

1 + y

1 + x2)

dy

dx− 2xy = x

3)dy

dx− y tanx = cosx 4)

dy

dx=

yx

x2 − y2

5)dy

dx=

2y + x− 2

y − x+ 16)

dy

dx+

1

xy =

cosx

y2

7)dy

dx+ x2y =

1

x2y4 8)

dy

dxcos y + 2x sin y = −2x

9)dy

dx= y2 + 2x− x4 10)

dy

dx+

(1

x− 1

)y =

e2x

x

11)dy

dx− 4y = 2exy1/2 12)

dy

dx=y + 1

x+ 2+ exp

(y + 1

x+ 2

)

13)dy

dx= 3 |y|2/3 14)

dy

dx=

e−y2

y (2x+ x2)

15)dy

dx= (x− y + 3)2 16) x

dy

dx= y +

√x2 + y2

17)dy

dx+ y sinx = sin3 x 18)

dy

dx+

y

1 + x= (1 + x) y4

19) y = xdy

dx+

(dy

dx

)2

26. Suponga que un gran tanque de mezclado tiene inicialmente 600 galones de salmuera.

Otra solucion de salmuera se bombea hacia en tanque a una rapidez de 4 galones

por minuto, la concentracion de sal en el flujo de entrada es de 2 libras por galon.

Cuando la solucion en el tanque esta bien agitada se bombea a 2 galones por minuto.

Determinar una EDO cuya solucion corresponda a la cantidad de sal en el tanque en

el momento t.

27. Identificar el campo de direcciones con la ecuacion correspondiente;

(A) y′ = x+ y

(B) y′ = −xy

(C) y′ = y + 1

77

Page 80: Apuntes MAT-023 USM

Apuntes Mat023 (Segundo semestre 2014)

(D) y′ = y2 − x2

Obs: el campo de direcciones nos indica la direccion de las rectas tangentes a las

soluciones en un punto (x, y) dado.

28. Considere la ecuacion diferencial autonoma

dS

dt= S3 − 2S2 + S

a) Hacer el diagrama de las lıneas de fase..

b) Usando el dibujo anterior delinear las graficas de las soluciones S (t) con las

condiciones iniciales siguientes: S (0) = 0;S (0) = 12;S (0) = 3

2y S (0) = −1

2.

c) ¿A que tienden sus soluciones cuando t→ +∞?

29. Considere la ecuaciondy

dt=

1

1− y

a) Determine los valores de y ∈ R en los cuales es aplicable el teorema de existencia

y unicidad.

b) Obtener las soluciones explıcitamente y mostrar que llegan la y = 1 en un

tiempo finito, determine los intervalos maximales de solucion

78

Page 81: Apuntes MAT-023 USM

Apuntes Mat023 (Segundo semestre 2014)

30. La ardilla negra es un pequeno mamıfero nativo de las montanas rocallosas. Esas

ardillas son muy territoriales, por lo que si su poblacion es grande, su razon de

crecimiento decrece y puede llegar a ser negativa. Por otra parte, si la poblacion es

demasiado pequena, los adultos fertiles corren el riesgo de no ser capaces de encontrar

parejas adecuadas y su tasa de crecimiento nuevamente es negativa. Muestre que el

modelodP

dt= kP

(1− P

N

)(P

M− 1

)donde k,M,N > 0 y M < N puede ser utilizado para describir la poblacion de

Ardillas ¿Que interpretacion tienen N y M?

79

Page 82: Apuntes MAT-023 USM

Capıtulo 2 : Ecuaciones diferenciales lineales de orden superior

Elementos de transformaciones lineales

Definiciones

En lo que sigue K representa R o bien C.

Definicion 2.1.1. Sean U y V espacios vectoriales sobre K. Una transformacion lineal

es una funcion T : U → V tal que para todo u, v ∈ U y α ∈ K cumple:

1. T (u+ v) = T (u) + T (v)

2. T (αu) = αT (u)

Observacion 2.1.1. Usualmente, los puntos (1) y (2) de la definicion anterior, pueden

escribirse como

1. ∀u, v ∈ U,∀α ∈ K, T (αu+ v) = αT (u) + T (v)

o bien

2. ∀u, v ∈ U,∀α, β ∈ K, T (αu+ βv) = αT (u) + βT (v)

las tres definiciones obtenidas son equivalentes.

Observacion 2.1.2. Note que si U, V son K espacios vectoriales, T : U → V es una

transformacion lineal, para i = 1, · · · , n, αi son escalares en K y ui ∈ U entonces

T

(n∑i=1

αiui

)=

n∑i=1

αiT (ui)

luego las transformaciones lineales envıan combinaciones lineales del espacio de partida a

combinaciones lineales del espacio de llegada.

Ejemplo 2.1.1. La funcion nula 0 y la funcion identidad 1 de un espacio vectorial en

sı mismo son transformaciones lineales. En efecto, Si V es un K espacio vectorial y

0 : V → V , v → 0 (v) = θV (el neutro de la adicion) entonces

0 (αu+ v) = θV

= αθv + θv

= α0 (u) + 0 (v)

80

Page 83: Apuntes MAT-023 USM

Apuntes Mat023 (Segundo semestre 2014)

de manera similar, si 1 : V → V , v → 1 (v) = v entonces

1 (αu+ v) = αu+ v

= α1 (u) + 1 (v)

Ejemplo 2.1.2. T : R2 → R4, (x, y) → T (x, y) = (x− y, 2x, x+ y, 2y) es una transfor-

macion lineal. En efecto:

1. Para cada (a, b), (u, v) ∈ R2 se tiene

T ((a, b) + (u, v))

= T (a+ u, b+ v)

= (a+ u− b− v, 2a+ 2u, a+ u+ v + b, 2b+ 2v)

= ((a− b) + (u− v) , 2a+ 2u, (a+ b) + (u+ v) , 2b+ 2v)

= (a− b, 2a, a+ b, 2b) + (u− v, 2u, u+ v, 2v)

= T (a, b) + T (u, v)

2. Para cada (u, v) ∈ R2 y α ∈ R se cumple

T (α (u, v)) = T (αu, αv)

= (αu− αv, 2αu, αu+ αv, 2αv)

= α (u− v, 2u, u+ v, 2v)

= αT (u, v)

de esta forma T verifica las condiciones de transformacion lineal.

Ejemplo 2.1.3. T : R→ R2, x→ T (x) = (x, 2x+ 1) no es una transformacion lineal. En

efecto,

T (x+ y) = (x+ y, 2x+ 2y + 1)

= (x, 2x+ 1) + (y, 2y)

6= T (x) + T (y)

basta usar un contraejemplo:

T (1 + 2) = T (3) = (3, 7)

pero

T (1) + T (2) = (1, 3) + (2, 5) = (3, 8)

de esta forma

T (1 + 2) 6= T (1) + T (2)

luego no es una transformacion lineal.

81

Page 84: Apuntes MAT-023 USM

Apuntes Mat023 (Segundo semestre 2014)

Ejemplo 2.1.4. Si R3 [x] es el espacio vectorial de los polinomios de grado menor igual a

3 entoncesT : R3 [x] → M2×2 (R)

p → T (p) =

(p (0) p′ (0)

p′′ (0) p′′′ (0)

)Es una transformacion lineal. En efecto:

1. Si p, q ∈ R3 [x] entonces

T (p+ q) =

((p+ q) (0) (p+ q)′ (0)

(p+ q)′′ (0) (p+ q)′′′ (0)

)pero recordar que (p+ q)′ (x) = p′ (x) + q′ (x), (p+ q)′′ (x) = p′′ (x) + q′′ (x) y

(p+ q)′′′ (x) = p′′′ (x) + q′′′ (x) se sigue

T (p+ q) =

((p+ q) (0) (p+ q)′ (0)

(p+ q)′′ (0) (p+ q)′′′ (0)

)=

(p (0) + q (0) p′ (0) + q′ (0)

p′′ (0) + q′′ (0) p′′′ (0) + q′′′ (0)

)=

(p (0) p′ (0)

p′′ (0) p′′′ (0)

)+

(q (0) q′ (0)

q′′ (0) q′′′ (0)

)= T (p) + T (q)

2. Si α ∈ R y p ∈ R3 [x] entonces

T (αp) =

((αp) (0) (αp)′ (0)

(αp)′′ (0) (αp)′′′ (0)

)=

(αp (0) αp′ (0)

αp′′ (0) αp′′′ (0)

)= α

(p (0) p′ (0)

p′′ (0) p′′′ (0)

)= αT (p)

Note que no es necesario considerar la expresion del polinomio p (x) = a3x3 + a2x

2 +

a1x+ a0 en esta demostracion de que es transformacion lineal, lo que utilizamos son

las propiedades de la derivada y matrices.

Ejemplo 2.1.5. Sea T : R3 → R2 la funcion definida por:

T (x, y, z) = (x+ y + z, x− y)

muestre que T es una transformacion lineal.

82

Page 85: Apuntes MAT-023 USM

Apuntes Mat023 (Segundo semestre 2014)

Solucion. Sean α ∈ R, (u, v, w) , (x, y, z) ∈ R3 entonces

T (α (u, v, w) + (x, y, z))

= T ((αu, αv, αw) + (x, y, z))

= T (αu+ x, αv + y, αw + z)

= ((αu+ x) + (αv + y) + (αw + z) , (αu+ x)− (αv + y))

= (αu+ αv + αw, αu− αv) + (x+ y + z, x− y)

= α (u+ v + w, u− v) + (x+ y + z, x− y)

= αT (u, v, w) + T (x, y, z)

Ejemplo 2.1.6. Sean I ⊆ R un intervalo abierto y C1 (I) el espacio vectorial real de las

funciones de clase C1 sobre I. Defina la transformacion D : C1 (I)→ C (I) mediante:

D (f) (x) =df (x)

dx, ∀x ∈ I

Entonces, D es una trasformacion lineal.

Solucion. Por propiedades de la derivada, si α ∈ R, f, g ∈ C1 (I) entonces

D (αf + g) (x) =d

dx(αf + g) (x)

= αdf (x)

dx+

dg (x)

dx= αD (f) (x) +D (g) (x)

= (αD (f) +D (g)) (x)

ası

D (αf + g) = αD (f) +D (g)

Ejemplo 2.1.7. Sea C el espacio vectorial de todas las funciones continuas de R en R y

sea T : C → C, f → T (f) donde T (f) es la funcion definida por:

T (f) (x) =

∫ x

1

f (t) dt

Entonces, T es una transformacion lineal.

Solucion. Sean α ∈ R, f, g ∈ C entonces

T (αf + g) (x) =

∫ x

1

(αf (t) + g (t)) dt

=

∫ x

1

αf (t) dt+

∫ x

1

g (t) dt

= α

∫ x

1

f (t) dt+

∫ x

1

g (t) dt

= αT (f) (x) + T (g) (x)

= (αT (f) + T (g)) (x)

83

Page 86: Apuntes MAT-023 USM

Apuntes Mat023 (Segundo semestre 2014)

ası

T (αf + g) = αT (f) + T (g)

Ejemplo 2.1.8. Sean x, y ∈ Rn. Considere la funcion P : Rn → R definida por:

x 7→ P (x) =n∑i=1

xiyi

Entonces, P es una transformacion lineal. Esto es directo por las propiedades del producto

punto en Rn.

Observacion 2.1.3. El conjunto de todas las transformaciones lineales de U en V se

denotara por LK (U, V ), o mas sencillamente por L (U, V ) si el cuerpo de escalares no

presenta confusion. Es decir:

L (U, V ) =T : U → V

∣∣∣T es transformacion lineal

Teorema 2.1.1. Sea T : U → V una transformacion lineal, entonces T (θU) = θV .

Demostracion. Note que

T (θU) = T (θU + θU)

= T (θU) + T (θU)

sumando el inverso aditivo −T (θU) ∈ V se tiene

T (θU) +−T (θU) = (T (θU) + T (θU)) +−T (θU)

luego

θV = T (θU) + (T (θU) +−T (θU))

= T (θU) + θV

y ası

T (θU) = θV

Ejemplo 2.1.9. Considere la funcion T : R2 → R3 definida por la ecuacion:

T (x, y) = (x+ y, 1− y, x)

En vista del teorema anterior, la funcion T no puede ser transformacion lineal, pues:

T (0, 0) = (0, 1, 0)

no es el vector nulo de R3.

84

Page 87: Apuntes MAT-023 USM

Apuntes Mat023 (Segundo semestre 2014)

Teorema 2.1.2. Sean U, V,W espacios vectoriales sobre K:

1. L (U, V ) es un espacio vectorial sobre K.

2. Si T ∈ L (U, V ) y S ∈ L (V,W ) entonces S T ∈ L (U,W ).

3. Si T ∈ L (U, V ) es biyectiva entonces T−1 ∈ L (V, U)

Demostracion. En efecto:

1. Mostraremos que es un subespacio del espacio de las funciones de U en V . L (U, V ) no

es vacıo pues existe la transformacion lineal θ : U → V definida por u→ θ (u) = θV .

Sean α, β ∈ K y T, L ∈ L (U, V ) entonces αT + βL es una transformacion lineal pues

si γ ∈ K, u1, u2 ∈ U entonces

(αT + βL) (γu1 + u2) = αT (γu1 + u2) + βL (γu1 + u2)

= α (γT (u1) + T (u2)) + β (γL (u1) + L (u2))

= γ (αT (u1) + βL (u1)) + (αT (u2) + βL (u2))

= γ (αT + βL) (u1) + (αT + βL) (u2)

luego L (U, V ) es no vacıo, cerrado para la suma y producto escalar.

2. Sean u, v ∈ U y α ∈ K, entonces:

(S T ) (αu+ v) = S (T (αu+ v))

= S (αT (u) + T (v))

= αS (T (u)) + S (T (v))

= α (S T ) (u) + (S T ) (v)

Luego, S T es una transformacion lineal.

3. Si T−1 : V → U existe entonces es una transformacion lineal, en efecto, sean v1, v2 ∈ Vy α ∈ K, por la biyectividad existen unicos u1, u2 ∈ U tales que

T (u1) = v1

T (u2) = v2

luego

T−1 (αv1 + v2) = T−1 (αT (u1) + T (u2))

= T−1 (T (αu1 + u2))

= αu1 + u2

= αT−1 (v1) + T−1 (v2)

85

Page 88: Apuntes MAT-023 USM

Apuntes Mat023 (Segundo semestre 2014)

Observacion 2.1.4. Con respecto al teorema anterior, denotaremos la composicion de

transformaciones lineales S T mediante la notacion producto ST .

Teorema 2.1.3. Sean U un espacio vectorial sobre K tal que u1, u2, . . . , un es una base

para U y v1, v2, . . . , vn un subconjunto cualquiera de un espacio vectorial V arbitrario (se

pueden repetir elementos), entonces existe una unica transformacion lineal T : U → V tal

que:

T (ui) = vi, ∀i = 1, 2, . . . , n (2.1)

Observacion 2.1.5. Note que si u1, u2, . . . , un es una base para un espacio vectorial U y

u ∈ U , entonces existen unicos escalares x1, x2, . . . , xn ∈ K tales que:

u =n∑i=1

xiui

Si, ademas, T esta definida por las ecuaciones (2.1), entonces se tiene que:

Tu = T

(n∑i=1

xiui

)

=n∑i=1

xiT (ui)

=n∑i=1

xivi

Ejemplo 2.1.10. Determinaremos la formula T (x, y, z) de la transformacion lineal T :

R3 → R3 [x] tal que:

T (1, 0, 2) = x3 + 2x2 + 1

T (−1, 1, 1) = −2x2 − 2 (2.2)

T (1,−2, 1) = x3 − 1

Notamos primeramente que u1 = (1, 0, 2) , u2 = (−1, 1, 1) y u3 = (1,−2, 1) forman una

base para R3, pues:

det

1 0 2

−1 1 1

1 −2 1

= 5

Por otro lado, si:

(a, b, c) = x1 · (1, 0, 2) + x2 · (−1, 1, 1) + x3 · (1,−2, 1)

86

Page 89: Apuntes MAT-023 USM

Apuntes Mat023 (Segundo semestre 2014)

entonces x1 = 35a+ 2

5b+ 1

5c, x2 = 2

5c− 1

5b− 4

5a y x3 = 1

5c− 3

5b− 2

5a, ası:

(x, y, z) =

(3

5a+

2

5b+

1

5c

)(1, 0, 2)+

(2

5c− 1

5b− 4

5a

)(−1, 1, 1)+

(1

5c− 3

5b− 2

5a

)(1,−2, 1)

por la linealidad de T y su definicion en la base, tenemos que:

T (a, b, c) =

(3

5a+

2

5b+

1

5c

)T (1, 0, 2) +

+

(2

5c− 1

5b− 4

5a

)T (−1, 1, 1) +

(1

5c− 3

5b− 2

5a

)T (1,−2, 1)

=

(3

5a+

2

5b+

1

5c

)(x3 + 2x2 + 1

)+

(2

5c− 1

5b− 4

5a

)(−2x2 − 2

)+

+

(1

5c− 3

5b− 2

5a

)(x3 − 1

)=

(1

5a− 1

5b+

2

5c

)x3 +

(14

5a+

6

5b− 2

5c

)x2 +

(13

5a+

7

5b− 4

5c

)Es decir, T : R3 → R3 [x] esta definida por:

T (a, b, c) =

(1

5a− 1

5b+

2

5c

)x3 +

(14

5a+

6

5b− 2

5c

)x2 +

(13

5a+

7

5b− 4

5c

)Ejemplo 2.1.11. Hallar la formula de una transformacion lineal T de R2 en:

U =

(x, y, z) ∈ R3 : x+ y + z = 0

tal que T (−1, 1) = (1, 2,−3) y T (2,−1) = (5,−3,−2).

Solucion. Note que (1, 2,−3), (5,−3,−2) pertenecen a U , como (−1, 1) y (2,−1) son

linealmente independiente, por el teorema existe una unica transformacion lineal T : R2 → U

tal que

T (−1, 1) = (1, 2,−3)

T (2,−1) = (5,−3,−2)

para determinar una formula, notemos que

(a, b) = α (−1, 1) + β (2,−1)

implica (−1 2 a

1 −1 b

)∼(

1 0 a+ 2b

0 1 a+ b

)87

Page 90: Apuntes MAT-023 USM

Apuntes Mat023 (Segundo semestre 2014)

ası

(a, b) = (a+ 2b) (−1, 1) + (a+ b) (2,−1)

ası

T (a, b) = (a+ 2b)T (−1, 1) + (a+ b)T (2,−1)

= (a+ 2b) (1, 2,−3) + (a+ b) (5,−3,−2)

= (6a+ 7b, b− a,−5a− 8b)

Nucleo e imagen

Definicion 2.1.2. Sean U y V espacios vectoriales sobre K y T : U → V una transforma-

cion lineal. Llamaremos:

1. Nucleo de T o Kernel de T al subconjunto de U definido como:

kerT = u ∈ U : T (u) = 0

2. Imagen de T al subconjunto de V definido como:

ImT = v ∈ V : ∃u ∈ U, T (u) = v

Observacion 2.1.6. El siguiente teorema indica que el nucleo y la imagen de T poseen

estructura de espacio vectorial.

Teorema 2.1.4. Sea T ∈ L (U, V ), entonces:

1. kerT ≤ U .

2. ImT ≤ V

Ejemplo 2.1.12. Hallar el nucleo y la imagen de T : R3 → R3 definida por:

T (x, y, z) = (2x− y + z, x− y + z, x)

Solucion. Primero determinaremos el nucleo

ker (T ) =

(x, y, z) ∈ R3 : T (x, y, z) = (0, 0, 0)

=

(x, y, z) ∈ R3 : (2x− y + z, x− y + z, x) = (0, 0, 0)

=

(x, y, z) ∈ R3 :

2x− y + z = 0

x− y + z = 0

x = 0

88

Page 91: Apuntes MAT-023 USM

Apuntes Mat023 (Segundo semestre 2014)

note que 2 −1 1 0

1 −1 1 0

1 0 0 0

∼ 1 0 0 0

0 1 −1 0

0 0 0 0

luego

ker (T ) =

(x, y, z) ∈ R3 :

x = 0

y − z = 0

=

(0, y, y) ∈ R3 : y ∈ R

= 〈(0, 1, 1)〉

un espacio de dimension 1. Determinemos la imagen

ImT =

(a, b, c) ∈ R3 : ∃ (x, y, z) ∈ R3, T (x, y, z) = (a, b, c)

=

(a, b, c) ∈ R3 : ∃ (x, y, z) ∈ R3, (2x− y + z, x− y + z, x) = (a, b, c)

=

(a, b, c) ∈ R3 : ∃ (x, y, z) ∈ R3,

2x− y + z = a

x− y + z = b

x = c

en otras palabras, para que valores de a, b, c ∈ R el sistema

2x− y + z = a

x− y + z = b

x = c

tiene solucion, notemos que 2 −1 1 a

1 −1 1 b

1 0 0 c

∼ 2 −1 1 a

0 −1 1 2b− a0 0 0 −c− b+ a

luego el sistema tiene solucion si y solo si

−c− b+ a = 0

se sigue

ImT =

(a, b, c) ∈ R3 : −c− b+ a = 0

=

(b+ c, b, c) ∈ R3 : b, c ∈ R

= 〈(1, 1, 0) , (1, 0, 1)〉

es un espacio de dimension 2.

89

Page 92: Apuntes MAT-023 USM

Apuntes Mat023 (Segundo semestre 2014)

Definicion 2.1.3. Sea T ∈ L (U, V ) con U y V espacios de dimension finita, entonces:

1. Llamaremos nulidad de T , denotado η (T ), al numero definido como:

η (T ) = dim kerT

2. Llamaremos rango de T , denotado ρ (T ) al numero definido como:

ρ (T ) = dim ImT

Teorema 2.1.5. Sean U, V un espacios vectoriales sobre K, u1, u2, . . . , un una base para

U y T ∈ L (U, V ), entonces:

ImT = 〈T (u1) , T (u2) , . . . , T (un)〉

Demostracion. Suponga que v ∈ ImT entonces, por definicion, existe un u ∈ U tal que

T (u) = v, como u1, u2, . . . , un es una base de U existiran escalares α1, α2, . . . , αn tales

que

u =n∑i=1

αiui

se sigue que

v = T (u) = T

(n∑i=1

αiui

)=

n∑i=1

αiT (ui)

es decir, v es una combinacion lineal de los elementos T (u1) , T (u2) , . . . , T (un), luego

T (u1) , T (u2) , . . . , T (un) genera ImT .

Ejemplo 2.1.13. Si T :M2×2 (R)→M2×2 (R)(x y

z w

)→ T

(x y

z w

)=

(1 1

1 1

)(x y

z w

)sabemos que (

1 0

0 0

),

(0 1

0 0

),

(0 0

1 0

)y

(0 0

0 1

)forman una base de M2×2 (R) luego

T

(1 0

0 0

)=

(1 1

1 1

)(1 0

0 0

)=

(1 0

1 0

)T

(0 1

0 0

)=

(1 1

1 1

)(0 1

0 0

)=

(0 1

0 1

)T

(0 0

1 0

)=

(1 1

1 1

)(0 0

1 0

)=

(1 0

1 0

)T

(0 0

0 1

)=

(1 1

1 1

)(0 0

0 1

)=

(0 1

0 1

)90

Page 93: Apuntes MAT-023 USM

Apuntes Mat023 (Segundo semestre 2014)

generan la imagen, se sigue

ImT =

⟨(1 0

1 0

),

(0 1

0 1

),

(1 0

1 0

),

(0 1

0 1

)⟩=

⟨(1 0

1 0

),

(0 1

0 1

)⟩de donde obtenemos ρ (T ) = 2.

Ejemplo 2.1.14. Demuestre que existe una transformacion lineal T :M2 (R)→ R3 tal

que:

ImT = (x, y, z) : x− 2y + z = 0

y la nulidad de T sea 2.

Solucion. Notemos que

ImT = (x, y, z) : x− 2y + z = 0= (x, y, z) : x = 2y − z= (2y − z, y, z) : y, z ∈ R= 〈(2, 1, 0) , (−1, 0, 1)〉

este espacio tiene dimension 2. Si necesitamos que la nulidad sea 2 entonces el Nucleo debe

estar generado por dos elementos L. I. Definamos

T

(1 0

0 0

)= (2, 1, 0)

T

(0 1

0 0

)= (−1, 0, 1)

T

(0 0

1 0

)= (0, 0, 0)

T

(0 0

0 1

)= (0, 0, 0)

existe una unica T : M2 (R) → R3 que cumple lo anterior, para esta transformacion se

tiene:

ImT =

⟨T

(1 0

0 0

), T

(0 1

0 0

), T

(0 0

1 0

), T

(0 0

0 1

)⟩= 〈(2, 1, 0) , (−1, 0, 1)〉

Note que

T

(a b

c d

)= (2a− b, a, b)

demuestre que el Nucleo tiene dimension 2..

91

Page 94: Apuntes MAT-023 USM

Apuntes Mat023 (Segundo semestre 2014)

Observacion 2.1.7. El siguiente resultado relaciona la nulidad y el rango de una trans-

formacion lineal:

Teorema 2.1.6. Sean U y V espacios vectoriales sobre K con dimU < ∞. Considere

T ∈ L (U, V ), entonces:

dimU = η (T ) + ρ (T )

Demostracion. Supongamos que dimU = n. Sea u1, u2, . . . , ur (r ≤ n) una base del nucleo

de T . Considere ur+1, ur+2, . . . , un una completacion de la base u1, u2, . . . , ur del nucleo de

T al espacio vectorial U . Luego, si v ∈ U , entonces:

v =n∑i=1

αiui

Ası:

T (v) = T

(n∑i=1

αiui

)

=n∑i=1

αiT (ui)

=n∑

i=r+1

αT (ui)

y por tanto, T (ur+1) , T (ur+2) , . . . , T (un) generan la imagen de T . Basta verificar, ahora,

que son linealmente independientes. En efecto, supongamos que:

n−r∑i=1

αiT (ur+i) = 0

Luego:

T

(n−r∑i=1

αiur+i

)= 0

es decir, si w =∑n−r

i=1 αiur+i, se tiene entonces que w ∈ kerT . Por tanto, existen escalares

β1, β2, . . . , βr ∈ K tales que:

w =r∑i=1

βiui

Ası:r∑i=1

βiui =n−r∑i=1

αiur+i

O bien:r∑i=1

βiui −n−r∑i=1

αiur+i = 0

92

Page 95: Apuntes MAT-023 USM

Apuntes Mat023 (Segundo semestre 2014)

Ahora bien, como u1, u2, . . . , ur, ur+1, . . . , un es una base de U se tiene que:

β1 = β2 = · · · = βr = α1 = α2 = · · · = αn−r = 0

Ası, T (ur+1) , T (ur+2) , . . . , T (un) es una base para la imagen de T . Por tanto, η (T ) = r

y ρ (T ) = n− r.

Definicion 2.1.4. Sea T ∈ L (U, V ). Diremos que:

1. T es inyectiva si:

T (u) = T (v) =⇒ u = v

para todos u, v ∈ U .

2. T es epiyectiva si:

ImT = V

3. T es biyectiva si es inyectiva y epiyectiva, simultaneamente.

Ejemplo 2.1.15. No existe transformacion lineal T : R2 → R3 que sea sobreyectiva. En

efecto, si T es sobreyectiva ImT = R3 luego ρ (T ) = 3 pero del teorema de las dimensiones

η (T ) + ρ (T ) = Dim(R2)

= 2

como ρ (T ) = 3 se obtiene

η (T ) = −1

esto es una contradiccion pues η (T ) ≥ 0.

Teorema 2.1.7. Sea T ∈ L (U, V ). Entonces, T es inyectiva, si y solo si, kerT = θU.

Demostracion. Supongamos que kerT = θU. Sean u, v ∈ U tales que T (u) = T (v) .

Como T es una transformacion lineal, se tiene T (u− v) = θV , es decir, u− v ∈ kerT . Ası,

u− v = θU , y por tanto, T es inyectiva.

Por otro lado, supongamos que T es inyectiva, como θU ⊆ kerT , bastara verificar

que kerT ⊆ 0. En efecto, sea u ∈ kerT , esto es T (u) = θV y como T es transformacion

lineal, se tiene que T (θU) = θV . Es decir, T (u) = T (θU), luego u = θU , pues T es inyectiva.

Ası, kerT = θU.

93

Page 96: Apuntes MAT-023 USM

Apuntes Mat023 (Segundo semestre 2014)

Isomorfismo

Definicion 2.1.5. Dos espacios vectoriales U y V sobre K se dicen isomorfos si existe una

transformacion lineal T : U → V biyectiva. Tal transformacion T biyectiva se llamara iso-

morfismo entre U y V . El concepto de isomorfismo entre dos espacios se representara por:

U ' V .

Ejemplo 2.1.16. Sea U un espacio vectorial sobre R tal que u1, u2, . . . , un es una base de

U , entonces L : Rn → U definida por:

(x1, x2, . . . , xn) 7→ L (x1, x2, . . . , xn) =n∑i=1

xiui

es un isomorfismo entre U y Rn, esto se debe a que

(x1, x2, . . . , xn) ∈ kerT ⇔n∑i=1

xiui = θU

⇔ ∀i, xi = 0

(pues u1, u2, . . . , un es una base de U) ası (x1, x2, . . . , xn) ∈ kerT ⇔ (x1, x2, . . . , xn) =

(0, 0, . . . , 0) luego L es inyectiva y por el teorema de las dimensiones

n = dimRn = 0 + ρ (L)

ası

ρ (L) = n

esto implica ImT = U .

Teorema 2.1.8. Sean U y V espacios vectoriales de dimension finita sobre un cuerpo K.

Entonces, U ' V si y solo si dimU = dimV .

Ejemplo 2.1.17. M2 (R) ' R3 [x] ' R4

Ejemplo 2.1.18. Kmn 'Mm×n (K)

Matriz asociada a una transformacion lineal

Definicion 2.1.6. Sean U un espacio vectorial sobre K y B = u1, u2, . . . , un una base

de U . Diremos que B es una base ordenada de U , si se considera B como la sucesion

finita de vectores en U :

B = (u1, u2, . . . , un)

94

Page 97: Apuntes MAT-023 USM

Apuntes Mat023 (Segundo semestre 2014)

Definicion 2.1.7. Sea B = u1, u2, . . . , un una base ordenada de un espacio vectorial U

y sea u ∈ U . Los coeficientes αi en la combinacion

u = α1u1 + α2u2 + · · ·+ αnun

son llamadas coordenadas de u con respecto a B y desde ahora usaremos la notacion

[u]B =

α1

α2

...

αn

Observacion 2.1.8. Estos escalares existen y son unicos por la definicion de base, el orden

de los elementos de la base es importante.

Ejemplo 2.1.19. Calcular las coordenadas del vector −1 + 2x+ 3x2 en la base de R2 [x]

dada por B = 1 + x, x2, 1 .

Solucion. Tenemos que encontrar escalares α, β, γ tales que

−1 + 2x+ 2x2 = α (1 + x) + βx2 + γ1

desarrollando

−1 + 2x+ 3x2 = (α + γ) + αx+ βx2

ası

−1 = α + γ

2 = α

3 = β

se sigue γ = −3 entonces

−1 + 2x+ 3x2 = 2 (1 + x) + 3(x2)− 3 (1)

de donde obtenemos [−1 + 2x+ 3x2

]B =

2

3

−3

Observacion 2.1.9. Dado en vector de coordenadas [v]B y conocida la base B se puede

recuperar el vector v.

95

Page 98: Apuntes MAT-023 USM

Apuntes Mat023 (Segundo semestre 2014)

Ejemplo 2.1.20. Si el determinado vector v en M2×2 (R) tiene vector de coordenadas

[v]B =

1

−1

2

−2

en la base B =

(1 1

1 1

),

(1 1

1 0

),

(1 1

0 0

),

(1 0

0 0

)entonces

v = 1

(1 1

1 1

)+ (−1)

(1 1

1 0

)+ 2

(1 1

0 0

)+ (−2)

(1 0

0 0

)=

(0 2

0 1

)Observacion 2.1.10. Sean U y V espacios vectoriales sobre K tales que B = u1, u2, . . . , uny D = v1, v2, . . . , vm son bases ordenadas de U y V , respectivamente. Note que para

cada i = 1, 2, . . . , n se tiene que T (ui) ∈ V , por tanto, existen escalares Aji ∈ K tales que:

T (ui) = A1 iv1 + A2 iv2 + · · ·+ Amivm =m∑j=1

Ajivj, ∀i = 1, 2, . . . , n

Es decir, los escalares A1 i, A2 i, . . . , Ami son las coordenadas de T (ui) en la base D. Ası:

[T (ui)]D =

A1 i

A2 i

...

Ami

, ∀i = 1, 2, . . . , n

Definicion 2.1.8. Con respecto a las notaciones de la observacion anterior, se define la

matriz de T respecto de las bases B y D como la matriz A tal que A = (Ai j) ∈Mm×n (R).

Esto es:

[T ]DB =(

[T (u1)D] [T (u2)D] · · · [T (un)D])

=

A11 A12 · · · A1n

A21 A22 · · · A2n

......

. . ....

Am1 Am2 · · · Amn

96

Page 99: Apuntes MAT-023 USM

Apuntes Mat023 (Segundo semestre 2014)

Ejemplo 2.1.21. Dada la transformacion lineal T : R3 → R2 definida por:

T (x, y, z) = (3x− 2y + z, x− y)

Determine la matriz asociada a T respecto de las bases:

1. canonicas de R3 y R2, respectivamente.

Solucion.

T (1, 0, 0) = (3, 1) = 3 (1, 0) + 1 (0, 1)

T (0, 1, 0) = (−2, −1) = −2 (1, 0)− 1 (0, 1)

T (0, 0, 1) = (1, 0) = 1 (1, 0) + 0 (0, 1)

ası

[T ]CR2CR3

=

(3 −2 1

1 −1 0

)2. las bases B = (1, 1, 0) ; (1, 0, 1) ; (1, 2, 3) y D = (2, 1) ; (−1, 1)

Solucion.

T (1, 1, 0) = (3− 2, 1− 1) = (1, 0) = α1 (2, 1) + β 1 (−1, 1)

T (1, 0, 1) = (3 + 1, 1) = (4, 1) = α2 (2, 1) + β 2 (−1, 1)

T (1, 2, 3) = (3− 4 + 3, 1− 2) = (2,−1) = α3 (2, 1) + β 3 (−1, 1)

determinemos los escalares, los determinaremos en un solo sistema(2 −1 1 4 2

1 1 0 1 −1

)∼(

1 0 13

53

13

0 1 −13−2

3−4

3

)se sigue

[T ]DB =

(13

53

13

−13−2

3−4

3

)Ejemplo 2.1.22. Sea D : R3 [x]→ R3 [x] el operador lineal derivada, es decir, p→ D (p) =

p′ y considere las bases de R3 [x] dadas por

B1 =

2x3, 3x− x2, 1− x, 1

B2 =

1, x, x2, x3

luego

D(2x3)

= 0 · 1 + 0x+ 6x2 + 0x3

D(3x− x2

)= 3 (1)− 2 (x) + 0x2 + 0x3

D (1− x) = −1 (1) + 0x+ 0x2 + 0x3

D (1) = 0 (1) + 0x+ 0x2 + 0x3

97

Page 100: Apuntes MAT-023 USM

Apuntes Mat023 (Segundo semestre 2014)

entonces

[D]B2B1 =

0 3 −1 0

0 −2 0 0

6 0 0 0

0 0 0 0

usted puede calcular [D]B1

B2 y vera que es una matriz diferente.

Ejemplo 2.1.23. Si B = v1, v2, . . . , vn es una base del espacio vectorial V y I : V → V

la transformacion identidad entonces

[I]BB = In×n (la matriz identidad)

En efecto, Iv1 = v1 = 1v1 + 0v2 + 0v3 + · · ·+ 0vn se sigue

[Iv1]B =

1

0...

0

similarmente Iv2 = v1 = 0v1 + 1v2 + 0v3 + · · ·+ 0vn se sigue

[Iv2]B =

0

1

0...

0

ası [I]BB es

[I]BB = In×n

Definicion 2.1.9. Sean U un espacio vectorial sobre K tales que B = u1, u2, . . . , un y

D = v1, v2, . . . , vn son bases ordenadas distintas de U . Sea 1 : U → U la transformacion

lineal identidad. Se define la matriz cambio de base como la matriz asociada a 1 respecto

de las bases B y D.

Teorema 2.1.9. Sean U y V espacios vectoriales finito dimensionales sobre K con bases By D, respectivamente. Sean, ademas, S, T : U → V dos transformaciones lineales, entonces:

[αT + S]DB = α [T ]DB + [S]DB

98

Page 101: Apuntes MAT-023 USM

Apuntes Mat023 (Segundo semestre 2014)

Teorema 2.1.10. Sean U, V y W espacios vectoriales de dimension finita sobre K tales que

B,D y E son bases de U, V y W , respectivamente. Suponga que T : U → V y S : V → W

son transformaciones lineales, entonces:

[ST ]EB = [S]ED · [T ]DB

Observacion 2.1.11. Sean U y V espacios vectoriales finito dimensionales sobre K con

bases B y E para U , y D y F para V . Considere, ademas, T : U → V una transformacion

lineal. Note que en vista del teorema anterior el cambio de representacion matricial puede

expresarse mediante el siguiente diagrama conmutativo:

UE[T ]FE−→ VF

[1]EB ↑ ↓ [1]DF

UB[T ]DB−→ VD

donde la notacion del tipo UE , por ejemplo, representa el espacio U considerado con la

base E . En efecto, utilizando la convencion usual para la composicion de funciones, tenemos

que:

[T ]DB = [1]DF · [T ]FE · [1]EB

Teorema 2.1.11. Sean U y V espacios vectoriales finito dimensionales sobre K con bases

B y D, respectivamente. Sea, ademas, T : U → V una transformacion lineal entonces T es

invertible si y solo si [T ]DB es invertible, ademas

[T−1

]BD =

([T ]DB

)−1

Teorema 2.1.12. Sean U y V espacios vectoriales sobre el mismo cuerpo K tales que

B = u1, u2, . . . , un es una base ordenada para U y D = v1, v2, . . . , vm es una base

ordenada para V , entonces:

[Tu]D = [T ]DB [u]B

para todo u ∈ U .

Ejemplo 2.1.24. Considere las bases de R3

B1 = (1, 1, 1) , (1, 1, 0) , (1, 0, 0)

y

B2 = (0, 0, 1) , (1, 0, 0) , (0, 1, 0)

99

Page 102: Apuntes MAT-023 USM

Apuntes Mat023 (Segundo semestre 2014)

1. Calcular [I]B2B1 y [I]B1B2 .

2. Calcular [v]Bi para i = 1, 2 donde v = (3, 1, 3)

1. Notemos que

I (1, 1, 1) = (1, 1, 1) = 1 (0, 0, 1) + 1 (1, 0, 0) + 1 (0, 1, 0)

I (1, 1, 0) = (1, 1, 0) = 0 (0, 0, 1) + 1 (1, 0, 0) + 1 (0, 1, 0)

I (1, 0, 0) = (1, 0, 0) = 0 (0, 0, 1) + 1 (1, 0, 0) + 0 (0, 1, 0)

entonces

[I]B2B1 =

1 0 0

1 1 1

1 1 0

como

([I]B1B2

)−1

= [I]B2B1 se sigue que [I]B1B2 =(

[I]B2B1

)−1

1 0 0 1 0 0

1 1 1 0 1 0

1 1 0 0 0 1

∼ 1 0 0 1 0 0

0 1 0 −1 0 1

0 0 1 0 1 −1

ası

[I]B1B2 =

1 0 0

−1 0 1

0 1 −1

2. Notemos que (3, 1, 3) = 3 (0, 0, 1) + 3 (1, 0, 0) + 1 (0, 1, 0)

[(3, 1, 3)]B2 =

3

3

1

luego

[(3, 1, 3)]B1 = [I]B1B2 [(3, 1, 3)]B2

=

1 0 0

−1 0 1

0 1 −1

3

3

1

=

3

−2

2

100

Page 103: Apuntes MAT-023 USM

Apuntes Mat023 (Segundo semestre 2014)

Ejemplo 2.1.25. Sean B = (1, 1, 1) ; (0, 0, 1) ; (1, 0, 1) y D = (1, 1) ; (0, 1) bases de

R3 y R2, respectivamente. Sea T una transformacion lineal tal que:

[T ]DB =

(4 2 −2

−3 −1 1

)Hallar explıcitamente T (x, y, z).

Solucion. Primero buscamos [(x, y, z)]B esto es

(x, y, z) = α (1, 1, 1) + β (0, 0, 1) + γ (1, 0, 1)

luego 1 0 1 x

1 0 0 y

1 1 1 z

∼ 1 0 0 y

0 1 0 z − x0 0 1 x− y

ası

[(x, y, z)]B =

y

z − xx− y

se sigue que

[T (x, y, z)]D = [T ]DB [(x, y, z)]B

=

(4 2 −2

−3 −1 1

) y

z − xx− y

=

(6y − 4x+ 2z

2x− 4y − z

)de donde obtenemos

T (x, y, z) = (6y − 4x+ 2z) (1, 1) + (2x− 4y − z) (0, 1)

= (6y − 4x+ 2z, 2y − 2x+ z)

Calculo con coordenadas

Observacion 2.1.12. Como se puede observar de la seccion anterior, las matrices asociadas

a las transformaciones lineales dependen de las coordenadas. Es decir, dependen de las

bases consideradas en los espacios vectoriales involucrados. Por consiguiente, el calculo

de los subespacios notables (nucleo e imagen) asociados a una transformacion lineal no

es directo por definicion. Sin embargo, con los procedimientos algebraicos adecuados se

pueden obtener los espacios notables asociados a las transformaciones a traves de calculos

con coordenadas. Veamos algunos ejemplos:

101

Page 104: Apuntes MAT-023 USM

Apuntes Mat023 (Segundo semestre 2014)

Ejemplo 2.1.26. Sean:

B = (1, 1, 1) ; (0, 1, 1) ; (1, 0, 1)

y:

D = (1, 1, 1) ; (1, 0, 1) ; (1, 2, 3)

bases de R3 y T ∈ L (R3,R3) tales que:

[T ]DB =

4 2 −2

−3 −1 1

1 2 −2

Calcule el nucleo y la imagen de T , sin explicitar T (x, y, z).

Solucion. Sea u un vector cualquiera en el nucleo de T , es decir T (u) = 0. Por tanto,

[T · u]D = [0]D = 0. Sin embargo, sabemos que [T · u]D = [T ]DB [u]B, luego: 4 2 −2

−3 −1 1

1 2 −2

a

b

c

=

0

0

0

en donde [u]B =

(a b c

)T. Escalonando la matriz de coeficientes: 4 2 −2

−3 −1 1

1 2 −2

→ 1 0 0

0 1 −1

0 0 0

se obtiene que a = 0 y b = c. Por lo tanto:

[u]B =

0

c

c

, c ∈ R

Luego:

u = 0 · (1, 1, 1) + c · (0, 1, 1) + c · (1, 0, 1) = (c, c, 2c) , c ∈ R

Por lo tanto, obtenemos de lo anterior que:

kerT = 〈 (1, 1, 2) 〉

Para calcular la imagen de T , debemos notar primeramente que de:

[T ]DB =

4 2 −2

−3 −1 1

1 2 −2

102

Page 105: Apuntes MAT-023 USM

Apuntes Mat023 (Segundo semestre 2014)

se obtiene que:

[T (1, 1, 1)]D =

4

−3

1

, [T (0, 1, 1)]D =

2

−1

2

, [T (1, 0, 1)]D =

−2

1

−2

Por consiguiente:

T (1, 1, 1) = 4 (1, 1, 1) + (−3) (1, 0, 1) + 1 (1, 2, 3) = (2, 6, 4)

T (0, 1, 1) = 2 (1, 1, 1) + (−1) (1, 0, 1) + 2 (1, 2, 3) = (3, 6, 7)

T (1, 0, 1) = (−2) (1, 1, 1) + 1 (1, 0, 1) + (−2) (1, 2, 3) = (−3,−6,−7)

Ahora bien:

ImT = 〈T (1, 1, 1) , T (0, 1, 1) , T (1, 0, 1)〉= 〈(2, 6, 4) , (3, 6, 7) , (−3,−6,−7)〉= 〈(2, 6, 4) , (3, 6, 7) 〉

note que el rango de T es 2.

Ejemplo 2.1.27. Sean:

B = (1, 1,−1) ; (0, 2,−1) ; (1, 0, 1)

y:

D =

1 + 2x+ x2; 1 + 3x+ 2x2; 2 + x+ 3x2

bases de R3 y R2 [x], respectivamente. Sea T ∈ L (R3,R2 [x]) tales que:

[T ]DB =

5 2 −2

−3 −1 1

1 4 −3

Verifique que T es un isomorfismo entre R3 y R2 [x]. Calcule, ademas, T−1.

Solucion. Para verificar que T es un isomorfismo, basta notar que det [T ]DB 6= 0. En efecto:

det [T ]DB =

∣∣∣∣∣∣5 2 −2

−3 −1 1

1 4 −3

∣∣∣∣∣∣ = 1

Por otro lado, para calcular T−1, procedemos por coordenadas. En efecto:[T−1 (a, b, c)

]B =

[T−1

]BD [(a, b, c)]D

=(

[T ]DB

)−1

[(a, b, c)]D

103

Page 106: Apuntes MAT-023 USM

Apuntes Mat023 (Segundo semestre 2014)

Luego:

[T−1

]BD =

5 2 −2

−3 −1 1

1 4 −3

−1

=

−1 −2 0

−8 −13 1

−11 −18 1

Necesitamos, ahora, las coordenadas de a+ bx+ cx2 respecto de la base D. Esto es:

α(1 + 2x+ x2

)+ β

(1 + 3x+ 2x2

)+ γ

(2 + x+ 3x2

)= a+ bx+ cx2

Resolviendo el sistema anterior, obtenemos:

[a+ bx+ cx2

]D =

α

β

γ

=

74a+ 1

4b− 5

4c

14b− 5

4a+ 3

4c

14c+ 1

4a− 1

4b

Luego:

[T−1

(a+ bx+ cx2

)]B =

−1 −2 0

−8 −13 1

−11 −18 1

74a+ 1

4b− 5

4c

14b− 5

4a+ 3

4c

14c+ 1

4a− 1

4b

=

34a− 3

4b− 1

4c

52a− 11

2b+ 1

2c

72a− 15

2b+ 1

2c

Ası:

T−1(a+ bx+ cx2

)=

(3

4a− 3

4b− 1

4c

)(1, 1,−1) +

(5

2a− 11

2b+

1

2c

)(0, 2,−1)

+

(7

2a− 15

2b+

1

2c

)(1, 0, 1)

=

(17

4a− 33

4b+

1

4c,

23

4a− 47

4b+

3

4c,

1

4a− 5

4b+

1

4c

)

Ejercicios resueltos de Transformaciones lineales

1. Considere la transformacion

T : M2×2 (R) → R4(a b

c d

)→ T

(a b

c d

)= (a+ b, a+ c− b, c, d)

104

Page 107: Apuntes MAT-023 USM

Apuntes Mat023 (Segundo semestre 2014)

a) Determine una base de Ker(T ) e Im(T ).

Solucion. Vamos a buscar el Nucleo de la transformacion,

ker (T ) =

(a b

c d

)∈M2×2 (R) : T

(a b

c d

)= (0, 0, 0, 0)

=

(a b

c d

)∈M2×2 (R) : (a+ b, a+ c− b, c, d) = (0, 0, 0, 0)

=

(a b

c d

)∈M2×2 (R) :

a+ b = 0

a− b+ c = 0

c = 0

d = 0

=

(a b

c d

)∈M2×2 (R) : a = b = c = d = 0

=

(0 0

0 0

)del teorema de las dimensiones se sigue que

Dim (ker (T )) + Dim (Im (T )) = Dim (M2×2 (R))

0 + Dim (Im (T )) = 4

ası Dim (Im (T )) = 4 y luego Im (T ) = R4 (T es biyectiva)

b) Si

B1 =

(1 −1

2 0

),

(0 −1

2 1

),

(0 1

1 1

),

(1 1

0 3

)y

B2 = (1, 0, 2, 0) , (1, 0, 2, 1) , (0, 0, 1, 1) , (1, 1, 0,−3)

son bases de M2×2 (R) y R4 respectivamente determinar [T ]B2B1 .

Solucion. Como

T

(1 −1

2 0

)= (0, 4, 2, 0)

T

(0 −1

2 1

)= (−1, 3, 2, 1)

T

(0 1

1 1

)= (1, 0, 1, 1)

T

(1 1

0 3

)= (2, 0, 0, 3)

105

Page 108: Apuntes MAT-023 USM

Apuntes Mat023 (Segundo semestre 2014)

se sigue que necesitamos resolver los sistemas

(0, 4, 2, 0) = α1 (1, 0, 2, 0) + β1 (1, 0, 2, 1) + γ1 (0, 0, 1, 1) + δ1 (1, 1, 0,−3)

(−1, 3, 2, 1) = α2 (1, 0, 2, 0) + β2 (1, 0, 2, 1) + γ2 (0, 0, 1, 1) + δ2 (1, 1, 0,−3)

(1, 0, 1, 1) = α3 (1, 0, 2, 0) + β3 (1, 0, 2, 1) + γ3 (0, 0, 1, 1) + δ3 (1, 1, 0,−3)

(2, 0, 0, 3) = α4 (1, 0, 2, 0) + β4 (1, 0, 2, 1) + γ4 (0, 0, 1, 1) + δ4 (1, 1, 0,−3)

y ası la matriz asociada sera

[T ]B2B1 =

α1 α2 α3 α4

β1 β2 β3 β4

γ1 γ2 γ3 γ4

δ1 δ2 δ3 δ4

vamos a resolver los 4 sistema de una sola vez

1 1 0 1 0 −1 1 2

0 0 0 1 4 3 0 0

2 2 1 0 2 2 1 0

0 1 1 −3 0 1 1 3

1 0 0 0 −6 −4 −1 −5

0 1 0 0 2 0 2 7

0 0 1 0 10 10 −1 −4

0 0 0 1 4 3 0 0

ası

[T ]B2B1 =

−6 −4 −1 −5

2 0 2 7

10 10 −1 −4

4 3 0 0

c) Si la transformacion es un isomorfismo (biyectiva) determine[

T−1]B1B2

Solucion. ya sabemos que la transformacion es biyectiva, como[T−1

]B1B2

=(

[T ]B2B1

)−1

se sigue

−6 −4 −1 −5

2 0 2 7

10 10 −1 −4

4 3 0 0

−1

=

322

− 322

− 922

1711

− 211

211

611

−1911

2311

2111

811

411

− 711

− 411

− 111

− 611

=

[T−1

]B1B2

106

Page 109: Apuntes MAT-023 USM

Apuntes Mat023 (Segundo semestre 2014)

2. Construir una transformacion lineal T : R3 → R3 que cumpla las siguientes condicio-

nes (simultaneamente)

a) Ker(T ) = (x, y, z) ∈ R3 : (x, y, z) = t (1, 0,−1) para t ∈ Rb) Im(T ) = (x, y, z) ∈ R3 : x− 2y + z = 0

Solucion. La transformacion tiene que cumplir las dos condiciones, notamos que

ker (T ) = 〈(1, 0,−1)〉

y

Im (T ) =

(x, y, z) ∈ R3 : x− 2y + z = 0

=

(x, y, z) ∈ R3 : x+ z = 2y

=

(x,x+ z

2, z

)∈ R3 : x, z ∈ R

=

⟨(1,

1

2, 0

),

(0,

1

2, 1

)⟩desde el punto de vista del teorema de las dimensiones estas dos condiciones no son

incompatibles puesto que

Dim (ker (T )) + Dim (Im (T )) = Dim(R3)

= 3

para la posible transformacion. Vamos a utilizar el teorema que nos permite construir

transformaciones si la conocemos en una base

T (1, 0, 0) =

(1,

1

2, 0

)T (0, 1, 0) =

(0,

1

2, 1

)T (1, 0,−1) = (0, 0, 0)

notamos que los vectores (1, 0, 0) , (0, 1, 0) y (1, 0,−1) son linealmente independientes

pues ∣∣∣∣∣∣1 0 1

0 1 0

0 0 −1

∣∣∣∣∣∣ = −1 6= 0

y luego forman una base de R3 estas condiciones determinan por completo una

transformacion lineal, note tambien que

Im (T ) = 〈T (1, 0, 0) , T (0, 1, 0) , T (1, 0,−1)〉

=

⟨(1,

1

2, 0

),

(0,

1

2, 1

), (0, 0, 0)

⟩=

⟨(1,

1

2, 0

),

(0,

1

2, 1

)⟩=

(x, y, z) ∈ R3 : x− 2y + z = 0

107

Page 110: Apuntes MAT-023 USM

Apuntes Mat023 (Segundo semestre 2014)

y (1, 0,−1) ∈ ker (T ) que tiene dimension 1 (por el teorema de las dimensiones) ası

ker (T ) = 〈(1, 0,−1)〉=

(x, y, z) ∈ R3 : (x, y, z) = t (1, 0,−1) para t ∈ R

podemos tambien determinar en forma explıcita esta transformacion

(x, y, z) = α (1, 0, 0) + β (0, 1, 0) + γ (1, 0,−1)

α + γ = x

β = y

−γ = z

de donde α = x+ z, β = y, γ = −z ası

(x, y, z) = (x+ z) (1, 0, 0) + y (0, 1, 0) + (−z) (1, 0,−1)

luego

T (x, y, z) = T ((x+ z) (1, 0, 0) + y (0, 1, 0) + (−z) (1, 0,−1))

= (x+ z)T (1, 0, 0) + yT (0, 1, 0) + (−z)T (1, 0,−1)

= (x+ z)

(1,

1

2, 0

)+ y

(0,

1

2, 1

)+ (−z) (0, 0, 0)

=

(x+ z,

1

2x+

1

2y +

1

2z, y

)3. Sea T ∈ L (R3,R2) con B = (0, 0, 1) ; (0, 2, 1) ; (3, 2, 1) y C = (1,−3) ; (2,−5)

bases ordenadas de R3 y R2, respectivamente. Supongamos que:

[T ]CB =

(−1 3 2

4 −12 −8

)∈M2×3 (R)

Con respecto a las hipotesis anteriores:

a) Hallar kerT sin calcular T (x, y, z).

Solucion. v ∈ ker (T )⇔ [v]B es solucion de [T ]CB x = 0 luego(−1 3 2 0

4 −12 −8 0

)∼(

1 −3 −2 0

0 0 0 0

)luego

[v]B =

α

β

γ

108

Page 111: Apuntes MAT-023 USM

Apuntes Mat023 (Segundo semestre 2014)

es solucion si y solo si

α− 3β − 2γ = 0

de esta forma

[v]B =

α

β

γ

=

3β + 2γ

β

γ

para β, γ ∈ R

luego

v ∈ ker (T )⇔ (x, y, z) = (3β + 2γ) (0, 0, 1) + β (0, 2, 1) + γ (3, 2, 1)

para β, γ ∈ R esto es

(x, y, z) = (3γ, 2β + 2γ, 4β + 3γ) para β, γ ∈ R

ası

ker (T ) = 〈(3, 2, 3) , (0, 2, 4)〉

b) Determinar T (x, y, z).

Solucion. Notemos que

[T (x, y, z)]C = [T ]CB [(x, y, z)]B

luego

(x, y, z) = α (0, 0, 1) + β (0, 2, 1) + γ (3, 2, 1)

ası

3γ = x

2β + 2γ = y

α + β + γ = z

luego α = z − 12y, β = 1

2y − 1

3x, γ = 1

3x de donde tenemos

[(x, y, z)]B =

z − 12y

12y − 1

3x

13x

ası

[T (x, y, z)]C =

(−1 3 2

4 −12 −8

) z − 12y

12y − 1

3x

13x

=

(2y − 1

3x− z

43x− 8y + 4z

)109

Page 112: Apuntes MAT-023 USM

Apuntes Mat023 (Segundo semestre 2014)

ası

T (x, y, z) =

(2y − 1

3x− z

)(1,−3) +

(4

3x− 8y + 4z

)(2,−5)

=

(7

3x− 14y + 7z, 34y − 17

3x− 17z

)

c) ¿Por que es falso que T (x, y, z) = [T ]CB ·

x

y

z

?

Solucion. Esta igualdad no es posible ni por los ordenes de las matrices.

d) ¿Cuando es valida la igualdad anterior?

Solucion. Al considerar las bases canonicas en los espacios de partida y llegada

se cumple una desigualdad del tipo

T (x, y, z)t = [T ]canonicacanonica ·

x

y

z

(ademas la transformacion este definida de Rn en Rm)

e) Hallar la nulidad de T y el rango de T sin hallar la Im(T )?

Solucion. El rango de la transformacion lineal es igual al rango de la matriz

asociada en este caso es 1.

f ) Hallar Im(T ) y una base para Im(T ).

Solucion. Como ya tenemos la transformacion podemos determinar la imagen

como

Im (T ) = 〈T (1, 0, 0) , T (0, 1, 0) , T (0, 0, 1)〉

=

⟨(7

3,−17

3

), (−14, 34) , (7,−17)

⟩= 〈(7,−17)〉

esto es una recta que pasa por el origen con direccion (7,−17).

4. Sean U = (x, y, z) ∈ R3 : 2x− 3y + 5z = 0 un espacio vectorial y A una base

ordenada de U y C la base canonica de R2. Suponga que T : U → R2 es una

transformacion lineal definida por T (x, y, z) = (x− y, x− z) tal que:

[T ]CA =

(1 2

1 1

)∈M2×2 (R)

Determine la base A.

110

Page 113: Apuntes MAT-023 USM

Apuntes Mat023 (Segundo semestre 2014)

Solucion. Sea A = (α, β, γ) , (ϕ, κ, λ) la base de U del ejercicio (el espacio tiene

dimension 2) entonces

[T (α, β, γ)]C =

(1

1

)[T (ϕ, κ, λ)]C =

(2

1

)ademas

2α− 3β + 5γ = 0

2ϕ− 3κ+ 5λ = 0

pero

[T (α, β, γ)]C =

(α− βα− γ

)[T (ϕ, κ, λ)]C =

(ϕ− κϕ− λ

)de donde tenemos el sistema de ecuaciones

α− β = 1

α− γ = 1

ϕ− κ = 2

ϕ− λ = 1

2α− 3β + 5γ = 0

2ϕ− 3κ+ 5λ = 0

que tiene solucion α = 12, κ = −9

4, β = −1

2, λ = −5

4, γ = −1

2, ϕ = −1

4ası

A =

(1

2,−1

2,−1

2

),

(−1

4,−9

4,−5

4

)5. Considere la transformacion lineal

T : M2×2 (R) → R4(x y

z w

)→ T

(x y

z w

)= (x− y, x− z, x− w, x)

y suponga que

B1 =

(1 0

0 1

),

(−2 0

0 0

),

(0 0

1 0

),

(0 −1

0 0

)B2 = (1, 1, 1, 1) , (1, 1, 0, 0) , (1, 0, 0, 0) , (1, 1, 1, 0)

111

Page 114: Apuntes MAT-023 USM

Apuntes Mat023 (Segundo semestre 2014)

a) Calcular [T ]B2B1 , muestre que T es invertible (isomorfismo) y calcule T−1 en forma

explıcita.

Solucion. Como

T

(x y

z w

)= (x− y, x− z, x− w, x)

se sigue

T

(1 0

0 1

)= (1, 1, 0, 1)

T

(−2 0

0 0

)= (−2,−2,−2,−2)

T

(0 0

1 0

)= (0,−1, 0, 0)

T

(0 −1

0 0

)= (1, 0, 0, 0)

luego tenemos que resolver los sistemas

(1, 1, 0, 1) = α1 (1, 1, 1, 1) + β1 (1, 1, 0, 0) + γ1 (1, 0, 0, 0) + δ1 (1, 1, 1, 0)

(−2,−2,−2,−2) = α2 (1, 1, 1, 1) + β2 (1, 1, 0, 0) + γ2 (1, 0, 0, 0) + δ2 (1, 1, 1, 0)

(0,−1, 0, 0) = α3 (1, 1, 1, 1) + β3 (1, 1, 0, 0) + γ3 (1, 0, 0, 0) + δ3 (1, 1, 1, 0)

(1, 0, 0, 0) = α4 (1, 1, 1, 1) + β4 (1, 1, 0, 0) + γ4 (1, 0, 0, 0) + δ4 (1, 1, 1, 0)

resolveremos los 4 sistemas1 1 1 1 1 −2 0 1

1 1 0 1 1 −2 −1 0

1 0 0 1 0 −2 0 0

1 0 0 0 1 −2 0 0

1 0 0 0 1 −2 0 0

0 1 0 0 1 0 −1 0

0 0 1 0 0 0 1 1

0 0 0 1 −1 0 0 0

se sigue

[T ]B2B1 =

1 −2 0 0

1 0 −1 0

0 0 1 1

−1 0 0 0

b) Encontrar

([T ]B2B1

)−1

112

Page 115: Apuntes MAT-023 USM

Apuntes Mat023 (Segundo semestre 2014)

Solucion. ([T ]B2B1

)−1

=[T−1

]B1B2

=

1 −2 0 0

1 0 −1 0

0 0 1 1

−1 0 0 0

−1

=

0 0 0 −1

−12

0 0 −12

0 −1 0 −1

0 1 1 1

c) Considere las bases canonicas deM2×2 (R) y R4

C1 =

(1 0

0 0

),

(0 1

0 0

),

(0 0

1 0

),

(0 0

0 1

)C2= ((1, 0, 0, 0)) , (0, 1, 0, 0) , (0, 0, 1, 0) , (0, 0, 0, 1)

calcular [T ]C2C1 usando la matrices de cambio de base adecuadas.

Solucion. Para calcular [T ]C2C1 necesitamos hacer

[I]C2B2 [T ]B2B1 [I]B1C1

ası

α

(1 0

0 1

)+ β

(−2 0

0 0

)+ γ

(0 0

1 0

)+ δ

(0 −1

0 0

)=

(α− 2β −δγ α

)de donde tenemos que resolver 4 sistemas

1 −2 0 0 1 0 0 0

0 0 0 −1 0 1 0 0

0 0 1 0 0 0 1 0

1 0 0 0 0 0 0 1

1 0 0 0 0 0 0 1

0 1 0 0 −12

0 0 12

0 0 1 0 0 0 1 0

0 0 0 1 0 −1 0 0

de aquı

[I]B1C1 =

0 0 0 1

−12

0 0 12

0 0 1 0

0 −1 0 0

113

Page 116: Apuntes MAT-023 USM

Apuntes Mat023 (Segundo semestre 2014)

(note que es mas facil pensarlo como(

[I]C1B1

)−1

= [I]B1C1 si la canonica esta en la

llegada es escribir los vectores como columna) y

[I]C2B2 =

1 1 1 1

1 1 0 1

1 0 0 1

1 0 0 0

ası

[T ]C2C1 = [I]C2B2 [T ]B2B1 [I]B1C1

=

1 1 1 1

1 1 0 1

1 0 0 1

1 0 0 0

1 −2 0 0

1 0 −1 0

0 0 1 1

−1 0 0 0

0 0 0 1

−12

0 0 12

0 0 1 0

0 −1 0 0

=

1 −1 0 0

1 0 −1 0

1 0 0 −1

1 0 0 0

6. Considere la transformacion lineal T : R2 [x]→ R2 [x] dada por:

T(ax2 + bx+ c

)= (a+ b)x2 + (a+ b+ c)x+ c

a) Encuentre una base para ker (T )

b) Encuentre una base para Im (T )

c) Considere en el dominio la base B = 1 + x2, 1 + x+ x2,−1 y en el codominio

la base D = 1− x, 1 + x, x2 encontrar la matriz asociada a T respecto a estas

bases.

a) Por definicion

ker (T ) =ax2 + bx+ c ∈ R2 [x] : T

(ax2 + bx+ c

)= 0

=ax2 + bx+ c ∈ R2 [x] : (a+ b)x2 + (a+ b+ c)x+ c = 0

=

ax2 + bx+ c ∈ R2 [x] : (a+ b = 0) ∧ (a+ b+ c = 0) ∧ (c = 0)

=

ax2 + bx+ c ∈ R2 [x] :

1 1 0

1 1 1

0 0 1

a

b

c

=

0

0

0

luego 1 1 0

1 1 1

0 0 1

∼ 1 1 0

0 0 1

0 0 0

114

Page 117: Apuntes MAT-023 USM

Apuntes Mat023 (Segundo semestre 2014)

ası

ker (T ) =ax2 + bx+ c ∈ R2 [x] : (a+ b = 0) ∧ (c = 0)

=

ax2 + bx+ c ∈ R2 [x] : (b = −a) ∧ (c = 0)

=

ax2 − ax ∈ R2 [x] : a ∈ R

=

⟨x2 − x

⟩luego el kernel tiene dimension 1 (la nulidad es 1).

b) Notemos que T (ax2 + bx+ c) = (a+ b)x2 + (a+ b+ c)x+ c y luego

(a+ b)x2 + (a+ b+ c)x+ c = a(x2 + x

)+ b(x2 + x

)+ c (x+ 1)

ası

T(ax2 + bx+ c

)∈⟨x2 + x, x2 + x, x+ 1

⟩como hay un elemento repetido, el conjunto es L.D. ası⟨

x2 + x, x2 + x, x+ 1⟩

=⟨x2 + x, x+ 1

⟩se sigue

Im (T ) ⊆⟨x2 + x, x+ 1

⟩por el teorema de las dimensiones se sigue

Dim (ker (T )) + Dim (Im (T )) = Dim (R2 [x])

ası, de la parte a) se tiene

1 + Dim (Im (T )) = 3

de donde se obtiene

Dim (Im (T )) = 2

y ası

Im (T ) =⟨x2 + x, x+ 1

⟩de esta forma, una base para la imagen es x2 + x, x+ 1

c) Como T (ax2 + bx+ c) = (a+ b)x2 + (a+ b+ c)x+ c se sigue que

T(1 + x2

)= x2 + 2x+ 1 = α (1− x) + β (1 + x) + γx2

= γx2 + (β − α)x+ (a+ β)

por determinar a, β, γ.

T(1 + x+ x2

)= 2x2 + 3x+ 1 = α (1− x) + β (1 + x) + γx2

= γx2 + (β − α)x+ (a+ β)

115

Page 118: Apuntes MAT-023 USM

Apuntes Mat023 (Segundo semestre 2014)

por determinar a, β, γ.

T (−1) = −x− 1 = α (1− x) + β (1 + x) + γx2

= γx2 + (β − α)x+ (a+ β)

por determinar a, β, γ.

Vamos a resolver esos 3 problemas de una sola vez, porque hay que resolver los

sistemas

γ = 1

(β − α) = 2

(a+ β) = 1

y

γ = 2

(β − α) = 3

(a+ β) = 1

y

γ = 0

(β − α) = −1

(a+ β) = −1

en todas la matriz de coeficientes es la misma, formamos una tripe matriz

ampliada y escalonamos 0 0 1 1 2 0

−1 1 0 2 3 −1

1 1 0 1 1 −1

Operaciones Elementales∼

1 0 0 −12−1 0

0 1 0 32

2 −1

0 0 1 1 2 0

[T ]DB =

−12−1 0

32

2 −1

1 2 0

(notar que T (1 + x2) = x2 + 2x + 1 =

(−1

2

)(1− x) +

(32

)(1 + x) + 1x2,

T (1 + x+ x2) = 2x2 + 3x + 1 = (−1) (1− x) + 2 (1 + x) + 2x2 y T (−1) =

−x− 1 = 0 (1− x) + (−1) (1 + x) + 0x2)

7. Suponga que S : R2 [x]→ R2 [x] es una transformacion lineal con

[T ]DB =

−12−1 0

32

2 −1

1 2 0

116

Page 119: Apuntes MAT-023 USM

Apuntes Mat023 (Segundo semestre 2014)

B = 1− x, 1 + x, x2 y D = 1 + x2, 1 + x+ x2,−1 (No es el ejercicio anterior ver

las bases)

a) Encontrar ker (T )

b) Encontrar Im (T )

c) Calcule T (2 + x− x2)

d) Encontrar una expresion para T (ax2 + bx+ c)

a) Note que p ∈ ker (T ) si y solo si T (p) = 0 si y solo si

[Tp]D =

0

0

0

pero

[Tp]D = [T ]DB [p]B

luego si resolvemos 0

0

0

= [T ]DB [p]B =

−12−1 0

32

2 −1

1 2 0

α

β

γ

estaremos encontrando las coordenadas α

β

γ

= [p]B

de los vectores de ker (T ). Pues bien −12−1 0

32

2 −1

1 2 0

∼ 1 0 −2

0 1 1

0 0 0

se sigue que p ∈ ker (T ) si y solo si α

β

γ

= [p]B

cumple

α = 2γ y β = −γ

117

Page 120: Apuntes MAT-023 USM

Apuntes Mat023 (Segundo semestre 2014)

ası p ∈ ker (T ) si y solo si

p (x) = (2γ) (1− x) + (−γ) (1 + x) + γx2

para algun γ ∈ R es decir

ker (T ) =⟨−3x+ x2 + 1

⟩b) De la matriz asociada tenemos

[T (1− x)]D =

−12

32

1

ası

T (1− x) =

(−1

2

)(1 + x2

)+

(3

2

)(1 + x+ x2

)+ (1) (−1)

= x2 +3

2x

tambien

[T (1 + x)]D =

−1

2

2

ası

T (1− x) = (−1)(1 + x2

)+ (2)

(1 + x+ x2

)+ (2) (−1)

= x2 + 2x− 1

y

[T(x2)]D =

0

−1

0

ası

T(x2)

= (0)(1 + x2

)+ (−1)

(1 + x+ x2

)+ (0) (−1)

= −x2 − x− 1

luego

Im (T ) =

⟨x2 +

3

2x, x2 + 2x− 1,−x2 − x− 1

⟩del teorema de las dimensiones

x2 + 3

2x, x2 + 2x− 1,−x2 − x− 1

deberıa ser

un conjunto L.D. en efecto 1 1 −132

2 −1

0 −1 −1

∼ 1 0 −2

0 1 1

0 0 0

118

Page 121: Apuntes MAT-023 USM

Apuntes Mat023 (Segundo semestre 2014)

(−2)

(x2 +

3

2x

)+ (1)

(x2 + 2x− 1

)= −x2 − x− 1

se sigue

Im (T ) =

⟨x2 +

3

2x, x2 + 2x− 1

⟩c) Para calcular T (2 + x− x2) podemos hacer lo siguiente

[2 + x− x2

]B =

α

β

γ

donde

2 + x− x2 = α (1− x) + β (1 + x) + γx2

γ = −1

β − α = 1

α + β = 2

α = 12, β = 3

2, γ = −1 luego

[T(2 + x− x2

)]D =

−12−1 0

32

2 −1

1 2 0

1232

−1

=

−74

19472

se obtiene

T(2 + x− x2

)=

(−7

4

)(1 + x2

)+

(19

4

)(1 + x+ x2

)+

(7

2

)(−1)

= 3x2 +19

4x− 1

2

d) Encontremos una expresion para

T(ax2 + bx+ c

)notemos que

ax2 + bx+ c = α (1− x) + β (1 + x) + γx2

si y solo si

γ = a

β − α = b

α + β = c

119

Page 122: Apuntes MAT-023 USM

Apuntes Mat023 (Segundo semestre 2014)

esto es 0 0 1 a

−1 1 0 b

1 1 0 c

∼ 1 0 0 1

2c− 1

2b

0 1 0 12b+ 1

2c

0 0 1 a

luego

ax2 + bx+ c =

(1

2c− 1

2b

)(1− x) +

(1

2b+

1

2c

)(1 + x) + ax2

podemos encontrar T (ax2 + bx+ c) en dos formas

T(ax2 + bx+ c

)=

(1

2c− 1

2b

)T (1− x) +

(1

2b+

1

2c

)T (1 + x) + aT

(x2)

=

(1

2c− 1

2b

)(x2 +

3

2x

)+

(1

2b+

1

2c

)(x2 + 2x− 1

)+a(−x2 − x− 1

)= (c− a)x2 +

(−a+

1

4b+

7

4c

)x+

(−1

2b− 1

2c− a

)la otra forma es

[T(ax2 + bx+ c

)]D =

−12−1 0

32

2 −1

1 2 0

(

12c− 1

2b)(

12b+ 1

2c)

a

=

−14b− 3

4c

14b− a+ 7

4c

12b+ 3

2c

y ası

T(ax2 + bx+ c

)=

(−1

4b− 3

4c

)(1 + x2

)+

(1

4b− a+

7

4c

)(1 + x+ x2

)+

(1

2b+

3

2c

)(−1)

= (c− a)x2 +

(−a+

1

4b+

7

4c

)x+

(−1

2b− 1

2c− a

)8. Muestre que T :M2×2 (R)→ R3 [x] dada por

T

[(a b

c d

)]= (a+ b)x3 + cx2 + (a+ d)x+ (c− d)

es una transformacion lineal ademas determine el ker (T ) e Im(T ).

120

Page 123: Apuntes MAT-023 USM

Apuntes Mat023 (Segundo semestre 2014)

Solucion. En general, si V y W son espacios vectoriales, una funcion F : V → W

es llamada transformacion lineal si cumple: i) ∀α ∈ K,∀v ∈ V , F (αv) = αF (v) y ii)

∀v1, v2 ∈ V , F (v1 + v2) = F (v1) +F (v2) (esto quiere decir, que envıa combinaciones

lineales en combinaciones lineales). En nuestro ejercicio los elementos del espacio

de partida son matrices de orden 2 × 2 luego para probar la primera propiedad

necesitamos considerar un escalar arbitrario y una matriz arbitraria y en la segunda

propiedad, dos matrices arbitrarias como sigue:

i) Sea α ∈ R y

(a b

c d

)∈M2×2 (R) se tiene

T

(a b

c d

)]= T

[(αa αb

αc αd

)]= T

[(A B

C D

)]donde A = αa,B = αb, C = αc,D = αd

= (A+B)x3 + Cx2 + (A+D)x+ (C −D)

= ((αa) + (αb))x3 + (αc)x2 + (αa+ αd)x+ (αc− αd)

= α((a+ b)x3 + cx2 + (a+ d)x+ (c− d)

)= αT

[(a b

c d

)](Explicacion: tenemos que mostrar que el escalar se puede “sacar” de la funcion y para

esto utilizamos solamente la definicion, en la linea T

(a b

c d

)]= T

[(αa αb

αc αd

)]no sabemos como actua la funcion T sobre el producto escalar α

(a b

c d

)es por

eso que aplicamos la definicion de producto escalar de matrices y ası

α

(a b

c d

)=

(αa αb

αc αd

)

y ahora podemos aplicar la definicion de T a la matriz

(αa αb

αc αd

)(en este caso si

sabemos como actua T ), para que se vea mas claro agregue la linea(αa αb

αc αd

)=

(A B

C D

)donde A = αa,B = αb, C = αc,D = αd

pero no es necesario, entonces por definicion

T

[(A B

C D

)]= (A+B)x3 + Cx2 + (A+D)x+ (C −D)

121

Page 124: Apuntes MAT-023 USM

Apuntes Mat023 (Segundo semestre 2014)

y ahora reemplazamos para obtener la igualdad deseada)

ii) Sean

(a11 a12

a21 a22

),

(b11 b12

b21 b22

)∈M2×2 (R) matrices arbitrarias, se tiene

T

[(a11 a12

a21 a22

)+

(b11 b12

b21 b22

)]= T

[(a11 + b11 a12 + b12

a21 + b21 a22 + b22

)]= T

[(A B

C D

)]donde A = a11 +b11, B = a12 +b12, C = a21 +b21, D = a22 +b22, aplicando la definicion

de T se sigue

T

[(A B

C D

)]= (A+B)x3 + Cx2 + (A+D)x+ (C −D)

= ((a11 + b11) + (a12 + b12))x3 + (a21 + b21)x2 +

((a11 + b11) + (a22 + b22))x+ ((a21 + b21)− (a22 + b22))

= ((a11 + a12) + (b11 + b12))x3 + (a21 + b21)x2 +

((a11 + a22) + (b11 + b22))x+ ((a21 − a22) + (b21 − b22))

=((a11 + a12)x3 + a21x

2 + (a11 + a22)x+ (a21 − a22))

+((b11 + b12)x3 + b21x

2 + (b11 + b22)x+ (b21 − b22))

= T

[(a11 a12

a21 a22

)]+ T

[(b11 b12

b21 b22

)]de esto concluimos

T

[(a11 a12

a21 a22

)+

(b11 b12

b21 b22

)]= T

[(a11 a12

a21 a22

)]+ T

[(b11 b12

b21 b22

)]de i) y ii) T es una transformacion lineal.

Vamos a buscar el ker (T )

ker (T ) = v ∈ V : T (v) = θW

=

(a b

c d

)∈M2×2 (R) : T

[(a b

c d

)]= 0 (el polinomio 0)

=

(a b

c d

)∈M2×2 (R) : (a+ b)x3 + cx2 + (a+ d)x+ (c− d) = 0

=

(a b

c d

)∈M2×2 (R) : (a+ b) = 0 ∧ c = 0 ∧ (a+ d) = 0 ∧ (c− d) = 0

122

Page 125: Apuntes MAT-023 USM

Apuntes Mat023 (Segundo semestre 2014)

de esta forma buscamos todas la matrices

(a b

c d

)tales que

a+ b = 0

c = 0

a+ d = 0

c− d = 0

o matricialmente 1 1 0 0

0 0 1 0

1 0 0 1

0 0 1 −1

a

b

c

d

=

0

0

0

0

resolvemos el sistema escalonando

1 1 0 0 0

0 0 1 0 0

1 0 0 1 0

0 0 1 −1 0

1 1 0 0 0

0 −1 0 1 0

0 0 1 0 0

0 0 0 −1 0

de esto se obtiene a = b = c = d = 0 es decir

ker (T ) =

(0 0

0 0

)(se concluye que la transformacion es inyectiva) Por el teorema de las dimensiones se

tiene lo siguiente

Dim (ker (T )) + Dim (Im (T )) = Dim (M2×2 (R)) = 4

pero

Dim (ker (T )) = 0

ası

Dim (Im (T )) = 4

como Dim (T ) < R3 [x] y R3 [x] tiene dimension 4, se sigue Im (T ) = R3 [x]. Otra

forma para determinar la imagen es utilizar el resultado que la imagenes de una base

generan la imagen de la transformacion, luego

Im (T ) =

⟨T

(1 0

0 0

), T

(0 1

0 0

), T

(0 0

1 0

), T

(0 0

0 1

)⟩pero notemos que

T

(1 0

0 0

)= (1 + 0)x3 + 0x2 + (1 + 0)x+ (0− 0)

= x3 + x

123

Page 126: Apuntes MAT-023 USM

Apuntes Mat023 (Segundo semestre 2014)

T

(0 1

0 0

)= (0 + 1)x3 + 0x2 + (0 + 0) x+ (0− 0)

= x3

T

(0 0

1 0

)= (0 + 0)x3 + 1x2 + (0 + 0) x+ (1− 0)

= x2 + 1

T

(0 0

0 1

)= (0 + 0)x3 + 0x2 + (0 + 1) x+ (0− 1)

= x− 1

ası

Im (T ) =⟨x3 + x, x3, x2 + 1, x− 1

⟩y mostrar que x3 + x, x3, x2 + 1, x− 1 es un conjunto L.I. luego Im (T ) tiene di-

mension 4 y por tanto tiene que ser R3 [x].

9. Encontrar por lo menos una transformacion lineal T :M2×2 (R)→ R3 tal que

ker (T ) =

(a b

c d

)∈M2×2 (R) : a+ d+ c = 0

Solucion. Notemos que

ker (T ) =

(a b

c d

)∈M2×2 (R) : a+ d+ c = 0

=

(a b

c d

)∈M2×2 (R) : a = −c− d

=

(−c− d b

c d

)∈M2×2 (R) : b, c, d ∈ R

=

c

(−1 0

1 0

)+ b

(0 1

0 0

)+ d

(−1 0

0 1

)∈M2×2 (R) : b, c, d ∈ R

⟨(

−1 0

1 0

),

(0 1

0 0

),

(−1 0

0 1

)⟩luego ker (T ) tiene dimension 3 (no puede ser base de M2×2 (R) que tiene dimension

4), sabemos que para conocer completamente una T.L. necesitamos conocerla sobre

una base, buscamos algun elemento L.I. con las tres matrices que forman el kernel,

para ello,

α

(−1 0

1 0

)+ β

(0 1

0 0

)+ γ

(−1 0

0 1

)+ δ

(a b

c d

)=

(0 0

0 0

)124

Page 127: Apuntes MAT-023 USM

Apuntes Mat023 (Segundo semestre 2014)

debe tener por unica solucion α = β = γ = δ = 0. Esto se traduce a

−α− γ + aδ = 0

β + bδ = 0

α + δc = 0

γ + δd = 0

matricialmente la ampliada−1 0 −1 a 0

0 1 0 b 0

1 0 0 c 0

0 0 1 d 0

∼−1 0 −1 a 0

0 1 0 b 0

0 0 −1 a+ c 0

0 0 0 a+ c+ d 0

la solucion es unica si y solo si a+ c+ d 6= 0, de esta forma, podemos considerar la

matriz (1 1

1 1

)pues 1 + 1 + 1 = 3 6= 0. Ahora para definir la transformacion (por ejemplo)

T

(−1 0

1 0

)= (0, 0, 0)

T

(0 1

0 0

)= (0, 0, 0)

T

(−1 0

0 1

)= (0, 0, 0)

T

(1 1

1 1

)= (1, 1, 1)

esto determina completamente quien es T (notar que

(−1 0

1 0

),

(0 1

0 0

),

(−1 0

0 1

)son elementos del kernel pero

(1 1

1 1

)no lo es). La T en forma explıcita se determina

de la siguiente manera, Como

B =

(−1 0

1 0

),

(0 1

0 0

),

(−1 0

0 1

),

(1 1

1 1

)

es base, dado una matriz

(x y

z w

)arbitraria, deben existir escalares α, β, γ, δ tales

que (x y

z w

)= α

(−1 0

1 0

)+ β

(0 1

0 0

)+ γ

(−1 0

0 1

)+ δ

(1 1

1 1

)125

Page 128: Apuntes MAT-023 USM

Apuntes Mat023 (Segundo semestre 2014)

para buscar tales escalares resolvemos el sistema−1 0 −1 1 x

0 1 0 1 y

1 0 0 1 z

0 0 1 1 w

1 0 0 0 23z − 1

3x− 1

3w

0 1 0 0 y − 13x− 1

3w − 1

3z

0 0 1 0 23w − 1

3x− 1

3z

0 0 0 1 13w + 1

3x+ 1

3z

de donde se tiene

α =2

3z − 1

3x− 1

3w

β = y − 1

3x− 1

3w − 1

3z

γ =2

3w − 1

3x− 1

3z

δ =1

3w +

1

3x+

1

3z

ası (x y

z w

)=

(2

3z − 1

3x− 1

3w

)(−1 0

1 0

)+

(y − 1

3x− 1

3w − 1

3z

)(0 1

0 0

)+

(2

3w − 1

3x− 1

3z

)(−1 0

0 1

)+

(1

3w +

1

3x+

1

3z

)(1 1

1 1

)luego

T

(x y

z w

)=

(2

3z − 1

3x− 1

3w

)(0, 0, 0)

+

(y − 1

3x− 1

3w − 1

3z

)(0, 0, 0)

+

(2

3w − 1

3x− 1

3z

)(0, 0, 0)

+

(1

3w +

1

3x+

1

3z

)(1, 1, 1)

=

(1

3w +

1

3x+

1

3z,

1

3w +

1

3x+

1

3z ,

1

3w +

1

3x+

1

3z

)ası

T : M2×2 (R)→ R3(x y

z w

)→ T

(x y

z w

)=

(1

3w +

1

3x+

1

3z,

1

3w +

1

3x+

1

3z ,

1

3w +

1

3x+

1

3z

)126

Page 129: Apuntes MAT-023 USM

Apuntes Mat023 (Segundo semestre 2014)

10. Sea B la base canonica de R2 y considere las transformaciones T : R2 → R2 y

S : R2 → R2 definidas por

T (a, b) = (2a+ 3b, 4a− 7b)

y

S (a, b) = (3a− 2b, a− b)

calcular [T ]B , [S]B y [T S]B comprobar que

[T ]B · [S]B = [T S]B

Solucion. Al ser la matriz con respecto a la base canonica se tiene

[T ]B =

(2 3

4 −7

)y [S]B =

(3 −2

1 −1

)ası

[T ]B · [S]B =

(2 3

4 −7

)(3 −2

1 −1

)=

(9 −7

5 −1

)por otro lado

T S (a, b) = T (3a− 2b, a− b)= (2 (3a− 2b) + 3 (a− b) , 4 (3a− 2b)− 7 (a− b))=

(9a− 7b 5a− b

)ası

[T S]B =

(9 −7

5 −1

)11. Sea T : R4 → R4 una transformacion lineal tal que

T (1, 1, 0, 0) = (0, 1, 0,−1)

T (1, 0, 1, 0) = (1, 1, 1, 0)

si ademas se tiene que T T = IR4 determinar la matriz asociada a T respecto a la

base canonica C de R4. ¿Es T un isomorfismo? Justifique.

Solucion. Por la propiedad T T = IR4 se sigue

T (T (1, 1, 0, 0)) = T (0, 1, 0,−1) = (1, 1, 0, 0)

T (T (1, 0, 1, 0)) = T (1, 1, 1, 0) = (1, 0, 1, 0)

127

Page 130: Apuntes MAT-023 USM

Apuntes Mat023 (Segundo semestre 2014)

ası

T (1, 1, 0, 0) = (0, 1, 0,−1)

T (1, 0, 1, 0) = (1, 1, 1, 0)

T (0, 1, 0,−1) = (1, 1, 0, 0)

T (1, 1, 1, 0) = (1, 0, 1, 0)

queremos determinar la matriz asociada a la base canonica

T (1, 1, 1, 0)− T (1, 1, 0, 0) = T (0, 0, 1, 0)

= (1, 0, 1, 0)− (0, 1, 0,−1)

= (1,−1, 1, 1)

T (0, 0, 1, 0) = (1,−1, 1, 1)

T (1, 0, 0, 0) = T (1, 0, 1, 0)− T (0, 0, 1, 0)

= (1, 1, 1, 0)− (1,−1, 1, 1)

= (0, 2, 0,−1)

T (0, 1, 0, 0) = T (1, 1, 0, 0)− T (1, 0, 0, 0)

= (0, 1, 0,−1)− (0, 2, 0,−1)

= (0,−1, 0, 0)

T (0, 0, 0, 1) = T (0, 1, 0, 0)− T (0, 1, 0,−1)

= (0,−1, 0, 0)− (1, 1, 0, 0)

= (−1,−2, 0, 0)

ası

[T ]CC =

0 0 1 −1

2 −1 −1 −2

0 0 1 0

−1 0 1 0

es facil ver que es isomorfismo, pues

T T = I

⇒ [T T ]CC = I4

⇒ [T ]CC [T ]CC = I4

ası [T ]CC es invertible y luego T es invertible. Otra forma∣∣∣∣∣∣∣∣∣0 0 1 −1

2 −1 −1 −2

0 0 1 0

−1 0 1 0

∣∣∣∣∣∣∣∣∣ = 1

128

Page 131: Apuntes MAT-023 USM

Apuntes Mat023 (Segundo semestre 2014)

y luego T es invertible.

Otra forma es utilizar la matriz de cambio de base

[I]CB =

1 1 0 1

1 0 1 1

0 1 0 1

0 0 −1 0

ası

[I]BC =

1 1 0 1

1 0 1 1

0 1 0 1

0 0 −1 0

−1

=

1 0 −1 0

1 −1 0 −1

0 0 0 −1

−1 1 1 1

luego

[T ]CC = [T ]CB [I]BC

=

0 1 1 1

1 1 1 0

0 1 0 1

−1 0 0 0

1 0 −1 0

1 −1 0 −1

0 0 0 −1

−1 1 1 1

=

0 0 1 −1

2 −1 −1 −2

0 0 1 0

−1 0 1 0

12. Sea T :M2×2(R)→ R3 la funcion definida por:

T

(a b

c d

)=(a+ b+ d, b− c, −a+ c+ d

)a) Demuestre que T es una transformacion lineal.

b) Calcule el nucleo y la imagen de T .

129

Page 132: Apuntes MAT-023 USM

Apuntes Mat023 (Segundo semestre 2014)

a) Sean(a bc d

),(x yz w

)∈M2(R) y α ∈ R, entonces:

T

(a b

c d

)+

(x y

z w

))= T

(αa+ x αb+ y

αc+ z αd+ w

)=(

(αa+ x) + (αb+ y) + (αd+ w),

, (αb+ y)− (αc+ z),

,−(αa+ x) + (αc+ z) + (αd+ w))

=(α(a+ b+ d) + (x+ y + z),

, α(b− c) + (y − z),

, α(−a+ c+ d) + (−x+ z + w))

= α(a+ b+ d, b− c,−a+ c+ d

)+

+(x+ y + w, y − z,−x+ z + w

)= αT

(a b

c d

)+ T

(x y

z w

)Se concluye, por tanto, que T es una transformacion lineal.

b) Sabemos, por definicion, que el nucleo de T es el conjunto:

kerT =(

a bc d

)∈M2(R) : T

(a bc d

)= (0, 0, 0)

Pero, T

(a bc d

)=(a+ b+ d, b− c, −a+ c+ d

). Entonces, ( a bc d

)∈ kerT , si y solo

si: (a+ b+ d, b− c, −a+ c+ d

)= (0, 0, 0)

S :

a+ b+ d = 0

b− c = 0−a+ c+ d = 0

Ası, mediante escalonamiento de la matriz asociada AS, se tiene:

AS =

1 1 0 1

0 1 −1 0

−1 0 1 1

−→1 0 1 1

0 1 −1 0

0 0 2 2

−→1 0 0 0

0 1 0 1

0 0 1 1

= EAS

130

Page 133: Apuntes MAT-023 USM

Apuntes Mat023 (Segundo semestre 2014)

de donde obtenemos que a = 0, b = −d y c = −d. Por lo tanto:

kerT =(

a bc d

): T(a bc d

)= (0, 0, 0)

=(

a bc d

): a = 0 d ∧ b = −d ∧ c = −d, d ∈ R

=(

0 −d−d d

): d ∈ R

=

⟨(0 −1−1 1

)⟩Finalmente, como dimM2(R) = 4 y la nulidad de T es 1 tenemos, por el Teorema

de la Dimension, que el rango de T es 3. Por tanto, ImT = R3.

13. Sean C1 y C2 las bases canonicas de R3 y R4 respectivamente (orden usual), T : R3 →R4, S : R4 → R4 y L : R4 → R3 transformaciones lineales tales que

[T ]C2C1 =

1 −1 0

−1 1 0

2 2 −1

0 2 −1

, [S]C2C2 =

1 −1 0 1

1 1 0 1

−1 1 1 1

−1 −1 0 1

y [L]C1C2 =

1 −1 0 1

0 1 0 1

−1 0 1 −1

a) Determine explıcitamente L S T : R3 → R3

Notemos que

[L S T ]C1C1 = [L]C1C2 [S]C2C2 [T ]C2C1

=

1 −1 0 1

0 1 0 1

−1 0 1 −1

1 −1 0 1

1 1 0 1

−1 1 1 1

−1 −1 0 1

1 −1 0

−1 1 0

2 2 −1

0 2 −1

=

2 0 −1

0 4 −2

−2 4 0

se sigue que

[(L S T ) (x, y, z)]C1 =

2 0 −1

0 4 −2

−2 4 0

x

y

z

=

2x− z4y − 2z

4y − 2x

ası

(L S T ) (x, y, z) = (2x− z, 4y − 2z, 4y − 2x)

131

Page 134: Apuntes MAT-023 USM

Apuntes Mat023 (Segundo semestre 2014)

b) ¿Es S una transformacion biyectiva?

Como ∣∣∣∣∣∣∣∣∣

1 −1 0 1

1 1 0 1

−1 1 1 1

−1 −1 0 1

∣∣∣∣∣∣∣∣∣ = 4

se sigue que la transformacion es biyectiva.

c) Determine ker (T ) e Im(T ).

Hay varias formas de hacer esto, pero como las bases son las canonicas1 −1 0

−1 1 0

2 2 −1

0 2 −1

1 0 0

0 1 0

0 0 1

0 0 0

ası ker (T ) = (0, 0, 0) e

Im (T ) =

1

−1

2

0

T

,

−1

1

2

2

T

,

0

0

−1

−1

T⟩

es de dimension 3.

14. Sean u = (1, 0, 1), v = (1, 0,−1) y w = u× v vectores en R3 y C la base canonica de

R3:

a) Si T : R3 → R3 es una transformacion lineal definida por T (u) = w, T (v) = v

y T (w) = u, determine [T ]CC y [T T ]CCNotemos que

w= u× v

= (1, 0, 1)× (1, 0,−1)

= (0, 2, 0)

se sigue

T (1, 0, 1) = (0, 2, 0)

T (1, 0,−1) = (1, 0,−1)

T (0, 2, 0) = (1, 0, 1)

132

Page 135: Apuntes MAT-023 USM

Apuntes Mat023 (Segundo semestre 2014)

notamos que ∣∣∣∣∣∣1 1 0

0 0 2

1 −1 0

∣∣∣∣∣∣ = 4

luego B = u,v,w es base de R3 ademas

[T ]CB =

0 1 1

2 0 0

0 −1 1

por otro lado

[I]CB =

1 1 0

0 0 2

1 −1 0

y (

[I]CB

)−1

= [I]BC =

1 1 0

0 0 2

1 −1 0

−1

=

12

0 12

12

0 −12

0 12

0

se sigue

[T ]CC = [T ]CB [I]BC

=

0 1 1

2 0 0

0 −1 1

12

0 12

12

0 −12

0 12

0

=

12

12−1

2

1 0 1

−12

12

12

y

[T T ]CC = [T ]CC [T ]CC

=

12

12−1

2

1 0 1

−12

12

12

2

=

1 0 0

0 1 0

0 0 1

b) Determine explıcitamente T−1 o argumente que no esta definida.

133

Page 136: Apuntes MAT-023 USM

Apuntes Mat023 (Segundo semestre 2014)

Note que [T ]CC tiene inversa luego la transformacion tiene inversa o bien T (B) = Bse sigue que es sobre y por teorema de las dimensiones es biyectiva, ademas

[T−1

]CC =

12

12−1

2

1 0 1

−12

12

12

y ası

[T−1 (x, y, z)

]C =

12

12−1

2

1 0 1

−12

12

12

x

y

z

=

12x+ 1

2y − 1

2z

x+ z12y − 1

2x+ 1

2z

esto es

T−1 (x, y, z) =

(1

2x+

1

2y − 1

2z, x+ z,

1

2y − 1

2x+

1

2z

)c) Si B = u,v,w determine [T ]BB

Desarrollo:

[T ]BB =

0 0 1

0 1 0

1 0 0

d) Encontrar una matriz A tal que A−1 [T ]BB A = [T ]CC

Desarrollo: Podemos usar A = [I]BC la matriz de cambio de base:([I]BC

)−1

[T ]BB [I]BC = [T ]CC

esto es 12

0 12

12

0 −12

0 12

0

−1 0 0 1

0 1 0

1 0 0

12

0 12

12

0 −12

0 12

0

=

12

12−1

2

1 0 1

−12

12

12

Ejercicios de la seccion

1. Determine si las siguientes funciones son transformaciones lineales:

a) T : R2 → R, T (x, y) = |x+ y|

b) T : R3 → R2, T (x, y, z) = (2x+ y, z − y)

134

Page 137: Apuntes MAT-023 USM

Apuntes Mat023 (Segundo semestre 2014)

c) T : R→ R3, T (x) = (x, 2x, x2)

d) T : R3 → R3, T (x, y, z) = (cos x, sin y, z)

2. ¿Existe un transformacion lineal de R2 en R2 que transforma los vectores (1, 4) y

(−3, 2) en los vectores (2, 1) y (4, 2), respectivamente? En caso afirmativo determine

la imagen del vector (−2, a), con a ∈ R.

3. Sea U un espacio vectorial tal que dimU = n < ∞. Suponga que u1, u2, . . . , un es

una base ordenada de U . Suponga que T : U → U es tal que:

T (ui) = ui, ∀i = 1, 2, . . . , n (2.3)

Entonces:

a) Demuestre que existen funciones T : U → U definidas por la condicion (2.3) que

son transformaciones lineales.

b) Verifique que si T es una transformacion lineal que satisface la condicion (2.3),

entonces T = IU , donde IU es la funcion identidad de U en U .

4. Sea T : R3 → R2 la transformacion lineal definida por:

T (x, y, z) = (x+ y − 2z, x+ y)

Calcule kerT e ImT .

5. Sea S ∈ L (R3,M2 (R)) definida por:

S (x, y, z) =

(x− 3y y − 2z

x z

)a) Calcule condiciones sobre a ∈ R de modo que:

3e1 + ae2 − e3 ∈ kerT

donde ei es el i–esimo vector de la base canonica de R3.

b) Calcule ImT

6. Sea T : R2 [x]→M2 (R) una funcion definida por:

T(p (x)

)=

(p′′ (0)

∫ 1

0p (x) dx

p (−1) 0

)

a) Demuestre que T es una transformacion lineal.

135

Page 138: Apuntes MAT-023 USM

Apuntes Mat023 (Segundo semestre 2014)

b) Hallar una base para el kernel de T .

c) Hallar ImT y su dimension.

7. Sea T ∈ L (U, V ) tal que kerT = 0. Suponga que B es un conjunto linealmente

independiente en U . Demuestre que:

T (B) = T (u) : u ∈ B

es linealmente independiente en V .

8. Sea T ∈ L (U,U). Entonces, las siguientes proposiciones son equivalentes:

(i) kerT ∩ ImT = 0.(ii) Si T 2 (u) = 0, entonces T (u) = 0.

9. Sea T : R3 → R3 la transformacion lineal definida por:

T (x, y, z) = (3x, x− y, 2x+ y + z)

¿Es T invertible? En caso afirmativo, hallar una formula para T−1.

10. Sean U y V espacios vectoriales sobre el mismo cuerpo K, tales que dimU = dimV =

n <∞. Sea, ademas, T ∈ L (U, V ) tal que:

Tu = 0 =⇒ u = 0

Entonces, existe una base u1, u2, . . . , un de U tal que Tu1, Tu2, . . . , Tun es una base

de V .

11. Sea B =

(1, 2, 1) ; (2, 1, 1) , (1, 1, 2)

una base de R3 y T : R3 → R3 una transfor-

macion lineal tal que:

[T ]BB =

−3 −3 −5

1 1 1

2 2 4

a) Determinar una base del kerT .

b) Determinar una base del kerT 2.

c) Determinar una base de ImT .

12. Sea T : R3 → R3 tal que:

[T ]DB =

1 2 0

1 1 1

3 −1 1

donde B = (1, 1, 2) ; (0, 1, 1) ; (−1, 0, 0) y D = (1, 1, 1) ; (0, 1, 1) ; (0, 0, 1) son

bases de R3. Entonces:

136

Page 139: Apuntes MAT-023 USM

Apuntes Mat023 (Segundo semestre 2014)

a) Encuentre T (1, 3, 1), sin calcular T (x, y, z).

b) Determine T−1 (x, y, z), si acaso existe.

c) Hallar [T ]FE , sin calcular T (x, y, z), si E = (0, 1, 2) ; (1, 1, 1) ; (0, 1, 0) y F =

(1, 1, 0) ; (1,−1, 1) ; (1, 0, 0) son, tambien, bases de R3.

13. Sean T : R3 [x]→ R3 [x] una funcion definida por:

T(p (x)

)= p′ (x)− x · p′′ (x)

y B1 = 1− x2, 1 + x2, 1− x3, 2 + x, B2 = 1, x2 + x3, x3 − x2, 1− 2x dos bases

ordenadas de R3 [x]. Entonces:

a) Pruebe que existe T es una transformacion lineal.

b) Hallar kerT y una base para ImT . ¿Es T invertible?

c) Calcule [T ]B2B1 .

14. Diremos que un espacio vectorial V es suma directa de los subespacios W1 y W2,

lo cual anotaremos, V = W1 ⊕W2, si W1 ∩W2 = 0, y para cada v ∈ V , existen

w1 ∈ W1 y w2 ∈ W2 tales que v = w1 +w2. Sean V un espacio vectorial de dimension

finita y T : V → V una transformacion lineal. Pruebe que:

V = kerT ⊕ ImT ⇐⇒ ker(T 2)

= kerT

15. Sean U y V espacios vectoriales sobre el mismo cuerpo K y sea S un isomorfismo de

U en V . Demuestre que:

T 7→ S T S−1

es un isomorfismo de L (U,U) en L (V, V ).

16. Hallar explıcitamente, esto es, calculando T (ax3 + bx2 + cx+ d), una transformacion

lineal T : R3 [x]→ R3 tal que:

ImT =

(x, y, z) ∈ R3 : x+ 2y − z = 0

y cuya nulidad sea 2.

17. Sean:

B = (1,−2, 1) , (0, 1,−2) , (1, 1, 1)

y:

D =

(1 2

−2 1

),

(0 0

1 −1

),

(1 −2

3 −3

),

(2 1

2 1

)

137

Page 140: Apuntes MAT-023 USM

Apuntes Mat023 (Segundo semestre 2014)

bases ordenadas de R3 y M2 (R), respectivamente. Considere T : R3 → M2 (R)

definida por:

T (x, y, z) =

(x+ 2y x− y + 3z

0 x+ y + z

)a) Hallar [T ]DB .

b) Determine condiciones sobre a ∈ R para que (x, y, z) ∈ kerT , sabiendo que:

[(x, y, z)]B =

−1

a

2

c) Hallar ImT , usando [T ]DB .

18. Sean T :M2 (R)→M2 (R) una transformacion lineal y:

B =

(1 0

0 1

),

(1 1

0 1

),

(1 0

1 1

),

(1 1

2 2

)una base ordenada de M2 (R) tal que:

[T ]BB =

1 0 1 0

1 2 3 0

1 1 2 0

1 1 2 1

Determinar una base D de M2 (R) de modo que [T ]DD sea una matriz diagonal.

(Ayuda: El polinomio caracterıstico de T es fT (λ) = λ (λ− 4) (λ− 1)2)

19. Sean:

B =

1 + x− x2, 2x− x2, 1 + x2

y:

D = (1, 2, 1) , (1, 3, 2) , (2, 1, 3)

bases ordenadas de R2 [x] y R3, respectivamente. Consideremos T : R2 [x]→ R3 tal

que:

[T ]DB =

5 2 −2

−3 −1 1

1 4 −3

a) Demuestre que T es un isomorfismo.

b) Calcule kerT−1 e ImT−1.

138

Page 141: Apuntes MAT-023 USM

Apuntes Mat023 (Segundo semestre 2014)

c) Hallar explıcitamente T−1.

20. Sean B = 1 + x, 1− x2, 1 y D = 1, 1 + x, 1 + x+ x2 bases ordenadas de R2 [x] .

Considere T : R2 [x]→ R2 [x] una transformacion lineal tal que:

[T ]DB =

1 0 1

0 1 1

1 1 2

a) Hallar condiciones sobre a ∈ R para que:

1− (a+ 1)x− 2x2 ∈ kerT

b) Hallar condiciones sobre b ∈ R para que:

1− 2x+ (a− 1)x2 ∈ kerT−1

21. Sea:

W =

(x, y, z) ∈ R3 : 2x− 3y + 5z = 0

un subespacio de R3 tal que B es una base W . Considere D = (1, 1) , (2, 1) una

base de R2. Sea T ∈ L (W,R2) definida por T (x, y, z) = (x+ y − z, 3x− y + 2z) tal

que:

[T ]DB =

(1 2

−3 −1

)Determine la base B.

22. Considere el plano en R3 dado por:

W =

(x, y, z) ∈ R3 : ax+ by + cz = 0

con a, b, c ∈ R. Para cualquier u ∈ R3, denotaremos por PW (u) al punto del plano W

que se encuentra a menor distancia de u. Tal funcion PW (u) se llama la proyeccion

ortogonal de u sobre W . Entonces:

a) Dado u ∈ R3, hallar una expresion para PW (u).

b) Demuestre que la funcion PW (u) : R3 → R3 definida por:

u 7→ PW (w)

es una transformacion lineal.

c) Calcule [PW ]CC, donde C es la base canonica de R3, y verifique que:([PW ]CC

)2

= [PW ]CC ∧(

[PW ]CC

)T= [PW ]CC

139

Page 142: Apuntes MAT-023 USM

Apuntes Mat023 (Segundo semestre 2014)

23. Considere el subespacio vectorial W < R2 [x] definido por:

W =⟨1− x2, x+ 2x2

⟩Demuestre que existe una transformacion lineal T : R2 [x]→ R2 [x] diagonalizable tal

que −1 y 1 sean sus valores propios, y que el espacio propio asociado a −1 sea W .

Ecuaciones diferenciales lineales

Definiciones

Definicion 2.2.1. Sean I ⊆ R un intervalo, A0, A1, . . . , An y R funciones continuas sobre

I tales que A (x) 6= 0, para cada x ∈ I. Una ecuacion diferencial de la forma:

An (x) y(n) + An−1 (x) y(n−1) + An−2 (x) y(n−2) + · · ·+ A1 (x) y′ + A0 (x) y = R (x) (2.4)

se denomina ecuacion diferencial lineal de orden n. Las funciones A0, A1, . . . , An son

llamados coeficientes de la ecuacion diferencial. Si R ≡ 0, la ecuacion lineal se denomina

ecuacion homogenea.

Observacion 2.2.1. En general, los puntos en los cuales An (x) = 0 se conocen como

puntos singulares y su estudio no sera considerado en este curso.

Observacion 2.2.2. La ecuacion (2.4) puede escribirse como:

y(n) + pn−1 (x) y(n−1) + pn−2 (x) y(n−2) + · · ·+ p1 (x) y′ + p0 (x) y = Q (x) (2.5)

al dividir la ecuacion (2.4) por An (x), de esta forma

pi (x) =Ai (x)

An (x)para i = 0, . . . , n− 1

y

Q (x) =R (x)

An (x)

esta forma de la ecuacion es llamada forma normal.

Ejemplo 2.2.1. Son ecuaciones diferenciales lineales las siguientes ecuaciones:

1. y(4) + 3xy′′ − 2 (cosx) y′ + y = e2x cos 3x

2. y′′ + y = 0

3. Las ecuaciones de Euler, definidas como:

(ax+ b)n y(n) + A1 (ax+ b)n−1 y(n−1) + · · ·+ An−1 (ax+ b) y′ + Any = Q (x)

con ax+ b > 0.

140

Page 143: Apuntes MAT-023 USM

Apuntes Mat023 (Segundo semestre 2014)

4. las ecuaciones de Legendre, definidas como:(1− x2

)y′′ − 2xy′ + α (α + 1) y = 0

con α ∈ R.

Ejemplo 2.2.2. Considere la ecuacion diferencial:

y′′ − 2y′ + 2y = 0

1. Verifique que las funciones u1 (x) = ex cosx y u2 (x) = ex sinx son soluciones de la

ecuacion diferencial.

2. Sean C1, C2 ∈ R. Verifique que y (x) = C1u1 (x) +C2u2 (x) es solucion de la ecuacion

diferencial.

3. Hallar una solucion y que satisfaga las condiciones iniciales y (0) = 1 e y′ (0) = 4.

Solucion. Por partes:

1. Notemos que

u′1 (x) =d

dx(ex cosx) = ex cosx− ex sinx

u′′1 (x) =d

dx(ex cosx− ex sinx) = −2ex sinx

se sigue

u′′1 − 2u′1 + 2u1 = (−2ex sinx)− 2 (ex cosx− ex sinx) + 2 (ex cosx)

= 0

de manera similar

u′2 (x) =d

dx(ex sinx) = ex (cosx+ sinx)

u′′2 (x) =d

dx(ex (cosx+ sinx)) = 2 (cosx) ex

luego

u′′2 − 2u′2 + 2u2 = (2 (cosx) ex)− 2 (ex (cosx+ sinx)) + 2 (ex sinx)

= 0

141

Page 144: Apuntes MAT-023 USM

Apuntes Mat023 (Segundo semestre 2014)

2. Notar que

y′ (x) = C1u′1 (x) + C2u

′2 (x)

y′′ (x) = C1u′′1 (x) + C2u

′′2 (x)

luego

y′′ − 2y′ + 2y = (C1u′′1 (x) + C2u

′′2 (x))− 2 (C1u

′1 (x) + C2u

′2 (x)) + 2 (C1u1 (x) + C2u2 (x))

= (C1u′′1 (x)− 2C1u

′1 (x) + 2C1u1 (x)) + (C2u

′′2 (x)− 2C2u

′2 (x) + 2C2u2 (x))

= C1 (u′′1 (x)− 2u′1 (x) + 2u1 (x)) + C2 (u′′2 (x)− 2u′2 (x) + 2u2 (x))

= C10 + C20

= 0

3. Sabemos que y (x) = C1ex cosx+ C2e

x sinx es solucion de la ecuacion, para cumplir

las condiciones y (0) = 1 e y′ (0) = 4 las constantes las debemos escoger en forma

adecuada

1 = y (0) = C1

4 = y′ (0) = C1 + C2

ası

1 = C1

4 = C1 + C2

que tiene solucion C1 = 1, C2 = 3. Ası

y (x) = ex cosx+ 3ex sinx

Observacion 2.2.3. Con el objeto de simplificar el estudio de las ecuaciones diferenciales

lineales y el de aprovechar la terminologıa de las transformaciones lineales es que consi-

deraremos la nocion de operador diferencial asociado a una ecuacion diferencial lineal de

orden n. Para ello recordemos que Cn (I) denota el espacio vectorial de todas las funciones

de clase Cn definidas sobre I, esto es el conjunto de todas las funciones f : I → R que son

n veces derivables con f (n) continua sobre I.

Definicion 2.2.2. Sean p0, p1, . . . , pn−1 funciones continuas sobre un intervalo I ⊆ R.

Llamaremos operador diferencial asociado a la ecuacion (2.5) a la funcion L : Cn (I)→ C (I)

definida por:

L (f) = f (n) + pn−1 f(n−1) + · · ·+ p0f (2.6)

142

Page 145: Apuntes MAT-023 USM

Apuntes Mat023 (Segundo semestre 2014)

Observacion 2.2.4. El operador diferencial dado por la ecuacion (2.6) puede escribirse

como:

L = Dn + pn−1Dn−1 + · · ·+ p0Id

en donde Dk representa la derivada de orden k y Id representa la transformacion lineal

identidad.

Teorema 2.2.1. Sea I ⊆ R un intervalo cualquiera. El operador diferencial L : Cn (I)→C (I) es una transformacion lineal.

Observacion 2.2.5. Con la introduccion del operador diferencial L toda ecuacion lineal:

y(n) + pn−1 (x) y(n−1) + · · ·+ p0 (x) y = Q (x)

puede escribirse como:

L (y) = Q (2.7)

Ası, la ecuacion:

L (y) = 0

se llamara ecuacion homogenea asociada a la ecuacion (2.7).

Observacion 2.2.6. La ecuacion (2.7) tiene un analogo finito dimensional con sistemas

de ecuaciones lineales y consecuentemente con la respectiva ecuacion matricial asociada.

En efecto, sea S un sistema de m ecuaciones lineales con n incognitas. Luego, existe una

matriz A ∈Mm×n (K) que contiene los coeficientes del sistema, una matriz X ∈Mm×1 (K)

con las incognitas y ademas, una matriz B ∈Mm×1 (K) con las constantes. El producto

de matrices permite representar el sistema S como la ecuacion matricial siguiente:

AX = B

Observe que las propiedades del algebra de matrices nos permiten escribir X como:

X = Xh +Xp

donde Xh representa la solucion general del sistema homogeneo AX = 0 y Xp representa

una solucion particular del sistema que se asume conocida. En efecto:

AX = A (Xh +Xp)

= AXh + AXp

= 0 +B

= B

Ahora bien, mediante el isomorfismo LK (U, V ) 'Mm×n (K), debe existir una transforma-

cion lineal T : U → V tal que:

A = [T ]CB

143

Page 146: Apuntes MAT-023 USM

Apuntes Mat023 (Segundo semestre 2014)

para ciertas bases ordenadas B y D, de U y V , respectivamente. Por tanto, el espacio

solucion de la ecuacion matricial es, salvo cambio de coordenadas, el nucleo o espacio nulo

de la transformacion lineal T . Por esta razon, nos interesara calcular el nucleo o kernel del

operador diferencial L.

Teorema 2.2.2. Suponga que yh representa la solucion de la ecuacion homogenea asociada

a L (y) = Q y que yp es una solucion particular (esto es, L (yp) = Q) de la misma ecuacion.

Entonces, la solucion general de:

L (y) = Q

es de la forma:

y = yh + yp

En efecto, siguiendo los razonamientos de la observacion anterior, tenemos que:

L (y) = L (yh + yp)

= L (yh) + L (yp)

= 0 +Q

= Q

Observacion 2.2.7. Consideremos la ecuacion diferencial:

y(n) + pn−1 (x) y(n−1) + pn−2 (x) y(n−2) + · · ·+ p1 (x) y′ + p0 (x) y = 0

Por lo expuesto anteriormente, el operador lineal asociado L : Cn (I)→ C (I) definido por:

L = Dn + pn−1Dn−1 + · · ·+ p1D + p0Id

permite expresar la ecuacion como L (y) = 0. Por tanto, una funcion y es solucion de la

ecuacion diferencial homogenea si y solo si y ∈ kerL, notamos ademas que:

kerL ≤ Cn (I)

y el espacio vectorial Cn (I) no es de dimension finita (basta ver que para cada n ≥ 1, el

conjunto 1, x, x2, . . . , xn es linealmente independiente). Una aplicacion muy importante

del Teorema de Existencia y Unicidad para ecuaciones diferenciales sera la de mostrar que el

nucleo de un operador diferencial no solo es de dimension finita, sino que, ademas, es de

dimension n, en donde n es el orden de la ecuacion diferencial lineal. Es decir, si:

L = Dn + pn−1Dn−1 + · · ·+ p1D + p0Id

es un operador diferencial asociado a una ecuacion diferencial lineal, entonces:

dim kerL = n

144

Page 147: Apuntes MAT-023 USM

Apuntes Mat023 (Segundo semestre 2014)

Teorema de Existencia y Unicidad

Observacion 2.3.1. El siguiente teorema juega un rol crucial en establecer que la dimen-

sion del nucleo del operador diferencial L asociado a una ecuacion diferencial lineal de

orden n del tipo:

y(n) + pn−1 (x) y(n−1) + · · ·+ p0 (x) y = Q (x)

es de dimension finita. En particular, en esta seccion se utilizara el Teorema de Existencia

y Unicidad para demostrar que no solo es finita esa dimension, sino que ademas es de

dimension coincide con el orden de la ecuacion lineal. Introduzcamos primeramente la

siguiente definicion:

Definicion 2.3.1. Diremos que la funcion ϕ : I ⊆ R→ R es solucion del problema de

valores iniciales de orden n:

y(n) = f(x, y, y′, y′′, . . . , y(n−1)

)y (x0) = y0, y

′ (x0) = y1, . . . , y(n−1) (x0) = yn−1

si ϕ : I ⊆ R→ R es solucion de la ecuacion en su intervalo de definicion, x0 ∈ I y:

ϕ (x0) = y0, ϕ′ (x0) = y1, . . . , ϕ

(n−1) (x0) = yn−1

Observacion 2.3.2. Para mayor claridad de los conceptos los teoremas seran expuestos

para ecuaciones de orden 2 y notar que la extension se hace por induccion sobre n. Ası, los

resultados se indicaran para orden 2 u orden n segun convenga.

Teorema 2.3.1. Sean I ⊆ R un intervalo abierto y p0, p1 dos funciones continuas sobre I.

Considere el operador diferencial:

L (y) = y′′ + p1y′ + p0y

Si x0 ∈ I e y0, y1 son numeros reales cualesquiera, entonces existe una unica funcion

f : I → R que es solucion de la ecuacion diferencial:

L (y) = 0

y que satisface las condiciones iniciales:

f (x0) = y0 ∧ f ′ (x0) = y1

Teorema 2.3.2. Consideremos el operador diferencial L (y) = y′′ + p1y′ + p0y, con p0 y

p1 funciones continuas sobre un intervalo abierto I ⊆ R. Suponga que u1 y u2 son dos

funciones no nulas, linealmente independientes sobre I, y que son solucion de la ecuacion

diferencial L (y) = 0. Entonces, para todo par de constantes c1 y c2, la combinacion lineal:

c1u1 + c2u2

145

Page 148: Apuntes MAT-023 USM

Apuntes Mat023 (Segundo semestre 2014)

es solucion de la ecuacion diferencial L (y) = 0 en I. Recıprocamente, si y es una solucion

de L (y) = 0 en I, entonces existen constantes c1 y c2 tales que:

y = c1u1 + c2u2

Demostracion. En primer lugar, note que como L es una transformacion lineal, se tiene

que:

C1u1 + C2u2 ∈ kerL

luego, C1u1 + C2u2 es solucion de L (y) = 0.

Por otro lado, como u1 y u2 son linealmente independientes en I, bastara verificar que:

kerL = 〈u1, u2〉

En efecto, sean f ∈ kerL y x0 ∈ I. Por el Teorema de Unicidad, es suficiente verificar que

existen constantes C1 y C2 tales que:C1u1 (x0) + C2u2 (x0) = f (x0)

C1u′1 (x0) + C2u

′2 (x0) = f ′ (x0)

es decir, que las funciones C1u1 +C2u2 y f coinciden en las condiciones iniciales. Entonces,

para que tales constantes existan, es suficiente a su vez que el determinante:∣∣∣∣ u1 (x0) u2 (x0)

u′1 (x0) u′2 (x0)

∣∣∣∣ = u1 (x0)u′2 (x0)− u2 (x0)u′1 (x0)

sea no nulo. Consideremos la funcion W : I ⊆ R→ R definida por:

W (x) =

∣∣∣∣ u1 (x) u2 (x)

u′1 (x) u′2 (x)

∣∣∣∣ = u1 (x)u′2 (x)− u2 (x)u′1 (x)

Supongamos que W (x) = 0, para todo x ∈ I, entonces:

d

dx

u2

u1

(x) =

u1 (x)u′2 (x)− u2 (x)u′1 (x)

u21 (x)

=W (x)

u21 (x)

= 0

luego, por el Teorema del Valor Medio el cociente u2/u1 debe ser constante, pues I es un

intervalo. Esto contradice la hipotesis de que u1 y u2 son linealmente independientes sobre

I. Por tanto, debe existir al menos un x0 ∈ I tal que W (x0) 6= 0, y esto ultimo implica

que existen constantes C1 y C2 tales que:

f (x) = C1u1 (x) + C2u2 (x)

en otras palabras, u1 y u2 forman una base para el nucleo del operador L, y ası:

dim kerL = 2

146

Page 149: Apuntes MAT-023 USM

Apuntes Mat023 (Segundo semestre 2014)

Observacion 2.3.3. Ası, entonces, el operador L asociado a la ecuacion diferencial lineal:

y(n) + pn−1 (x) y(n−1) + · · ·+ p0 (x) y = Q (x)

tiene nucleo kerL con dimension finita dado por el orden de la ecuacion diferencial. Es

decir:

dim kerL = n

vemos entonces que para determinar la solucion general de una ecuacion diferencial

lineal homogenea, necesitamos una familia de n funciones linealmente independientes si

la ecuacion es de orden n. Antes de continuar, veamos algunos ejemplos de funciones

linealmente independientes:

Ejemplo 2.3.1. Sean r1, r2, . . . , rn ∈ R tales que ri 6= rj, para todo i 6= j. Considere la

familia de n funciones definidas por:

ui (x) = eri x, ∀i = 1, 2, . . . , n

con x ∈ U ⊆ R. Note que si n = 2, entonces u1 y u2 son linealmente independientes, pues:

u1 (x)

u2 (x)= e(r1−r2)x

no es constante. Supongase que el resultado es cierto para k funciones exponenciales. Sea:

k+1∑i=1

Cierix = 0, ∀x ∈ U (2.8)

Luego, multiplicando por e−rk+1x y derivando la ecuacion anterior respecto de x, obtenemos:

k∑i=1

Ci (ri − rk+1) e(ri−rk+1)x = 0

Los k numeros ri − rk+1, con 1 ≤ i ≤ k son distintos. Utilizando la hipotesis de induccion,

las k exponenciales de la ecuacion anterior son linealmente independientes en U . Por tanto,

se debe tener que:

Ci (ri − rk+1) = 0, ∀i = 1, 2, . . . , k

pero ri 6= rk+1, para cada i ≤ k, se tiene que Ci = 0 para i ≤ k. Utilizando esto ultimo

junto con la ecuacion (2.8) se concluye tambien que Ck+1 = 0.

Ejemplo 2.3.2. Sea r ∈ R. Las n funciones:

ui (x) = xi−1erx, ∀i = 1, 2, . . . , n

son linealmente independientes en todo intervalo I ⊆ R.

147

Page 150: Apuntes MAT-023 USM

Apuntes Mat023 (Segundo semestre 2014)

Observacion 2.3.4. Si el cociente entre dos funciones u y v es identicamente una constante,

entonces u y v son linealmente dependientes. En efecto, suponga que:

u

v≡ λ, λ constante

entonces u = λv. Es decir, u ∈ 〈v〉, y por tanto, no puede ser linealmente independiente.

Ejemplo 2.3.3. Tres polinomios de primer grado cualesquiera son linealmente dependientes

en (−∞,+∞).

Ejemplo 2.3.4. Cuatro polinomios de segundo grado cualesquiera son linealmente depen-

dientes en (−∞,+∞).

Definicion 2.3.2. Diremos que las funciones u1, u2, . . . , un forman un sistema funda-

mental de soluciones de la ecuacion diferencial L (y) = 0, de orden n, si:

1. L (ui) = 0, para todo i = 1, 2, . . . , n.

2. u1, u2, . . . , un son linealmente independientes (sobre I).

Observacion 2.3.5. Note que un sistema fundamental de soluciones u1, u2, . . . , un es una

base para el nucleo de L. Es decir:

kerL = 〈u1, u2, . . . , un〉 ≤ Cn (I)

Ejemplo 2.3.5. Considere la ecuacion y′′+y = 0. Verifique que las funciones u1 (x) = cosx

y u2 (x) = sin x, forman un sistema fundamental de soluciones de la ecuacion diferencial.

Solucion. Las funciones sinx y cosx son linealmente independientes y son soluciones de

la ecuacion y′′ + y = 0, se sigue que ker (D2 + 1) = 〈sinx, cosx〉.

El wronskiano

Observacion 2.4.1. En el teorema anterior, se demostro que el operador diferencial L,

asociado a la ecuacion diferencial lineal homogenea de orden 2 siguiente:

y′′ + p1 (x) y′ + p0 (x) y = 0

tiene nucleo:

kerL = 〈u1, u2〉

en el cual el determinante: ∣∣∣∣ u1 (x0) u2 (x0)

u′1 (x0) u′2 (x0)

∣∣∣∣juega un papel importante en relacion con la independencia lineal de las funciones, tenemos

la siguiente definicion:

148

Page 151: Apuntes MAT-023 USM

Apuntes Mat023 (Segundo semestre 2014)

Definicion 2.4.1. Llamaremos wronskiano de las funciones u1 (x) , u2 (x) , . . . , un (x) a:

W (x) =

∣∣∣∣∣∣∣∣∣u1 (x) u2 (x) · · · un (x)

u′1 (x) u′2 (x) · · · u′n (x)...

.... . .

...

u(n−1)1 (x) u

(n−1)2 (x) · · · u

(n−1)n (x)

∣∣∣∣∣∣∣∣∣n×n

Se anota tambien W [u1, u2, . . . , un] (x).

Ejemplo 2.4.1. Calcule el wronskiano de u1 (x) = cos x y u2 (x) = sinx.

Solucion. W (x) =

∣∣∣∣ cosx sinx

− sinx cosx

∣∣∣∣ = cos2 x+ sin2 x = 1

Teorema 2.4.1. Sean u1 (x) y u2 (x) dos soluciones de la ecuacion diferencial:

y′′ + p1 (x) y′ + p0 (x) y = 0 (2.9)

Entonces, el wronskiano:

W (x) =

∣∣∣∣ u1 (x) u2 (x)

u′1 (x) u′2 (x)

∣∣∣∣satisface la ecuacion diferencial:

W ′ + p1 (x)W = 0

Demostracion. Note que:

W ′ = (u1u′2 − u′1u2)

= u′1u′2 + u1u

′′2 − u′′1u2 − u′1u′2

= u1u′′2 − u′′1u2

luego:

W ′ + p1 (x)W = u1u′′2 − u′′1u2 + p1 (x) u1u

′2 − u′1u2

= u1 u′′2 + p1 (x)u′2 − u2 u′′1 + p1 (x)u′1= u1 u′′2 + p1 (x)u′2 + p0 (x)u2 − u2 u′′1 + p1 (x)u′1 + p0 (x)u1= u1 · 0− u2 · 0= 0

Por tanto, W ′ + p1 (x)W = 0, como se querıa demostrar.

149

Page 152: Apuntes MAT-023 USM

Apuntes Mat023 (Segundo semestre 2014)

Observacion 2.4.2. En vista del teorema anterior, si u1 (x) y u2 (x) son dos soluciones

de la ecuacion (2.9), la ecuacion diferencial asociada al wronskiano:

W ′ + p1 (x)W = 0

es de variable separable, luego:∫dW

W= −

∫p1 (x) dx+ C

Por tanto:

W (x) = W (x0) e−∫p1(x)dx

para algun x0 ∈ I. Por tanto se tiene el siguiente teorema:

Teorema 2.4.2 (Formula de Abel para el wronskiano). Sea W = W (x) el wronskiano

asociado a dos soluciones linealmente independientes de la ecuacion diferencial de segundo

orden:

y′′ + p1 (x) y′ + p0 (x) y = 0

entonces:

W (x) = W (x0) e−∫p1(x)dx

Teorema 2.4.3. Sean u1, u2, . . . , un soluciones de la ecuacion diferencial:

y(n) + pn−1 (x) y(n−1) + · · ·+ p0 (x) y = 0

donde las funciones p1, p2, . . . , pn−1 son continuas en un intervalo I ⊆ R. Entonces,

u1, u2, . . . , un son linealmente independientes sobre I si y solamente si W [u1, u2, . . . , un] (x) 6=0 en I.

Ejemplo 2.4.2. Sean u1 (x) = x y u2 (x) = 1x

soluciones de la ecuacion diferencial:

y′′ +1

xy′ − 1

x2y = 0

definida para x > 0. Hallar la solucion general.

Solucion. Para hallar la solucion general, es suficiente probar que u1 (x) y u2 (x) son

linealmente independientes. Por el teorema anterior:

W

[x,

1

x

]=

∣∣∣∣ x 1x

1 − 1x2

∣∣∣∣ = −2 6= 0

Por tanto:

y (x) = C1x+ C21

x

150

Page 153: Apuntes MAT-023 USM

Apuntes Mat023 (Segundo semestre 2014)

Teorema 2.4.4 (Formula de Abel para una segunda solucion). Sea u1 (x) una solucion no

trivial de la ecuacion diferencial:

y′′ + p1 (x) y′ + p0 (x) y = 0

entonces, una segunda solucion u2 (x), linealmente independiente con u1 (x), esta dada por:

u2 (x) = u1 (x)

∫e−

∫p1(x)dx

u21 (x)

dx

Demostracion. Como:

W (x) =

∣∣∣∣ u1 (x) u2 (x)

u′1 (x) u′2 (x)

∣∣∣∣= u1u

′2 − u′1u2

y:

W (x) = C e−∫p1(x)dx

obtenemos la ecuacion diferencial lineal de primer orden en la variable u2 siguiente:

u1u′2 − u′1u2 = C e−

∫p1(x)dx

Dividiendo la ecuacion anterior por u21, se tiene que:

u1u′2 − u′1u2

u21

=C

u21

e−∫p1(x)dx

Por consiguiente:d

dx

u2

u1

=C

u21

e−∫p1(x)dx

integrando y eligiendo valores adecuados para las constantes de integracion, se obtiene

finalmente que:

u2 (x) = u1 (x)

∫e−

∫p1(x)dx

u21 (x)

dx

Ejemplo 2.4.3. Hallar la solucion general de la ecuacion de Legendre, con α = 1:(1− x2

)y′′ − 2xy′ + 2y = 0, |x| < 1

Solucion. Notamos que u1 (x) = x es una solucion particular de la ecuacion diferencial.

Luego, de:

y′′ − 2x

1− x2y′ +

2

1− x2y = 0

151

Page 154: Apuntes MAT-023 USM

Apuntes Mat023 (Segundo semestre 2014)

y de la formula de Abel, se obtiene que:

y2 (x) = x

∫e∫

2x1−x2

dx

x2dx

= x

∫e− ln(1−x2)

x2dx

= x

∫dx

x2 (1− x2)

= x

∫ (1

x2+

1

2 (1− x)+

1

2 (1 + x)

)dx

mediante un desarrollo en fracciones parciales del penultimo integrando. Por tanto:

y2 (x) =1

2x ln

∣∣∣∣1 + x

1− x

∣∣∣∣− 1

Ası, la solucion general de la ecuacion diferencial es:

y (x) = C1x+ C2

x

2ln

∣∣∣∣1 + x

1− x

∣∣∣∣− 1

Ejemplo 2.4.4. Hallar la solucion general de la ecuacion diferencial:

xy′′ − 2 (x+ 1) y′ + (x+ 2) y = 0

para x > 0, bajo el supuesto de que la ecuacion homogenea tiene una solucion de la forma

y = emx, con m ∈ R una constante adecuada.

Solucion. Por hipotesis, la ecuacion diferencial posee una solucion de la forma y = emx.

Ası, al reemplazar en:

xy′′ − 2 (x+ 1) y′ + (x+ 2) y = 0

obtenemos la ecuacion:

xm2emx − 2m (x+ 1) emx + (x+ 2) emx = 0

para todo x > 0. Pero:

xm2 − 2m (x+ 1) + (x+ 2) = xm2 − 2m (x+ 1) + (x+ 1) + 1

= xm2 + (x+ 1) (1− 2m) + 1

= xm2 + (x+ 1) (1− 2m) + 1−m2 +m2

= (x+ 1)m2 + (x+ 1) (1− 2m) + 1−m2

= (x+ 1)(m2 − 2m+ 1

)+(1−m2

)= (x+ 1) (m− 1)2 +

(1−m2

)152

Page 155: Apuntes MAT-023 USM

Apuntes Mat023 (Segundo semestre 2014)

entonces:

(x+ 1) (m− 1)2 +(1−m2

)= 0

para todo x > 0. Por tanto, m = 1. Ası, y1 (x) = ex es una solucion de la ecuacion. Para

hallar una segunda solucion linealmente independiente con y1 (x) = ex, utilizaremos la

formula de Abel. Primeramente escribimos la ecuacion en su forma normal:

y′′ − 2

(1 +

1

x

)y′ +

(1 +

2

x

)y = 0

entonces, si p1 (x) = −2(1 + 1

x

), tenemos que:

y2 (x) = y1 (x)

∫e−

∫p1(x)dx

[y1 (x)]2dx

= ex∫e2∫(1+ 1

x)dx

e2xdx

= ex∫e2(x+lnx)

e2xdx

= ex∫x2e2x

e2xdx

= ex∫x2 dx

=1

3x3ex

por tanto, la solucion general de la ecuacion diferencial esta dada por:

y (x) = C1ex + C2x

3ex

Ecuaciones diferenciales a coeficientes constantes

Definiciones

Definicion 2.5.1. Una ecuacion diferencial lineal a coeficientes constantes es una ecuacion

de la forma:

y(n) + an−1 y(n−1) + an−2 y

(n−2) + · · ·+ a1 y′ + a0 y = Q (x)

en donde a0, a1, . . . , an−1 ∈ R y Q es una funcion continua en R. Las constantes ai ∈ R,

i = 0, 1, . . . , n− 1, se llaman coeficientes de la ecuacion.

Observacion 2.5.1. Comenzamos el estudio de este tipo de ecuaciones con el caso n = 2.

153

Page 156: Apuntes MAT-023 USM

Apuntes Mat023 (Segundo semestre 2014)

La ecuacion de orden 2

Observacion 2.5.2. Consideremos la ecuacion diferencial de segundo orden siguiente:

y′′ + ay′ + by = 0 (2.10)

Supongamos que la ecuacion (2.10) posee una solucion y = y (x) de la forma y (x) = eλx,

con λ ∈ R. Note que:

y′ (x) = λeλx ∧ y′′ (x) = λ2eλx

Por tanto, reemplazando lo anterior en la ecuacion (2.10) obtenemos:

λ2eλx + aλeλx + beλx = 0

Ası, en vista de la ecuacion anterior, las condiciones sobre λ ∈ R de modo que y (x) = eλx

sea solucion es que:

λ2 + aλ+ b = 0 (2.11)

Esto es, que λ sea raız de la ecuacion de segundo grado anterior. La ecuacion (2.11) se

llama ecuacion caracterıstica asociada a la ecuacion (2.10).

Por tanto, tenemos tres casos dados por ∆ = a2 − 4b, el discriminante de (2.11). En

efecto:

1. Si ∆ = a2 − 4b > 0, entonces la ecuacion (2.11), tiene dos soluciones reales λ1 y λ2

distintas, dadas por:

λ1 =−a+

√a2 − 4b

2∧ λ2 =

−a−√a2 − 4b

2

las que a su vez definen las funciones:

u1 (x) = eλ1x ∧ u2 (x) = eλ2x

Luego, como u2/u1 no es constante, la solucion general de la ecuacion diferencial

lineal homogenea de orden 2 es:

y (x) = C1eλ1x + C2e

λ2x

Ejemplo 2.5.1. Resuelva la ecuacion diferencial:

y′′ + y′ − 2y = 0

Solucion. En este caso la ecuacion caracterıstica es

λ2 + λ− 2 = 0

⇔(λ+ 2) (λ− 1) = 0

154

Page 157: Apuntes MAT-023 USM

Apuntes Mat023 (Segundo semestre 2014)

luego tenemos dos soluciones u1 (x) = e−2x y u1 (x) = ex de y′′ + y′ − 2y = 0 las

cuales son linealmente independientes pues

W(e−2x, ex

)=

∣∣∣∣ e−2x ex

−2e−2x ex

∣∣∣∣ = 3e−x

la solucion general de la ecuacion sera entonces

y (x) = c1ex + c2e

−2x

2. Si ∆ = a2 − 4b = 0, entonces la ecuacion (2.11) tiene una raız real λ de multiplicidad

2, dada por:

λ = −a2

Luego, u1 (x) = eλx es una solucion, pero dim kerL = 2 por tanto, debemos hallar

otra solucion u2 (x) linealmente independiente con u1 (x), podemos usar la formula

de Abel para la segunda solucion

u2 (x) = e−a2x

∫e−

∫adx

e−axdx

= xe−a2x

se sigue que la segunda solucion es u2 (x) = xe−a2x y ası la solucion general es

y (x) = c1eλx + c2xe

λx

donde λ es la raız repetida de la ecuacion caracterıstica.

Ejemplo 2.5.2. Resuelva la ecuacion diferencial:

y′′ + 4y′ + 4y = 0

Solucion. La ecuacion caracterıstica es

λ2 + 4λ+ 4 = 0

la cual tiene raız repetida λ = −2 se sigue que la solucion general de la ecuacion es

y (x) = c1e−2x + c2xe

−2x

3. Si ∆ = a2 − 4b < 0, entonces la ecuacion (2.11) no tiene raıces reales. Sin embargo,

sea:

λ = α + iβ

una raız compleja de (2.11). Recordemos el siguiente teorema:

155

Page 158: Apuntes MAT-023 USM

Apuntes Mat023 (Segundo semestre 2014)

Teorema 2.5.1 (Formula de Euler). Sea θ ∈ R, entonces:

eiθ = cos θ + i sin θ

Entonces:

eλx = e(α+iβ)x

= eαxeiβx

= eαx cos (βx) + i sin (βx)= eαx cos (βx) + i eαx sin (βx)

Consideremos ahora:

Teorema 2.5.2. Sean L (y) = 0 una ecuacion diferencial lineal de orden 2 e y (x) =

u (x) + iv (x) una solucion compleja de la ecuacion tal que u 6= v. Entonces, u (x) y

v (x) son dos soluciones reales y linealmente independientes de L (y) = 0.

De:

eλx = eαx cos (βx) + i eαx sin (βx)

se obtiene que:

u1 (x) = eαx cos (βx) ∧ u2 (x) = eαx sin (βx)

son dos soluciones reales y linealmente independientes de la ecuacion diferencial

lineal:

y′′ + ay′ + by = 0

Observacion 2.5.3. Debemos notar que eλx y eλx son efectivamente funciones linealmente

independientes como C espacio vectorial.

Ejemplo 2.5.3. Resuelva la ecuacion:

y′′ + 2y′ + 5y = 0

Solucion. La ecuacion caracterıstica es

λ2 + 2λ+ 5 = 0

la cual tiene raıces

λ = −1± 2i

se sigue que la solucion general es

y (x) = c1e−x cos (2x) + c2e

−x sin (2x)

156

Page 159: Apuntes MAT-023 USM

Apuntes Mat023 (Segundo semestre 2014)

Ahora daremos una segunda mirada a la ecuacion de segundo orden, en relacion al

operador diferencial asociado. En general los operadores diferenciales no conmutan, esto es,

si L1 y L2 son dos operadores diferenciales en general no es cierto que

L1L2 = L2L1

sin embargo, si los operadores diferenciales son con coeficientes constantes, estos se com-

portan igual que polinomios. Por ejemplo, considere los operadores diferenciales D − 1 y

D + 2 entonces

(D − 1) (D + 2) y = (D − 1) (y′ + 2y)

= y′′ + 2y′ − y′ − 2y

= y′′ + y′ − 2y

=(D2 +D − 2

)y

por otro lado

(D + 2) (D − 1) y = (D + 2) (y′ − y)

= y′′ − y′ + 2y′ − 2y

= y′′ + y′ − 2y

=(D2 +D − 2

)y

se sigue

(D − 1) (D + 2) = (D + 2) (D − 1) =(D2 +D − 2

)note que si (D − 1) y = 0 entonces (D + 2) (D − 1) y = 0 y ası y ∈ ker (D2 +D − 2) en

otras palabras una solucion de la ecuacion

y′ − y = 0

tambien es solucion de

y′′ + y′ − 2y = 0

de manera similar, si (D + 2) y = 0 entonces (D − 1) (D + 2) y = 0 y ası y ∈ ker (D2 +D − 2)

en otras palabras una solucion de la ecuacion

y′ + 2y = 0

tambien es solucion de

y′′ + y′ − 2y = 0

pero la soluciones y′ − y = 0 son u1 (x) = cex y las soluciones de y′ + 2y = 0 son

u2 (x) = ke−2x (son de primer orden de variables separadas) ası podemos obtener las

soluciones ex y e−2x de la ecuacion y′′+ y′− 2y = 0 y como son linealmente independientes,

se sigue que la solucion general de la ecuacion es dada por

y (x) = c1ex + c2e

−2x

son estas ideas las que utilizaremos para resolver la ecuacion diferencial lineal de orden n.

157

Page 160: Apuntes MAT-023 USM

Apuntes Mat023 (Segundo semestre 2014)

La ecuacion de orden superior

Observacion 2.5.4. En general, respecto de la ecuacion diferencial:

y(n) + an−1 y(n−1) + an−2 y

(n−2) + · · ·+ a1 y′ + a0 y = Q (x)

tenemos:

Definicion 2.5.2. Sean I ⊆ R un intervalo abierto y L : C(n) (I) → C (I) un operador

diferencial de orden n a coeficientes constantes, es decir:

L = Dn + an−1Dn−1 + · · ·+ a1D + a01 (2.12)

El polinomio caracterıstico de la ecuacion diferencial L (y) = 0 es el polinomio en R [λ]

siguiente:

fL (λ) = λn + an−1λn−1 + · · · a1λ+ a0

Diremos, ademas, que la ecuacion caracterıstica asociada a la ecuacion diferencial

L (y) = 0 es la ecuacion siguiente:

fL (λ) = 0

O bien:

λn + an−1λn−1 + · · · a1λ+ a0 = 0

Ejemplo 2.5.4. La ecuacion diferencial:

y(4) − 3y′′ + y′ − 3y = 0

tiene polinomio caracterıstico:

f (λ) = λ4 − 3λ2 + λ− 3

Note que, el operador diferencial asociado es:

L = D4 − 3D2 +D − 3

Ejemplo 2.5.5. La ecuacion diferencial:

y(5) + 2y(4) − 3y′′′ − y′′ − 2y′ + 3y = 0

tiene polinomio caracterıstico:

f (λ) = λ5 + 2λ4 − 3λ3 − λ2 − 2λ+ 3

Note que:

D5 + 2D4 − 3D3 −D2 − 2D + 3 =(D2 +D + 1

)(D − 1)2 (D + 3)

158

Page 161: Apuntes MAT-023 USM

Apuntes Mat023 (Segundo semestre 2014)

Teorema 2.5.3. Sean L y S dos operadores a coeficientes constantes con polinomios

caracterısticos fL y fS, respectivamente, y sea α ∈ R, entonces:

1. L = S, si y solo si, fL = fS

2. fL+S = fL + fS

3. fLS = fL · fS

4. fαL = αfL

Observacion 2.5.5. Del teorema anterior se deduce que toda relacion algebraica consti-

tuıda por sumas, productos y productos por escalares de polinomios caracterısticos son

tambien validas para los operadores L y S. En particular, se tienen las leyes conmutativas:

L+ S = S + L ∧ LS = SL ∧ α (βL) = β (αL)

Note, ademas, que si L y S son operadores tales que LS = SL, entonces:

kerL+ kerS ⊆ ker (LS) (2.13)

Por otro lado, recordemos que en el conjunto R [x] los elementos irreducibles monicos

son los polinomios de la forma x− c y x2 + αx+ β, con α2 − 4β < 0. Luego, el Teorema

Fundamental del Algebra implica el siguiente resultado:

Teorema 2.5.4. Sea p (x) ∈ R [x]. Entonces, p (x) se factoriza como

p (x) =r∏i=1

(x− λi)ni ·s∏j=1

(x2 + αjx+ βj

)mj (2.14)

donde λi son las raıces reales de p (x) con multiplicidad ni, para cada i = 1, 2, . . . , r,

α2j −4βj < 0 y mj es la multiplicidad de la raız compleja asociada, para cada j = 1, 2, . . . , s.

Obtenemos lo siguiente para los operadores diferenciales dados por la factorizacion de

la ecuacion caracterıstica en (2.14):

1. ker (D − λ)m =⟨eλx, xeλx, x2eλx, . . . , xm−1eλx

⟩2.

ker((D − α)2 + β2

)m=⟨eαx cos (βx) , xeαx cos (βx) , . . . , xm−1eαx cos (βx) ; eαx sin (βx) ,

, xeαx sin (βx) , . . . , xm−1eαx sin (βx)⟩

Por tanto, mediante el uso del wronskiano y la ecuacion (2.14), la solucion general de la

ecuacion diferencial esta dada por las combinaciones lineales (con las constantes indexadas

adecuadamente) de los sistemas fundamentales de soluciones de cada factor.

159

Page 162: Apuntes MAT-023 USM

Apuntes Mat023 (Segundo semestre 2014)

Ejemplo 2.5.6. Resuelva la ecuacion diferencial:

y′′′ − y′′ − 8y′ + 12y = 0

Solucion. Consideremos la ecuacion caracterıstica:

λ3 − λ2 − 8λ+ 12 = 0

Como:

λ3 − λ2 − 8λ+ 12 = (λ+ 3) (λ− 2)2

se sigue que

ker (D − 2)2 ⊂ ker(D3 −D2 − 8D + 12

)ker (D + 3) ⊂ ker

(D3 −D2 − 8D + 12

)pero ker (D − 2)2 = 〈e2x, xe2x〉 y ker (D + 3) = 〈e−3x〉 luego

e2x, xe2x, e−3x⊂ ker

(D3 −D2 − 8D + 12

)sabemos que el espacio ker (D3 −D2 − 8D + 12) tiene dimension tres luego, la solucion

general de la ecuacion

y′′′ − y′′ − 8y′ + 12y = 0

es dada por

yG (x) = c1e2x + c2xe

2x + c3e−3x

Ejemplo 2.5.7. Suponga que L (y) = 0 es una ecuacion diferencial con operador:

L = D5 + 2D4 − 3D3 −D2 − 2D + 3

Hallar la solucion general.

Solucion. Note que:

L = D5 + 2D4 − 3D3 −D2 − 2D + 3

=(D2 +D + 1

)(D − 1)2 (D + 3)

Ahora bien:

ker(D2 +D + 1

)=

⟨e−x/2 cos

(√3x

2

); e−x/2 sin

(√3x

2

)⟩ker (D − 1)2 = 〈ex; xex〉

y ademas:

ker (D + 3) =⟨e−3x

⟩Por tanto, la solucion general de la ecuacion L (y) = 0 esta dada por:

y (x) = C1e−x/2 cos

(√3x

2

)+ C2e

−x/2 sin

(√3x

2

)+ C3e

x + C4xex + C5e

−3x

160

Page 163: Apuntes MAT-023 USM

Apuntes Mat023 (Segundo semestre 2014)

Ejemplo 2.5.8. Hallar la solucion general de una ecuacion diferencial lineal a coeficientes

constantes cuya ecuacion caracterıstica es:

λ5 − 2λ4 + 6λ3 − 9λ2 + 8λ− 4 = 0

sabiendo que y = ex/2 cos(√

32x)

es una solucion de dicha ecuacion.

Solucion. Sea ε una ecuacion diferencial lineal a coeficientes constantes homogenea de la

cual se sabe que:

λ5 − 2λ4 + 6λ3 − 9λ2 + 8λ− 4 = 0 (2.15)

es la ecuacion caracterıstica asociada, y que ademas:

y = ex/2 cos

(√3

2x

)

es solucion de la ecuacion. Ahora bien, como y = ex/2 cos(√

32x)

es solucion:

λ =1

2+

√3

2i

es una raız de la ecuacion caracterıstica (2.15), entonces:

λ =1

2−√

3

2i

tambien es raız, luego la expresion:λ−

(1

2+

√3

2i

)λ−

(1

2−√

3

2i

)= λ2 − λ+ 1

factoriza a la ecuacion caracterıstica. Ası:

λ5 − 2λ4 + 6λ3 − 9λ2 + 8λ− 4 =(λ3 − λ2 + 4λ− 4

) (λ2 − λ+ 1

)Notamos, ademas, que λ = 1 es raız de la ecuacion λ3 − λ2 + 4λ− 4 = 0. Entonces, por el

teorema del factor, se tiene que:

λ3 − λ2 + 4λ− 4 =(λ2 + 4

)(λ− 1)

entonces

λ5 − 2λ4 + 6λ3 − 9λ2 + 8λ− 4 =(λ2 − λ+ 1

) (λ2 + 4

)(λ− 1)

Por tanto, la ecuacion (2.15) tiene las siguientes raıces:1

2+

√3

2i,

1

2−√

3

2i, 1, 2i,−2i

Finalmente, la solucion general de la ecuacion ε esta dada por:

y (x) = C1ex + ex/2

C2 cos

(√3

2x

)+ C3 sin

(√3

2x

)+ C4 cos 2x+ C5 sin 2x

161

Page 164: Apuntes MAT-023 USM

Apuntes Mat023 (Segundo semestre 2014)

Metodo de variacion de parametros

Observacion 2.6.1. En las secciones anteriores obtuvimos metodos para encontrar el

espacio solucion de ecuaciones diferenciales, para los siguientes casos:

1. Es conocida una solucion particular y1 (x) de la ecuacion:

y′′ + p1 (x) y′ + p0 (x) y = 0

En particular, para la ecuacion anterior, obtuvimos que la solucion general esta dada

por:

y (x) = C1y1 (x) + C2y2 (x)

donde:

y2 (x) = y1 (x)

∫e−

∫p1(x)dx

y21 (x)

dx

2. La ecuacion diferencial es de la forma:

y(n) + an−1y(n−1) + · · ·+ a1y

′ + a0y = 0

con ai ∈ R, para cada i = 0, 1, . . . , n− 1. En este caso, las soluciones dependen de

las raıces del polinomio caracterıstico:

fL (λ) = λn + an−1λn−1 + · · ·+ a1λ+ a0

Por otro lado, dada la estructura lineal de la ecuacion L (y) = Q, donde L es el operador

diferencial asociado a la ecuacion y Q es una funcion continua, entonces la solucion general

de la ecuacion lineal se descompone como:

y = yh + yp

donde yh es la solucion general de la ecuacion L (y) = 0. El objetivo de esta seccion es

hallar una solucion particular yp para la ecuacion L (y) = Q.

Observacion 2.6.2. En este curso, se ven dos de tales metodos: el metodo de variacion de

parametros y el metodo de los anuladores. Como se acostumbra, haremos los razonamientos

para n = 2, y luego se presentara el resultado para n ≥ 2. Veamos primeramente:

Teorema 2.6.1 (Variacion de parametros). Sean u1 y u2 dos soluciones linealmente

independientes de la ecuacion:

L (y) = y′′ + p1 (x) y′ + p0 (x) y = 0

162

Page 165: Apuntes MAT-023 USM

Apuntes Mat023 (Segundo semestre 2014)

en un intervalo I ⊆ R. Entonces, la ecuacion no homogenea L (y) = Q tiene una solucion

particular de la forma:

yp (x) = C1 (x)u1 (x) + C2 (x)u2 (x)

donde:

C1 (x) = −∫u2 (x)Q (x)

W (x)dx ∧ C2 (x) =

∫u1 (x)Q (x)

W (x)dx (2.16)

y W (x) es el wronskiano de u1 (x) y u2 (x).

Demostracion. Supongamos que yp (x) = C1 (x)u1 (x) + C2 (x)u2 (x), luego:

y′p = C1u′1 + C2u

′2 + (C ′1u1 + C ′2u2)

y′′p = C1u′′1 + C2u

′′2 + (C ′1u

′1 + C ′2u

′2) + (C ′1u1 + C ′2u2)

Ahora bien, al evaluar yp en L se obtiene que:

L (yp) = y′′p + p1 (x) y′p + p0 (x) yp

= (C ′1u′1 + C ′2u

′2) + (C ′1u1 + C ′2u2)

′+ p1 (x) (C ′1u1 + C ′2u2)

pues todos los terminos que contienen a C1 (x) y C2 (x) desaparecen dado que L (u1) =

L (u2) = 0. Como deseamos que L (yp) = Q, entonces de la ultima ecuacion imponemos

que:

C ′1u1 + C ′2u2 = 0 ∧ C ′1u′1 + C ′2u

′2 = Q

Ası, las funciones C1 (x) y C2 (x), deben satisfacer que:C ′1 (x)u1 (x) + C ′2 (x)u2 (x) = 0

C ′1 (x)u′1 (x) + C ′2 (x)u′2 (x) = Q (x)

para cada x ∈ I. Sin embargo, note que el ultimo sistema tiene solucion si y solo si la matriz

de coeficientes de el tiene determinante no nulo. Es decir, si:∣∣∣∣ u1 (x) u2 (x)

u′1 (x) u′2 (x)

∣∣∣∣ 6= 0

para cada x ∈ I. Pero el determinante anterior es el wronskiano W (x) de u1 (x) y u2 (x), y

como u1, u2 son linealmente independientes W (x) 6= 0, para cada x ∈ I. Por lo tanto, el

sistema tiene unica solucion, y ademas, por la regla de Cramer, obtenemos finalmente:

C ′1 (x) =

∣∣∣∣ 0 u2 (x)

Q (x) u′2 (x)

∣∣∣∣∣∣∣∣ u1 (x) u2 (x)

u′1 (x) u′2 (x)

∣∣∣∣ = −u2 (x)Q (x)

W (x)

163

Page 166: Apuntes MAT-023 USM

Apuntes Mat023 (Segundo semestre 2014)

y:

C ′1 (x) =

∣∣∣∣ u1 (x) 0

u′1 (x) Q (x)

∣∣∣∣∣∣∣∣ u1 (x) u2 (x)

u′1 (x) u′2 (x)

∣∣∣∣ =u1 (x)Q (x)

W (x)

Integrando, se obtiene el resultado deseado.

Observacion 2.6.3. En base a los razonamientos en la demostracion del teorema anterior,

tenemos:

Teorema 2.6.2. Sean u1, u2, . . . , un una familia de n soluciones linealmente independientes

de la ecuacion lineal de orden n:

L (y) = y(n) + pn−1 (x) y(n−1) + · · ·+ p1 (x) y′ + p0 (x) y = 0

definida en un intervalo abierto I ⊆ R. Entonces, la ecuacion no homogenea L (y) = Q,

tiene una solucion particular:

yp (x) =n∑i=1

Ci (x)ui (x)

donde Ci (x), con i = 1, 2, . . . , n son funciones que se obtienen como soluciones del sistema

de ecuaciones:

C ′1 u1 + C ′2 u2 + · · ·+ C ′n un = 0

C ′1 u′1 + C ′2 u

′2 + · · ·+ C ′n u

′n = 0

C ′1 u′′1 + C ′2 u

′′2 + · · ·+ C ′n u

′′n = 0

......

C ′1 u(n−1)1 + C ′2 u

(n−1)2 + · · ·+ C ′n u

(n−1)n = Q (x)

(2.17)

Ejemplo 2.6.1. Resuelva la ecuacion diferencial:

y′′ + y = tanx

Solucion. Consideremos un intervalo en el que el segundo miembro sea continuo, sea este

(−a, a), siendo 0 < a < π/2. La ecuacion homogenea tiene solucion general:

yh (x) = C1 cosx+ C2 sinx

pues la ecuacion caracterıstica de dicha ecuacion homogenea es:

λ2 + 1 = 0

164

Page 167: Apuntes MAT-023 USM

Apuntes Mat023 (Segundo semestre 2014)

En particular, notamos que:

W (x) =

∣∣∣∣ cosx sinx

− sinx cosx

∣∣∣∣ = cos2 x+ sin2 x = 1

Entonces, en vista de las ecuaciones en (2.16) las funciones C1 (x) y C2 (x) satisfacen:

C1 (x) = −∫

sinx tanx

W (x)dx

= −∫

sinx tanxdx

= sin x− ln |secx+ tanx|

y ademas:

C2 (x) =

∫cosx tanx

W (x)dx

=

∫cosx tanxdx

= − cosx

Por tanto, por el metodo de variacion de parametros, una solucion particular yp viene dada

por:

yp = C1 (x) cosx+ C2 (x) sinx

= sin x cosx− cosx ln |secx+ tanx| − sinx cosx

= − cosx ln |secx+ tanx|

y la solucion general es:

y = C1 cosx+ C2 sinx+ yp

= C1 cosx+ C2 sinx− cosx ln |secx+ tanx|

Solucion. Hallar la solucion general de la ecuacion diferencial:

y′′ − 2y′ + y =ex

(1− x)2

Solucion. Considere la ecuacion diferencial:

y′′ − 2y′ + y =ex

(1− x)2

Notamos que la ecuacion anterior es una ecuacion diferencial lineal a coeficientes constantes

no homogenea. Ası, si y = y (x) es la solucion general, entonces:

y (x) = yh (x) + yp (x)

165

Page 168: Apuntes MAT-023 USM

Apuntes Mat023 (Segundo semestre 2014)

donde yh (x) es la solucion de la ecuacion homogenea asociada e yp (x) es una solucion

particular. Consideremos, entonces, la ecuacion homogenea asociada:

y′′ − 2y′ + y = 0

Luego, la ecuacion caracterıstica:

λ2 − 2λ+ 1 = 0

tiene raız λ = 1 con multiplicidad 2. Por tanto, la solucion homogenea yh (x) esta dada por:

yh (x) = C1 ex + C2 xe

x

Ahora bien, por el metodo de variacion de parametros, tenemos que la solucion particular

yp (x) tiene la forma:

yp (x) = C1 (x) ex + C2 (x)xex

donde C1 (x) y C2 (x) son soluciones del sistema de ecuaciones:C ′1 (x) ex + C ′2 (x)xex = 0

C ′1 (x) ex + C ′2 (x) (1 + x) ex = ex

(1−x)2

Ahora bien, como el wronskiano:

W (x) =

∣∣∣∣ ex xex

ex (1 + x) ex

∣∣∣∣ = e2x

es no nulo, por la regla de Cramer, se tiene que:

C ′1 (x) =

∣∣∣∣∣ 0 xex

ex

(1−x)2(1 + x) ex

∣∣∣∣∣W (x)

= − x

(x− 1)2

y ademas:

C ′2 (x) =

∣∣∣∣∣ ex 0

ex ex

(1−x)2

∣∣∣∣∣W (x)

=1

(x− 1)2

Ası:

C1 (x) = −∫

x

(x− 1)2dx =1

x− 1ln (x− 1)− x ln (x− 1) + 1

y:

C2 (x) =

∫1

(x− 1)2dx = − 1

x− 1

Por tanto, la solucion general y (x) de la ecuacion diferencial esta dada por:

y (x) = C1 ex + C2 xe

x +ex

x− 1ln (x− 1)− x ln (x− 1) + 1 − xex

x− 1= C1 e

x + C2 xex − ex ln (x− 1) + 1

166

Page 169: Apuntes MAT-023 USM

Apuntes Mat023 (Segundo semestre 2014)

Ejemplo 2.6.2. Hallar la solucion general de la ecuacion

x2y′′ + xy′ +

(x2 − 1

4

)y = x3/2

en el intervalo (0,+∞) sabiendo que y1(x) = x−1/2 cosx es solucion de la ecuacion ho-

mogenea asociada.

Solucion. Usando la formula de Abel obtenemos una segunda solucion de a homogenea

y2 (x) = x−1/2 cosx

∫e−

∫dxx

x−1 cos2 xdx

= x−1/2 cosx

∫e− lnx

x−1 cos2 xdx

= x−1/2 cosx

∫1

cos2 xdx

= x−1/2 cosx tanx

= x−1/2 sinx

ahora buscamos la solucion particular usando variacion de parametros

yp (x) =(x−1/2 cosx

)C1 (x) +

(x−1/2 sinx

)C2 (x)

donde

C ′1 (x) =

∣∣∣∣∣ 0 x−1/2 sinx

x−1/2(x−1/2 sinx

)′ ∣∣∣∣∣∣∣∣∣ x−1/2 cosx x−1/2 sinxddx

(x−1/2 cosx

)ddx

(x−1/2 sinx

) ∣∣∣∣ =− 1x

sinx1x

= − sinx

ası C1 (x) = cos x

C ′2 (x) =

∣∣∣∣ x−1/2 cosx 0ddx

(x−1/2 cosx

)x−1/2

∣∣∣∣∣∣∣∣ x−1/2 cosx x−1/2 sinxddx

(x−1/2 cosx

)ddx

(x−1/2 sinx

) ∣∣∣∣ =x−1 cosx

x−1

= cos x

ası C2 (x) = sinx se sigue

yp (x) =(x−1/2 cosx

)cosx+

(x−1/2 sinx

)sinx

= x−1/2

la solucion general de la ecuacion no homogenea es

yg (x) = αx−1/2 cosx+ βx−1/2 sinx+ x−1/2

167

Page 170: Apuntes MAT-023 USM

Apuntes Mat023 (Segundo semestre 2014)

Observacion 2.6.4. Las integrales que aparecen en las formulas (2.16) o al despejar

C1 (x) y C2 (x) del sistema (2.17), son integrales indefinidas, luego las funciones quedan

determinadas salvo una constante de integracion. Suponga entonces que tenemos:

vi (x) = Ci (x) +Ki

con Ki ∈ R, i = 1, 2, constantes de integracion. Luego, si u1 y u2 son las soluciones

linealmente independientes de L (y) = 0, considere:

y1 (x) = A1u1 (x) + A2u2 (x) + v1 (x)u1 (x) + v2 (x)u2 (x)

= (A1 +K1)u1 (x) + (A2 +K2)u2 (x) + C1 (x)u1 (x) + C2 (x)u2 (x)

Entonces:

y1 (x) = α1u1 (x) + α2u2 (x) + C1 (x)u1 (x) + C2 (x)u2 (x)

que era la solucion que ya tenıamos.

Metodo del anulador

Definicion 2.7.1. Diremos que el operador diferencial L es un anulador o aniquilador de

ϕ si se cumple L (ϕ) = 0.

Ejemplo 2.7.1. El operador diferencial L = (D − 1)2 es un anulador de ϕ (x) = 2ex+5xex

pues ker (D − 1)2 = 〈ex, xex〉, es decir, toda combinacion lineal de ex y xex esta en el

nucleo de (D − 1)2. Notar que (D − 1)2 no es un anulador de x2 pues

(D − 1)2 (x2)

= (D − 1) (D − 1)(x2)

= (D − 1)(2x− x2

)= (2− 2x)−

(2x− x2

)= x2 − 4x+ 2

6≡ 0

Observacion 2.7.1. Recordemos que kerL + kerL ⊆ ker (LS) para los operadores con

coeficientes constantes.

Ejemplo 2.7.2. (D2 + 1)2

es anulador de x sinx y D2 es anulador de x luego D2 (D2 + 1)2

es un anulador de 5x sinx− 3x

Suponga que buscamos resolver la ecuacion L (y) = Q donde L es un operador con

coeficientes constantes y LQ es un anulador de Q (con coeficientes constantes tambien)

entonces

L (y) = Q

⇒LQL (y) = LQ (Q) = 0

168

Page 171: Apuntes MAT-023 USM

Apuntes Mat023 (Segundo semestre 2014)

luego la solucion de L (y) = Q se encuentra entre las soluciones de la ecuacion homogenea

LQL (y) = 0.

Ejemplo 2.7.3. Resolver la ecuacion diferencial:

y′′ − 5y′ + 6y = xex

Solucion. Notamos que el operador asociado a la ecuacion diferencial es:

L = D2 − 5D + 6

= (D − 2) (D − 3)

Ası, la ecuacion diferencial puede escribirse como L (y) = Q, con Q (x) = xex. Por otro lado,

la ecuacion homogenea L (y) = 0 tiene soluciones linealmente independientes u1 (x) = e2x

y u2 (x) = e3x. Sabemos que la solucion general de la ecuacion es de la forma:

y (x) = C1e2x + C2e

3x + yp (x)

donde yp es una solucion de la ecuacion no homogenea L (y) = xex. Notamos que:

xex ∈ ker (D − 1)2

por tanto, aplicamos el operador diferencial (D − 1)2 a la ecuacion diferencial para obtener

:

(D − 1)2 (D − 2) (D − 3) (y) = (D − 1)2 (xex) = 0

Ası, hemos obtenido la ecuacion diferencial homogenea:

S (y) = (D − 1)2 (D − 2) (D − 3) (y) = 0

por tanto, la solucion general de S (y) = 0 es:

yS (x) = aex + bxex + ce2x + de3x

donde a, b, c y d son constantes reales. Ahora bien, se deben elegir los valores de a, b, c y d

de tal modo que:

L(aex + bxex + ce2x + de3x

)= xex

sin embargo, ce2x + de3x ∈ kerL, luego bastara elegir a y b tales que:

L (aex + bxex) = xex

ası, la ecuacion:

L (aex + bxex) = (D − 2) (D − 3) (aex + bxex)

= (2a− 3b) ex + 2bxex

= xex

169

Page 172: Apuntes MAT-023 USM

Apuntes Mat023 (Segundo semestre 2014)

implica que a = 3/4 y b = 1/2. Finalmente:

yp (x) =3

4ex +

1

2xex

y por tanto:

y (x) =

yh︷ ︸︸ ︷C1e

2x + C2e3x +

yp︷ ︸︸ ︷3

4ex +

1

2xex

Observacion 2.7.2. Recalcamos que el operador (D − 1)2 se eligio de tal modo que:

xex ∈ ker (D − 1)2

Luego, al aplicar dicho operador a la ecuacion diferencial L (y) = xex, vemos que el

termino de la derecha se anula bajo la aplicacion de (D − 1)2. Por este hecho, el metodo

anterior se llama el metodo del anulador. El metodo del anulador, funciona correctamente

para ecuaciones que son anulables, en el sentido de que son funciones que aparecen como

soluciones de ecuaciones diferenciales a coeficientes constantes. En particular, funciones

que aparecen como combinaciones lineales de funciones de la forma:

xm−1eαx ∨ xm−1eαx cos βx ∨ xm−1eαx sin βx

donde m es un numero natural y α y β son constantes reales. A continuacion detallamos

algunas de las funciones anteriores con sus respectivos anuladores:

Funcion Anulador

y = xm−1 Dm

y = eαx D − αy = xm−1eαx (D − α)m

y = cos βx o bien y = sin βx D2 + β2

y = xm−1 cos βx o bien y = xm−1 sin βx (D2 + β2)m

y = eαx cos βx o bien y = eαx sin βx D2 − 2αD + (α2 + β2)

y = xm−1eαx cos βx o bien y = xm−1eαx sin βx D2 − 2αD + (α2 + β2)m

Observacion 2.7.3. Un resultado que utiliza las propiedades lineales del operador dife-

rencial asociado a una ecuacion, es el principio de superposicion, que facilita en algunos

casos el calculo de la solucion particular. Tenemos, entonces:

Teorema 2.7.1 (Principio de superposicion). Sean L un operador diferencial no necesaria-

mente a coeficientes constantes y f1, f2 funciones continuas. Suponga que yi es solucion de

la ecuacion diferencial L (y) = fi, con i = 1, 2. Entonces:

y = y1 + y2

es solucion de la ecuacion L (y) = f1 + f2.

170

Page 173: Apuntes MAT-023 USM

Apuntes Mat023 (Segundo semestre 2014)

Este principio nos permite separar la busqueda de la solucion particular.

Ejemplo 2.7.4. Resolver

(D − 1) (D − 2) y = ex + x

o en otras palabrasd2y

dx2− 3

dy

dx+ 2y = ex + x

Solucion. Sabemos que

(D − 1) (ex) = 0 y D2 (x) = 0

se sigue que

(D − 1)D2 (ex + x) = 0

ası

(D − 1)D2 (D − 1) (D − 2) y = 0

D2 (D − 1)2 (D − 2) = 0

luego

yp (x) = c1 + c2x+ c3ex + c4xe

x + c5e2x

y como

(D − 1) (D − 2) yp (x) = ex + x

se sigue

(D − 1) (D − 2)[c1 + c2x+ c3e

x + c4xex + c5e

2x]

= (D − 1) (D − 2) [c1 + c2x] + (D − 1) (D − 2) [c4xex]

=(D2 − 3D + 2

)[c1 + c2x] + (D − 1) (D − 1− 1) [c4xe

x]

= 2c1 − 3c2 + 2c2x− (D − 1) [c4xex]

= 2c1 − 3c2 + 2c2x− (c4ex + c4xe

x − c4xex)

= 2c1 − 3c2 + 2c2x− c4ex

de esto obtenemos que las constantes cumplen

2c1 − 3c2 = 0

2c2 = 1

−c4 = 1

se sigue

c4 = −1, c2 = 1/2, c1 = 3/4

171

Page 174: Apuntes MAT-023 USM

Apuntes Mat023 (Segundo semestre 2014)

ası

yG (x) = (3/4) + (1/2)x− xex + c3ex + c5e

2x

= [(3/4) + (1/2)x− xex] +[c3e

x + c5e2x]

= yp (x) + yh (x)

es la solucion general.

Note que

(D − 1) (D − 2) ((3/4) + (1/2)x) =d2

dx2((3/4) + (1/2)x)

−3d

dx((3/4) + (1/2)x) + 2 ((3/4) + (1/2)x)

= x

(D − 1) (D − 2) (−xex) =d2

dx2(−xex)− 3

d

dx(−xex) + 2 (−xex)

= 3ex (x+ 1)− ex (x+ 2)− 2xex

= ex

luego vale el principio de superposicion.

Veamos como podemos trabajar, en algunos casos, si no conocemos el anulador:

Ejemplo 2.7.5. Hallar la solucion general de la ecuacion diferencial:

y′′ − 3y′ + 2y = xex+x2

Solucion. Notamos que:

L = D2 − 3D + 2

= (D − 1) (D − 2)

Buscamos, primeramente, una solucion particular yp. Considere el cambio de variables:

u = (D − 2) y (2.18)

la ecuacion queda:

(D − 1)u = xex+x2

Mediante la formula de Leibnitz, obtenemos:

ue−x =

∫xex

2

dx+ C =1

2ex

2

+ C

Ahora bien, como buscamos yp, podemos elegir C = 0. Ası:

u =1

2ex+x2

172

Page 175: Apuntes MAT-023 USM

Apuntes Mat023 (Segundo semestre 2014)

Sustituyendo en (2.18), se obtiene:

(D − 2) y =1

2ex+x2

Luego, nuevamente por la formula de Leibnitz:

yp =1

2e2x

∫ex

2−xdx

Finalmente, la solucion general de la ecuacion diferencial es:

y = C1ex + C2e

2x +1

2e2x

∫ex

2−xdx

Ejemplo 2.7.6. Resuelva la ecuacion diferencial:

y(6) + 2y(5) − y(4) + 4y′′′ − 5y′′ + 2y′ − 3y = x

sabiendo que λ=−3 es una raız de la ecuacion caracterıstica y que u1(x) = cosx es una

solucion de la ecuacion homogenea asociada.

Solucion. Consideremos la ecuacion diferencial:

y(6) + 2y(5) − y(4) + 4y′′′ − 5y′′ + 2y′ − 3y = x (2.19)

En primer lugar, notamos que la ecuacion caracterıstica de la ecuacion homogenea asociada

a (2.19) es:

f(λ) = λ6 + 2λ5 − λ4 − 5λ2 − 3 = 0 (2.20)

Ahora bien, como u1(x) = cosx es una solucion de la ecuacion homogenea, entonces λ = i

y λ = −i son raıces de la ecuacion (2.20). Ademas, por hipotesis, λ=−3 es una raız de la

ecuacion (2.20). Por tanto, por el Teorema del Factor, el polinomio f(λ) es divisible por(λ2 + 1

)(λ+ 3). Ası, se obtiene la siguiente factorizacion:

f(λ) = (λ2 + 1)(λ+ 3)(λ3 − λ2 + λ− 1

)Por otro lado, por inspeccion, notamos que λ = 1 es raız de λ3 − λ2 + λ − 1, entonces

λ3 − λ2 + λ− 1 = (λ2 + 1)(λ− 1). Por tanto, la factorizacion completa de f(λ) es:

f(λ) =(λ2 + 1

)2(λ− 1)(λ+ 3) (2.21)

Ası, la solucion de la ecuacion homogenea esta dada por:

yh(x) = C1 cosx+ C2 sinx+ C3 x cosx+ C4 x sinx+ C5 ex + C6 e

−3x

173

Page 176: Apuntes MAT-023 USM

Apuntes Mat023 (Segundo semestre 2014)

Debemos, ahora, calcular una solucion particular de la ecuacion:(D2 + 1

)2(D − 1)(D + 3) · y(x) = x

Un anulador para la funcion Q(x) = x es el operador diferencial D2. Entonces, por el

metodo del anulador, sabemos que una solucion particular yp(x) de la ecuacion anterior,

satisface:

yp ∈ kerD2(D2 + 1

)2(D − 1)(D + 3)

Luego:

yp(x) = a+ bx+ ε(x)

donde ε(x) ∈ ker(D2 + 1

)2(D − 1)(D + 3). Entonces:(

D2 + 1)2

(D − 1)(D + 3) · yp(x) =(D2 + 1

)2(D − 1)(D + 3) ·

[a+ bx+ ε(x)

]=(D2 + 1

)2(D − 1)(D + 3) ·

[a+ bx

]=(D2 + 1

)2(D − 1) ·

[b+ 3a+ 3bx

]=(D2 + 1

)2 ·[2b− 3a− 3bx

]= 2b− 3a− 3bx

Ahora bien, como(D2 + 1

)2(D− 1)(D+ 3) · yp(x) = x, obtenemos el sistema de ecuaciones:

−3b = 1

2b− 3a = 0

de donde a = −2/9 y b = −1/3. Ası, la solucion particular de la ecuacion (2.19) es

yp(x) = −29x− 1

3.

Finalmente, la solucion y(x) = yh(x) + yp(x) de la ecuacion diferencial en (2.19) es:

y(x) = C1 cosx+ C2 sinx+ C3 x cosx+ C4 x sinx+ C5 ex + C6 e

−3x − 2

9x− 1

3

Finalmente, como una aplicacion de los metodos de variacion de parametros y del

anulador resolveremos la ecuacion de Euler.

Definicion 2.7.2. Una ecuacion de Euler es una ecuacion de la forma:

(ax+ b)n y(n) + An−1 (ax+ b)n−1 y(n−1) + · · ·+ A1 (ax+ b) y′ + A0y = Q (x) (2.22)

donde ax+ b > 0 y A0, A2, . . . , An−1 son constantes cualesquiera.

Observacion 2.7.4. El metodo de resolucion de la ecuacion de Euler consiste en considerar

el cambio de variables:

ax+ b = et

174

Page 177: Apuntes MAT-023 USM

Apuntes Mat023 (Segundo semestre 2014)

Como se puede observar, el cambio de variable es un cambio en la variable independiente.

Ası, por la regla de la cadena:

dy

dx=

dy

dt

dt

dx= ae−t

dy

dtd2y

dx2=

d

dx

dy

dx

=

d

dt

ae−t

dy

dt

dt

dx= a2e−2t

d2y

dt2− dy

dt

d3y

dx3=

d

dx

d2y

dt

=

d

dt

a2e−2t

d2y

dt2− dy

dt

= a3e−3t

d3y

dt3− 3

d2y

dt2+ 2

dy

dt

y ası sucesivamente. Reemplazando todas las derivadas de orden superior en la ecuacion de

Euler en (2.22) se obtiene la siguiente ecuacion a coeficientes constantes:

y(n) (t) +Bn−1y(n−1) (t) + · · ·+B1y

′ (t) +B0y (t) = Q (t)

Ejemplo 2.7.7. Resuelva la ecuacion diferencial:

x2y′′ + xy′ + y = 1

Solucion. Utilizando el cambio de variables x = et, se tiene que:

dy

dx= e−t

dy

dt∧ d2y

dx= e−2t

(d2y

dt2− dy

dt

)al reemplazar en la ecuacion original, obtenemos:

d2y

dt2+ y = 1

luego:

y = C1 cos t+ C2 sin t+ 1

donde yp = 1, se obtiene inmediatamente por medio del metodo del anulador. Por tanto:

y = C1 cos (lnx) + C2 sin (lnx) + 1

es la solucion general de la ecuacion diferencial.

Ejemplo 2.7.8. Resolver las ecuacion de Euler

x2y′′ + 2xy′ − 2y = 0

Solucion. Haciendo el cambio de variables x = et se sigue

x2y′′ =

(d2y

dt2− dy

dt

)xy′ =

dy

dt

175

Page 178: Apuntes MAT-023 USM

Apuntes Mat023 (Segundo semestre 2014)

entonces la ecuacion se transforma en

D (D − 1) y + 2Dy − 2y = 0

donde y = y (et) entonces(D2 +D − 2

)y = 0⇔ (D + 2) (D − 1) y = 0

se sigue

y(et)

= c1e−2t + c2e

t

se sigue

y (x) =c1

x2+ c2x

Ejemplo 2.7.9. Determine una solucion y = y(x) de la ecuacion diferencial:

x3 y′′ + 5x2 y′ + 4xy = x2 + 1, x > 0

de modo que la recta tangente a esta solucion en el punto (1,−1) tenga una inclinacion de

45 con respecto al eje X.

Solucion. Consideremos la ecuacion:

x3 y′′ + 5x2 y′ + 4xy = x2 + 1, x > 0 (2.23)

Como x > 0, tenemos que la ecuacion (2.23) se escribe de forma equivalente como la

ecuacion:

x2y′′ + 5xy′ + 4y = x+1

x, x > 0 (2.24)

la cual es una ecuacion de Euler. En efecto, consideramos x = et, y entonces:

i) dx = et dt ii)dy

dt=dy

dt

dt

dxiii)

d2y

dx2=

d

dx

(dydx

)= e−t

dy

dt=

d

dt

(e−t

dy

dt

) dtdx

= e−2t

(d2y

dt2− dy

dt

)Luego, al cambiar la variable y reemplazar ii) y iii) en la ecuacion (2.24), obtenemos:

d2y

dt2+ 4

dy

dt+ 4y = 2 cosh t (2.25)

Ahora bien, la ecuacion caracterıstica asociada a la ecuacion anterior:

λ2 + 4λ+ 4 = 0

176

Page 179: Apuntes MAT-023 USM

Apuntes Mat023 (Segundo semestre 2014)

tiene raız λ = −2 con multiplicidad 2. Luego, la solucion de la ecuacion homogenea asociada

a (2.25) es:

yh(t) = C1 e−2t + C2 te

−2t

Ahora, por el metodo de variacion de parametros, la solucion particular de la ecuacion a

coeficientes constantes (2.25) esta dada por:

yp(t) = C1(t) e−2t + C2(t) te−2t

donde C1(t) y C2(t) estan dadas por el sistema de ecuaciones:C ′1(t)e−2t + C ′2(t)te−2t = 0

−2C ′1(t)e−2t + C ′2(t)(1− 2t)e−2t = 2 cosh t

Ası, entonces, por la regla de Cramer, tenemos que:

C ′1(t) =

∣∣∣∣ 0 te−2t

2 cosh t (1− 2t)e−2t

∣∣∣∣W (t)

y:

C ′1(t) =

∣∣∣∣ e−2t 0

−2−2t 2 cosh t

∣∣∣∣W (t)

donde:

W (t) =

∣∣∣∣ e−2t te−2t

−2e−2t (1− 2t)e−2t

∣∣∣∣= e−4t

∣∣∣∣ 1 t

−2 1− 2t

∣∣∣∣= e−4t

Luego:

C1(t) = −∫ (

te3t + tet)dt = (1− t)et +

e3t

9(1− 3t)

y:

C2(t) =

∫ (e3t + et

)dt = et +

e3t

3

Por tanto:

yp(t) =[(1− t)et +

e3t

9(1− 3t)

]e−2t +

[et +

e3t

3

]te−2t

=et

9+ e−t

177

Page 180: Apuntes MAT-023 USM

Apuntes Mat023 (Segundo semestre 2014)

Ası, la solucion general (en la variable t) de la ecuacion (2.25) esta dada por:

y(t) = C1 e−2t + C2te

−2t +et

9+ e−t

Ahora bien, como x = et, se tiene que t = lnx, luego:

y(x) = C11

x2+ C2

lnx

x2+x

9+

1

x

Finalmente, como la curva y = y(x) debe pasar por el punto (1,−1), y ademas en este

punto la recta tangente a la curva forma un angulo de 45 con respecto al eje X, se tiene,

entonces, que las condiciones iniciales son y(1) = −1 e y′(1) = 1. Por tanto, C1 = −199

y

C2 = 559

.

Ası, y finalmente, la curva y = y(x) del problema es:

y(x) = − 19

9x2+

55 lnx

9x2+x

9+

1

x

En las ecuaciones diferenciales de orden superior (al igual que en las de primer orden)

podemos hacer los cambios de variables que consideremos convenientes, sin embargo,

cualquier cambio de variables no asegura el poder resolver la ecuacion:

Ejemplo 2.7.10. Haciendo el cambio de variables x = t3 resolver la ecuacion

9x4/3 d2y

dx2+(6 3√x− 9x2/3

) dy

dx+ 2y = x2/3

Solucion. Usando el cambio de variables

dy

dx=

dy

dt

dt

dx

d2y

dx2=

d2y

dt2

(dt

dx

)2

+dy

dt

d2t

dx2

pero

dt

dx=

1

3x−2/3

d2t

dx2=−2

9x−5/3

se sigue

dy

dx=

dy

dt

(1

3t−2

)d2y

dx2=

d2y

dt2

(1

3t−2

)2

+dy

dt

(−2

9t−5

)178

Page 181: Apuntes MAT-023 USM

Apuntes Mat023 (Segundo semestre 2014)

reemplazando

9t4

(d2y

dt2

(1

3t−2

)2

+dy

dt

(−2

9t−5

))+(6t− 9t2

)(dy

dt

(1

3t−2

))+ 2y = t2

esto esd2y

dt2+

dy

dt

(−2t−1

)+ 2t−1 dy

dt− 3

dy

dt+ 2y = t2

es decir,d2y

dt2− 3

dy

dt+ 2y = t2

esta ecuacion es

(D − 2) (D − 1) y = t2

aplicamos anuladores

D3 (D − 2) (D − 1) y = 0

la solucion es de la forma

y = αe2t + βet + C1 + C2t+ C3t2

determinamos las constantes C1, C2 y C3(D2 − 3D + 2

) C1 + C2t+ C3t

2

= t2

2C3 − 3C2 − 6C3t+ 2C1 + 2C2t+ 2C3t2 = t2

obtenemos el sistema

2C3 − 3C2 + 2C1 = 0

−6C3 + 2C2 = 0

2C3 = 1

que tiene solucion C1 = 74, C2 = 3

2, C3 = 1

2ası

y = αe2t + βet +7

4+

3

2t+

1

2t2

volvemos a la variable original y la solucion es

y (x) = αe2 3√x + βe3√x +

7

4+

3

23√x+

1

2x2/3

con α, β ∈ R.

Ejemplo 2.7.11. Use z = sin (x) para resolver la ecuacion

y′′ + tan (x) y′ + cos2 (x) y = sin (x) cos2 (x)

179

Page 182: Apuntes MAT-023 USM

Apuntes Mat023 (Segundo semestre 2014)

Solucion. Las derivadas son con respecto a x la idea es cambiar las derivadas ahora con

respecto a z, para ello usamos la regla de la cadena: Notemos que

dy

dz=

dy

dx

dx

dz

o biendy

dz

dz

dx=

dy

dx

reemplazandody

dz=

(dydx

)(cosx)

= secxdy

dx

luego

d

dz

(dy

dz

)=

d

dz((secx) y′)

=

(secx tanx

dx

dz

)(dy

dx

)+ secx

(d2y

dx2

dx

dz

)=

sinx

cos2 x

1

cosx

(dy

dx

)+

1

cos2 x

d2y

dx2

se sigue

cos2 x

(d2y

dz2

)= tanx

dy

dx+

d2y

dx2

reemplazando en la ecuacion original

(cos2 x

)(d2y

dz2

)+(cos2 x

)y = 0

luegod2y

dz2+ y = 0

se sigue

y (z) = c1 sin z + c2 cos z

es decir la solucion general de la es

y (x) = c1 sin (sinx) + c2 cos (sinx) para c1, c2 ∈ R

180

Page 183: Apuntes MAT-023 USM

Apuntes Mat023 (Segundo semestre 2014)

Movimiento vibratorio

Observacion 2.8.1. Consideremos un sistema mecanico compuesto por un resorte de

longitud inicial l0 con su extremo superior sujeto firmemente y una masa m atada en su

extremo inferior. Nos interesa hallar la posicion x (t) de la masa en cualquier instante

t. Mas precisamente, nos interesa hallar la ecuacion que describa el movimiento de la

partıcula a la cual se le ha dado un desplazamiento x0 y una velocidad inicial v0. Para ello

consideramos los siguientes supuestos:

1. La masa se mueve a lo largo de la vertical que pasa por el centro de gravedad de la

masa y en el sistema de referencia x = 0 indica la posicion de reposo de la masa en

el resorte.

2. En cualquier tiempo t la magnitud de la fuerza ejercida sobre la masa es proporcional

a la diferencia entre la longitud l del resorte y su posicion de equilibrio l0. La constante

positiva de proporcionalidad k se llama constante del resorte, y el principio anterior

se conoce como la ley de Hooke.

Recordemos que la segunda ley del movimiento de Newton establece que la sumatoria

de fuerzas que actua sobre la partıcula es igual a la variacion instantanea del momemtum

mv respecto del tiempo, pero como la masa m es constante, se tiene:∑i

Fi = ma

donde a (t) = d2x(t)dt2

.

Suponemos, por ahora, que la unica fuerza que actua sobre la partıcula es la fuerza

restauradora inducida por el resorte, es decir, no se consideran las fuerzas de friccion, por

ejemplo. Por la ley de Hooke, tenemos que:

md2x

dt2= −kx

y el signo negativo es debido a que la fuerza restauradora del resorte se opone al movimiento.

Asumiendo, que la posicion inicial (es decir, en t = 0) de la masa es x0 y que su velocidad

inicial es v0, obtenemos el siguiente problema de valor inicial:

x′′ +k

mx = 0, x (0) = x0, x′ (0) = v0 (2.26)

La ecuacion caracterıstica de la ecuacion diferencial anterior es:

λ2 +k

m= 0

181

Page 184: Apuntes MAT-023 USM

Apuntes Mat023 (Segundo semestre 2014)

Ası:

λ = ±ω0

en donde ω0 =√k/m, luego:

x (t) = C1 cosω0t+ C2 sinω0t

Usando las condiciones iniciales, obtenemos que C1 = x0 y C2 = v0/ω0. Por tanto:

x (t) = x0 cosω0t+ (v0/ω0) sinω0t

= A sin (ω0 t+ φ)

donde A =√x2

0 + (v0/ω0)2 y φ es tal que tanφ = x0ω0/v0.

Definicion 2.8.1. El movimiento de la masa m bajo las condiciones anteriores, se denomina

movimiento armonico simple. La frecuencia natural f del movimiento es el numero

de oscilaciones completas por unidad de tiempo:

f =ω0

Ejemplo 2.8.1. Considere un resorte, sujeto en su extremo superior, que sostiene una

pesa de 10 libras en su extremo inferior. Suponga que la pesa estira el resorte 6 pulgadas.

Halle la ecuacion del movimiento de la pesa si esta se lleva a una posicion de 4 pulgadas

por debajo de su punto de equilibrio y se suelta. Calcule la ecuacion del movimiento su

amplitud y la frecuencia natural.

Ayuda: Considere que 1 [pie] = 12 [pulgadas] y que g = 32[

pieseg2

].

Observacion 2.8.2. En los supuestos que dan origen al movimiento armonico simple

no se consideran las fuerzas, que en una situacion mas realista, actuan sobre la masa.

Considerando por ejemplo resistencia del medio, ya sea por accion del aire o realizacion

de la oscilacion en un medio viscoso, es que se tienen las vibraciones amortiguadas. El

supuesto es simple, la fuerza amortiguadora del movimiento se presupone proporcional a

la velocidad del movimiento, ası el problema de valor inicial de la ecuacion (2.26) queda

como:

x′′ = − kmx− c

mx′ (2.27)

con las condiciones iniciales x (0) = x0 y x′ (0) = v0. La ecuacion caracterıstica de la

ecuacion (2.27) es:

λ2 +c

mλ+

k

m= 0

cuya solucion es:

λ =−c±

√c2 − 4km

2m

182

Page 185: Apuntes MAT-023 USM

Apuntes Mat023 (Segundo semestre 2014)

Por tanto, el comportamiento del movimiento depende del valor que tome el discriminante:

∆ = c2 − 4km

Tenemos, por tanto, tres casos:

Caso 2.8.1 (c2 − 4km > 0). La solucion de la ecuacion (2.27) en este caso esta dada por:

x (t) = C1 exp

(−c+

√c2 − 4km

2m

)+ C2 exp

(−c−

√c2 − 4km

2m

)Note que 4km < c2, lo que implica que

√c2 − 4km < c, y entonces:

−c+√c2 − 4km < 0 ∧ −c−

√c2 − 4km < 0

Ası, en este caso tenemos que:

lımt→∞

x (t) = 0

Caso 2.8.2 (c2 − 4km = 0). La solucion de la ecuacion (2.27) es:

x (t) = (C1 + C2 t) e(−c/2m)t

y al igual que en el caso anterior:

lımt→∞

x (t) = 0

Observacion 2.8.3. Entonces, se observa por la forma de las soluciones que si:

c2 − 4km ≥ 0

no se producen oscilaciones en el sistema mecanico masa-resorte considerando amortiguacion.

Este tipo de movimiento tıpicamente ocurre en un medio viscoso como agua o aceite.

Caso 2.8.3 (c2 − 4km < 0). Como (−c/2m) 6= 0, obtenemos:

x (t) = exp(− c

2m

)C1 cos

√4km− c2

2mt+ C2 sin

√4km− c2

2mt

Observacion 2.8.4. Finalmente, podemos suponer una fuerza g = g (t) aplicada sobre

extremo superior del resorte (que inicialmente se supuso sujeto firmemente, esto es con

g ≡ 0) en la direccion del movimiento de la masa m. El movimiento de la masa obtenido bajo

este supuesto se conoce como vibracion forzada. Una vibracion forzada tambien puede

contener fuerzas amortiguadoras. Este es el caso mas general del movimiento vibratorio, el

problema de valor inicial queda en su forma mas general como:

x′′ +c

mx′ +

k

mx =

g (t)

m(2.28)

con x (0) = x0 e x′ (0) = v0.

183

Page 186: Apuntes MAT-023 USM

Apuntes Mat023 (Segundo semestre 2014)

Ejemplo 2.8.2. Considere un resorte que en su extremo superior esta sometido a una

fuerza externa:

f (t) =10

32sin t

y sostiene, en su parte inferior una masa de 10 libras. Suponga que la masa estira el resorte

6 pulgadas. Hallar la ecuacion del movimiento de la masa si esta se lleva a una posicion de

4 pulgadas por debajo del punto de equilibrio y se suelta.

Observacion 2.8.5. Un caso muy importante es cuando g es una funcion periodica, es

decir, supongamos que:

g (t) = F0 sinωt

Ası, la ecuacion (2.28) queda:

x′′ +c

mx′ +

k

mx =

F0

msinωt (2.29)

Sabemos que el anulador de g (t) = F0 sinωt es el operador:

D2 + ω2

Aplicando este ultimo operador al operador D2 + cmD + k

m, obtenemos:

(D2 + ω2

)(D2 +

c

mD +

k

m

)· x =

(D2 + ω2

)· F0

msinωt = 0

Por tanto, la ecuacion (2.29) tiene una solucion particular de la forma:

xp = b1 cosωt+ b2 sinωt (2.30)

donde b1 y b2 son soluciones del sistema:(ω2

0 − ω2) b1 + cωmb2 = 0

− cωmb1 + (ω2

0 − ω2) b2 = F0

m

donde ω0 =√

km

. Ası:

b1 = − F0 cω

m2 (ω20 − ω2)

2+ (cω)2

b2 =F0m (ω2

0 − ω2)

m2 (ω20 − ω2)

2+ (cω)2

Ası, a partir de lo anterior y mediante el procedimiento de transformacion a sinusoide de

la ecuacion (2.30) encontramos que:

xp = A sin (ωt+ φ)

184

Page 187: Apuntes MAT-023 USM

Apuntes Mat023 (Segundo semestre 2014)

donde la amplitud A esta dada por:

A =F0/k√

1−(ωω0

)22

+(

2 cc0

ωω0

)2

y el angulo de fase φ satisface:

tanφ =2 cc0(

ωω0

)2

− 1

con c0 = 2mω0. Notamos que de la ecuacion que define la amplitud los coeficientes

importantes son:

1. c/c0 llamado el cociente de amortiguacion, y

2. ω/ω0 llamado el cociente de frecuencia del movimiento.

Se observa que el comportamiento de la solucion particular xp depende entonces del

cociente de amortiguacion si este es cero o despreciable, entonces tenemos el movimiento

armonico simple:

x′′ + ω20x = 0

y por tanto, hay dos casos para el cociente de frecuencia:

Caso 2.8.4 (ω2 6= ω20). En este caso se tiene:

x (t) = C1 cosω0t+ C2 sinω0t+F0/k

1− (ω/ω0)2 sinωt

donde:

C1 = x0 ∧ C2 =v0

ω0

− (F0/k) (ω/ω0)

1− (ω/ω0)2

Caso 2.8.5 (ω2 = ω20). En este caso debemos buscar una solucion particular de la forma:

xp = b1t cosωt+ b2t sinωt

pues la ecuacion (2.30) se anula con el operador D2 + ω2, luego el anulador a considerar es

(D2 + ω2)2. Ası, luego de los calculo habituales del metodo del anulador se obtiene:

b1 = − F0

2mω∧ b2 = 0

Por tanto, la solucion general tiene la forma:

x (t) = C1 cosωt+ C2 sinωt− F0

2mωt cosωt

185

Page 188: Apuntes MAT-023 USM

Apuntes Mat023 (Segundo semestre 2014)

Observacion 2.8.6. Observe en el caso anterior que al aumenta t, las vibraciones causadas

por el ultimo termino de la ecuacion anterior aumentarıan sin cota. En este caso, se dice

que la fuerza externa esta en resonancia con la masa en vibracion.

Ejemplo 2.8.3. Una masa de 2 [kg] se sujeta de un resorte suspendido del techo. Esto

ocasiona que el resorte se estire 4965

[mt] al llegar a su estado de equilibrio. En el instante

t = 0 la masa se desplaza 1[mt] hacia abajo y se suelta. En ese mismo instante se aplica

una fuerza externa 5 sin(2t) [newton] al sistema. Si la constante de amortiguacion es de

8[newton seg

mt

]. Determine el desplazamiento x(t) de la masa para t > 0. Considere g=9, 8[

mtseg2

].

Solucion.

mx′′ + cx′ + kx = 5 sin(2t)

donde

2 (9,8) =49

65k

de donde k = 26 se sigue

2x′′ + 8x′ + 26x = 5 sin(2t)

x (0) = 1

x′ (0) = 0

la solucion de la ecuacion es

x (t) =9

58sin 2t− 4

29cos 2t+ C1 (cos 3t) e−2t + C2e

−2t (sin 3t)

reemplazando la C.I

1 = − 4

29+ C1

C1 = 3329

x (t) =9

58sin 2t− 4

29cos 2t+

33

29(cos 3t) e−2t + C2e

−2t (sin 3t)

derivando y reemplazando

0 =9

29− 66

29+ 3C2

C2 = 1929

se sigue

x (t) =

(9

58

)sin 2t−

(4

29

)cos 2t+

((33

29

)cos 3t+

(19

29

)sin 3t

)e−2t

186

Page 189: Apuntes MAT-023 USM

Apuntes Mat023 (Segundo semestre 2014)

Ejercicios del capıtulo

1. En los problemas siguientes se muestra que el principio de superposicion generalmente

no se cumple para ecuaciones no lineales

a) Muestre que y = 1/x es solucion de la ecuacion y′ + y2 = 0 pero si c 6= 0 y c 6= 1

entonces y = c (1/x) no es solucion.

b) Muestre que φ1 (x) = 1 e φ2 (x) =√x son soluciones de yy′′ + (y′)2 = 0 pero la

suma no es solucion.

2. Busque una solucion polinomial de la ecuacion de Legendre(1− x2

)y′′ − 2xy′ + 2y = 0

y usando la formula de Abel determine la solucion general de esta ecuacion.

3. Verificar que φ1 (x) = x−1/2 cosx es solucion (para x > 0) de la ecuacion de Bessel

de orden 12

x2y′′ + xy′ +

(x2 − 1

4

)y = 0

y obtener su solucion general.

4. Muestre que y1 (x) = x+ 1 es solucion de la ecuacion(1− 2x− x2

)y′′ + 2 (1 + x) y′ − 2y = 0

y resolverla completamente (encontrar solucion general).

5. Considere la ecuacion diferencial:

y′′ + P1 (x) y′ + P2 (x) y = Q (x) (2.31)

y sean u1 (x) , u2 (x) soluciones linealmente independientes de la ecuacion homogenea

asociada. Pruebe que la funcion:

f (x) = C1 (x)u1 (x) + C2 (x)u2 (x)

es una solucion particular de la ecuacion (2.31), si se satisface el sistema de ecuaciones:C ′1 (x)u1 (x) + C ′2 (x)u2 (x) = 0

C ′1 (x)u′1 (x) + C ′2 (x)u′2 (x) = Q (x)

6. Compruebe que:

y =1

ω

∫ x

0

f (t) sinω (x− t) dt

es una solucion particular de y′′ + ω2y = f (x).

187

Page 190: Apuntes MAT-023 USM

Apuntes Mat023 (Segundo semestre 2014)

7. Suponga que ex y e−x son soluciones de una ecuacion diferencial lineal ordinaria

homogenea, muestre que las funciones sinhx y coshx tambien son soluciones de esa

ecuacion.

8. Muestre que la solucion particular de una EDO lineal de segundo orden no homogenea

y′′ + a1 (x) y′ + a0 (x) y = f (x)

se puede escribir en la forma

yp (x) =

∫ x

x0

G (x, t) f (t) dt

donde G es una funcion adecuada.

9. Hallar la solucion general de

1) y′′′ + 3y′′ − y′ − 3y = 0 2) y(4) + 16y = 0 3) d4ydx4

+ d2ydx2

+ y = 0

4) y′′ − 36y = 0 5) y′′ + 2y′ − 3y = 0 6) y′′ + 8y′ + 16y = 0

7) y′′′′ + 4y′′ + 3y = 0 8) y′′ − 4y′ + 5y = 0 9) y′′′ − 5y′′ + 3y′ + 9y = 0

10. Resuelva la ecuacion diferencial sujeta a las condiciones indicadas.

a) y′′ + 16y = 0 ; y(0) = 2, y′(0) = −2

b) y′′ + 6y′ + 25y = 0 ; y(0) = 2, y′(0) = 6

c) y′′ − 4y = 0 ; y(0) = 4, y′ (0) = 16

d) y′′′ + 2y′′ − 5y′ − 6y = 0 ; y(1) = 0, y′(1) = 0, y′′(1) = 1

11. Resuelva las siguientes ecuaciones por anuladores.

a) y′′ + y′ = 2t+ sin t

b) y′′ + 4y′ + 2y = te−2t

c) y′′ − 4y′ + 5y = 4 + sin(√

7t)

d) y′′ + 3y = x2e3x

e) y′′ − 2y′ + 2y = e2x(cosx− 3 sinx)

f ) y′′′ − y′′ − 4y′ + 4y = 5− ex + e2x

g) y′′′ − 2y′′ + 2y′ − y = ex + cosx

12. Resuelva la ecuacion diferencial sujeta a las condiciones indicadas.

a) 2y′′ + 3y′ − 2y = 14x2 − 4x− 11 , y(0) = 0, y′(0) = 0

188

Page 191: Apuntes MAT-023 USM

Apuntes Mat023 (Segundo semestre 2014)

b) y′′ + 4y′ + 4y = (3 + x)e−2x , y(0) = 2, y′(0) = 5

c) y′′ − 2y′ + 2y = 2x− 2 , y(π) = 0, y′(π) = 0

d)d2x

dt2+ ω2x = F0 cos γt , x(0) = 0, x′(0) = 1

e)d2x

dt2+ ω2x = F0 sinωt , x(0) = 0, x′(0) = 0

13. Hallar una EDO lineal con coeficientes constantes que tenga por solucion general:

1) y (x) = c1xex + c2e

x + x+ 1

2) y (x) = c1e2x cos

(√3x)

+ c2e2x sin

(√3x)

+ c3xe2x cos

(√3x)

+ c4xe2x sin

(√3x)

+ ex

14. Encontrar un anulador de

1) f (x) = x cos(2x+ π

3

)2) f (x) = (x2 + 1)

2ex + x sinh (x)

3) f (x) = (x2 − 1)2

(1 + sin x) e√

3x

15. Encontrar la solucion general de

1) x2y′′ + xy′ + 9y = 0

2) x2y′′ + xy′ − p2y = 0 con p ∈ R

16. Encontrar la solucion general de

1) x2y′′ + xy′ − 9y = x2 + 1 2) x2y′′

+ xy′ − 9y = x3 + 1

3) x2y′′ + 4xy′ + 2y = 2 ln x 4) x2y′′

+ xy′+ 9y = sin(lnx3)

5) x3y′′′ + 4x2y′′ + xy′ − y = x 6) x2y′′ − xy′ + 2y = 16

7) y′′ + 1xy′ − 1

x2y = 1

x2+x38) y′′ − 2

xy′ + 2

x2y = lnx

x

9) x4y′′ + x3y′ − 4x2y = 1 10) x3y′′′ + 6x2y′′ + 4xy′ − 4y = x

17. Encontrar una solucion general de cada una de las ecuaciones diferenciales, usando el

metodo de variacion de parametros.

a) y′′′ − y′′ + y′ − y = 4xex

b) y′′′ − y′ = sinx

c) y′′′ − 2y′′ = 4(x+ 1)

d) y′′′ − 3y′′ − y′ + 3y = 1 + ex

e) y′′′ − 7y′ + 6y = 2 sin x

f ) 3y′′ − 6y′ + 30y = ex tan 3x

g) y′′ − 2y′ + y = 4x2 − 3 +ex

x

189

Page 192: Apuntes MAT-023 USM

Apuntes Mat023 (Segundo semestre 2014)

h) y′′′ − 2y′ − 4y = 2e−x secx (puede dejar integrales finales sin calcular)

18. Una solucion y = u(x) de la ecuacion diferencial y′′ − 3y′ − 4y = 0 intersecta a una

solucion de la ecuacion diferencial y′′ + 4y′ − 5y = 0 en el origen. Determinar las

funciones u y v si las dos soluciones tienen la misma pendiente en el origen y si

lımx→∞

(v(x))4

u(x)=

5

6.

19. Sean y1 = u(x) y y2 = xu(x) dos soluciones de la ecuacion y′′ + 4xy′ + p(x)y = 0. Si

u(0) = 1, determine u(x) y p(x) explıcitamente.

20. Se sabe que para x > 0 la ecuacion homogenea asociada a

4xy′′ +(2− 8

√x)y′ − 5y = 1

tiene una solucion

y1 (x) = e−√x

determine la solucion general.

21. Determine la solucion general de la ecuacion diferencial

(2x− 1)2 d2y

dx2+ 4 (2x− 1)

dy

dx− 8y =

√2x− 1

Ind.: Considere el cambio de variables u = 2x− 1.

22. Dada la ecuacion diferencial

(x− a)d2y

dx2− xdy

dx+ a2y = a (x− 1)2 ex

determine el valor del parametro de forma que ex sea una solucion de la homogenea

asociada y determine la solucion general de la ecuacion no homogenea para tales

valores del parametro.

23. Use z = sin (x) para resolver la ecuacion

y′′ + tan (x) y′ + cos2 (x) y = sin (x) cos2 (x)

24. Considere la ecuacion diferencial

x2y′′ − 2xy′ + 2y = 6

con condiciones iniciales y (0) = 3, y′ (0) = 1 tiene soluciones yc (x) = cx2 + x + 3

para todo c ∈ R, este problema no tiene solucion unica, ¿contradice esto el teorema

de existencia y unicidad.?

190

Page 193: Apuntes MAT-023 USM

Apuntes Mat023 (Segundo semestre 2014)

25. Resolver

(xD + 1) (D − x) (xD) y = x

donde D = ddx

.

26. Un cuerpo con masa m = 12

kilogramo (kg) esta unido en el extremo de un resorte

estirado 2 metros (m) debido a una fuerza de 100 newtons (N) y es puesto en

movimiento a partir de la posicion inicial x0 = 1 (m) y velocidad inicial v0 = 5 (m/s).

(Notese que estas condiciones iniciales indican que el cuerpo se desplaza a la derecha

y a la izquierda en el tiempo t = 0). Encuentrese la funcion de la posicion del cuerpo,

ası como su amplitud, frecuencia, periodo de oscilacion y el tiempo de retardo de su

movimiento.

27. Determine el periodo y la frecuencia del movimiento armonico simple de una masa

de 4 kg unida al extremo de un resorte con constante de 16 N/m.

28. Establezca el periodo y la frecuencia del movimiento armonico simple de un cuerpo

con una masa de 0.75 kg unida al extremo de un resorte con constante de 48 N/m.

29. Un cuerpo con masa de 250 g esta unido al extremo de un resorte estirado 25 cm por

una fuerza de 9 N. En el tiempo t0 el cuerpo es movido 1 m a la derecha, estirando el

resorte y aplicando un movimiento con una velocidad inicial de 5 m/s a la izquierda.

(a) Encuentre x(t). (b) Obtenga la amplitud y el periodo de movimiento del cuerpo.

30. Asuma que la ecuacion diferencial de un pendulo simple de longitud L es Lθ′′+gθ = 0,

donde g = GM/R2 es la aceleracion gravitacional en el lugar donde este se encuentra

(a una distancia R del centro de la Tierra; M significa la masa de la Tierra). Dos

pendulos de longitudes L1 y L2 —ubicados a una distancia R1y R2 respecto del centro

de la Tierra— tienen periodos p1 y p2. Muestre que

p1

p2

=R1

√L1

R2

√L2

191

Page 194: Apuntes MAT-023 USM

Capıtulo 3 : Sistemas de ecuaciones diferenciales

Definiciones

Considere los dos tanques que se ilustran en la figura. Suponga que el tanque A contiene

50 galones de agua en los que hay disueltas 25 libras de sal. Suponga que el tanque B

contiene 50 galones de agua pura. A los tanques entra y sale lıquido como se indica en

la figura; se supone que tanto la mezcla intercambiada entre los tanques como el lıquido

bombeado hacia fuera del tanque B estan bien mezclados. Se desea construir un modelo

matematico que describa la cantidad de libras x1 (t) y x2 (t) de sal en los tanques A y B

respectivamente en el tiempo t.

se tiene

dx1

dt=

(3

[gal

min

]· 0[

lb

gal

]+ 1

[gal

min

]· x2

50

[lb

gal

])− 4

[gal

min

]· x1

50

[lb

gal

]= − 2

25x1 +

x2

50

y

dx2

dt= 4

x1

50− 3

x2

50− 1

x2

50

=2

25x1 −

2

25x2

ası obtenemos el sistema

dx1

dt= − 2

25x1 +

1

50x2

dx2

dt=

2

25x1 −

2

25x2

192

Page 195: Apuntes MAT-023 USM

Apuntes Mat023 (Segundo semestre 2014)

y las condiciones x1 (0) = 25 y x2 (t) = 0

Un sistema de ecuaciones de la forma

dX

dt= A (t)X (t) +B (t)

esto es x′1 (t)

x′2 (t)...

x′n (t)

=

a11 (t) a12 (t) · · · a1n (t)

a21 (t) a22 (t) · · · a2n (t)...

.... . .

...

an1 (t) an2 (t) · · · ann (t)

x1 (t)

x2 (t)...

xn (t)

+

b1 (t)

b2 (t)...

bn (t)

donde X (t) = (x1 (t) , x2 (t) , . . . , xn (t))T , A (t) = (aij (t)) es una matriz de orden n × ny B (t) = (b1 (t) , b2 (t) , . . . , bn (t))T es llamado sistema de ecuaciones diferenciales

lineal de primer orden (se supone que las funciones aij (t) y bi (t) son continuas en un

intervalo abierto I). Si adicionalmente se debe cumplir X (t0) = X0 para t0 ∈ I, X0 ∈ Rn

entonces tenemos el P.V.I

dX

dt= A (t)X (t) +B (t)

X (t0) = X0

Ejemplo 3.1.1. Son sistemas de ecuaciones lineales:

1.

dxdt

= x+ 2y + et

dydt

= −x+ 2y + cos t

2.

dxdt

= etx+ e−ty + t

dydt

= −x+ (sin t) y

3.

dxdt

= x+ y + z

dydt

= −x+ 2y + 3z

dzdt

= 2x+ 3y + z

mientras que el sistema

dx

dt= x2 + ety2

dy

dt= cos t+

√x2 + y2

no es lineal.

193

Page 196: Apuntes MAT-023 USM

Apuntes Mat023 (Segundo semestre 2014)

Teorema 3.1.1. Si las funciones aij (t) y bi (t) son continuas en el abierto I ⊆ R, t0 ∈ I,

X0 ∈ Rn entonces el problema de valores iniciales

dX

dt= A (t)X (t) +B (t)

X (t0) = X0

tiene solucion unica.

Ejemplo 3.1.2. Resolver el sistema

dx1

dt= x2

dx2

dt= −x1

con x1 (0) = 1, x2 (0) = 0.

Solucion. Note que derivando la primera ecuacion respecto a t obtenemos

d2x1

dt2=dx2

dt= −x1

de donde x1 debe cumplird2x1

dt2+ x1 = 0

ası x1 (t) = c1 cos t+ c2 sin t para algunas constantes c1 y c2 ahora bien, x1 (0) = 1 = c1 se

sigue

x1 (t) = cos t+ c2 sin t

ademas

x2 (t) =dx1

dt= − sin t+ c2 cos t

luego

0 = x2 (0) = c2

se sigue

x1 (t) = cos t

x2 (t) = − sin t

194

Page 197: Apuntes MAT-023 USM

Apuntes Mat023 (Segundo semestre 2014)

Observacion 3.1.1. Toda ecuacion diferencial ordinaria lineal de orden n puede ser

transformada en un sistema de ecuaciones lineales, en efecto si tenemos

y(n) + an−1 (t) y(n−1) + · · ·+ a1 (t) y′ + a0 (t) y = h (t)

poniendo

x1 (t) = y (t)

x2 (t) = y′ (t)...

xn (t) = y(n−1) (t)

entonces

d

dt

x1

x2

...

xn−1

xn

=

0 1 0 0 · · · 0

0 0 1 0 · · · 0...

......

......

...

0 0 0 0. . . 1

−a0 −a1 · · · · · · · · · −an−1

x1

x2

...

xn−1

xn

+

0

0...

0

h (t)

Ejemplo 3.1.3. Considere la ecuacion

d2x

dt2+ (cos t)

dx

dt+ 2x = et

esta se transforma en el sistema

d

dt

(x1

x2

)=

(0 1

−2 − cos t

)(x1

x2

)+

(0

et

)donde x1 (t) = x (t) y x2 (t) = x′ (t)

La teorıa de los sistemas lineales es similar a la que hemos visto hasta ahora en el caso

de una ecuacion. La ecuacion Homogenea asociada a

dX

dt= A (t)X (t) +B (t)

esdX

dt= A (t)X (t)

su conjunto solucion

S =

X (t) ∈ C1 (I,Rn) :

dX

dt= A (t)X (t)

es un espacio vectorial de dimension n (si el sistema es de orden n) y la solucion de la no

homogenea sera una solucion particular mas una combinacion lineal arbitraria de elementos

de la base de S.

195

Page 198: Apuntes MAT-023 USM

Apuntes Mat023 (Segundo semestre 2014)

Ecuacion con coeficientes constantes

Suponga que trabajamos con el sistema

dX

dt= AX

donde A ∈Mn×n (R) (coeficientes constantes).

Teorema 3.2.1. Si λ es un valor propio de A con vector propio v entonces

x (t) = veλt

es solucion de dXdt

= AX.

Matriz A diagonalizable

Si A es una matriz diagonalizable entonces

P−1AP = D

donde D es una matriz diagonal, la ecuacion

dX

dt= AX

la podemos ver comodX

dt= PDP−1X

multiplicando la ecuacion por P−1 por la izquierda

d

dt

(P−1X

)= DP−1X

cambiamos la variable

Y = P−1X

nos quedadY

dt= DY

que es un sistema de la forma

dyidt

= λiyi para i = 1, 2, . . . , n

196

Page 199: Apuntes MAT-023 USM

Apuntes Mat023 (Segundo semestre 2014)

tiene soluciones

yi (t) = cieλit

luego la solucion es

X (t) = PY (t) =n∑i=1

civieλit

donde vi son los vectores propios asociados a los valores propios correspondientes.

Observacion 3.2.1. Si el valor propio λi es complejo vieλit y vie

λit apareceran como

soluciones, al buscar soluciones reales se tiene que Re(vie

λit)

y Im(vie

λit)

son soluciones

l.i. reales correspondientes a combinaciones lineales de las dos complejas luego generan lo

mismo.

Ejemplo 3.2.1. Resolver

d

dt

x1

x2

x3

=

−1 1 1

1 −1 1

1 1 1

x1

x2

x3

Solucion. Primero buscamos el polinomio caracterıstico de la matriz

A =

−1 1 1

1 −1 1

1 1 1

esta dado por

PA (λ) = |A− λI| =

∣∣∣∣∣∣−1− λ 1 1

1 −1− λ 1

1 1 1− λ

∣∣∣∣∣∣ = (λ− 2) (λ+ 2) (λ+ 1)

luego tiene 3 valores propios distintos y es una matriz de orden 3 luego es diagonalizable,

ahora buscamos los vectores propios:⟨ −1

−1

1

⟩↔ −1,

⟨ −1

1

0

⟩↔ −2,

⟨ 1212

1

⟩↔ 2

ası la solucion del sistema es

X (t) = c1

−1

−1

1

e−t + c2

−1

1

0

e−2t + c3

1212

1

e2t =

12c3e

2t − c2e−2t − c1e

−t

c2e−2t − c1e

−t + 12c3e

2t

c1e−t + c3e

2t

197

Page 200: Apuntes MAT-023 USM

Apuntes Mat023 (Segundo semestre 2014)

Ejemplo 3.2.2. Resolver

d

dt

x1

x2

x3

=

5 2 2

2 2 −4

2 −4 2

x1

x2

x3

Si

A =

5 2 2

2 2 −4

2 −4 2

entonces es diagonalizable por ser una matriz simetrica

|A− λI| =

∣∣∣∣∣∣5− λ 2 2

2 2− λ −4

2 −4 2− λ

∣∣∣∣∣∣ = − (λ+ 3) (λ− 6)2

luego hay 1 valor propio de multiplicidad 2 buscamos los vectores propios⟨ −12

1

1

⟩↔ −3,

⟨ 2

1

0

,

2

0

1

⟩↔ 6

se sigue que la solucion del sistema es

X (t) = c1

−12

1

1

e−3t + c2

2

1

0

e6t + c3

2

0

1

e6t =

2c2e6t − 1

2c1e−3t + 2c3e

6t

c1e−3t + c2e

6t

c1e−3t + c3e

6t

Ejemplo 3.2.3. Resolver

d

dt

x1

x2

x3

=

0 2 0

−2 0 0

0 0 1

x1

x2

x3

Solucion. Si

A =

0 2 0

−2 0 0

0 0 1

entonces la ecuacion caracterıstica es (λ− 1) (λ2 + 4) = 0 de donde tenemos que la matriz

es diagonalizable, 3 valores propios distintos aunque hay dos complejos, veamos los espacios

propios asociados⟨ 0

0

1

⟩↔ 1,

⟨ i

1

0

⟩↔ −2i,

⟨ −i10

⟩↔ 2i

198

Page 201: Apuntes MAT-023 USM

Apuntes Mat023 (Segundo semestre 2014)

el resultado nos dice que i

1

0

e−2it y

−i10

e2it

son soluciones pero buscamos soluciones reales y para ello

Re

i

1

0

e−2it

y Im

i

1

0

e−2it

generan lo mismo que las dos anteriores i

1

0

e−2it =

i

1

0

(cos (2t)− i sen (2t))

=

i cos (2t) + sen (2t)

cos (2t)− i sen (2t)

0

=

sen (2t)

cos (2t)

0

+ i

cos (2t)

− sen (2t)

0

se sigue

Re

i

1

0

e−2it

=

sen (2t)

cos (2t)

0

y

Im

i

1

0

e−2it

=

cos (2t)

− sen (2t)

0

luego la solucion es

X (t) = c1

0

0

1

et + c2

sin (2t)

cos (2t)

0

+ c3

cos (2t)

− sin (2t)

0

=

c3 cos 2t+ c2 sin 2t

c2 cos 2t− c3 sin 2t

c1et

Si la matriz no es diagonalizable no hemos desarrollado una teorıa para poder resolverla

(mas adelante resolveremos estos sistemas utilizando la transformada de Laplace) sin

embargo podemos enfrentar directamente el sistema a traves de eliminacion, considere el

siguiente ejemplo:

199

Page 202: Apuntes MAT-023 USM

Apuntes Mat023 (Segundo semestre 2014)

Ejemplo 3.2.4. Resolver el sistema

d

dt

(x

y

)=

(−2 −1

1 −4

)(x

y

)Solucion. En este caso el polinomio es

|A− λI| =∣∣∣∣ −2− λ −1

1 −4− λ

∣∣∣∣ = λ2 + 6λ+ 9 = (λ+ 3)2

se sigue que hay un solo valor propio de multiplicidad algebraica 2, Si calculamos vemos

que la multiplicidad geometrica (dimension del espacio propio asociado) es 1 luego la matriz

no es diagonalizable

Wλ=−3 =

⟨(1

1

)⟩Lo resolveremos directamente

x′ = −2x− yy′ = x− 4y

si derivamos la primera ecuacion obtenemos

x′′ = −2x′ − y′

pero de la segunda ecuacion sabemos

y′ = x− 4y

reemplazando

x′′ = −2x′ − (x− 4y)

= 4y − x− 2x′

pero de la primera ecuacion

y = −x′ − 2x

ası

x′′ = 4 (−x′ − 2x)− x− 2x′

= −9x− 6x′

ası

x′′ + 6x′ + 9x = 0⇔ (D + 3)2 x = 0

luego

x (t) = c1e−3t + c2te

−3t

200

Page 203: Apuntes MAT-023 USM

Apuntes Mat023 (Segundo semestre 2014)

ademas

y (t) = −x′ − 2x

= − d

dt

(c1e−3t + c2te

−3t)− 2

(c1e−3t + c2te

−3t)

= e−3t (c1 − c2 + tc2)

= c1e−3t + c2 (−1 + t) e−3t

luego

X (t) =

(x (t)

y (t)

)=

(c1e−3t + c2te

−3t

c1e−3t + c2 (−1 + t) e−3t

)= c1

(1

1

)e−3t + c2

(t

−1 + t

)e−3t

es la solucion. Note que la estructura de la solucion es similar a las ecuaciones con raız

repetida.

Variacion de parametros en sistemas

Suponga que queremos determinar la solucion particular de la ecuacion

dX

dt= AX +B (t)

donde la solucion de la homogenea es

Xh (t) = c1X1 (t) + c2X2 (t) + · · ·+ cnXn (t)

entonces proponemos una solucion particular de la forma

Xp (t) =n∑i=1

ci (t)Xi (t)

se sigue que

X ′p (t) =n∑i=1

c′i (t)Xi (t) +n∑i=1

ci (t)X′i (t)

=n∑i=1

c′i (t)Xi (t) +n∑i=1

ci (t)AXi (t)

=n∑i=1

c′i (t)Xi (t) + AXp (t)

201

Page 204: Apuntes MAT-023 USM

Apuntes Mat023 (Segundo semestre 2014)

entoncesn∑i=1

c′i (t)Xi (t) = B (t)

se sigue

c′i (t) =

∣∣ X1 (t) X2 (t) · · · Xi−1 (t) B (t) Xi+1 (t) · · · Xn (t)∣∣∣∣ X1 (t) X2 (t) · · · Xi−1 (t) Xi (t) Xi+1 (t) · · · Xn (t)∣∣

es un cociente de determinantes por la regla de Cramer.

Ejemplo 3.3.1. Determine la solucion general del sistema(dxdtdydt

)=

(1 −3

−3 1

)(x

y

)+

(e2t

e4t

)Solucion. Primero resolvemos el sistema homogeneo, la matriz(

1 −3

−3 1

)tiene valores propios −2, 4 y

Wλ=−2 =

⟨(1

1

)⟩Wλ=4 =

⟨(−1

1

)⟩se sigue que la solucion general del sistema homogeneo es

Xh (t) = α

(1

1

)e−2t + β

(−1

1

)e4t para α, β ∈ R

para determinar la solkucion particular usamos variacion de parametros, existe una solucion

de la forma

Xp (t) = C1 (t)

(1

1

)e−2t + C2 (t)

(−1

1

)e4t

donde

C ′1 (t) =

∣∣∣∣ e2t −e4t

e4t e4t

∣∣∣∣∣∣∣∣ e−2t −e4t

e−2t e4t

∣∣∣∣ =e6t + e8t

2e2t

=1

2e4t +

1

2e6t

202

Page 205: Apuntes MAT-023 USM

Apuntes Mat023 (Segundo semestre 2014)

se sigue

C1 (t) =

∫ (1

2e4t +

1

2e6t

)dt =

1

8e4t +

1

12e6t

por otro lado

C ′2 (t) =

∣∣∣∣ e−2t e2t

e−2t e4t

∣∣∣∣∣∣∣∣ e−2t −e4t

e−2t e4t

∣∣∣∣ =e2t − 1

2e2t

=1

2− 1

2e−2t

se sigue

C2 (t) =

∫ (1

2− 1

2e−2t

)dt

=1

2t+

1

4e−2t

ası

Xp (t) = C1 (t)

(1

1

)e−2t + C2 (t)

(−1

1

)e4t

=

(1

8e4t +

1

12e6t

)(1

1

)e−2t +

(1

2t+

1

4e−2t

)(−1

1

)e4t

=

(18e2t + 1

12e4t

18e2t + 1

12e4t

)+

(−(

12te4t + 1

4e2t)(

12te4t + 1

4e2t) )

=

(18e2t + 1

12e4t − 1

2te4t − 1

4e2t

18e2t + 1

12e4t + 1

2te4t + 1

4e2t

)=

(18e2t − 1

4e2t + 1

12e4t − 1

2te4t

18e2t + 1

4e2t + 1

12e4t + 1

2te4t

)la solucion general es

XG = α

(1

1

)e−2t + β

(−1

1

)e4t +

(−1

8e2t +

(112− 1

2t)e4t

38e2t +

(112

+ t2

)e4t

)para α, β ∈ R

Ejemplo 3.3.2. Resolver el sistema dxdtdydtdzdt

=

1 −4 0

4 1 0

0 0 2

x

y

z

+

1

0

t

203

Page 206: Apuntes MAT-023 USM

Apuntes Mat023 (Segundo semestre 2014)

Solucion. Este sistema tiene la forma

dX

dt= AX +B (t)

donde

B (t) =

1

0

t

y A =

1 −4 0

4 1 0

0 0 2

primero resolvemos el sistema homogeneo

dX

dt= AX

es decir dxdtdydtdzdt

=

1 −4 0

4 1 0

0 0 2

x

y

z

buscamos los valores y vectores propios asociados, en este caso son:

0

0

1

↔ 2,

−i1

0

↔ 1− 4i,

i

1

0

↔ 1 + 4i

se sigue que las solucion general del sistema homogeneo es de la forma

Xh (t) = c1

0

0

1

e2t + c2Re

−i10

e(1−4i)t

+ c3Im

−i10

e(1−4i)t

donde −i1

0

e(1−4i)t =

−i10

ete−4ti

=

−i10

et (cos 4t− i sin 4t)

=

−iet cos 4t− et sin 4t

et (cos 4t− i sin 4t)

0

= et

− sin 4t

cos 4t

0

+ iet

− cos 4t

− sin 4t

0

204

Page 207: Apuntes MAT-023 USM

Apuntes Mat023 (Segundo semestre 2014)

entonces

Xh (t) = c1

0

0

1

e2t + c2et

− sin 4t

cos 4t

0

+ c3et

− cos 4t

− sin 4t

0

ahora buscamos una solucion particular de la ecuacion no homogenea

dX

dt= AX +B (t)

de la forma

Xp (t) = c1 (t)X1 (t) + c2 (t)X2 (t) + c3 (t)X3 (t)

(mediante variacion de parametros) donde

X1 (t) =

0

0

e2t

, X2 (t) =

−et sin 4t

et cos 4t

0

y X3 (t) =

−et cos 4t

−et sin 4t

0

se sigue las funciones c1, c2 y c3 deben cumplir 0 −et sin 4t −et cos 4t

0 et cos 4t −et sin 4t

e2t 0 0

c′1 (t)

c′2 (t)

c′3 (t)

=

1

0

t

sistema que podemos resolver por ejemplo mediante determinantes:

c′1 (t) =

∣∣∣∣∣∣1 −et sin 4t −et cos 4t

0 et cos 4t −et sin 4t

t 0 0

∣∣∣∣∣∣∣∣∣∣∣∣0 −et sin 4t −et cos 4t

0 et cos 4t −et sin 4t

e2t 0 0

∣∣∣∣∣∣=te2t

e4t= te−2t

:

c′2 (t) =

∣∣∣∣∣∣0 1 −et cos 4t

0 0 −et sin 4t

e2t t 0

∣∣∣∣∣∣∣∣∣∣∣∣0 −et sin 4t −et cos 4t

0 et cos 4t −et sin 4t

e2t 0 0

∣∣∣∣∣∣=−e3t sin 4t

e4t= −e−t sin 4t

y

c′3 (t) =

∣∣∣∣∣∣0 −et sin 4t 1

0 et cos 4t 0

e2t 0 t

∣∣∣∣∣∣∣∣∣∣∣∣0 −et sin 4t −et cos 4t

0 et cos 4t −et sin 4t

e2t 0 0

∣∣∣∣∣∣=−e3t (cos 4t)

e4t= − (cos 4t) e−t

205

Page 208: Apuntes MAT-023 USM

Apuntes Mat023 (Segundo semestre 2014)

integramos :

c1 (t) =

∫te−2tdt = −1

4e−2t (2t+ 1)

c2 (t) =

∫−e−t sin 4tdt =

1

17e−t (4 cos 4t+ sin 4t)

c3 (t) =

∫− (cos 4t) e−t)dt =

1

17e−t (cos 4t− 4 sin 4t)

entonces la solucion particular es

Xp (t) =

(−1

4e−2t (2t+ 1)

) 0

0

e2t

+

(1

17e−t (4 cos 4t+ sin 4t)

) −et sin 4t

et cos 4t

0

+

1

17e−t (cos 4t− 4 sin 4t)

−et cos 4t

−et sin 4t

0

=

− 117

417

−12t− 1

4

verifiquemos que es correcto

X ′p (t) =

0

0

−12

=

1 −4 0

4 1 0

0 0 2

− 117

417

−12t− 1

4

+

1

0

t

=

−1

0

−t− 12

+

1

0

t

se sigue que la solucion del sistema es

X (t) = Xh (t) +Xp (t)

= c1

0

0

1

e2t + c2et

− sin 4t

cos 4t

0

+ c3et

− cos 4t

− sin 4t

0

+

− 117

417

−12t− 1

4

206

Page 209: Apuntes MAT-023 USM

Apuntes Mat023 (Segundo semestre 2014)

Analisis cualitativo de sistemas

Vamos a analizar el comportamiento de los sistemas de orden 2 de la forma

dx

dt= ax+ by

dy

dt= cx+ dy

donde a, b, c y d son constantes, basandonos en los valores propios de la matriz asociada al

sistema

A =

(a b

c d

)la cual supondremos diagonalizable y adicionalmente que det (A) = ad− bc 6= 0 de esta

forma la unica solucion constante del sistema corresponde al origen, esta solucion es llamada

solucion de equilibrio, las demas soluciones seran entonces curvas parametricas (x (t) , y (t))

y segun su comportamiento clasificaremos la solucion de equilibrio.

Valores propios reales y distintos (no nulos)

1. Punto silla: En el caso en que los valores propios son reales y distintos con signos

contrarios, el origen se dice que es un punto de silla. Considere el sistema

d

dt

(x

y

)=

(−1 1

3 1

)(x

y

)En este caso la matriz de coeficientes es

A =

(−1 1

3 1

)la cual tiene determinante distinto de cero. Buscamos sus valores propios y vectores

propios asociados, en este caso son:(−1

1

)↔ −2,

(13

1

)↔ 2

luego las soluciones del sistema son de la forma

X (t) = c1

(13

1

)e2t + c2

(−1

1

)e−2t

Note que si c1 = 1 y c2 = 0 obtenemos una solucion recta

X1 (t) =

(13

1

)e2t

207

Page 210: Apuntes MAT-023 USM

Apuntes Mat023 (Segundo semestre 2014)

en esta solucion x (t) = 13e2t y y (t) = e2t se sigue que los puntos de esta curva estan

sobre la rectay

x=

e2t

13e2t

= 3

es decir y = 3x y se alejan del origen cuando t→ +∞ por el primer cuadrante. De

manera similar, −X1 (t) = −(

13

1

)e2t es la curva parametrica que esta sobre la

recta y = 3x y si t → +∞ la solucion se aleja del origen por el tercer cuadrante.

Existe otra solucion recta, al poner c1 = 0 y c2 = 1 se obtiene

X2 (t) =

(−1

1

)e−2t

es decir la curva parametrica

x = −e−2t

y = e−2t

esta curva esta sobre la recta

y

x= −1 es decir y = −x

note que cuando t → +∞ los puntos se acercan al origen sobre tal recta por el

segundo cuadrante. Note que −X2 (t) tambien es solucion, en este caso

x = e−2t

y = −e−2t

esta curva tambien esta sobre la recta y = −x los puntos se acercan al origen pero

por el cuarto cuadrante. Las solucion general es

X (t) = c1

(13

1

)e2t + c2

(−1

1

)e−2t

note que si t → +∞ entonces c2

(−1

1

)e−2t ≈ 0 y la solucion se comporta como

c1

(13

1

)e2t esto es, cuando el tiempo avanza la curva solucion se acerca a la recta

y = 3x (el cuadrante depende de c1) y si analizamos el comportamiento cuando

t→ −∞ las curvas soluciones tienden a c2

(−1

1

)e−2t esto se interpreta como que

las soluciones parten cercana a esa recta y se aproximan a y = 3x. Recuerde que

208

Page 211: Apuntes MAT-023 USM

Apuntes Mat023 (Segundo semestre 2014)

por teorema de existencia y unicidad dos soluciones distintas no se pueden cortar.

Grafiquemos algunas soluciones y el campo de direcciones asociado a este sistema

2. Sumidero: Si los valores propios son reales y distintos, ambos negativos, el origen

es llamado sumidero. Considere el sistema

d

dt

(x

y

)=

(−6

525

25−9

5

)(x

y

)en este caso la matriz de coeficientes es

A =

(−6

525

25−9

5

)que tiene vectores y valores propios asociados(

2

1

)↔ −1,

(−1

2

1

)↔ −2

luego las soluciones son de la forma

X (t) = c1

(2

1

)e−t + c2

(−1

2

1

)e−2t

209

Page 212: Apuntes MAT-023 USM

Apuntes Mat023 (Segundo semestre 2014)

grafiquemos el campo de direcciones y diagrama de fase asociados. Note que en este

caso tenemos dos soluciones rectas y que las soluciones tienden al origen si t→ +∞

3. Fuente: Si los valores propios son distintos pero ambos positivos entonces el origen

es llamado fuente (las soluciones se alejan del origen) Considere el sistema

d

dt

(x

y

)=

(0 6

−1 5

)(x

y

)en este caso la matriz de coeficientes es

A =

(0 6

−1 5

)que tiene vectores y valores propios(

3

1

)↔ 2,

(2

1

)↔ 3

luego las soluciones son de la forma

X (t) = c1

(3

1

)e2t + c2

(2

1

)e3t

210

Page 213: Apuntes MAT-023 USM

Apuntes Mat023 (Segundo semestre 2014)

note que si t → ∞ entonces ‖X (t)‖ → ∞ se sigue que las soluciones se alejan del

origen y tenemos dos soluciones rectas.

Valores propios complejos

1. Sumidero espiral: En el caso en que los valores propios son complejos y la parte

real del valor propio es negativa, el origen es un sumidero espiral. Considere el sistema

d

dt

(x

y

)=

(1 2

−4 −3

)(x

y

)en este caso la matriz asociada es

A =

(1 2

−4 −3

)tiene por vectores y valores propios(

−12

+ 12i

1

)↔ −1− 2i,

(−1

2− 1

2i

1

)↔ −1 + 2i

se sigue que la solucion del sistema esta generada por

Re

((−1

2+ 1

2i

1

)e(−1−2i)t

)e Im

((−1

2+ 1

2i

1

)e(−1−2i)t

)211

Page 214: Apuntes MAT-023 USM

Apuntes Mat023 (Segundo semestre 2014)

entonces

X (t) = c1e−t(

12

sin 2t− 12

cos 2t

cos 2t

)+ c2e

−t(

12

sin 2t+ 12

cos 2t

− sin 2t

)ahora el campo de vectores y algunas curvas (note que en este caso no hay soluciones

rectas) las soluciones se acercan al origen

2. Fuente espiral: En el caso en que los valores propios son complejos con parte real

positiva las soluciones se alejan del origen y este es llamado fuente espiral. Considere

el sistemad

dt

(x

y

)=

(4 3

−6 −2

)(x

y

)en este caso la matriz asociada es

A =

(4 3

−6 −2

)los valores y vectores propios son(

−12

+ 12i

1

)↔ 1− 3i,

(−1

2− 1

2i

1

)↔ 1 + 3i

212

Page 215: Apuntes MAT-023 USM

Apuntes Mat023 (Segundo semestre 2014)

luego la solucion es generada por

Re

((−1

2+ 1

2i

1

)e(1−3i)t

)e Im

((−1

2+ 1

2i

1

)e(1−3i)t

)es decir las soluciones son de la forma

X (t) = c1et

(12

sin 3t− 12

cos 3t

cos 3t

)+ c2e

t

(12

sin 3t+ 12

cos 3t

− sin 3t

)grafiquemos el campo de direcciones y algunas soluciones el comportamiento es el

mismo pero las soluciones se alejan del origen. (no tenemos soluciones rectas)

3. Centro: Cuando los valores propios son complejos con parte real igual a cero el

origen es llamado centro, en este caso las soluciones no se acercan al origen sino que

se mantienen alrededor de el, el origen es estable en este caso pero no asintoticamente

estable. Considere el sistema

d

dt

(x

y

)=

(3 3

−6 −3

)(x

y

)en este caso la matriz asociada es

A =

(3 3

−6 −3

)213

Page 216: Apuntes MAT-023 USM

Apuntes Mat023 (Segundo semestre 2014)

que tiene por valores y vectores propios(−1

2+ 1

2i

1

)↔ −3i,

(−1

2− 1

2i

1

)↔ 3i

luego la solucion del sistema es generada por

Re

((−1

2+ 1

2i

1

)e(−3i)t

)e Im

((−1

2+ 1

2i

1

)e(−3i)t

)es decir las soluciones son de la forma

X (t) = c1

(12

sin 3t− 12

cos 3t

cos 3t

)+ c2

(12

sin 3t+ 12

cos 3t

− sin 3t

)grafiquemos el campo de vectores y algunas curvas solucion.

Valores propios repetidos (no nulos)

1. Fuente: Cuando hay solo un valor propio y es positivo el origen es llamado fuente,

las soluciones se alejan del origen. Considere el sistema

d

dt

(x

y

)=

(2 0

0 2

)(x

y

)214

Page 217: Apuntes MAT-023 USM

Apuntes Mat023 (Segundo semestre 2014)

en este caso la matriz asociada es

A =

(2 0

0 2

)y el valor propio es λ = 2 se ve que las soluciones son de la forma

X (t) = c1

(1

0

)e2t + c2

(0

1

)e2t

es decir

X (t) =

(c1e

2t

c2e2t

)las soluciones son todas rectas que se alejan del origen.

2. Sumidero: Si hay solo un valor propio y es negativo las curvas se acercan al origen

y este es llamado sumidero. Considere el sistema

d

dt

(x

y

)=

(−2 0

0 −2

)(x

y

)en este caso la matriz asociada es

A =

(−2 0

0 −2

)los valores propios son iguales a −3 pero la matriz no es diagonalizable, las soluciones

son de la forma

X (t) =

(c1e−2t

c2e−2t

)son rectas que se acercan al origen.

Teorema 3.4.1. Para un sistema lineal X′ = AX en el cual det (A) 6= 0, sea X = X (t)

la solucion que satisface X (0) = X0 con X0 6= 0.

1. lımt→+∞X (t) = 0 si y solo si los valores propios de A tienen partes reales negativas.

2. X (t) es periodica si y solo si las valores propios de A son imaginarios puros.

Ejemplo 3.4.1. Sea α ∈ R−1, 2. Considere el sistema de ecuaciones

d

dt

(x

y

)=

(α α− 2

1− α α

)(x

y

)

215

Page 218: Apuntes MAT-023 USM

Apuntes Mat023 (Segundo semestre 2014)

1. Clasificar la solucion de equilibrio (silla, atractor, repulsor, etc.) para los distintos

valores de α.

Primero notemos que ∣∣∣∣ α α− 2

1− α α

∣∣∣∣ = 2α2 − 3α + 2 > 0

para todo α ∈ R, luego la unica solucion de equilibrio es el origen. Para clasificar

calculamos los valores propios

λ2 − 2αλ+(2α2 − 3α + 2

)luego

λ =2α±

√4α2 − 4 (2α2 − 3α + 2)

2

=2α±

√4α2 − 4 (2α2 − 3α + 2)

2

= α±√− (α− 1) (α− 2)

a) Si − (α− 1) (α− 2) > 0 es decir α ∈ ]1, 2[ tenemos soluciones reales y notemos

que el producto de las dos es 2α2 − 3α + 2 > 0 luego las dos son positivas o

negativas,

α−√− (α− 1) (α− 2) > 0

⇔ (para α > 0)

α2 > − (α− 1) (α− 2)

⇔α2 + (α− 1) (α− 2) > 0

lo que tiene solucion todo R+ pero por la restriccion se sigue α ∈ ]1, 2[ . Ası,

para α ∈ ]1, 2[ tenemos dos valores propios reales y positivos, el punto es un

repulsor.

b) Si − (α− 1) (α− 2) < 0 esto es α ∈ ]−∞, 1[∪ ]2,+∞[ tenemos raıces complejas

conjugadas. Para α = 0 tenemos un centro, para α < 0 tenemos un atractor

espiral y para α ∈ ]0, 1[ ∪ ]2,+∞[ tenemos un repulsor espiral.

2. Para α = 32

determine la solucion general del sistema y bosquejar el diagrama de

fases.

si α = 32

entonces

λ =3

2± 1

2

216

Page 219: Apuntes MAT-023 USM

Apuntes Mat023 (Segundo semestre 2014)

es decir, los valores propios son 2, 1 buscamos los espacios propios(32

32− 2

1− 32

32

)=

(32−1

2

−12

32

)se sigue

Wλ=1 =

⟨(1

1

)⟩y

Wλ=2 =

⟨(−1

1

)⟩la solucion general es(

x (t)

y (t)

)= C1

(1

1

)et + C2

(−1

1

)e2t

el diagrama de fases es

Ejemplo 3.4.2. Considere el sistema de ecuaciones diferenciales lineales(x′

y′

)=

(−a a

1 −a

)(x

y

)1. Bosquejar los diagramas de fases para todo a ∈ R−0, 1

Buscamos los valores propios, determinamos si son reales y complejos para clasificar.

El polinomio caracterıstico es

λ2 + 2aλ+(a2 − a

)luego como a2 − a = a (a− 1) 6= 0 se sigue que los valores propios son no nulos

y la unica solucion de equilibrio es el cero. El discriminante de esta cuadratica es

217

Page 220: Apuntes MAT-023 USM

Apuntes Mat023 (Segundo semestre 2014)

4a2 − 4 (a2 − a) = 4a se sigue que para a < 0 las raıces son complejas y para a > 0

las raıces son reales.

Supongamos a < 0 entonces las raıces son

−2a±√

4a

2= −a±

√a

= −a± i√−a

complejas con parte real positiva es una fuente espiral.

Para a > 0 las raıces son reales, su producto es a (a− 1) el cual es positivo si a > 1

y negativo si 0 < a < 1. Este signo nos dice que para 0 < a < 1 los valores propios

tienen distinto signo (es una silla) y para a > 1 tienen igual signo veamos si es

positivo o negativo

−a+√a y − a−

√a

son negativas por la segunda, se sigue que para a > 1 es un sumidero.

Los graficos son los siguientes: Para a < 0

218

Page 221: Apuntes MAT-023 USM

Apuntes Mat023 (Segundo semestre 2014)

para 0 < a < 1

para a > 1

2. Resolver el sistema no homogeneo(x′

y′

)=

(−4 4

1 −4

)(x

y

)+

(et

e2t

)Resolvemos el sistema homogeneo, la matriz(

−4 4

1 −4

)

219

Page 222: Apuntes MAT-023 USM

Apuntes Mat023 (Segundo semestre 2014)

tiene valores propios −2 y −6 y vectores propios asociados

(2

1

)y

(−2

1

)respec-

tivamente, se sigue que la solucion del sistema homogeneo es

X (t) = α

(2

1

)e−2t + β

(−2

1

)e−6t

la solucion particular la determinamos usando variacion de parametros

Xp (t) = C1 (t)

(2

1

)e−2t + C2 (t)

(−2

1

)e−6t

donde

C ′1 (t) =

∣∣∣∣ et −2e−6t

e2t e−6t

∣∣∣∣∣∣∣∣ 2e−2t −2e−6t

e−2t e−6t

∣∣∣∣ =2e−4t + e−5t

4e−8t

=1

2e4t +

1

4e3t

ası

C1 (t) =

∫ (1

2e4t +

1

4e3t

)dt

=1

12e3t +

1

8e4t

de manera similar

C ′2 (t) =

∣∣∣∣ 2e−2t et

e−2t e2t

∣∣∣∣∣∣∣∣ 2e−2t −2e−6t

e−2t e−6t

∣∣∣∣ =1

2e8t − 1

4e7t

luego

C2 (t) =

∫ (1

2e8t − 1

4e7t

)dt

=1

16e8t − 1

28e7t

se sigue

Xp (t) =

(1

12et +

1

8e2t

)(2

1

)+

(1

16e2t − 1

28et)(

−2

1

)=

(521et + 1

8e2t

121et + 3

16e2t

)220

Page 223: Apuntes MAT-023 USM

Apuntes Mat023 (Segundo semestre 2014)

la solucion general es

XG (t) = α

(2

1

)e−2t + β

(−2

1

)e−6t +

(521et + 1

8e2t

121et + 3

16e2t

)con α, β ∈ R.

Ejemplo 3.4.3. Resolver el sistema de ecuaciones

x′ = −3x+ 25y

y′ = −x− 3y

y bosquejar el diagrama de fases.

Solucion. El sistema se puede escribir matricialmente como(x′

y′

)=

(−3 25

−1 −3

)(x

y

)buscamos los valores propios y vectores propios. La ecuacion caracterıstica es

P (λ) =

∣∣∣∣ −3− λ 25

−1 −3− λ

∣∣∣∣= λ2 + 6λ+ 34

= 0

que tiene soluciones λ1 = −3 + 5i y λ2 = −3− 5i.

Wλ=−3+5i =

(a

b

)∈ R2 :

(−3− (−3 + 5i) 25

−1 −3− (−3 + 5i)

)(a

b

)=

(0

0

)=

(a

b

)∈ R2 :

(−5i 25

−1 −5i

)(a

b

)=

(0

0

)=

(a

b

)∈ R2 : a+ 5ib = 0

=

⟨(−5i

1

)⟩ası las soluciones base son (

−5i

1

)e(−3+5i)t y

(5i

1

)e(−3−5i)t

para las soluciones reales calculamos

Re

((−5i

1

)e(−3+5i)t

)e Im

((−5i

1

)e(−3+5i)t

)221

Page 224: Apuntes MAT-023 USM

Apuntes Mat023 (Segundo semestre 2014)

donde (−5i

1

)e(−3+5i)t = e−3t

((−5i) (cos (5t) + i sin (5t))

(cos (5t) + i sin (5t))

)= e−3t

(5 sin 5t− 5i cos 5t

cos 5t+ i sin 5t

)=

(5 sin 5t

cos 5t

)e−3t + i

(−5 cos 5t

sin 5t

)e−3t

ası la solucion general es

X = α

(5 sin 5t

cos 5t

)e−3t + β

(−5 cos 5t

sin 5t

)e−3t

para α, β ∈ R. El diagrama de fases es

Ejemplo 3.4.4. Considere el sistema de ecuaciones

d

dtX = AX + B

donde X =

(x (t)

y (t)

), A =

(2 4

−1 −3

)y B =

(et

e−2t

)222

Page 225: Apuntes MAT-023 USM

Apuntes Mat023 (Segundo semestre 2014)

1. Resolver el sistema homogeneo ddt

X = AX, bosquejar el diagrama de fases y clasificar

la solucion de equilibrio. Si

(x (0)

y (0)

)=

(3

2

)determine la ecuacion de la recta a

la cual tiende la curva solucion cuando t→ +∞.

Buscamos los valores propios∣∣∣∣ 2− λ 4

−1 −3− λ

∣∣∣∣ = λ2 + λ− 2

= (λ+ 2) (λ− 1)

luego los valores propios son λ = −2 y λ = 1. Como los valores propios son reales

con distinto signo, la solucion de equilibrio corresponde a un punto silla. Buscamos

los vectores propios

Wλ=−2 =

(x

y

):

(4 4

−1 −1

)(x

y

)=

(0

0

)⟨(

−1

1

)⟩y

Wλ=1 =

(x

y

):

(1 4

−1 −4

)(x

y

)=

(0

0

)⟨(

−4

1

)⟩se sigue que la solucion del sistema homogeneo es(

x (t)

y (t)

)= c1

(−1

1

)e−2t + c2

(−4

1

)et

Note que independiente del punto por donde pase las soluciones se aproximan a la

recta (x

y

)= c2

(−4

1

)et

(a menos que sea una solucion de la forma c2

(−4

1

)et) se sigue que la recta es

x = −4c2et

y = etc2

asıy

x= −1

4

223

Page 226: Apuntes MAT-023 USM

Apuntes Mat023 (Segundo semestre 2014)

esto es

y = −x4

2. Resolver el sistema no homogeneo ddt

X = AX + B

Dado que tenemos la solucion de ddt

X = AX, necesitamos la solucion particular la

cual tiene la forma

Xp (t) = c1 (t)

(−1

1

)e−2t + c2 (t)

(−4

1

)et

(por variacion de parametros) donde

c′1 (t) =

∣∣∣∣ et −4et

e−2t et

∣∣∣∣∣∣∣∣ −e−2t −4et

e−2t et

∣∣∣∣ =1

3e3t +

4

3

⇒ c1 (t) =

∫ (1

3e3t +

4

3

)dt

=4

3t+

1

9e3t

224

Page 227: Apuntes MAT-023 USM

Apuntes Mat023 (Segundo semestre 2014)

de manera similar

c′2 (t) =

∣∣∣∣ −e−2t et

e−2t e−2t

∣∣∣∣∣∣∣∣ −e−2t −4et

e−2t et

∣∣∣∣ = −e−3t

3− 1

3

⇒ c2 (t) =

∫ (−e−3t

3− 1

3

)dt =

1

9e−3t − 1

3t

ası

Xp (t) =

(4

3t+

1

9e3t

)(−1

1

)e−2t +

(1

9e−3t − 1

3t

)(−4

1

)et

=

(−(

43te−2t + 1

9et)(

43te−2t + 1

9et) )

+

(−4(

19e−2t − 1

3tet)(

19e−2t − 1

3tet) )

=

43tet − 4

9e−2t − 4

3te−2t − 1

9et

19et + 1

9e−2t + 4

3te−2t − 1

3tet

ası la solucion es(

x (t)

y (t)

)= c1

(−1

1

)e−2t + c2

(−4

1

)et +

43tet − 4

9e−2t − 4

3te−2t − 1

9et

19et + 1

9e−2t + 4

3te−2t − 1

3tet

Ejercicios del capıtulo

1. Escribir los siguientes sistemas en forma matricial

a) dxdt

= 3x− 2y b) dxdt

= x− 2ty + t2

dydt

= 4x+ 8y dydt

= 4t2x+ 8y + sin t

c) dxdt

= 3x− y + 3z d) dxdt

= etx− cos ty + 3tz + cos tdydt

= 4x+ 3y + z dydt

= x+ et

dzdt

= 2x+ y − z dzdt

= 2x+ y − z − 3t+ 1

2. Escribir los siguientes sistemas sin usar matrices

a) ddt

(x

y

)=

(−2 1

t 0

)(x

y

)+

(sin t

et

)

b) X′ =

−1 1 1

0 1 2

−1 −1 1

X+

−1

0

−1

et +

−1 + t

1

−1

et sin t

225

Page 228: Apuntes MAT-023 USM

Apuntes Mat023 (Segundo semestre 2014)

3. En los siguientes ejercicios verificar si el vector X dado es solucion del sistema

a) X =

(1

2

)e−5t de

dxdt

= 3x− 4ydydt

= 4x− 7y

b) X =

1

6

13

de

dxdt

= x+ 2y + zdydt

= 6x− ydzdt

= −x− 2y − z

c) X =

sin t

−12

sin t− 12

cos t

− sin t+ cos t

de

X′=

1 0 1

1 1 0

−2 0 −1

X

4. Determine si los vectores dados son linealmente independientes

a) X1=

(1

−1

)et y X2=

(2

6

)et +

(8

−8

)tet

b) X1=

1

6

−13

, X2=

1

−2

−1

e−4t y X3=

2

3

−2

e3t

5. Muestre que la funcion Xp =

(1

1

)et+

(1

−1

)tet es solucion particular del sistema

X′=

(2 1

1 −1

)X−

(1

7

)et

6. Demostrar que la solucion general del sistema

X′=

0 6 0

1 0 1

1 1 0

X

es de la forma

X =c1

6

−1

−5

e−t + c2

−3

1

1

e−2t + c2

2

1

1

e3t

7. Determinar la solucion general de los sistemas:

226

Page 229: Apuntes MAT-023 USM

Apuntes Mat023 (Segundo semestre 2014)

a) dxdt

= x+ 2y b) dxdt

= 2x+ 2ydydt

= 4x+ 3y dydt

= x+ 3y

c) dxdt

= x+ y − z d) dxdt

= 2x− 7ydydt

= 2y dydt

= 5x+ 10y + 4zdzdt

= y − z dzdt

= 5y + 2z

e) dxdt

= 6x− y f) dxdt

= 2x+ y + 2zdydt

= 5x+ 2y dydt

= 3x+ 6zdzdt

= −4x− 3z

8. Resolver los siguientes sistemas usando variacion de parametros

a) dxdt

= 3x− 3y + 4 b) dxdt

= 2x− ydydt

= 2x− 2y − 1 dydt

= 3x− 2y + 4t

c) dxdt

= x+ y − z + t d) dxdt

= −y + sec tdydt

= 2y + t dydt

= xdzdt

= y − z + t

e) dxdt

= x+ 2y + et csc t f) dxdt

= 3x− y − zdydt

= −12x+ y + et sec t dy

dt= x+ y − z + t

dzdt

= x− y + z + 2et

9. Sea X = X (t) la solucion al sistema de ecuaciones

x′ = αx− βyy′ = βx+ αy

que satisface la condicion inicial X (0) = X0. Determine condiciones sobre α, β que

aseguren

lımt→+∞

X (t) =

(0

0

)10. Determine condiciones sobre µ tal que (0, 0) sea un centro para el sistema lineal

x′ = −µx+ y

y′ = −x+ µy

11. Sea A ∈Mn×n (R) defina

eAt = I + At+ A2 t2

2!+ A3 t

3

3!+ · · ·

=∞∑k=0

Ak tk

k!

227

Page 230: Apuntes MAT-023 USM

Apuntes Mat023 (Segundo semestre 2014)

a) Calcular eAt para A =

(a c

0 b

)b) Muestre que si A = PDP−1 con D =diag(λ1, λ2, . . . , λn) diagonal entonces

etA = P(diag

(eλ1t, eλ2t, . . . , eλnt

))P−1

c) Muestre que la solucion general de

d

dtX =

(4 3

−2 −1

)X

es

X =etA(c1

c2

)

donde A =

(4 3

−2 −1

)d) Conjeturar una formula para la solucion general de

X′ = AX + B (t)

en terminos de etA.

12. Clasificar la solucion de equilibrio de los siguientes sistemas y bosquejar el diagrama

de fases:

a) dxdt

= −x− 5y b) dxdt

= 4x+ 3ydydt

= x+ 3y dydt

= 2x− 3y

c) dxdt

= −x+ 2y d) dxdt

= x+ 4ydydt

= x− 4y dydt

= −x+ y

e) dxdt

= −3x+ y f) dxdt

= 2x+ ydydt

= −2x− y dydt

= 3x+ 6y

228

Page 231: Apuntes MAT-023 USM

Capıtulo 4 : Transformacion integral de Laplace

Definiciones y teoremas fundamentales

Observacion 4.1.1. Las ecuaciones diferenciales describen formas en las cuales ciertas

cantidades cambian respecto al tiempo, cantidades tales como la corriente en un circuito

electrico, las oscilaciones de una membrana, flujos de calor, etc. esas ecuaciones generalmente

estan acompanadas de condiciones iniciales que describen el estado del sistema en el

tiempo t = 0. Una herramienta muy poderosa para resolver este tipo de problemas es la

transformada de Laplace la cual transforma la ecuacion diferencial original en una ecuacion

algebraica la cual puede ser transformada nuevamente en la solucion del problema original.

Esta tecnica es conocida como “el metodo de la transformada de Laplace”. La clave del

proceso anterior radica en la formula de integracion por partes. Si f, g ∈ C1 recordemos

que la formula de integracion por partes corresponde a:∫ b

a

f (x) g′ (x) dx = f (x) g (x)∣∣∣ba−∫ b

a

f ′ (x) g (x) dx

y se puede interpretar del modo siguiente: En un producto de funciones bajo el sımbolo de

la integral es posible intercambiar la derivada presente en una funcion. Sin embargo, tal

procedimiento implica al final el tener que evaluar el producto de tales funciones en los

extremos del intervalo y trabajar con una nueva derivada.

La idea, entonces, es trabajar con una funcion que tenga un buen comportamiento de la

derivada y la evaluacion en los extremos del intervalo de integracion, esto se puede lograr

mediante una funcion que no cambie esencialmente al derivarla i.e. una exponencial y que

se pueda anular en los extremos. Lo anterior motiva la siguiente definicion:

Definicion 4.1.1. Una funcion f : [0,+∞)→ R se dice localmente integrable si para

cada T > 0, la integral∫ T

0f (t) dt existe. Llamaremos transformacion integral de

Laplace o simplemente la transformada de Laplace a la funcion L [f (t)] : D ⊆ R→ Rdefinida mediante la integral impropia:

s→ L [f (t)] (s) =

∫ ∞0

f (t) e−st dt

para todos los valores de s ∈ R para los cuales la integral sea convergente. El conjunto de

los valores de s ∈ R para los cuales la integral converge se llamara dominio de L [f (t)] y se

denotara por D (L [f ]).

Ejemplo 4.1.1. Sea f : R→ R, t→ f (t) = 1. Calcular su transformada de Laplace.

229

Page 232: Apuntes MAT-023 USM

Apuntes Mat023 (Segundo semestre 2014)

Solucion. Note que: ∫ ∞0

1 e−st dt = lımT→∞

∫ T

0

e−st dt

= lımT→∞

−1

se−st

∣∣∣T0

=1

s

para cada s > 0. Por tanto:

L [1] (s) =1

s, s > 0

Ejemplo 4.1.2. Sea f (t) = eat, con a una constante real. Calcular su transformada de

Laplace

Solucion. Por definicion:

L[eat]

(s) =

∫ ∞0

eat e−st dt = lımT→∞

∫ T

0

e(a−s)tdt

= lımT→∞

1

a− se(a−s)t

∣∣∣T0

= lımT→∞

1

a− s(e(a−s)T − 1

)ası, si a− s < 0, entonces lımT→∞

1a−s

(e(a−s)T − 1

)= 1

s−a , luego:

L[eat]

(s) =1

s− aPara s > a

Observacion 4.1.2. Con respecto al ejemplo anterior, dos observaciones deben hacerse.

En primer lugar, respecto a lo notacional, en lo sucesivo escribiremos:

lımT→∞

f (t)∣∣∣T0

= f (t)∣∣∣∞0

para evitar anotar cada vez que se calcule una transformada de Laplace, el lımite relacionado

con la integral impropia. En segundo lugar, notamos que D (L [eat]) = ]a,+∞[, luego la

transformada no necesariamente estara definida para todos los valores de s ∈ R.

Ejemplo 4.1.3. Calcule, si acaso existe, la transformada de Laplace de f (t) = 1t, con

t > 0.

Solucion. Sea t > 0. Luego:∫ ∞0

e−st

tdt =

∫ 1

0

e−st

tdt+

∫ ∞1

e−st

tdt

230

Page 233: Apuntes MAT-023 USM

Apuntes Mat023 (Segundo semestre 2014)

pero si 0 < t ≤ 1 y s > 0, entonces e−s ≤ e−st y la integral:∫ 1

0

e−s

tdt

diverge. Por tanto, la transformada de Laplace de f (t) = 1/t no existe para ningun valor

de s ∈ R.

En vista del ejemplo anterior, trataremos de dar respuesta a la siguiente pregunta:

¿Para que tipo de funciones podemos asegurar la existencia de la transformada de Laplace?

Definicion 4.1.2. Sean f una funcion real. Anotaremos:

f(a+)

= lımt→a+

f (t) ∧ f(a−)

= lımt→a−

f (t)

si acaso estos lımites existen. Diremos que una funcion f tiene una discontinuidad de

salto en a si: ∣∣f (a+)− f

(a−)∣∣ <∞

o bien si f (a+) = f (a−), pero estos valores son distintos de f (a) (puede ser incluso que

f (a) no este definida). Una funcion f : [0,+∞)→ R se dice seccionalmente continua

si para cada T > 0 la funcion f |[0,T ] es continua y tiene a lo sumo un numero finito de

discontinuidades de salto.

Ejemplo 4.1.4. La funcion

f (t) =

1/t t 6= 0

0 t = 0

no es seccionalmente continua en [0,+∞), lo mismo para

f (t) =

cos (1/t) t > 0

0 t ≤ 0

mientras que g (t) = btc si es seccionalmente continua.

Definicion 4.1.3. Diremos que una funcion f : [0,+∞)→ R es de orden exponencial

β si existen constantes M > 0, β ∈ R y un valor t0 ≥ 0 tales que:

|f (t)| ≤M eβt

para todo t ≥ t0.

Ejemplo 4.1.5. Las siguientes funciones son seccionalmente continuas y de orden expo-

nencial:

1. f (t) = sinh t, pues:

sinh t =et − e−t

2<et

2

231

Page 234: Apuntes MAT-023 USM

Apuntes Mat023 (Segundo semestre 2014)

2. f (t) = tn. En efecto, sabemos que et =∑∞

n=0tn

n!. Luego, para t > 0 , tn

n!< et. Por

tanto:

tn ≤ n! et

Teorema 4.1.1 (Existencia). Sea f : [0,+∞)→ R una funcion seccionalmente continua y

de orden exponencial β entonces la transformada de Laplace L [f (t)] (s) existe para s > β.

Demostracion. En primer lugar note que, como f : [0,+∞) → R es seccionalmente

continua, la funcion t 7→ f (t) e−st es localmente integrable en [0,+∞). Por otro lado, por

la desigualdad triangular tenemos que:∣∣∣∣∫ ∞0

f (t) e−st dt

∣∣∣∣ ≤ ∣∣∣∣∫ t0

0

f (t) e−st dt

∣∣∣∣+

∫ ∞t0

|f (t)| e−st dt

Note que∣∣∣∫ t00

f (t) e−st dt∣∣∣ <∞ y ademas, como f : [0,+∞)→ R es de orden exponencial,

existen constantes M,β > 0 y un valor t0 > 0 tales que:

|f (t)| ≤M eβt

para todo t ≥ t0. Luego:∫ ∞t0

|f (t)| e−st dt ≤M

∫ ∞t0

eβt e−st dt = M

∫ ∞t0

e(β−s)tdt =Me(β−s)

β − s

∣∣∣∞t0

en donde el ultimo termino converge si y solo si β − s < 0. Por tanto, como:∫ ∞0

f (t) e−st dt <

∣∣∣∣∫ ∞0

f (t) e−st dt

∣∣∣∣se tiene que Lf (t) (s) existe para cada s > β.

Observacion 4.1.3. La funcion f (t) = 2tet2

cos(et

2)

es continua en [0,+∞[ pero no de

orden exponencial sin embargo su transformada de Laplace existe para s > 0, por otro

lado la funcion g (t) = 1√t

no es seccionalmente continua en [0,+∞[ sin embargo posee

transformada de Laplace.

Teorema 4.1.2 (Linealidad de la transformada). Sean f, g : [0,+∞) → R dos funciones

localmente integrables y α ∈ R, entonces para cada s ∈ D (L [f ]) ∩D (L [g]) se tiene que:

L [αf + g] (s) = αL [f ] (s) + L [g] (s)

232

Page 235: Apuntes MAT-023 USM

Apuntes Mat023 (Segundo semestre 2014)

Calculo de transformadas

Observacion 4.2.1. En esta seccion haremos el calculo de transformadas de Laplace

basicas, para ası obtener una tabla elemental de transformadas.

Ejemplo 4.2.1. Transformadas de f (t) = sinωt y f (t) = cosωt.

Solucion. Integrando por partes obtenemos∫eαx cos (βx) dx =

α cos (βx) eαx

α2 + β2+β sin (βx) eαx

α2 + β2+ C∫

eαx sin (βx) dx = −β cos (βx) eαx

α2 + β2+α sin (βx) eαx

α2 + β2+ C

luego

L [sinωt] (s) =

∫ ∞0

sin (ωt) e−stdt

= −ω cos (ωt) e−st

s2 + ω2− s sin (ωt) e−st

s2 + ω2

∣∣∣∣+∞0

s2 + ω2

si s > 0. De manera similar

L [cosωt] (s) =

∫ ∞0

cos (ωt) e−stdt

=−s cos (ωt) e−st

s2 + ω2+ω sin (ωt) e−st

s2 + ω2

∣∣∣∣+∞0

=s

s2 + ω2

para s > 0.

Ejemplo 4.2.2. Usemos variable compleja para calcular las mismas transformadas: Por la

formula de Euler:

eiωt = cosωt+ i sinωt

y la linealidad de la transformada, se obtiene que:

cosωt =eiωt + e−iωt

2

y:

sinωt =eiωt − e−iωt

2i

233

Page 236: Apuntes MAT-023 USM

Apuntes Mat023 (Segundo semestre 2014)

Entonces:

L [cosωt] (s) = L[eiωt + e−iωt

2

](s)

=1

2

(L[eiωt]

(s) + L[e−iωt

](s))

=1

2

(1

s− iω+

1

s+ iω

)=

1

2

(s+ iω + s− iωs2 − (iω)2

)=

s

s2 + ω2

para s > 0. Analogamente, se obtiene que:

L [sinωt] (s) =ω

s2 + ω2, s > 0

Ejemplo 4.2.3. Transformadas de f (t) = sinhωt y f (t) = coshωt. Sabemos que:

coshωt =eωt + e−ωt

2∧ sinhωt =

eωt − eωt

2

Por la linealidad de la transformada de Laplace se obtiene que:

L [coshωt] (s) =s

s2 − ω2, s > |ω|

y:

L [sinhωt] (s) =ω

s2 − ω2, s > |ω|

Ejemplo 4.2.4. Transformada de f (t) = t. Notemos que:∫ ∞0

te−st dt = −1

s

∫ ∞0

t(e−st

)′dt

= −1

s

te−st

∣∣∣∞0−∫ ∞

0

e−st dt

= −1

s

0− 1

s

=

1

s2

para todo s > 0. Por tanto:

L [t] (s) =1

s2, s > 0

usando integracion por partes, es facil mostrar que

L [tn] (s) =n!

sn+1, s > 0

234

Page 237: Apuntes MAT-023 USM

Apuntes Mat023 (Segundo semestre 2014)

Observacion 4.2.2. Resumimos las funciones con sus respectivas transformadas de Laplace

y sus dominios en la siguiente tabla:

f (t) L [f (t)] (s) Dominio

11

ss > 0

t1

s2s > 0

eαt1

s− αs > α

cosωts

s2 + ω2s > 0

sinωtω

s2 + ω2s > 0

coshωts

s2 − ω2s > |ω|

sinhωtω

s2 − ω2s > |ω|

tnn!

sn+1s > 0

Ejemplo 4.2.5. Calcule la transformada de Laplace de la funcion:

f (t) = 3− 2t+ 4 cos 2t

Solucion. Por la linealidad de la transformada y la tabla anterior, se obtiene:

L [f (t)] (s) = L [3− 2t+ 4 cos 2t] (s)

= 3L [1] (s)− 2L [t] (s) + 4L [cos 2t] (s)

=3

s− 2

s2+

4s

s2 + 4

Ejemplo 4.2.6. si f (t) = a0 + a1t+ · · ·+ antn entonces

L [f (t)] (s) =n∑k=0

akL[tk]

(s)

=n∑k=0

akk!

sk+1

Teorema 4.2.1. Si f es seccionalmente continua en [0,+∞) y de orden exponencial β

entonces

lıms→+∞

L [f (t)] (s) = 0

235

Page 238: Apuntes MAT-023 USM

Apuntes Mat023 (Segundo semestre 2014)

El resultado anterior se debe a la desigualdad∣∣∣∣∫ ∞0

e−stf (t) dt

∣∣∣∣ ≤ M

s− βtomando s→ +∞ se obtiene el resultado.

Observacion 4.2.3. Las funciones s−1s+1

, es

sno pueden ser transformadas de una funcion

seccionalmente continua de orden exponencial.

Primer Teorema de la Traslacion

Observacion 4.3.1. El siguiente resultado nos permite calcular la transformada de Laplace

de eαtf (t) cuando es conocida la transformada de Laplace de f (t), en este resultado se

pone de manifiesto la primera propiedad de la traslacion de la transformada de Laplace.

En efecto, tenemos el siguiente teorema:

Teorema 4.3.1. Suponga que F (s) = L [f (t)] (s) existe para cada s > β. Si α ∈ R,

entonces:

L[eαt f (t)

](s) = F (s− α)

para todo s > α + β.

Ejemplo 4.3.1. Calcule L [e−3t cos 2t] (s)

Solucion. Notamos que F (s) = L [cos 2t] (s) = ss2+4

. Luego:

L[e−3t sin 2t

](s) = F (s+ 3) =

s+ 3

(s+ 3)2 + 4

Observacion 4.3.2. La primera propiedad de traslacion de la transformada de Laplace,

deja nuestra tabla basica de transformadas como sigue:

f (t) L [f (t)] (s) Dominio

eαttnn!

(s− α)n+1 s > α

eαt cosωts− α

(s− α)2 + ω2s > α

eαt sinωtω

(s− α)2 + ω2s > α

eαt coshωts− α

(s− α)2 − ω2s > α + |ω|

eαt sinhωtω

(s− α)2 − ω2s > α + |ω|

236

Page 239: Apuntes MAT-023 USM

Apuntes Mat023 (Segundo semestre 2014)

Transformada de la derivada

Observacion 4.4.1. El origen y el objetivo de la transformada de Laplace es la resolucion

de ecuaciones diferenciales. Para lograr lo anterior, debemos calcular la transformada de

Laplace de la derivada de una funcion. Entonces, sea y = y (t) una funcion derivable, note

que: ∫ ∞0

y′ (t) e−st dt = y (t) e−st∣∣∣∞0−∫ ∞

0

y (t)(e−st

)′dt

= y (0) + s

∫ ∞0

y (t) e−st dt

bajo el supuesto que:

lımt→∞

y (t) e−st = 0

Ası, si y = y (t) es una funcion derivable y de orden exponencial, se tiene que y′ (t) existe

para cada t ≥ 0 y:

|y (t)| ≤Meβt, t ≥ t0

Entonces:

−Me(β−s)t ≤ y (t) e−st ≤Me(β−s)t

donde los extremos convergen a cero si β − s < 0, cuando t→∞. Ası, bajo las condiciones

anteriores se obtiene que:

L [y′ (t)] (s) = sL [y (t)] (s)− y(0+)

Por tanto, tenemos el siguiente teorema:

Teorema 4.4.1. Suponga que f es continua en ]0,∞[ y de orden exponencial β, supongamos

ademas que f ′ es seccionalmente continua en [0,+∞) entonces:

L [f ′ (t)] (s) = sL [f (t)] (s)− f(0+)

Corolario 4.4.1. Suponga que f (t) , f ′ (t) , · · · , f (n−1) (t) son continuas en ]0,∞[ y de

orden exponencial, suponga tambien que f (n) (t) es seccionalmente continua en [0,+∞).

Entonces

L[f (n) (t)

](s) = snL [f (t)] (s)− sn−1f

(0+)− sn−2f ′

(0+)− · · · − f (n−1)

(0+)

Ejemplo 4.4.1. Calcule la transformada de Laplace de f (t) = tn, para n ≥ 2.

Solucion. Note que:

f ′ (t) = ntn−1

237

Page 240: Apuntes MAT-023 USM

Apuntes Mat023 (Segundo semestre 2014)

ası, por la transformada de la derivada, se tiene que:

L[ntn−1

](s) = L [f ′ (t)] (s) = sL [f (t)] (s)− f

(0+)

= sL [tn] (s)

ası:

L [tn] (s) =n

sL[tn−1

](s) , n ≥ 2

entonces:

L [tn] (s) =

1s2

n = 1nsL [tn−1] (s) n > 1

s > 0

se sigue:

L [tn] (s) =n!

sn+1, s > 0

Ejemplo 4.4.2. Como aplicacion de la transformada de la derivada, calcular la transfor-

mada de Laplace de sin2 (αt)

Solucion. Derivando

f ′ (t) = 2α sin (αt) cos (αt)

= α sin (2αt)

luego

sL [f (t)] (s)− f (0) = L [α sin (2αt)] (s)

ası

sL [f (t)] (s) =2α2

s2 + 4α2

finalmente

L [f (t)] (s) =2α2

s (s2 + 4α2)

Ejemplo 4.4.3. Sea f : [0,+∞)→ R una funcion continua. Se define:

F (t) =

∫ t

0

f (u) du

Calcule L [F (t)] (s).

Solucion. Como f es continua, por el Teorema Fundamental del Calculo, F es derivable

con:

F ′ (t) = f (t)

Entonces, por la transformada de la derivada tenemos que:

L (F ′ (t)) (s) = sL [F (t)] (s)− F(0+)

Ası:

L[∫ t

0

f (t) dt

](s) = L [F (t)] (s) =

1

sL [f (t)] (s)

pues F (0) = 0.

238

Page 241: Apuntes MAT-023 USM

Apuntes Mat023 (Segundo semestre 2014)

Teorema 4.4.2. Sea n ∈ N, suponga que f es seccionalmente continua en [0,+∞) de

orden exponencial β y que F (s) = L [f (t)] (s) entonces:

L [tn f (t)] (s) = (−1)ndn

dsnF (s) (4.1)

para s > β.

Observacion 4.4.2. El siguiente teorema, habla sobre la inyectividad de la transformacion

de Laplace, lo que nos permite hablar de inversa.

Teorema 4.4.3. Dos funciones con la misma transformada de Laplace no pueden diferir

en todo un intervalo de longitud positiva. Es decir, si L [f (t)] (s) = L [g (t)] (s) entonces

f (t) = g (t), para cada t tal que f y g simultaneamente sean continuas.

Observacion 4.4.3. Funciones continuas diferentes tienen transformadas de Laplace

diferentes.

Definicion 4.4.1. Sea F : [0,+∞)→ R una funcion tal que lıms→∞ F (s) = 0. Diremos

que una funcion f : [0,+∞)→ R, para la cual existe su transformada de Laplace, es una

transformada de Laplace inversa de F si satisface:

L [f (t)] (s) = F (s)

Por el Teorema de Lerch, tal transformada inversa esta unicamente determinada solo sobre

sus puntos de continuidad. Abusando del lenguaje, se escribe:

f (t) = L−1 [F (s)] (t)

Observacion 4.4.4. El operador L−1 tambien es lineal. Es decir:

L−1 [αf + g] = αL−1 [f ] + L−1 [g]

Ejemplo 4.4.4. Calcule L−1[

s+9s2+6s+13

](t).

Solucion. Observe que:

s+ 9

s2 + 6s+ 13=

s+ 9

(s+ 3)2 + 4

=s+ 3

(s+ 3)2 + 4+

6

(s+ 3)2 + 4

Entonces:

L−1

[s+ 9

s2 + 6s+ 13

](t) = L−1

[s+ 3

(s+ 3)2 + 4

](t) + 3L−1

[2

(s+ 3)2 + 4

](t)

= e−3t cos 2t+ 3e−3t sin 2t

239

Page 242: Apuntes MAT-023 USM

Apuntes Mat023 (Segundo semestre 2014)

Ejemplo 4.4.5. Calcular

L−1

[s

(s+ 1) (s+ 2) (s+ 3)

](s)

Solucion. Usando la tecnica de fracciones parciales

s

(s+ 1) (s+ 2) (s+ 3)=

2

s+ 2− 1

2 (s+ 1)− 3

2 (s+ 3)

luego

L−1

[s

(s+ 1) (s+ 2) (s+ 3)

](s)

= L−1

[2

s+ 2− 1

2 (s+ 1)− 3

2 (s+ 3)

](s)

= 2e−2t − 1

2e−t − 3

2e−3t

Ejemplo 4.4.6. Calcular

L−1

[ln

(s+ 1

s+ 2

)](s)

Solucion. Queremos determinar f (t) tal que

L [f (t)] (s) = ln

(s+ 1

s+ 2

)notamos que

L [f (t)] (s) = ln (s+ 1)− ln (s+ 2)

derivando

L [f (t)]′ (s) =1

s+ 1− 1

s+ 2pero

L [f (t)]′ (s) = L [−tf (t)] (s)

se sigue

L [−tf (t)] (s) = L[e−t − e−2t

](s)

ası

−tf (t) = e−t − e−2t

de donde obtenemos

f (t) =e−2t − e−t

t

Ahora veremos algunos ejemplos de como funciona el metodo de la transformada de

Laplace para resolver ecuaciones:

240

Page 243: Apuntes MAT-023 USM

Apuntes Mat023 (Segundo semestre 2014)

Ejemplo 4.4.7. Resuelva la ecuacion diferencial:

ty′′ − ty′ − y = 0, y (0) = 0, y′ (0) = 3 (4.2)

Solucion. Sea Y (s) = L [ y (t)] (s). Aplicando la transformada de Laplace a la ecuacion

(4.2), obtenemos:

L [ty′′] (s)− L [ty′] (s)− L [y] (s) = 0

pero:

L [ty′′] (s) = − d

ds

s2L [y] (s)− sy (0)− y′ (0)

= −s2Y ′ − 2sY

y

L [ty′] (s) = − d

dssL [y] (s)− y (0) = −sY ′ − Y

reemplazando estas expresiones en la ecuacion original obtenemos:

−s2Y ′ − 2sY + sY ′ + Y − Y = 0

rs decir:

s (s− 1)Y ′ + 2sY = 0

entonces:

Y ′ +2

s− 1Y = 0

la cual es una ecuacion de variable separable. Separando, entonces, las variables, obtenemos:

dY

Y= − 2

s− 1ds

de donde se sigue que:

Y (s) =C

(s− 1)2

se sigue que

L [y (t)] (s) = Y (s) =C

(s− 1)2 = L[Ctet

](s)

usando el teorema de Lerch, se sigue

y (t) = Ctet

(note que se busca una funcion derivable, luego continua). Ahora bien, como y′ (0) = 3, se

obtiene que C = 3. Por tanto, la solucion de la ecuacion diferencial es:

y (t) = 3tet

241

Page 244: Apuntes MAT-023 USM

Apuntes Mat023 (Segundo semestre 2014)

Ejemplo 4.4.8. Resolver el P.V.I.

ty′′ − ty′ + y = 2(et − 1

)y (0) = 0

y′ (0) = −1

Solucion. Aplicando la transformada de Laplace

− d

ds

(s2Ly − sy (0)− y′ (0)

)+

d

ds(sLy − y (0)) + Ly = 2

(1

s− 1− 1

s

)reemplazando las C.I.

− d

ds

(s2Ly + 1

)+

d

ds(sLy) + Ly =

2

s (s− 1)

se sigue

−(2sLy + s2L′y

)+(Ly + sL′y

)+ Ly =

2

s (s− 1)⇔(

2sLy + s2L′y)−(Ly + sL′y

)− Ly =

−2

s (s− 1)⇔(

s2L′y − sL′y)

+ 2sLy − 2Ly =−2

s (s− 1)⇔

L′y(s2 − s

)+ 2 (s− 1)Ly =

−2

s (s− 1)

se sigue

s (s− 1)L′y + 2 (s− 1)Ly =−2

s (s− 1)⇔

L′y +2

sLy =

−2

s2 (s− 1)2

es una EDO lineal con factor integrante µ (s) = e∫

2sds = e2 ln s = s2

d

ds

(s2Ly

)=

−2

(s− 1)2

integrando

s2Ly =

∫−2

(s− 1)2ds =2

s− 1+ C

se sigue

Ly =2

(s− 1) s2+C

s2

=2

s− 1− 2

s− 2

s2+C

s2

242

Page 245: Apuntes MAT-023 USM

Apuntes Mat023 (Segundo semestre 2014)

ası

y (t) = 2et − 2− 2t+ Ct

como y (0) = 0 e y′ (0) = −1 se sigue

−1 = C

ası

y (t) = 2et − 2− 3t

Ejemplo 4.4.9. Utilizando la transformada de Laplace, resuelva la siguiente ecuacion

diferencial:

y′′ − 4y = 0, y (0) = 1, y′ (0) = 2

Solucion. Aplicando la transformada de Laplace

s2L [y] (s)− sy (0)− y′ (0)− 4L [y] (s) = 0

reemplazando los valores iniciales(s2 − 4

)L [y] (s) = s+ 2

ası

L [y] (s) =s+ 2

s2 − 4=

1

s− 2= L

[e2t]

(s)

finalmente la solucion de la ecuacion es:

y (t) = e2t

Funciones escalonadas y Segundo Teorema de la Traslacion

Observacion 4.5.1. Cuando se estudio el movimiento vibratorio en el caso mas general,

se obtuvo una ecuacion diferencial de la forma:

x′′ +c

mx′ +

k

mx = F (t)

para constantes fısicas c, k y m. La funcion forzadora F (t), que actuaba como fuerza

externa al sistema, se supuso continua. En las aplicaciones, no siempre se puede pedir tal

condicion; muchas veces, tal funcion es una funcion escalonada o con discontinuidades de

salto.

Definicion 4.5.1. La funcion de Heaviside con salto en 0 se define como la funcion:

H (t) =

1 t ≥ 0

0 t < 0

La funcion de Heaviside con salto en a > 0 esta dada por Ha (t) = H (t− a), con

t ∈ R.

243

Page 246: Apuntes MAT-023 USM

Apuntes Mat023 (Segundo semestre 2014)

Observacion 4.5.2. Para las funciones anteriores, se tiene:

L [H (t)] (s) =

∫ ∞0

H (t) e−st dt =1

s, s > 0

y para todo a > 0:

L [Ha (t)] (s) =

∫ ∞0

Ha (t) e−st dt

=

∫ ∞a

e−st dt

=e−as

s

para s > 0.

Observacion 4.5.3. Note que si a < b la funcion:

f (t) = Ha (t)−Hb (t)

corresponde a la funcion

f (t) =

1 t ∈ [a, b[

0 t ∈ [a, b[c

Observacion 4.5.4. Para introducir la segunda propiedad de traslacion de la transformada

de Laplace, requerimos truncar una funcion. Mas precisamente, consideremos f : R→ Runa funcion y a > 0. Entonces, Ha fa : R→ R es la funcion definida como:

(Ha fa) (t) =

f (t− a) t ≥ a

0 t < a

Note que (Ha fa) (t) = f (t− a)H (t− a).

Tenemos ası el siguiente teorema, que muestra que multiplicar por una funcion escalon

unitario tiene el efecto de multiplicar su transformada por una funcion exponencial:

Teorema 4.5.1 (Segundo Teorema de la Traslacion). Sea a > 0 y f una funcion con

transformada de Laplace, entonces:

L [(Ha fa) (t)] (s) = e−asL [f (t)] (s)

Ejemplo 4.5.1. L [sin a (t− b)H (t− b)] (s) = e−bsL [sin at] (s) = ae−bs

s2+a2.

Ejemplo 4.5.2. Calcule L [f (t)] (s), si:

f (t) =

et 0 ≤ t < 2π

et + cos t t > 2π

244

Page 247: Apuntes MAT-023 USM

Apuntes Mat023 (Segundo semestre 2014)

Solucion. Note que

f (t) = et +H (t− 2π) (cos t)

= et +H (t− 2π) (cos (t− 2π + 2π))

= et +H (t− 2π) (cos (t− 2π))

ası

L [f (t)] (s) =1

s− 1+se−2πs

s2 + 1

Ejemplo 4.5.3. Calcule:

L−1

[1− e−πs/2

1 + s2

](t)

Solucion. Notamos que

1− e−πs/2

1 + s2=

1

s2 + 1− e−πs/2 1

s2 + 1

= L [sin t] (s)− e−πs/2L [sin t] (s)

= L [sin t] (s)− L[H(t− π

2

)sin(t− π

2

)](s)

= L[sin t−H

(t− π

2

)sin(t− π

2

)](s)

luego

L−1

[1− e−πs/2

1 + s2

](t) = sin t+H

(t− π

2

)cos t

Ejemplo 4.5.4. Resolver el P.V.I. dado por

y′′ − 2y = f (t) , y (0) = y′ (0) = 0

donde

f (t) =

t 0 ≤ t < 1

t2 t > 1

Solucion. Usando la funcion salto podemos escribir la ecuacion como

y′′ − 2y = t+H (t− 1)(t2 − t

)para poder usar el segundo teorema de la traslacion manipulamos la funcion

t2 − t = (t− 1 + 1)2 − (t− 1 + 1)

= (t− 1)2 + 2 (t− 1) + 1− (t− 1)− 1

= (t− 1)2 + (t− 1)

245

Page 248: Apuntes MAT-023 USM

Apuntes Mat023 (Segundo semestre 2014)

ası

y′′ − 2y = t+H (t− 1) (t− 1)2 +H (t− 1) (t− 1)

se sigue (s2 − 2

)L [y] (s) =

1

s2+ e−s

2

s3+ e−s

1

s2

luego

L [y] (s) =1

s2 (s2 − 2)+ e−s

2

s3 (s2 − 2)+ e−s

1

s2 (s2 − 2)

usando fracciones parciales

1

s2 (s2 − 2)=

1

2

(1

s2 − 2− 1

s2

)y

2

s3 (s2 − 2)=

1

2

s

s2 − 2− 1

2s− 1

s3

luego

1

s2 (s2 − 2)=

1

2

(1

s2 − 2− 1

s2

)= L

[1

2√

2sinh

(√2t)− t

2

](s)

y

2

s3 (s2 − 2)=

1

2

s

s2 − 2− 1

2s− 1

s3

= L[

1

2cosh

(√2t)− 1

2− t2

2

](s)

se sigue

L [y] (s) =1

s2 (s2 − 2)+ e−s

2

s3 (s2 − 2)+ e−s

1

s2 (s2 − 2)

= L[

1

2√

2sinh

(√2t)− t

2

](s)

+e−sL[

1

2cosh

(√2t)− 1

2− t2

2

](s)

+e−sL[

1

2√

2sinh

(√2t)− t

2

](s)

= L[

1

2√

2sinh

(√2t)− t

2

](s)

+L

[H (t− 1)

(1

2cosh

(√2 (t− 1)

)− 1

2− (t− 1)2

2

)](s)

+L[H (t− 1)

(1

2√

2sinh

(√2 (t− 1)

)− (t− 1)

2

)](s)

246

Page 249: Apuntes MAT-023 USM

Apuntes Mat023 (Segundo semestre 2014)

la solucion es entonces

y (t) =1

2√

2sinh

(√2t)− t

2

+H (t− 1)

(1

2cosh

(√2 (t− 1)

)− 1

2− (t− 1)2

2+

1

2√

2sinh

(√2 (t− 1)

)− (t− 1)

2

)

La Transformada de integrales de convolucion

Definicion 4.6.1. Sean f y g funciones seccionalmente continuas en [0,+∞). Se define la

convolucion de f y g como la funcion:

(f ∗ g) (t) =

∫ t

0

f (t− u) g (u) du

Ejemplo 4.6.1. La convolucion entre f (t) = t y g (t) = sin t, esta dada por:

t ∗ sin t =

∫ t

0

(t− u) sinu du

= t

∫ t

0

sinu du−∫ t

0

u sinu du

= t− t cos t− sin t+ t cos t

= t− sin t

Ejemplo 4.6.2. Calcular t ∗ t2

Solucion.

t ∗ t2 =

∫ t

0

(t− u)u2du

=1

12t4

Observacion 4.6.1. Dentro de las propiedades algebraicas del producto de convolucion,

destacaremos la propiedad de conmutatividad. En efecto, observe que:

(f ∗ g) (t) =

∫ t

0

f (t− u) g (u) du

= −∫ 0

t

f (z) g (t− z) dz ←− z = t− u

=

∫ t

0

g (t− z) f (z) dz

= (g ∗ f) (t)

otras de sus propiedades basicas son:

247

Page 250: Apuntes MAT-023 USM

Apuntes Mat023 (Segundo semestre 2014)

1. α (f ∗ g) = (αf) ∗ g = f ∗ (αg) para α constante.

2. f ∗ (g ∗ h) = (f ∗ g) ∗ h

3. f ∗ (g + h) = (f ∗ g) + (f ∗ h)

Teorema 4.6.1. Sean f y g funciones seccionalmente continuas en [0,+∞) y de orden

exponencial β entonces:

L [(f ∗ g) (t)] (s) = L [f (t)] (s) · L [g (t)] (s)

o bien

L−1 [L [f ] (s)L [g] (s)] (t) = (f ∗ g) (t)

Observacion 4.6.2. El resultado anterior, desde una perspectiva practica se utiliza

diciendo que la transformada de Laplace inversa de un producto de transformadas es la

convolucion.

Ejemplo 4.6.3. Calcule:

L−1

s+ 1

(s− 2)2 ((s+ 1)2 + 32)

Solucion. Note que:

s+ 1

(s− 2)2 ((s+ 1)2 + 32) =

1

(s− 2)2 ·s+ 1

(s+ 1)2 + 32

Luego:

L−1

[s+ 1

(s− 2)2 ((s+ 1)2 + 32)] (t) = L−1

[1

(s− 2)2

](t) ∗ L−1

[s+ 1

(s+ 1)2 + 32

](s)

= te2t ∗ e−t cos (3t)

=

∫ t

0

(t− u) e2(t−u)e−u cos (3u) du

=1

6te2t − 1

18e−t sin 3t

Ejemplo 4.6.4. Calcular

L−1

1

(s− 2) (s+ 2) (s− 1)

(t)

248

Page 251: Apuntes MAT-023 USM

Apuntes Mat023 (Segundo semestre 2014)

Solucion. Esto lo podemos calcular con fracciones parciales pero

1

(s− 2) (s+ 2) (s− 1)=

1

s− 2

1

s+ 2

1

s− 1

= L[e2t ∗ e−2t ∗ et

](s)

donde

e2t ∗ e−2t =

∫ t

0

e2(t−u)e−2udu =1

4e2t − e−2t

4y

e2t ∗ e−2t ∗ et =

∫ t

0

et−u(

1

4e2u − e−2u

4

)du

=e−2t

12− 1

3et +

1

4e2t

se sigue

L−1

1

(s− 2) (s+ 2) (s− 1)

(t) =

e−2t

12− 1

3et +

1

4e2t

Definicion 4.6.2. Sean f (t) , g (t) funciones continuas. Llamaremos ecuacion de Volte-

rra a una ecuacion de la forma:

x (t) = f (t) +

∫ t

0

g (t− u)x (u) du

Observacion 4.6.3. Note que, mediante el producto de convolucion, podemos escribir la

ecuacion anterior como:

x (t) = f (t) + (g ∗ x) (t)

Ahora bien, si ponemos X (s) = L [x (t)] (s) , F (s) = L [f (t)] (s) y G (s) = L [g (t)] (s) y

aplicamos luego la transformada de Laplace a la ecuacion anterior, obtenemos:

X (s) = F (s) +G (s)X (s)

de donde se obtiene finalmente:

X (s) =F (s)

1−G (s)

de donde podemos obtener

x (t) = L−1

[F (s)

1−G (s)

](t)

Ejemplo 4.6.5. Resuelva la ecuacion integral:

x (t) = t2 +

∫ t

0

sin (t− u)x (u) du

249

Page 252: Apuntes MAT-023 USM

Apuntes Mat023 (Segundo semestre 2014)

Solucion. Supongamos que X (s) = L [x (t)] (s). Entonces, aplicando la transformada de

Laplace a la ecuacion anterior, tenemos:

X (s) =2

s3+

1

s2 + 1X (s)

Es decir:

X (s) =2

s3+

2

s5

=2

s3+

2

4!

4!

s5

Por lo tanto:

x (s) = L−1

[2

s3

](t) +

2

4!L−1

[4!

s5

](t)

= t2 +t4

12

Ejemplo 4.6.6. Resolver el sistema de ecuaciones

x′ = x− y + t

y′ = x+ y + et

con las condiciones iniciales x (0) = y (0) = 0.

Solucion. Pongamos X (s) = L [x (t)] (s) y Y (s) = L [y (t)] (s) entonces

sX = X − Y +1

s2

sY = X + Y +1

s− 1

luego

X =2s− 1

2s2 − 4s3 + 3s4 − s5

=s− 1

2

s2 − 2s+ 2− 1

s− 1− 1

2s2

Y =s2 + 1

2s2 − 2s3 + s4

=1

2s−

12s− 3

2

s2 − 2s+ 2+

1

2s2

de donde obtenemos

x (t) = et cos t+1

2et sin t− et − 1

2t

y (t) =1

2t+

1

2− 1

2et (cos t− 2 sin t)

250

Page 253: Apuntes MAT-023 USM

Apuntes Mat023 (Segundo semestre 2014)

Ejemplo 4.6.7. Resolver el problema

d2y

dt2= t+ 2

∫ t

0

e−(t−u)y (u) du

y (0) = 0

y′ (0) = 0

Solucion. Aplicando la transformada de Laplace

s2L [y] (s) =1

s2+ 2

1

s+ 1L [y] (s)

luego (s2 − 2

s+ 1

)L [y] (s) =

1

s2

despejando

L [y] (s) =1s2(

s2 − 2s+1

) =1

s2

s+ 1

s3 + s2 − 2

=s+ 1

s2 (s− 1) (2s+ s2 + 2)

hacemos fracciones parciales

s+ 1

s2 (s− 1) (2s+ s2 + 2)

=2

5 (s− 1)+

110s+ 3

10

s2 + 2s+ 2− 1

2s− 1

2s2

y reconocemos las transformadas

2

5 (s− 1)= L

[2

5et]

(s)

110s+ 3

10

s2 + 2s+ 2= L

[1

10e−t (cos t+ 2 sin t)

](s)

− 1

2s− 1

2s2= L

[−1

2t− 1

2

](s)

se sigue por Lerch

y (t) =2

5et +

1

10e−t (cos t+ 2 sin t)− 1

2t− 1

2

otra forma es usar convoluciones, note que

s+ 1

s2 (s− 1) (2s+ s2 + 2)=

1

s2

1

(s− 1)

s+ 1

(s+ 1)2 + 1

= L [t] (s)L[et]

(s)L[e−t cos t

](s)

= L[(t ∗ et

)∗(e−t cos t

)](s)

251

Page 254: Apuntes MAT-023 USM

Apuntes Mat023 (Segundo semestre 2014)

de donde

y (t) =(t ∗ et

)∗(e−t cos t

)calculando

1

s2 (s− 1)= L [t] (s)L

[et]

(s)

= L[t ∗ et

](s)

ası

t ∗ et =

∫ t

0

(t− u) eudu

= et − t− 1

luego1

s2 (s− 1)

s+ 1

(s+ 1)2 + 1= L

[et − t− 1

](s)L

[e−t cos t

](s)

ası

y (t) =

∫ t

0

(et−u − t+ u− 1

)e−u cosudu

=2

5et − 1

2t+

1

10(cos t) e−t +

1

5(sin t) e−t − 1

2

Observacion 4.6.4. Como apendice calcularemos la transformada de la “funcion” delta

de Dirac. Muy informalmente, la funcion delta de Dirac se describe como una funcion que

es cero en todas partes excepto en t = 0 y cumple ademas con:∫ ∞−∞

δ (t) dt = 1

Claramente, ninguna funcion real puede satisfacer tales condiciones. Sin embargo, considere

la familia de funciones:

fε (t) =

12ε

, −ε < t < ε

0 , |t| > ε

Note que, para cada ε > 0 las funciones fε (t) son seccionalmente continuas, y ademas:∫ ∞−∞

fε (t) dt =

∫ ε

−εfε (t) dt = 1

De este modo, podemos pensar a δ como:

δ (t) = lımε→0+

fε (t)

252

Page 255: Apuntes MAT-023 USM

Apuntes Mat023 (Segundo semestre 2014)

Definicion 4.6.3. Sea a > 0. La funcion delta de Dirac con salto en t = a se define

como:

δa (t) = δ (t− a)

Observacion 4.6.5. Notemos que

fε (t− a) =1

2εH (t− a+ ε)− 1

2εH (t− a− ε)

ası

L [fε (t− a)] (s) =e(−a+ε)s − e−(ε+a)s

2εsse sigue

L [δ (t− a)] (s) = lımε→0L [fε (t− a)] (s)

= e−as lımε→0

eεs − e−εs

2εs

= e−as lımε→0

seεs + se−εs

2s= e−as

Teorema 4.6.2. Sea a > 0, entonces:

L [δa (t)] (s) = e−as s > 0

y

L [δ (t)] (s) = 1

Ejemplo 4.6.8. Resolver el P.V.I.

y′′ − y = δ2 (t)

y (0) = 0

y′ (0) = 0

Solucion. Aplicando la transformada de Laplace(s2 − 1

)L [y] (s) = e−2s

ası

L [y] (s) =e−2s

s2 − 1= e−2sL [sinh (t)] (s)

= L [H (t− 2) sinh (t− 2)] (s)

ası

y (t) = H (t− 2) sinh (t− 2)

253

Page 256: Apuntes MAT-023 USM

Apuntes Mat023 (Segundo semestre 2014)

Observacion 4.6.6. Una aplicacion interesante es estudiar la ecuacion diferencial:

LdI

dt+RI +

Q

C= δa (t)

que es la ecuacion diferencial asociada a un circuito RLC en serie. Es decir, un circuito

electrico compuesto por una resistencia R [Ω], un capacitor C [F], un inductor L [H] y

una fuente de voltaje modelada por la funcion delta de Dirac como funcion forzadora.

Naturalmente, las herramientas de las ecuaciones diferenciales a coeficientes constantes no

bastan para resolver este sistema.

254

Page 257: Apuntes MAT-023 USM

Apuntes Mat023 (Segundo semestre 2014)

Ejercicios del capıtulo

1. Sean α, β > 0. Encontrar la transformada de Laplace de:

1) sin (t+ α) + cos2 (βt) 2) (αt+ β) e5t + t cos(

2t+π

4

)3) t2 cos t+ 2e−3t sin (2t) 4) t3e2t cos (6t) + 7e−4t sin t cos2 t

5) te2t

∫ t

0

u

(d

due3u cosu

)du 6) t2

∫ t

3

cos2 (u) du

7) f (t) =

t si t < 2

0 si t > 28) f (t) =

cos t si 0 ≤ t < 2π

sin t si t > 2π

9) f (t) =

cos t si 0 ≤ t ≤ 2π

sin t si 2π < t ≤ 4π

et si 4π < t

10)

∫ t

0

sinh [2 (t− u)] cosh (2u) du

11)

∫ t

0

(t− u)3 sinu du 12)

∫ t

0

eu−t cos (3u) du

2. Si ω > 0, calcular las transformadas de:

1)sinωt

t2)

∫ t

0

sin (ωu)

udu 3)

cosωt− 1

t

4)1− coshωt

t5)

∫ t

0

1− cos (ωu)

udu 6)

∫ t

0

1− cosh (ωu)

udu

7) erf (t) =2√π

∫ t

0

e−u2

du 8)∞∑n=0

H (t− nω)

255

Page 258: Apuntes MAT-023 USM

Apuntes Mat023 (Segundo semestre 2014)

3. Encontrar la transformada inversa de las siguientes funciones:

1)2

s (s+ 1)2)

3

(s+ 1)3 3)s+ 1

s (s+ 4)2

4)2s

s (s− 7)5)

s

s2 + 6s+ 256)

2s

(s2 + 4)2

7)e−s

(s2 + 1)2 8)e−3s

(s− 1) (s+ 2)9)

1

s3 + 23

10)s2

s3 + 2311)

n∑k=1

e−ks

s12) ln

(s2 + 1

s (s− 3)

)

13) e−2s

(1

s2+

1

s4

)14) arctan

(1

s

)15)

e−πss

s2 + 3

16)e−3s

s2 − 917)

s

(s2 + 3) (s2 + 1)18)

1

sarctan

(1

s

)

19)1

(s2 + 1)3 20) ln

(s

s+ 1

)21) ln

(1 +

52

s2

)

4. Suponga que α, β ∈ R, n,m ∈ N. Calcular explıcitamente las siguientes convoluciones:

a) exp (αt) ∗ exp (βt)

b) tn ∗ exp (αt)

c) tn ∗ tm

d) exp (αt) ∗ sin (βt)

e) exp (αt) ∗ cos (βt)

f ) sin (αt) ∗ sin (βt)

g) sin (αt) ∗ cos (βt)

h) cos (αt) ∗ cos (βt)

i) tn ∗ cos (αt)

j ) tn ∗ sin (αt)

5. Escriba las transformadas inversas de las siguientes funciones en terminos de convo-

luciones:

256

Page 259: Apuntes MAT-023 USM

Apuntes Mat023 (Segundo semestre 2014)

a)s3

(s2 + 4) (s2 + 9) (s2 + 16)

b)s

(s− 2) (s+ 2) (s− 1)

c) ln

(s+ 1

s+ 2

)ln

(1 +

1

s2

)arctan

(1

s

)6. Resuelva las siguientes ecuaciones integrales o integro-diferenciales.

a) x (t) = e−t − 2

∫ t

0

cos (t− u)x (u) du

b) x (t) = t+1

6

∫ t

0

(t− u)3 x (u) du

c) x (t) = t+

∫ t

0

sin (t− u)x (u) du

d) x (t) = sin t+

∫ t

0

cos (t− u)x (u) du

e) x′ + 2x+

∫ t

0

x (u) du = f (t) donde

f (t) =

cos t si 0 ≤ t < 2π

sin t si t > 2π

con la condicion inicial x (0) = 1

f )

∫ t

0

sin (t− u) y (u) du = y (t) + et cos (3t)

7. Suponga que f, f ′ : [0,+∞[ → R son localmente integrables y de orden exponen-

cial: Calcular la transformada de

∫ t

0

(∫ u

0

f (v) dv

)du y tet

df

dten terminos de la

transformada de f .

8. La funcion Gamma esta definida por:

Γ (x) =

∫ ∞0

e−uux−1du, x > 0

a) Demuestre que Γ (x+ 1) =∫∞

0e−uux dx.

b) Integrando por partes, muestre que:

Γ (x+ 1) = xΓ (x)

c) Demuestre que Γ (1) = 1

257

Page 260: Apuntes MAT-023 USM

Apuntes Mat023 (Segundo semestre 2014)

d) Usando los resultados de (b) y (c), pruebe que si n es un entero positivo,

entonces:

Γ (n+ 1) = n!

e) Calcule, usando la sustitucion u = st en la parte (a), demuestre que:

L [tx] (s) =Γ (x+ 1)

sx+1, s > 0, x > −1

f ) Se sabe que Γ(

12

)=√π. Calcule:

1) L

1√t

2) L

√t

3) Lt5/2

9. Resuelva las siguientes ecuaciones diferenciales mediante Transformada de Laplace:

a) y′′ + y = 0, y (0) = 1, y′ (0) = 3

b) y′′ − 4y = 0, y (0) = 2, y′ (0) = −5

c) y′′ + 4y′ + 4y = 0, y (0) = −1, y′ (0) = 2

d) y′′ + 2y′ + 2y = H (t− 3) , y (0) = y′ (0) = 0

e) y′′′ + y = 0, y (0) = y′′ (0) = 1, y′ (0) = −1

f ) y(4) − y = 0, y (0) = y′′ (0) = 0, y′ (0) = y′′′ (0) = 1

g) y(4) − y = sinh t, y (0) = y′′ (0) = 0, y′ (0) = y′′′ (0) = 1

10. Resolver los siguientes problemas de valores iniciales

a) y′′ − 4y = f (t) donde

f (t) =

t2 si 0 ≤ t < 1

1− t si 1 < t < 3

et 3 < t

con las condiciones iniciales y (0) = 0, y′ (0) = 1.

b) y′′ + 4y′ + 13y = et cos t con valores iniciales y (0) = 1, y′ (0) = 0.

c) y′′ + y = 1 con y (1) = y′ (1) = 1.

d)

y′′ + y = 3 sin (4t)−H (t− 2π) sin (8t)

y (0) = 1

y′ (0) = −1

258

Page 261: Apuntes MAT-023 USM

Apuntes Mat023 (Segundo semestre 2014)

e)

y′′ + 3y′ + 2y = 3e4tH (t− 2)

y (0) = 2

y′ (0) = 1

f )

ty′′ + 2 (t− 1) y′ − 2y = 0

y (0) = 0

y′ (0) = 0

g)

ty′′ + 2 (t− 1) y′ + (t− 2) y = 0

y (0) = 1

y′ (0) = −1

h)

ty′′ − ty′ + y = 0

y (0) = 0

y′ (0) = 1

11. Resolver el siguiente sistema utilizando transformada de Laplace:

2x′1 + x1 + x′′2 = e6t

2x1 + x′2 = 0

donde x1 (0) = 1, x2 (0) = 2 y x′2 (0) = −2.

12. Resolver los siguientes sistemas utilizando transformada de Laplace:

1)

x′ − 4x− y + t = 0

y′ + 2x− y + et = 0con x (0) = 0, y (0) = 1

2)

x′ + x+ y′ + cos t = 0

y′ + x− y + et = 0con x (0) = 0, y (0) = 0

3)

x′ + x+ y = cos t

y′ + x− y = sin tcon x (0) = 1, y (0) = 2

4)

x′′ + y′′ + y′ = t2

x′′ − y′′ = 2tcon x (0) = 1, x′ (0) = 0, y (0) = 0, y′ (0) = 1

259

Page 262: Apuntes MAT-023 USM

Apuntes Mat023 (Segundo semestre 2014)

13. Calcular la transformada de Laplace de f (t) = abtc donde btc es la parte entera de t

y a ∈ R+ y con esa informacion resolver la ecuacion

y (t+ 2)− 3y (t+ 1) + 2y (t) = 0

si se sabe que y (t) es constante e igual a 0 en el intervalo [0, 1[ y es constante e igual

a 1 en [1, 2[.

14. Resolver el problema de valores iniciales

y′′ + ty′ − 2y = 4

y (0) = −1

y′ (0) = 0

15. Resolver la ecuacion diferencial

d2y

dt2+ 2t

dy

dt− 4y = 2− 2t

con y (0) = 0 y y′ (0) = 1.

16. Muestre que si f : R→ R es una funcion periodica de periodo T > 0 entonces

L [f (t)] (s) =1

1− e−sT

∫ T

0

e−stf (t) dt

Ind.: L [f (t)] (s) =∫ T

0f (t) e−stdt+

∫ +∞T

f (t) e−stdt y cambie variable.

17. Calcular la transformada de las funciones f (t) = |sinωt| y f (t) = t− btc

260

Page 263: Apuntes MAT-023 USM

Apuntes Mat023 (Segundo semestre 2014)

18. Calcular las transformadas de las funciones de la figura

261

Page 264: Apuntes MAT-023 USM

Capıtulo 5 : Series de Fourier

Definiciones

El espacio de las funciones continuas en un intervalo [a, b] a valores reales representado

por C [a, b], es un espacio de dimension infinita, pues para todo m ∈ N se cumple1.x.x2, . . . , xm

⊂ C [a, b]

y el conjunto 1.x.x2, . . . , xm es linealmente independiente. Sabemos que si un espacio V

tiene una base B = v1,v2, . . . ,vm entonces todo elemento del espacio se puede escribir

como combinacion lineal, esto es,

v =m∑j=1

αjvj

al ser un espacio de dimension infinita, debemos esperar que los elementos sean mas

complicados que un polinomio, por ejemplo sabemos que ex no es un polinomio. Es natural

preguntarse entonces, si todos los elementos se pueden escribir como una “combinacion

lineal infinita” de los elementos de este conjunto, esto es, una serie de la forma

f (x) =+∞∑k=0

akxk = a0 + a1x+ a2x

2 + · · ·

lo que llamamos serie de Taylor. Sabemos de Mat022 que las funciones que se pueden

escribir de esta forma son funciones de clase C∞ en su intervalo de convergencia, luego

no es de esperar que toda funcion continua se pueda escribir como una serie de Taylor

¿existira otra forma de escribir estas funciones en forma de serie? La respuesta a esta

pregunta es afirmativa y viene dada por las series de Fourier, en algun sentido que vamos a

especificar en breve, toda funcion continua f ∈ C [a, b] se puede escribir en la forma

a0

2+

+∞∑n=1

an cos

(2nπx

b− a

)+

+∞∑n=1

bn sin

(2nπx

b− a

)para elecciones adecuadas de las constantes a0, an y bn.

El espacio SC [a, b]

Definicion 5.1.1. Una funcion a valores reales f es llamada funcion seccionalmente

continua en [a, b] si:

1. f esta definida y es continua en todos los puntos de [a, b] salvo quizas un numero

finito de ellos.

262

Page 265: Apuntes MAT-023 USM

Apuntes Mat023 (Segundo semestre 2014)

2. Los lımites

f(x+

0

)= lım

x→x+0f (x)

f(x−0)

= lımx→x−0

f (x)

existen en cada punto de [a, b] (note que en los extremos solo un lımite es relevante).

Observacion 5.1.1. Si f ∈ SC [a, b] entonces

∫ b

a

f (x) dx existe y es independiente de

los valores (si los toma) de la funcion en los puntos de discontinuidad. En particular, si

f, g ∈ SC [a, b] son iguales salvo en los puntos de discontinuidad entonces∫ b

a

f (x) dx =

∫ b

a

g (x) dx

diremos entonces que dos funciones en SC [a, b] son iguales si f (x) = g (x) salvo quizas en

los puntos de discontinuidad.

Observacion 5.1.2. Si α ∈ R, f, g ∈ SC [a, b] entonces αf , f + g y fg son funciones en

SC [a, b].

Observacion 5.1.3. C [a, b] ⊆ SC [a, b].

Definicion 5.1.2. Sea V un espacio vectorial real. Diremos que la funcion 〈·, ·〉 : V ×V −→R es un producto interior si cumple:

1. ∀f ∈ V , 〈f, f〉 ≥ 0, Ademas 〈f, f〉 = 0⇔ f = 0

2. ∀f, g ∈ V , 〈f, g〉 = 〈g, f〉

3. ∀α ∈ R,∀f ∈ V , 〈αf, g〉 = α 〈f, g〉

4. ∀f, g, h ∈ V , 〈f + g, h〉 = 〈f, h〉+ 〈g, h〉

Ejemplo 5.1.1. En V = Rn, la funcion 〈·, ·〉 : V × V −→ R definida por

< x,y >=n∑i=1

xiyi

es un producto interior.

263

Page 266: Apuntes MAT-023 USM

Apuntes Mat023 (Segundo semestre 2014)

Ejemplo 5.1.2. En SC[a, b] la funcion 〈·, ·〉 : SC[a, b]× SC[a, b] −→ R definida por

(f, g)→ 〈f, g〉 =

∫ b

a

f(x)g(x) dx

es un producto interior. Llamaremos a este producto, el producto usual en SC[a, b].

Ejemplo 5.1.3. En C[a, b], si ρ ∈ C [a, b] es tal que ρ (x) > 0 para x ∈ [a, b] la funcion

〈·, ·〉 : C[a, b]× C[a, b] −→ R definida por

(f, g)→ 〈f, g〉 =

∫ b

a

ρ (x) f(x)g(x) dx

es un producto interior. Este producto es llamado producto con peso ρ.

En todo espacio vectorial con producto interior es posible definir una norma

‖ · ‖ : V −→ R

como

‖f‖ =√〈f, f〉

y esta a su vez, nos permite definir una funcion distancia en el espacio V mediante la

formula

d(f, g) = ‖f − g‖

Teorema 5.1.1. Sea (V, 〈·, ·〉) un espacio vectorial real con producto interior, para toda

f, g ∈ V se tiene

|〈f, g〉| ≤√〈f, f〉

√〈g, g〉

Ejemplo 5.1.4. Si f(x) = ex, g(x) = e−x son funciones en C [0, 1], calcular d (f, g), con

la distancia inducida por el producto usual del ejemplo.

Tenemos

d (f, g) =√〈f − g, f − g〉

=

√∫ 1

0

(f (x)− g (x))2 dx

=

√∫ 1

0

(ex − e−x)2 dx

=√

sinh 2− 2

264

Page 267: Apuntes MAT-023 USM

Apuntes Mat023 (Segundo semestre 2014)

Proposicion 5.1.1. Propiedades basicas del producto interior 〈·, ·〉 en un espacio vectorial

real V :

1. ∀f, g ∈ V, ∀α ∈ R, 〈f, αg〉 = α 〈f, g〉

2. ∀f, g, h ∈ V, 〈f, g + h〉 = 〈f, g〉+ 〈f, h〉

3. ∀fk, gj ∈ V, con k = 1, 2, . . . , n y j = 1, 2, . . . ,m y ∀αk, βj ∈ R, con k = 1, 2, . . . , n y

j = 1, 2, . . . ,m ⟨n∑k=1

αkfk,m∑j=1

βjgj

⟩=

n∑k=1

m∑j=1

αkβj 〈fk, gj〉

Definicion 5.1.3. Sea V un espacio con producto interior < ·, · >.

1. Diremos que f y g en V son ortogonales si < f, g >= 0

2. Diremos que fkk∈I es un conjunto ortogonal si < fk, fj >= 0, para k 6= j

3. Diremos que fkk∈I es un conjunto ortonormal si es ortogonal y la norma de cada

elemento es 1

Ejemplo 5.1.5. En SC[0, L], la familia

sin(nπxL

), n ∈ N

es ortogonal.

En efecto, si n 6= m entonces∫sin(nπxL

)sin(mπx

L

)dx

=L

2π (m2 − n2)

((m+ n) sin

(m− n) πx

L+ (n−m) sin

(m+ n) πx

L

)(usar prostaferesis) luego∫ L

0

sin(nπxL

)sin(mπx

L

)dx

=L

2π (m2 − n2)

((m+ n) sin

(m− n) πx

L+ (n−m) sin

(m+ n) πx

L

)∣∣∣∣L0

=L

2π (m2 − n2)

((m+ n) sin

(m− n) πL

L+ (n−m) sin

(m+ n) πL

L

)=

L

2π (m2 − n2)((m+ n) sin [(m− n) π] + (n−m) sin [(m+ n) π])

= 0

265

Page 268: Apuntes MAT-023 USM

Apuntes Mat023 (Segundo semestre 2014)

Ejemplo 5.1.6. En SC[a, b] la familia

1, cos

(2πx

b− a

), sin

(2πx

b− a

), cos

(4πx

b− a

), sin

(4πx

b− a

), cos

(6πx

b− a

), sin

(6πx

b− a

), . . .

= 1, cos

(2nπx

b− a

), sin

(2nπx

b− a

)con n ∈ N

es una familia ortogonal.

Ejemplo 5.1.7. Si (V, 〈·, ·〉) es un espacio con producto interior y u1,u2,u3 ⊆ V es un

conjunto ortonormal, determine la norma del vector

u1 − u2 + u3

Solucion. Usando las propiedades del producto punto y la ortogonalidad

‖u1 − u2 + u3‖2 = 〈u1 − u2 + u3,u1 − u2 + u3〉= ‖u1‖2 + ‖u2‖2 + ‖u3‖2

= 3

(los vectores tienen norma 1) entonces

‖u1 − u2 + u3‖ =√

3

Ejemplo 5.1.8. Muestre que el conjunto B =

1√2π, cosx√

π, sin 2x√

π

es ortonormal en C [−π, π]

y calcular (∫ π

−π

(1√2π− cosx√

π+

sin 2x√π

)2

dx

)1/2

Solucion. ⟨1√2π,cosx√π

⟩=

∫ π

−π

cosx√2π

dx

=1√2π

∫ π

−πcosxdx

=1√2π

sin (x)

∣∣∣∣π−π

= 0

y ⟨1√2π,sinx√π

⟩=

∫ π

−π

sin 2x√2π

dx = 0

266

Page 269: Apuntes MAT-023 USM

Apuntes Mat023 (Segundo semestre 2014)

(funcion impar) ⟨sin 2x√

π,cosx√π

⟩=

∫ π

−π

sin 2x cosx

πdx = 0

funcion impar. Es facil ver que∥∥∥∥ 1√2π

∥∥∥∥ =1√2π‖1‖ =

√2π√2π

= 1∥∥∥∥sin 2x√π

∥∥∥∥ =1√π

(∫ π

−πsin2 2xdx

)1/2

=1√π

(2

∫ π

0

1− cos 4x

2dx

)1/2

=1√π

(√π)1/2

= 1∥∥∥∥cosx√π

∥∥∥∥ =1√π

(∫ π

−πcos2 xdx

)1/2

=1√π

(2

∫ π

0

cos2 xdx

)1/2

=1√π

(2

∫ π

0

1 + cos 2x

2dx

)1/2

= 1

usando el ejercicio anterior(∫ π

−π

(1√2π− cosx√

π+

sin 2x√π

)2

dx

)1/2

=√

3

Teorema de la mejor aproximacion

Ahora vamos a analizar cuando tenemos familia ortonormales como se logra la mejor

aproximacion de un elemento en un espacio dado. La idea es obtener una aproximacion

con distancia cero.

Teorema 5.1.2. Sea fkNk=1 un conjunto ortogonal en V . Si λk ∈ R, ∀k = 1, . . . N

entonces ∥∥∥∥∥N∑k=1

λkfk

∥∥∥∥∥2

=N∑k=1

|λk|2‖fk‖2

267

Page 270: Apuntes MAT-023 USM

Apuntes Mat023 (Segundo semestre 2014)

Demostracion. ∥∥∥∥∥N∑k=1

λkfk

∥∥∥∥∥2

=

⟨N∑k=1

λkfk,N∑j=1

λjfj

=N∑k=1

N∑j=1

λkλj 〈fk, fj〉

=N∑k=1

λ2k ‖fk‖

2

Note que si el conjunto fkNk=1 es ortogonal (y funciones no nulas) en V entonces es

linealmente independienteN∑k=1

λkfk = θV

implica

0 = 〈θV , fi〉 =

⟨N∑k=1

λkfk, fi

=N∑k=1

λk 〈fk, fi〉

= λi 〈fi, fi〉

de donde obtenemos λi = 0.

Sea f ∈ V dado, ¿Cual es la mejor aproximacion que podemos hacer en V mediante un

elemento de de la formaN∑k=1

λkfk

esto es un elemento del subespacio generado por f1, f2, ..., fN

S = 〈f1, f2, ..., fN〉

donde fk es un conjunto ortonormal.

268

Page 271: Apuntes MAT-023 USM

Apuntes Mat023 (Segundo semestre 2014)

d

(f,

N∑k=1

λkfk

)2

=

∥∥∥∥∥f −N∑k=1

λkfk

∥∥∥∥∥2

= ‖f‖2 − 2N∑k=1

λk 〈f, fk〉+N∑k=1

|λk|2 ‖fk‖2

= ‖f‖2 − 2N∑k=1

λk 〈f, fk〉+N∑k=1

|λk|2

= ‖f‖2 −N∑k=1

〈f, fk〉2 +N∑k=1

(〈f, fk〉 − λk)2

se sigue, independiente de los escalares λ1, λ2, . . . , λN

d

(f,

N∑k=1

λkfk

)≥

√√√√‖f‖2 −N∑k=1

〈f, fk〉2

luego el mınimo valor de esta distancia corresponde a√√√√‖f‖2 −N∑k=1

〈f, fk〉2

que se alcanza en la combinacion lineal de la forma

N∑k=1

〈f, fk〉 fk

se sigue que

N∑k=1

〈f, fk〉 fk

es la mejor aproximacion de f ∈ V en el espacio.

S = 〈f1, f2, ..., fN〉

y

mın(λ1,...,λN )∈Rn

d

(f,

N∑k=1

λkfk

)=

√√√√‖f‖2 −N∑k=1

< f, fk >2

269

Page 272: Apuntes MAT-023 USM

Apuntes Mat023 (Segundo semestre 2014)

Teorema 5.1.3. Sea fkNk=1 un conjunto ortonormal en un espacio vectorial V con

producto interior, f ∈ V y S = 〈f1, f2, . . . , fN〉 se tiene

d (f, S) = mın(λ1,...,λN )∈Rn

d

(f,

N∑k=1

λkfk

)

=

√√√√‖f‖2 −N∑k=1

〈f, fk〉2

y esta distancia mınima se logra para λi = 〈f, fi〉 con i = 1, 2, . . . , N . Es decir, el elemento

de S a menor distancia de f esN∑k=1

〈f, fk〉 fk

de este resultado obtenemos:

Teorema 5.1.4 (Desigualdad de Bessel). Si fkNk=1 es un conjunto ortonormal en V

entonces ∀f ∈ VN∑k=1

〈f, fk〉2 ≤ ‖f‖2

Demostracion. De los calculos anteriores

0 ≤ d

(f,

N∑k=1

λkfk

)2

= ‖f‖2 −N∑k=1

〈f, fk〉2 +N∑k=1

(〈f, fk〉 − λk)2

si tomamos 〈f, fk〉 − λk = 0 entonces

0 ≤ ‖f‖2 −N∑k=1

〈f, fk〉2

que es la desigualdad deseada.

Teorema 5.1.5. Si fk∞k=1 es un conjunto ortonormal en V entonces para cada f en V

se cumple

lımk→∞〈f, fk〉 = 0

Demostracion. Por la desigualdad de Bessel se tiene ∀N ∈ N

N∑k=1

〈f, fk〉2 ≤ ‖f‖2

270

Page 273: Apuntes MAT-023 USM

Apuntes Mat023 (Segundo semestre 2014)

luego la serie∑+∞

k=1 〈f, fk〉2 converge (sucesion de sumas parciales creciente y acotada

superiormente) luego

lımk→∞〈f, fk〉 = 0

Ejemplo 5.1.9. Muestre que el conjunto

1√2π, sinx√

π

es ortonormal en C [−π, π] y deter-

minar el elemento de la formaα√2π

+ βsinx√π

(con α, β ∈ R) mas cercano a f (x) = x. ¿Cual es la mınima distancia?.

Solucion. Primero calculamos las normas∥∥∥∥ 1√2π

∥∥∥∥ =

(∫ π

−π

1

2πdx

)1/2

=

(1

∫ π

−πdx

)1/2

=

(1

∫ π

−πdx

)1/2

= (1)1/2

= 1

y ∥∥∥∥sinx√π

∥∥∥∥ =

(∫ π

−π

sin2 x

πdx

)1/2

=

(1

π

∫ π

−π

1− cos 2x

2dx

)1/2

=

(1

ππ

)1/2

= 1

que son ortogonales es facil⟨1√2π,sinx√π

⟩=

1√2π

∫ π

−πsinxdx

= 0

(por imparidad). El elemento mas cercano es

g (x) =

⟨1√2π, x

⟩1√2π

+

⟨sinx√π, x

⟩sinx√π

271

Page 274: Apuntes MAT-023 USM

Apuntes Mat023 (Segundo semestre 2014)

donde ⟨1√2π, x

⟩=

∫ π

−π

x√2πdx = 0⟨

sinx√π, x

⟩=

∫ π

−π

x sinx√π

dx = 2√π

ası

g (x) = 2 sinx

y la distancia mınima es

d =

√‖x‖2 − 02 −

(2√π)2

donde

‖x‖2 =

∫ π

−πx2dx =

2

3π3

entonces

d =

√2

3π3 − 4π

Observacion 5.1.4. El conjunto

B =

1

‖1‖,

cos(

2kπxb−a

)∥∥cos(

2kπxb−a

)∥∥ , sin(

2kπxb−a

)∥∥sin(

2kπxb−a

)∥∥ con k ∈ N

es un conjunto ortonormal en SC[a, b], notemos que(∫ b

a

dx

)1/2

=√

(b− a)

∫ b

a

cos2

(2kπx

b− a

)dx =

∫ b

a

(1 + cos

(4kπxb−a

)2

)dx

=

(b− a

2

)+

1

2

∫ b

a

cos

(4kπx

b− a

)dx

=

(b− a

2

)+

1

2sin

(4kπx

b− a

)(b− a4kπ

)∣∣∣∣ba

=

(b− a

2

)+

1

2

(sin

(4kπb

b− a

)− sin

(4kπa

b− a

))(b− a4kπ

)=

(b− a

2

)donde hemos usado la formula de prostaferesis

sin (x)− sin (y) = −2 sin

(x− y

2

)cos

(x+ y

2

)272

Page 275: Apuntes MAT-023 USM

Apuntes Mat023 (Segundo semestre 2014)

para ver

sin

(4kπb

b− a

)− sin

(4kπa

b− a

)= 0

ahora bien ∫ b

a

sin2

(2kπx

b− a

)dx =

∫ b

a

(1− cos2

(2kπx

b− a

))dx

= (b− a)−(b− a

2

)=

(b− a

2

)ası

‖1‖ =√b− a∥∥∥∥cos

(2kπx

b− a

)∥∥∥∥ =

√b− a

2∥∥∥∥sin

(2kπx

b− a

)∥∥∥∥ =

√b− a

2

Definicion 5.1.4. Sea f ∈ SC [a, b]. Llamaremos serie de Fourier de f a la serie de la

formaa0

2+∞∑k=1

ak cos

(2kπ

b− ax

)+∞∑k=1

bk sin

(2kπ

b− ax

)donde los coeficientes son dados por la mejor aproximacion.

Calculemos los coeficientes en forma explıcita:

〈f (x) , 1〉

=

⟨a0

2+∞∑k=1

ak cos

(2kπ

b− ax

)+∞∑k=1

bk sin

(2kπ

b− ax

), 1

=⟨a0

2, 1⟩

+

⟨∞∑k=1

ak cos

(2kπ

b− ax

), 1

⟩+

⟨∞∑k=1

bk sin

(2kπ

b− ax

), 1

=a0

2〈1, 1〉+

∞∑k=1

ak

⟨cos

(2kπ

b− ax

), 1

⟩+∞∑k=1

bk

⟨sin

(2kπ

b− ax

), 1

⟩=

a0

2(b− a)

esto es

a0 =2

b− a

∫ b

a

f (x) dx

273

Page 276: Apuntes MAT-023 USM

Apuntes Mat023 (Segundo semestre 2014)

de manera similar⟨f, cos

(2nπ

b− ax

)⟩= an

⟨cos

(2nπ

b− ax

), cos

(2nπ

b− ax

)⟩ası

an =

⟨f, cos

(2nπb−ax

)⟩⟨cos(

2nπb−ax

), cos

(2nπb−ax

)⟩=

2

b− a

∫ b

a

f (x) cos

(2nπ

b− ax

)dx para n ∈ N

y

bn =

⟨f, sin

(2nπb−ax

)⟩⟨sin(

2nπb−ax

), sin

(2nπb−ax

)⟩=

2

b− a

∫ b

a

f (x) sin

(2nπ

b− ax

)dx para n ∈ N

Teorema 5.1.6. Si f ∈ SC [a, b] entonces

lımN→+∞

∥∥∥∥∥f (x)−

(a0

2+

N∑k=1

ak cos

(2kπ

b− ax

)+

N∑k=1

bk sin

(2kπ

b− ax

))∥∥∥∥∥ = 0

si a0, ak y bk corresponden a los coeficientes de Fourier de f .

Podemos entonces escribir

f (x) =a0

2+∞∑k=1

ak cos

(2kπ

b− ax

)+∞∑k=1

bk sin

(2kπ

b− ax

)pero la convergencia de la serie no es necesariamente al evaluar en los puntos, sino en el

sentido de la distancia definida.

Ejemplo 5.1.10. Determinar la serie de Fourier de f (x) = x en C [−π, π]

Solucion. En este intervalo la serie tiene la forma

a0

2+∞∑k=1

ak cos (kx) +∞∑k=1

bk sin (kx)

274

Page 277: Apuntes MAT-023 USM

Apuntes Mat023 (Segundo semestre 2014)

note que al ser una funcion impar a0 = ak = 0 y

bk =1

π

∫ π

−πx sin (kx) dx

=2

π

∫ π

0

x sin (kx) dx

=2

πk2((sin kx− kx cos kx)|π0 )

=2

πk2

(−k (−1)k

)=

2 (−1)k+1

πk

ası

x =∞∑k=1

(2 (−1)k+1

πk

)sin (kx)

275

Page 278: Apuntes MAT-023 USM

Apuntes Mat023 (Segundo semestre 2014)

en media. Veamos algunos graficos de la aproximacionN∑k=1

(2 (−1)k+1

πk

)sin (kx)

∑3k=1

(2(−1)k+1

πk

)sin (kx)

∑6k=1

(2(−1)k+1

πk

)sin (kx)

∑15k=1

(2(−1)k+1

πk

)sin (kx)

∑60k=1

(2(−1)k+1

πk

)sin (kx)

Ejemplo 5.1.11. Determinar la serie de Fourier de

f (x) = ex

para x ∈ [−1, 1] .

Solucion. La serie de Fourier de f en este intervalo es de la forma

a0

2+∞∑n=1

an cos

(2nπx

2

)+∞∑n=1

an sin

(2nπx

2

)=

a0

2+∞∑n=1

an cos (nπx) +∞∑n=1

an sin (nπx)

276

Page 279: Apuntes MAT-023 USM

Apuntes Mat023 (Segundo semestre 2014)

donde

a0 =2

2

∫ 1

−1

exdx = 2 sinh 1

an =

∫ 1

−1

ex cos (nπx) dx =(2 sinh 1) (−1)n

π2n2 + 1para n ∈ N

y

bn =

∫ 1

−1

ex sin (nπx) dx =(2nπ sinh 1) (−1)n+1

π2n2 + 1para n ∈ N

se sigue

ex = (sinh 1) +∞∑n=1

(2 sinh 1) (−1)n

π2n2 + 1cos (nπx) +

∞∑n=1

(2nπ sinh 1) (−1)n+1

π2n2 + 1sin (nπx)

= (sinh 1)

(1 +

∞∑n=1

2 (−1)n

π2n2 + 1cos (nπx) +

∞∑n=1

2nπ (−1)n+1

π2n2 + 1sin (nπx)

)ası por ejemplo, la grafica de la aproximacion

(sinh 1)

(1 +

10∑n=1

2 (−1)n

π2n2 + 1cos (nπx) +

10∑n=1

2nπ (−1)n+1

π2n2 + 1sin (nπx)

)es:

en rojo esta la grafica de la exponencial.

Teorema 5.1.7 (Identidad de Parseval). Si f ∈ SC [a, b] entonces

2

b− a

∫ b

a

f 2 (x) dx =a2

0

2+∞∑n=1

a2n +

∞∑n=1

b2n

donde

an =2

b− a

∫ b

a

f (x) cos

(2nπ

b− ax

)dx para n ∈ N∪0

277

Page 280: Apuntes MAT-023 USM

Apuntes Mat023 (Segundo semestre 2014)

y

bn =2

b− a

∫ b

a

f (x) sin

(2nπ

b− ax

)dx para n ∈ N

Ejemplo 5.1.1. Calcular la serie de Fourier de f (x) = |x| para x ∈ [−1, 1] y usando

Parseval calcular∞∑n=1

1

(2n+ 1)4

Convergencia Puntual de series de Fourier

Si bien hemos enunciado que la serie de Fourier converge en norma la funcion dada, se

presenta la siguiente situacion: Determinemos la serie de Fourier de

f (x) =

1 si x ∈ [0, π]

−1 si x ∈ [−π, 0[

se tiene:

a0 =1

π

∫ π

−πf (x) dx = 0 (la funcion es impar)

an =1

π

∫ π

−πf (x) cos (nx) dx = 0 (la funcion f (x) cos (nx) es impar) para cada n ∈ N

bn =1

π

∫ π

−πf (x) sin (nx) dx = − 1

πn(2 (−1)n − 2) para n ∈ N

se sigue que

f (x) =∞∑n=1

(− 1

πn(2 (−1)n − 2)

)sin (nx) (en media)

=2

π

∞∑n=1

(− 1

n((−1)n − 1)

)sin (nx) (en media)

pero

f (0) = 1

pero si evaluamos2

π

∞∑n=1

(− 1

n((−1)n − 1)

)sin (nx)

en x = 0 nos queda2

π

∞∑n=1

(− 1

n((−1)n − 1)

)sin (n0) = 0

278

Page 281: Apuntes MAT-023 USM

Apuntes Mat023 (Segundo semestre 2014)

luego el valor al cual converge la serie no es el valor de la funcion en el punto. Como se ve

en las siguiente graficas, los combinaciones trigonometricas

2

π

N∑n=1

(− 1

n((−1)n − 1)

)sin (nx)

cuando N crece se va pareciendo mas a la grafica pero todos estas funciones pasan por el 0.

El siguiente teorema nos dice a que valor converge la serie de Fourier

a0

2+∞∑k=1

ak cos

(2kπ

b− ax

)+∞∑k=1

bk sin

(2kπ

b− ax

)de f ∈ SC [a, b] cuando evaluamos en un x0 ∈ [a, b].

Teorema 5.2.1 (Convergencia puntual). Sea f, f ′ ∈ SC [a, b] entonces la serie de Fourier

converge en todos los puntos del intervalo [a, b], ademas si x0 ∈ [a, b] la serie de Fourier

converge af(x+

0

)+ f

(x−0)

2

cuando x0 ∈ ]a, b[ y af (b−) + f (a+)

2

si x0 = a o x0 = b.

Por convergencia puntual se entiende

lımN→∞

(a0

2+

N∑k=1

(ak cos

(2kπ

b− ax0

)+ bk sin

(2kπ

b− ax0

)))= lım

N→∞SN (x0)

=f(x+

0

)+ f

(x−0)

2

(no es la convergencia en norma, la cual esta garantizada sin mirar la derivada).

Ejemplo 5.2.1. Sabemos que

ex = (sinh 1)

(1 +

∞∑n=1

2 (−1)n

π2n2 + 1cos (nπx) +

∞∑n=1

2nπ (−1)n+1

π2n2 + 1sin (nπx)

)(en media)

para x ∈ [−1, 1]. Usando esa serie, determinar el valor de

∞∑n=1

(−1)n

π2n2 + 1

279

Page 282: Apuntes MAT-023 USM

Apuntes Mat023 (Segundo semestre 2014)

Solucion. Como

ex = (sinh 1)

(1 +

∞∑n=1

2 (−1)n

π2n2 + 1cos (nπx) +

∞∑n=1

2nπ (−1)n+1

π2n2 + 1sin (nπx)

)

evaluando en x = 0 que es un punto de continuidad se obtiene

1 = (sinh 1)

(1 +

+∞∑n=1

2 (−1)n

π2n2 + 1

)

ası1

2

(1

sinh 1− 1

)=

+∞∑n=1

(−1)n

π2n2 + 1

Note que si f, f ′ ∈ SC [a, b] la serie de Fourier

a0

2+∞∑k=1

ak cos

(2kπ

b− ax

)+∞∑k=1

bk sin

(2kπ

b− ax

)converge en cada punto del intervalo, si denotamos por

SF (x) =a0

2+∞∑k=1

ak cos

(2kπ

b− ax

)+∞∑k=1

bk sin

(2kπ

b− ax

)del resultado anterior se tiene que SF (x0) = f (x0) en los puntos en los cuales f es continua,

pero se obtiene una propiedad adicional, dada que las funciones 1, cos(

2kπb−ax

), sin

(2kπb−ax

)son periodicas de periodo (b− a) se sigue que

SF (x) =a0

2+∞∑k=1

ak cos

(2kπ

b− ax

)+∞∑k=1

bk sin

(2kπ

b− ax

)es una funcion periodica de periodo (b− a) y ası SF (x) esta bien definida en todo R,

SF : R→ Rx → SF (x)

donde SF : R→ R es una funcion periodica de periodo (b− a) tal que

SF (x0) =f(x+

0

)+ f

(x−0)

2para x0 ∈ ]a, b[

=f (b−) + f (a+)

2para x0 = a, b

280

Page 283: Apuntes MAT-023 USM

Apuntes Mat023 (Segundo semestre 2014)

Ejemplo 5.2.2. La serie de Fourier de

f (x) =

1 si x ∈ [0, π]

−1 si x ∈ [−π, 0[

es2

π

∞∑n=1

(− 1

n((−1)n − 1)

)sin (nx)

entonces

1 =2

π

∞∑n=1

(− 1

n((−1)n − 1)

)sin (nx) para x ∈ ]0, π[

−1 =2

π

∞∑n=1

(− 1

n((−1)n − 1)

)sin (nx) para x ∈ ]−π, 0[

1 + (−1)

2= 0 para x = 0,−π, π

y por periodicidad podemos decir por ejemplo que

2

π

∞∑n=1

(− 1

n((−1)n − 1)

)sin(n(

2π +π

4

))=

2

π

∞∑n=1

(− 1

n((−1)n − 1)

)sin(πn

4

)= 1

Ejemplo 5.2.3. Determinar la serie de Fourier de f ∈ C [−2, 2] definida por

f (x) =

1 si 0 ≤ x ≤ 2

−1 si −2 ≤ x < 0

Determine el valor al cual converge la serie en x = 6 y bosquejar un grafico de la serie

poniendo especial cuidado en los puntos de discontinuidad.

Solucion. La funcion es impar, esto permitira simplifica los calculos, la serie de Fourier

tiene la forma

F (x) =a0

2+∞∑n=1

an cos

(2nπx

4

)+∞∑n=1

bn sin

(2nπx

4

)=

a0

2+∞∑n=1

an cos(nπx

2

)+∞∑n=1

bn sin(nπx

2

)donde

a0 =2

4

∫ 2

−2

f (x) dx = 0

an =1

2

∫ 2

−2

f (x) cos(nπx

2

)dx = 0

281

Page 284: Apuntes MAT-023 USM

Apuntes Mat023 (Segundo semestre 2014)

y

bn =1

2

∫ 2

−2

f (x) sin(nπx

2

)dx

=

∫ 2

0

sin(nπx

2

)dx

= − 2

nπcos(nπx

2

)∣∣∣∣20

= − 2

nπ(cos (nπ)− 1)

= −2(−1)n − 1

= 2(−1)n+1 + 1

ası

F (x) = 2∞∑n=1

((−1)n+1 + 1

)sin(nπx

2

)por los teoremas vistos en clases, la serie es periodica de periodo 4 se sigue que

F (6) = F (2) =1 + (−1)

2= 0

la grafica de la serie es:

1.

Series de Fourier de senos y cosenos

Definicion 5.3.1. Si f : [0, L]→ R es una funcion en SC [0, L], llamaremos:

282

Page 285: Apuntes MAT-023 USM

Apuntes Mat023 (Segundo semestre 2014)

1. Extension par de f a la funcion

fp (x) =

f (x) si x ∈ [0, L]

f (−x) si x ∈ [−L, 0]

2. Extension impar de f a la funcion

fI (x) =

f (x) si x ∈ [0, L]

−f (−x) si x ∈ [−L, 0[

Observacion 5.3.1. fp ∈ SC [−L,L] es una funcion par tal que

fp (x) = f (x) para x ∈ [0, L]

y fI (x) ∈ SC [−L,L] es una funcion impar tal que

fI (x) = f (x) para x ∈ [0, L]

Al desarrollar en serie de Fourier la funcion fp en [−L,L] se tiene

fp (x) =a0

2+∞∑k=1

ak cos

(kπx

L

)+∞∑k=1

bk sin

(kπx

L

)donde

a0 =1

L

∫ L

−Lfp (x) dx

ak =1

L

∫ L

−Lfp (x) cos

(kπx

L

)dx para k ∈ N

bk =1

L

∫ L

−Lfp (x) sin

(kπx

L

)dx para k ∈ N

notemos que al ser fp una funcion par fp (x) cos(kπxL

)es par y fp (x) sin

(kπxL

)es impar,

luego

a0 =1

L

∫ L

−Lfp (x) dx =

2

L

∫ L

0

fp (x) dx

=2

L

∫ L

0

f (x) dx

283

Page 286: Apuntes MAT-023 USM

Apuntes Mat023 (Segundo semestre 2014)

y

ak =1

L

∫ L

−Lfp (x) cos

(kπx

L

)dx para k ∈ N

=2

L

∫ L

0

fp (x) cos

(kπx

L

)dx para k ∈ N

=2

L

∫ L

0

f (x) cos

(kπx

L

)dx para k ∈ N

y

bk =1

L

∫ L

−Lfp (x) sin

(kπx

L

)dx para k ∈ N

= 0

ası la serie es de la forma

fp (x) =a0

2+∞∑k=1

ak cos

(kπx

L

)con

a0 =2

L

∫ L

0

f (x) dx

ak =2

L

∫ L

0

f (x) cos

(kπx

L

)dx para k ∈ N

de manera similar, si al desarrollar en serie de Fourier la funcion fI en [−L,L] se tiene

fI (x) =a0

2+∞∑k=1

ak cos

(kπx

L

)+∞∑k=1

bk sin

(kπx

L

)donde

a0 =1

L

∫ L

−LfI (x) dx

ak =1

L

∫ L

−LfI (x) cos

(kπx

L

)dx para k ∈ N

bk =1

L

∫ L

−LfI (x) sin

(kπx

L

)dx para k ∈ N

notemos que al ser fI una funcion impar fI (x) cos(kπxL

)es impar y fI (x) sin

(kπxL

)es par,

luego

a0 =1

L

∫ L

−LfI (x) dx = 0

284

Page 287: Apuntes MAT-023 USM

Apuntes Mat023 (Segundo semestre 2014)

y

ak =1

L

∫ L

−LfI (x) cos

(kπx

L

)dx para k ∈ N

= 0 para k ∈ N

y

bk =1

L

∫ L

−LfI (x) sin

(kπx

L

)dx para k ∈ N

=2

L

∫ L

0

fI (x) sin

(kπx

L

)dx para k ∈ N

=2

L

∫ L

0

f (x) sin

(kπx

L

)dx para k ∈ N

ası la serie es de la forma

fI (x) =∞∑k=1

bk sin

(kπx

L

)con

bk =2

L

∫ L

0

f (x) sin

(kπx

L

)dx para k ∈ N

como fp (x) = fI (x) = f (x) para x ∈ [0, L] se obtiene que es posible desarrollar f ∈SC [0, L] en series de la forma

f (x) =a0

2+∞∑k=1

ak cos

(kπx

L

)donde

a0 =2

L

∫ L

0

f (x) dx

ak =2

L

∫ L

0

f (x) cos

(kπx

L

)dx para k ∈ N

llamada serie cosenoidal de f o serie de cosenos de f y

f (x) =∞∑k=1

bk sin

(kπx

L

)donde

bk =2

L

∫ L

0

f (x) sin

(kπx

L

)dx para k ∈ N

llamada serie senoidal de f o serie de senos de f .

285

Page 288: Apuntes MAT-023 USM

Apuntes Mat023 (Segundo semestre 2014)

Ejemplo 5.3.1. Obtener la serie senoidal de

f (x) = cos x

en [0, π].

Solucion. Hacemos uso de la extension impar de f al intervalo [−π, π] entonces

fI (x) =

cosx si x ∈ [0, π]

− cos (−x) si x ∈ [−π, 0[

entonces

fI (x) =a0

2+∞∑n=1

an cos (nx) +∞∑n=1

bn sin (nx)

donde

a0 =1

π

∫ π

−πfI (x) = 0

an =1

π

∫ π

−πfI (x) cos (nx) dx = 0

bn =2

π

∫ π

0

cosx sin (nx) dx

si n = 1 entonces

b1 =2

π

∫ π

0

cosx sinxdx = 0

pero

sin (x+ nx) = sinx cosnx+ sinnx cosx

sin (x− nx) = sinx cosnx− sinnx cosx

ası

sin (x+ nx)− sin (x− nx) = 2 cosx sinnx

luego

−cos ((n+ 1)x)

n+ 1+

cos ((1− n)x)

1− n= 2

∫cosx sinnxdx

luego (−cos ((n+ 1)x)

n+ 1+

cos ((1− n)x)

1− n

)∣∣∣∣π0

= 2

∫ π

0

cosx sinnxdx(−cos ((n+ 1) π)

n+ 1+

cos ((1− n) π)

1− n

)−(− 1

n+ 1+

1

1− n

)= 2

∫ π

0

cosx sinnxdx

(−1)n

n+ 1+

(−1)n

n− 1+

2n

n2 − 1= 2

∫ π

0

cosx sinnxdx

((−1)n + 1) 2n

n2 − 1= 2

∫ π

0

cosx sinnxdx

286

Page 289: Apuntes MAT-023 USM

Apuntes Mat023 (Segundo semestre 2014)

se sigue

fI (x) =

(2

π

) +∞∑n=2

(((−1)n + 1)n

n2 − 1

)sin (nx)

ası en [0, π] obtenemos la representacion

cosx =

(2

π

) +∞∑n=2

(((−1)n + 1)n

n2 − 1

)sin (nx)

la grafica de la aproximacion (en verde)(2

π

) 100∑n=2

(((−1)n + 1)n

n2 − 1

)sin (nx)

es

en rojo esta la grafica de coseno en el intervalo deseado.

Ejemplo 5.3.2. Obtener la serie cosenoidal de la funcion

f (x) = x

para x ∈ [0, 4] .

Solucion. En este caso hacemos uso de la extension par de f al intervalo [−4, 4] se tiene

fp (x) =

x si x ∈ [0, 4]

(−x) si x ∈ [−4, 0[

287

Page 290: Apuntes MAT-023 USM

Apuntes Mat023 (Segundo semestre 2014)

entonces

fp (x) =a0

2+∞∑n=1

an cos

(2nπx

8

)+∞∑n=1

an sin

(2nπx

8

)donde

a0 =2

8

∫ 4

−4

fp (x) dx

=1

2

∫ 4

0

xdx = 4

an =1

4

∫ 4

−4

fp (x) cos(nπx

4

)dx

=1

2

∫ 4

0

x cos(nπx

4

)dx

=8 ((−1)n − 1)

π2n2

y

bn =1

4

∫ 4

−4

fp (x) sin(nπx

4

)dx = 0

se sigue

fp (x) = 2 +∞∑n=1

(8 ((−1)n − 1)

π2n2

)cos(nπx

4

)en particular en [0, 4] se tiene

x = 2 + 8∞∑n=1

(((−1)n − 1)

π2n2

)cos(nπx

4

)

Ejemplo 5.3.3. Sea f : [0, π]→ R definida por f (x) = x (π − x)

1. Encontrar una funcion F periodica y par tal que F (x) = f (x) para x ∈ [0, π] y

desarrollar en serie de Fourier F .

2. Encontrar una funcion G periodica e impar tal que G (x) = f (x) para x ∈ [0, π] y

desarrollar en serie de Fourier G.

3. Use los puntos anteriores para probar

∞∑n=1

(−1)n+1

n2=π2

12y∞∑n=1

(−1)n+1

(2n− 1)3 =π3

32

288

Page 291: Apuntes MAT-023 USM

Apuntes Mat023 (Segundo semestre 2014)

Solucion. Por punto pero calculando las sumas pedidas.

1. F es la serie de cosenos

F (x) =a0

2+∞∑n=1

an cos (nx)

donde

a0 =2

π

∫ π

0

x (π − x) dx =π2

3

an =2

π

∫ π

0

x (π − x) cos (nx) dx = −2 ((−1)n + 1)

n2

ası

F (x) =π2

6− 2

∞∑n=1

((−1)n + 1)

n2cos (nx)

si consideramos la suma sobre los pares (en los impares los coeficientes son nulos)

F (x) =π2

6− 2

∞∑n=1

2

(2n)2 cos (2nx)

=π2

6−∞∑n=1

cos (2nx)

n2

evaluando en π2

obtenemos

(π2

)(π − π

2

)=

π2

6−∞∑n=1

cos(2n(π2

))n2

=π2

6−∞∑n=1

(−1)n

n2

esto es

π2

4− π2

6=

∞∑n=1

(−1)n+1

n2

π2

12=

∞∑n=1

(−1)n+1

n2

2. G es la serie senoidal

G (x) =∞∑n=1

bn sin (nx)

289

Page 292: Apuntes MAT-023 USM

Apuntes Mat023 (Segundo semestre 2014)

donde

bn =2

π

∫ π

0

x (π − x) sin (nx) dx

= − 2

πn3(2 (−1)n − 2)

=−4 ((−1)n − 1)

πn3

ası

G (x) =∞∑n=1

(−4 ((−1)n − 1)

πn3

)sin (nx)

note que ahora en los pares los coeficientes son nulos, al considerar en los impares

tenemos

G (x) =8

π

∞∑n=1

sin ((2n− 1)x)

(2n− 1)3

evaluando en π2

tenemos

(π2

)(π − π

2

)=

8

π

∞∑n=1

sin((2n− 1) π

2

)(2n− 1)3

=8

π

∞∑n=1

(−1)n+1

(2n− 1)3

pues

sin(

(2n− 1)π

2

)= (−1)n+1

asıπ2

4=

8

π

∞∑n=1

(−1)n+1

(2n− 1)3

de donde obtenemosπ3

32=∞∑n=1

(−1)n+1

(2n− 1)3

Ejemplo 5.3.4. Sea f (x) = exp (− [x]) definida en [0, 2]. Obtenga su serie de Fourier de

cosenos y use la serie para calcular el valor de

∞∑n=1

(−1)n−1

2n− 1

Obs.: [x] es la parte entera de x.

290

Page 293: Apuntes MAT-023 USM

Apuntes Mat023 (Segundo semestre 2014)

Solucion. Tenemos que encontrar la serie de cosenos, usamos la extension par

fp (x) =a0

2+∞∑n=1

an cos

(2πnx

4

)donde

a0 =2

4

∫ 2

−2

fp (x) dx

=

∫ 2

0

f (x) dx

=

∫ 2

0

exp (− [x]) dx

=

∫ 1

0

1dx+

∫ 2

1

edx

= e−1 + 1

y

an =2

4

∫ 2

−2

fp (x) cos

(2πnx

4

)dx

=

∫ 2

0

exp (− [x]) cos(πnx

2

)dx

=

∫ 1

0

cos(πnx

2

)dx+

∫ 2

1

e−1 cos(πnx

2

)dx

=1

πn

(2e−1 sin πn− 2e−1 sin

1

2πn

)+

2

πnsin

1

2πn

= − 2

πn

(sin

πn

2

) (e−1 − 1

)luego

fp (x) =e−1 + 1

2+∞∑n=1

2

πn

(sin

πn

2

) (1− e−1

)cos(πnx

2

)evaluando en x = 0 se tiene

1 =e−1 + 1

2+∞∑n=1

2

πn

(sin

πn

2

) (1− e−1

)de donde obtenemos

1− e−1 + 1

2=

2 (1− e−1)

π

∞∑n=1

sin πn2

n

luegoπ(

12− 1

2e−1)

2 (1− e−1)=∞∑n=1

sin πn2

n

291

Page 294: Apuntes MAT-023 USM

Apuntes Mat023 (Segundo semestre 2014)

luegoπ

4=∞∑n=1

sin πn2

n

si n es par sin πn2

es cero, se sigue

π

4=∞∑n=1

sin π(2n−1)2

2n− 1

pero

sinπ (2n− 1)

2= (−1)n+1

es decirπ

4=∞∑n=1

(−1)n+1

2n− 1

Derivacion e integracion de Series de Fourier

Si para x ∈ ]−π, π[

x =∞∑k=1

2 (−1)k+1

ksin (kx)

es cierto que

1 =∞∑k=1

2 (−1)k+1 cos (kx)

para x ∈ ]−π, π[? En otras palabras ¿es posible derivar termino a termino una serie de

Fourier y asegurar que converge a la derivada de la funcion?, la respuesta es no, ya que

2 (−1)k+1 no corresponde al coeficiente de Fourier de la funcion, el cual tiene que cumplir

lımk→∞

Ck = 0

Para que sea valida la derivacion termino a termino tenemos el siguiente resultado:

Teorema 5.4.1. Si f ∈ C [a, b] , f (a) = f (b) y f ′ ∈ SC [a, b] entonces la serie de Fourier

para f ′ puede ser obtenida derivando termino a termino la serie de f . La serie obtenida

converge puntualmente a f ′ (x) en los puntos en los cuales f ′′ existe.

para la integracion de las series tenemos el siguiente resultado:

Teorema 5.4.2. Si f ∈ SC [a, b] y

f (x) =a0

2+∞∑n=1

(ak cos

(2nπx

b− a

)+ bk sin

(2nπx

b− a

))(media)

292

Page 295: Apuntes MAT-023 USM

Apuntes Mat023 (Segundo semestre 2014)

entonces ∫ x

a

f (t) dt = a0(x− a)

2

+∞∑n=1

ak

∫ x

a

cos

(2nπt

b− a

)dt

+∞∑n=1

bk

∫ x

a

sin

(2nπt

b− a

)dt (media)

(Esto es, es posible integrar la serie termino a termino)

Ejercicios del capıtulo

1. Suponga que

x (L− x) =∞∑k=1

ak sin

(kπx

L

)para x ∈ [0, L]. Haciendo uso de la propiedad de ortogonalidad de la familia

sin(kπxL

)k∈N en C[0, L] encontrar los coeficientes de la serie.

2. Muestre que en C[a, b] = V se cumple

a) ∀f, g ∈ V‖f − g‖2 + ‖f + g‖2 = 2 ‖f‖2 + 2 ‖g‖2

b) Si 〈f, g〉 = 〈f, h〉 para todo f ∈ V entonces g = h.

3. Muestre que para todo f ∈ C [−π, π]

lımn→∞

∫ π

−πsin (nx) f (x) dx = 0

Ind.: Identidad de Parseval.

4. (Calculando Productos) Calcular para n,m ∈ N∫ π

−πsin (mx) sin (nx) dx,

∫ π

−πsin (nx) dx∫ π

−πcos (nx) dx,

∫ π

−πcos (nx) cos (mx) dx∫ π

−πcos (nx) sin (mx) dx

¿Que dicen estos calculos del conjunto B = 1, cos (nx) , sin (nx) con n ∈ N en

SC[−π, π]?

293

Page 296: Apuntes MAT-023 USM

Apuntes Mat023 (Segundo semestre 2014)

5. (De lo general a lo particular) En SC [a, b] hemos definido el producto interior

〈f, g〉 =

∫ b

a

f (x) g (x) dx

y enunciado que el conjunto

C =

1, cos

(2nπx

b− a

), sin

(2nπx

b− a

)con n ∈ N

es una conjunto ortogonal. Escribir en los casos especiales SC[−π, π], SC[−L,L] y

SC[0, L]

6. (Calculando series de Fourier) Encontrar las series de Fourier para las funciones

siguientes en SC[−π, π]

a) x+ sinx

b) ex

c)

1 si x ≤ 0

x2 si x > 0

d)

x si x ≤ 0

x3 si x > 0

e) sin3x

7. (La identidad de Parseval) Utilizar la serie de Fourier de f (x) = x en [−π, π]

para calcular el valor de∞∑k=1

1

k2

8. (Funciones pares e impares) Clasificar las siguientes funciones en par, impar o

ninguna de las anteriores

A) tan x

B) xex2

C)x+ 1

x− 1

D) ln |x|

E) arcsinx

F) f (|x|) definida en [−1, 1] donde f : R→ R es una funcion cualquiera.

G) x cosx− cos 2x

294

Page 297: Apuntes MAT-023 USM

Apuntes Mat023 (Segundo semestre 2014)

9. (Parte par e impar de una serie) En Mat021 se mostro que toda funcion f :

[−a, a] → R (donde a ∈ R+) se puede descomponer en su parte par mas su parte

impar, esto es

f (x) = fp (x) + fi (x)

donde

fp (x) =f (x) + f (−x)

2

fi (x) =f (x)− f (−x)

2

Suponiendo que no hay problemas de convergencia, encontrar las partes par e impar

dea0

2+∞∑k=1

(ak cos (kx) + bk sin (kx))

donde ak, bk son constantes.

10. (Convergencia puntual) Encontrar el desarrollo en serie de Fourier de:

f (x) =

−1 −π < x < 0

1 0 < x < π

y mostrar que la serie evaluada en x = 0 converge a 0. Utilizando esta serie mostrar

queπ

4= 1− 1

3+

1

5− 1

7+ · · ·

11. (Series de Fourier en intervalos arbitrarios) Encontrar el desarrollo en serie de

Fourier para las siguientes funciones:

A) f (x) = x para x ∈ ]−π, π[

B) f (x) = ex para x ∈ ]0, 2π[

C) f (x) =

1 −π < x < 0

1/2 0 < x < π

D) f (x) = |sinx| para x ∈ ]−2π, 2π[

E) f (x) = x (L− x) para x ∈ ]0, L[

12. (Series de Fourier y series numericas) Encontrar el desarrollo en serie de Fourier

de la funcion

f (x) = cos (αx) para x ∈ ]−π, π[

para todos los valores de α ∈ R. Usar esta serie para demostrar que

cot (απ) =1

π

(1

α−∞∑k=1

k2 − α2

)siempre que α 6∈ Z.

295

Page 298: Apuntes MAT-023 USM

Apuntes Mat023 (Segundo semestre 2014)

13. (Combinaciones lineales de series) Si f, g ∈SC[a, b] son tales que

f (x) =a0

2+∞∑k=1

(ak cos

(2kπx

b− a

)+ bk sin

(2kπx

b− a

))y

g (x) =A0

2+∞∑k=1

(Ak cos

(2kπx

b− a

)+Bk sin

(2kπx

b− a

))A) Determinar la serie de Fourier de αf + βg.

B) Es posible mostrar que las series

1

2+

2

π

∞∑k=1

sin (2k − 1)x

2k − 1

y2

π

∞∑k=1

(−1)k+1

ksin (kx)

son las series de Fourier de las funciones

f1 (x) =

0 −π < x < 0

1 0 < x < π

y

f2 (x) =x

πpara x ∈ ]−π, π[

respectivamente, usar estas series y el la parte anterior del ejercicio para obtener

las series de las siguientes funciones:

1) f (x) =

1/2 −π < x < 0

−1/2 0 < x < π

2) f (x) =

x+ π −π < x < 0

x 0 < x < π

3) f (x) =

−x −π < x < 0

−x+ 2π 0 < x < π

14. (Ojo serie truncada) ¿Cual es el desarrollo en serie de Fourier de f (x) = 2 +

5 cos (3x)− 4 sin (7x) en SC[−π, π]?.

15. (Para trigonometricas mejor identidades) Encontrar la serie de Fourier de

f (x) = cos4 x en SC[−π, π]

16. (Extensiones pares e impares) Encontrar las extensiones pares e impares de las

siguientes funciones y trazar su grafica:

296

Page 299: Apuntes MAT-023 USM

Apuntes Mat023 (Segundo semestre 2014)

A) f (x) = 1 definida en ]0, 5[

B) f (x) = x2 definida en ]0, π[

C) f (x) = sin (x) definida en]0, π

4

[D) f (x) = ex definida en ]0, π[

17. (Series de senos) Encontrar la serie de Fourier de senos de las funciones

A) f (x) = cos x para x ∈ ]0, π[

B) f (x) = ex para x ∈ ]0, π[

C) f (x) = x2 para x ∈ ]0, π[

D) f (x) = x2 − x para x ∈ ]0, 1[

18. (Series de cosenos) Encontrar la serie de Fourier de cosenos de las funciones

A) f (x) = sinx para x ∈ ]0, π[

B) f (x) = ex para x ∈ ]0, π[

C) f (x) = x2 para x ∈ ]0, π[

D) f (x) = x− x3 para x ∈ [0, 4]

19. (Derivando Series) Si para x ∈ ]−π, π[

x =∞∑k=1

2 (−1)k+1

ksin (kx)

¿Es cierto que

1 =∞∑k=1

2 (−1)k+1 cos (kx)

para x ∈ ]−π, π[? En otras palabras ¿es posible derivar termino a termino una serie de

Fourier y asegurar que converge a la derivada de la funcion?. (Obs: Para poder derivar

una serie de Fourier y garantizar convergencia basta que la funcion sea continua

f (a) = f (b) y f ′, f ′′ ∈ SC[a, b] derivar por ejemplo la serie de Fourier de |x| para

x ∈ ]−π, π[)

20. (¿Cual serie usar?) Calcular el valor de

∞∑k=1

1

k2,∞∑k=1

1

k4,∞∑k=1

1

k6

usando series de Fourier adecuadas.

297

Page 300: Apuntes MAT-023 USM

Parte II

Calculo diferencial en varias variables

298

Page 301: Apuntes MAT-023 USM

Capıtulo 6 : Elementos de topologıa de Rn

El espacio euclidiano Rn

Definicion 6.1.1. Se define el espacio n−dimensional sobre el conjunto de los numeros

reales por:

Rn = x = (x1, x2, . . . , xn) : xi ∈ R, i = 1, 2, . . . , n

junto con la suma en Rn dada por:

x + y = (x1, x2, . . . , xn) + (y1, y2, . . . , yn)

= (x1 + y1, x2 + y2, . . . , xn + yn)

y la multiplicacion por escalar definida por:

αx = α (x1, x2, . . . , xn)

= (αx1, αx2, . . . , αxn)

para todos x,y ∈ Rn y α ∈ R.

Teorema 6.1.1. El espacio Rn es un espacio vectorial real de dimension n.

Producto interno y norma

Definicion 6.2.1. Sean x = (x1, . . . , xn) e y = (y1, . . . , yn) en Rn. Se define el producto

interno o producto escalar de x e y como el numero real:

〈x,y〉 =n∑i=1

xi yi

Observacion 6.2.1. El producto interno es el que confiere la nocion de distancia y de

perpendicularidad al espacio Rn.

Proposicion 6.2.1. Sean x,y, z ∈ Rn y α ∈ R, entonces:

1. Bilinealidad: 〈αx + y, z 〉 = α 〈x, z〉+ 〈y, z〉 y 〈x, αy + z〉 = α 〈x,y〉+ 〈x, z〉

2. Simetrıa: 〈x,y〉 = 〈y,x〉

3. Definido positivo: 〈x,x〉 ≥ 0 y 〈x,x〉 = 0 ⇐⇒ x = 0.

299

Page 302: Apuntes MAT-023 USM

Apuntes Mat023 (Segundo semestre 2014)

Definicion 6.2.2. Sea x = (x1, . . . , xn) ∈ Rn. Se define la norma (euclidiana) de x

como el numero real:

‖x‖ =√〈x,x〉

=

n∑i=1

x2i

1/2

Proposicion 6.2.2. Sean x ∈ Rn y α ∈ R, entonces:

1. ‖x‖ ≥ 0

2. ‖αx‖ = |α| ‖x‖

Teorema 6.2.1 (Desigualdad de Cauchy-Schwarz). Sean x,y ∈ Rn, entonces:

|〈x,y〉| ≤ ‖x‖ ‖y‖

Demostracion. Consideremos ϕ : R→ R definida por:

ϕ (λ) = 〈x+ λy, x+ λy〉

Note que:

ϕ (λ) = ‖x‖2 + 2λ 〈x, y〉+ λ2 ‖y‖2

Ahora bien, ϕ (λ) ≥ 0 es equivalente a que el discriminante de la expresion cuadratica

anterior sea negativa o cero. Es decir:

4 〈x, y〉2 − 4 ‖x‖2 ‖y‖2 ≤ 0

Luego, extrayendo raız cuadrada se obtiene:

|〈x, y〉| ≤ ‖x‖ ‖y‖

Observacion 6.2.2. La nocion de perpendicularidad caracterıstica de los espacios eucli-

dianos se obtiene de la desigualdad de Cauchy-Schwarz. En efecto, sean x,y ∈ Rn r 0,luego:

|〈x,y〉| ≤ ‖x‖ ‖x‖ ⇐⇒ |〈x,y〉|‖x‖ ‖y‖

≤ 1 ⇐⇒ −1 ≤ 〈x,y〉‖x‖ ‖y‖

≤ 1

Por tanto, debe existir un angulo θ ∈ [0, π) tal que:

cos θ =〈x,y〉‖x‖ ‖y‖

O bien, en su forma clasica:

〈x,y〉 = ‖x‖ ‖y‖ cos θ

300

Page 303: Apuntes MAT-023 USM

Apuntes Mat023 (Segundo semestre 2014)

Definicion 6.2.3. Sean x,y ∈ Rn. Se define el angulo entre x e y como:

∠ (x,y) = arc cos〈x,y〉‖x‖ ‖y‖

Ademas, diremos que x e y son perpendiculares u ortogonales si ∠ (x,y) = 0. En tal

caso, se anota x ⊥ y.

Teorema 6.2.2 (Pitagoras). Sean x,y ∈ Rn tales que x ⊥ y, entonces:

‖x + y‖2 = ‖x‖2 + ‖y‖2

Teorema 6.2.3. Sean x1,x2, . . . ,xm ∈ Rn r 0 con xi ⊥ xj, i 6= j y m ≤ n, entonces

x1,x2, . . . ,xm son linealmente independientes.

Observacion 6.2.3. Otra consecuencia de la desigualdad de Cauchy-Schwarz es la de-

sigualdad triangular. La desigualdad triangular es una propiedad muy importante de la

norma euclidiana y es la piedra angular de la nocion de distancia sobre el espacio Rn.

Teorema 6.2.4 (Desigualdad triangular). Sean x,y ∈ Rn, entonces:

‖x + y‖ ≤ ‖x‖+ ‖y‖

Demostracion. Por la desigualdad de Cauchy-Schwarz:

‖x+ y‖2 = 〈x+ y, x+ y〉= ‖x‖2 + 2 〈x, y〉+ ‖y‖2

≤ ‖x‖2 + 2 ‖x‖ ‖y‖+ ‖y‖2

(‖x‖+ ‖y‖)2

Lo cual implica el resultado.

Definicion 6.2.4. Sean x,y ∈ Rn. Se define la distancia entre x e y como el numero

real dado por:

d (x,y) = ‖x− y‖

Teorema 6.2.5. Sean x,y, z ∈ Rn, entonces:

1. d (x,y) ≥ 0; d (x,y) = 0 ⇐⇒ x = y

2. d (x,y) = d (y,x)

3. d (x, z) ≤ d (x,y) + d (y, z) (desigualdad triangular)

Observacion 6.2.4. La nocion de norma euclidiana se puede generalizar al concepto de

norma, como una funcion mas general N : Rn → R la cual debe cumplir:

301

Page 304: Apuntes MAT-023 USM

Apuntes Mat023 (Segundo semestre 2014)

1. N (x) ≥ 0

2. N (αx) = |α| N (x)

3. N (x + y) ≤ N (x) +N (y)

A modo de ejemplo, las normas mas comunes en Rn son:

‖x‖∞ = max |x1| , |x2| , . . . , |xn|

‖x‖1 =n∑i=1

|xi|

‖x‖p =

n∑i=1

|xi|p1/p

Ademas, se tiene que:

lımp→∞

‖x‖p = ‖x‖∞

Ası:

Teorema 6.2.6. Sean N y N1 dos normas cualesquiera sobre Rn. Entonces, existen

constantes α, β > 0 tales que:

αN1 (x) ≤ N (x) ≤ βN1 (x)

para todo x ∈ Rn.

Elementos de topologıa de Rn

Definicion 6.3.1. Sean a ∈ Rn y ε > 0. Llamaremos bola abierta de radio ε con

centro en a al conjunto definido por:

B (a, ε) = x ∈ Rn : ‖x− a‖ < ε

Ejemplo 6.3.1. Si n = 1, entonces:

B (a,ε) = (a− ε, a+ ε)

302

Page 305: Apuntes MAT-023 USM

Apuntes Mat023 (Segundo semestre 2014)

Si n = 2 y a = (x0, y0), entonces:

B (a, ε) =

(x, y) ∈ R2 :

√(x− x0)2 + (y − y0)2 < ε

y si n = 3 y a = (x0, y0, z0) entonces

B (a, ε) =

(x, y, z) ∈ R3 :

√(x− x0)2 + (y − y0)2 + (z − z0)2 < ε

303

Page 306: Apuntes MAT-023 USM

Apuntes Mat023 (Segundo semestre 2014)

Definicion 6.3.2. Sea U ⊆ Rn. Diremos que a es un punto interior de U si:

∃ε > 0, B (a, ε) ⊆ U

Llamaremos interior de U al conjunto de todos los puntos interiores de U , este conjunto

es representado por U .

Ejemplo 6.3.2. Muestre que (1, 2) es punto interior de

U =

(x, y) ∈ R2 : x+ y ≥ 1

pero (1,0) no lo es.

304

Page 307: Apuntes MAT-023 USM

Apuntes Mat023 (Segundo semestre 2014)

Solucion. El conjunto U corresponde a la region de la figura

notemos que la distancia de (1, 2) a la recta x+ y = 1 es

|1 + 2− 1|√1 + 4

=2

5

√5 >

4

5

se sigue que existe ε = 45

tal que

B ((1, 2) ; ε) = B

((1, 2) ;

4

5

)⊂ U

305

Page 308: Apuntes MAT-023 USM

Apuntes Mat023 (Segundo semestre 2014)

para el punto (1, 0) note que (1, 0) ∈ U pero para todo ε > 0, B ((1, 0) ; ε) 6⊆ U toda bola

abierta con centro (1, 0) contiene puntos que no pertenecen a U, por ejemplo el punto(1,− ε

2

)

Definicion 6.3.3. Sea U ⊆ Rn. Diremos que U es un conjunto abierto en Rn, o

simplemente abierto en Rn si U = U .

Observacion 6.3.1. U es abierto si:

∀a ∈ U, ∃ε > 0, B (a, ε) ⊆ U

Ejemplo 6.3.3. Los conjuntos ∅ y Rn son abiertos.

Ejemplo 6.3.4. Un conjunto finito A ⊆ Rn no es abierto.

Teorema 6.3.1. Toda bola abierta es un conjunto abierto.

Demostracion. Sean a ∈ Rn y ε > 0. Considere x ∈ B (a, ε) y:

δ = ε− ‖x− a‖ > 0

Sea, ahora, y ∈ B (x, δ). Luego, ‖y − x‖ < δ y ademas:

‖y − a‖ = ‖(y − x) + (x− a)‖≤ ‖y − x‖+ ‖x− a‖< δ + ‖x− a‖= ε− ‖x− a‖+ ‖x− a‖ = ε

Ası, ‖y − a‖ < ε, y por tanto, y ∈ B (a, ε). Esto implica que, B (x, δ) ⊆ B (a, ε). Es decir,

que todo punto x ∈ B (a, ε) es interior.

306

Page 309: Apuntes MAT-023 USM

Apuntes Mat023 (Segundo semestre 2014)

Ejemplo 6.3.5. Todo intervalo abierto en R es un conjunto abierto.

Ejemplo 6.3.6. Sea:

U =

(x, y) ∈ R2 : x > 2

Entonces, U es abierto.

Ejemplo 6.3.7. Si U = (x, y) ∈ R2 : x2 ≤ y

entonces U = (x, y) ∈ R2 : x2 < y

se sigue que el conjunto no es abierto.

307

Page 310: Apuntes MAT-023 USM

Apuntes Mat023 (Segundo semestre 2014)

Ejemplo 6.3.8. Considere el conjunto

A =

(x, y) ∈ R2 : y > x2

muestre que (0, 2) ∈A y (0, 0) ∈ ∂A.

Solucion. El conjunto A es la parte achurada del dibujo siguiente

En esta figura se muestran los puntos (2, 0) y (0, 0)

Primero mostraremos que (0, 2) ∈A. Consideremos r = 1 y mostremos

B ((0, 2) ; 1) ⊂ A

si (x, y) ∈ B ((0, 2) ; 1) entonces x2 + (y − 2)2 < 1 entonces

x2 ≤ x2 + (y − 2)2 < 1⇒ |x| < 1

tambien

(y − 2)2 ≤ x2 + (y − 2)2 < 1⇒ |y − 2| < 1

308

Page 311: Apuntes MAT-023 USM

Apuntes Mat023 (Segundo semestre 2014)

se sigue 1 < y < 3 entonces

|x|2 = x2 < 1 < y

ası

x2 < y

de donde obtenemos (x, y) ∈ A, ası B ((0, 2) ; 1) ⊂ A luego (0, 2) ∈A

Ahora mostremos que (0, 0) ∈ ∂A, sea r > 0 se tiene

B ((0, 0) , r) ∩ A 6= ∅B ((0, 0) , r) ∩ Ac 6= ∅

esto es claro pues el punto (0, r/2) ∈ B ((0, 0) , r) ∩ A y (0,−r/2) ∈ B ((0, 0) , r) ∩ Ac

Ejemplo 6.3.9. Sea U = (x, y) : y = 2x muestre queU = ∅.

Solucion. Sabemos queU ⊆ U , mostremos que un (x0, y0) ∈ U no puede ser punto interior.

Si r > 0 entonces

B ((x0, y0) , r) 6⊂ U

309

Page 312: Apuntes MAT-023 USM

Apuntes Mat023 (Segundo semestre 2014)

note que el punto(x0, y0 + r

2

)∈ B ((x0, y0) , r) pero

(x0, y0 + r

2

)6∈ U pues

y0 +r

2= 2x0 ⇒

r

2= 0

lo que es una contradiccion

Proposicion 6.3.1. Los conjuntos abiertos de Rn poseen las siguientes propiedades:

1. ∅ y Rn son abiertos.

2. La union arbitraria de conjuntos abiertos en Rn es un conjunto abierto en Rn.

3. La interseccion de un numero finito de conjuntos abiertos en Rn es un conjunto

abierto en Rn.

Ejemplo 6.3.10. Sea Hn =(a− 1

n, b+ 1

n

)⊆ R, para cada n ∈ N. Notamos que cada Hn

es un conjunto abierto en R, pero:

H =∞⋂n=1

Hn = [a, b]

310

Page 313: Apuntes MAT-023 USM

Apuntes Mat023 (Segundo semestre 2014)

el cual no es abierto.

Definicion 6.3.4. Sean U ⊆ Rn y a ∈ Rn. Diremos que a es un punto de adherencia

a U si:

∀ε > 0, B (a, ε) ∩ U 6= ∅

El conjunto de todos los puntos de adherencia de U se denomina la clausura de U y se

representa por U .

Observacion 6.3.2. Para todo U ⊆ Rn, U ⊆ U ⊆ U .

Ejemplo 6.3.11. La bola cerrada con centro en a y radio ε esta dada por:

B (a, ε) = x ∈ Rn : ‖x− a‖ ≤ ε

Definicion 6.3.5. Sea U ⊆ Rn. Diremos que U es un conjunto cerrado si U = U .

Ejemplo 6.3.12. Los conjuntos ∅ y Rn son cerrados.

Teorema 6.3.2. Sea U ⊆ Rn. Entonces, U es cerrado en Rn, si y solo si, UC es un conjunto

abierto.

Definicion 6.3.6. Sean U ⊆ Rn y a ∈ Rn. Diremos que a es un punto de acumulacion

de U si:

∀ε > 0, (B (a, ε) r a) ∩ U 6= ∅

El conjunto de todos los puntos de acumulacion de U se denomina el conjunto derivado

de U y se representa por U ′.

Ejemplo 6.3.13. Sea U = (0, 1] ∪ 2, 3. Luego, U ′ = [0, 1].

Ejemplo 6.3.14. Sea X =

1n

: n ∈ N

. Luego, X ′ = 0. Note que lo anterior quiere

decir que hay infinitos puntos distintos de X suficientemente cercanos a 0.

Observacion 6.3.3. Si U ⊆ Rn tal que U ′ 6= ∅, entonces U es infinito. Ası, para todo

conjunto finito A ⊆ Rn, se tiene que A′ = ∅.

Teorema 6.3.3. Sea U ⊆ Rn. Entonces, U es cerrado en Rn, si y solo si, U ′ ⊆ U .

Definicion 6.3.7. Sean U ⊆ Rn y a ∈ Rn. Diremos que a es un punto frontera de U si:

∀ε > 0, B (a, ε) ∩ U 6= ∅ ∧ B (a, ε) ∩ UC 6= ∅

El conjunto de todos los puntos frontera de U se llama la frontera de U y se anota como

∂U .

Observacion 6.3.4. Note que, desde la definicion, se obtiene que ∂U = U ∩ UC .

311

Page 314: Apuntes MAT-023 USM

Apuntes Mat023 (Segundo semestre 2014)

Ejemplo 6.3.15. Considere el conjunto

A = ]1, 2[× ]1, 3[

1. Muestre que A es abierto en R2.

El conjunto corresponde a un rectangulo sin los bordes como se muestra en la figura.

Supongamos que (u0, v0) ∈ A entonces 1 < u0 < 2 y 1 < v0 < 3 sea

r =1

2mın u0 − 1, 2− u0, v0 − 1, 3− v0

(la menor distancia a los bordes dividida por dos) entonces

B ((u0, v0) , r) ⊂ ]1, 2[× ]1, 3[

312

Page 315: Apuntes MAT-023 USM

Apuntes Mat023 (Segundo semestre 2014)

en efecto, si (x, y) ∈ B ((u0, v0) , r) entonces√(x− u0)2 + (y − v0)2 < r

luego

|x− u0| < r ⇔ x ∈ ]u0 − r, u0 + r[

y

|y − v0| < r ⇔ y ∈ ]v0 − r, v0 + r[

como r = 12

mın u0 − 1, 2− u0, v0 − 1, 3− v0 se sigue

r ≤ u0 − 1

2< u0 − 1⇒ 1 < u0 − r

r ≤ 2− u0

2< 2− u0 ⇒ r + u0 < 2

r ≤ v0 − 1

2< v0 − 1⇒ 2 < v0 − r

r ≤ 3− v0

2< 3− v0 ⇒ v0 + r < 3

se sigue

]u0 − r, u0 + r[ ⊂ ]1, 2[

]v0 − r, v0 + r[ ⊂ ]1, 3[

313

Page 316: Apuntes MAT-023 USM

Apuntes Mat023 (Segundo semestre 2014)

ası

(x, y) ∈ ]1, 2[× ]1, 3[

se sigue que todo punto de A es un punto interior luego es un conjunto abierto.

2. Determine A y ∂A.

Desarrollo:

A = [1, 2]× [1, 3]

corresponde al rectangulo con el borde y

∂A = 1 × [1, 3] ∪ 2 × [1, 3]

∪ [1, 2]× 1 ∪ [1, 2]× 3

son los bordes del rectangulo.

Ejemplo 6.3.16. La esfera con centro en a y radio ε > 0 se obtiene como:

S (a, ε) = ∂B (a, ε)

= x ∈ Rn : ‖x− a‖ = ε

Ası, la esfera unitaria Sn−1 en Rn se obtiene como:

Sn−1 = ∂B (0, 1)

= x ∈ Rn : ‖x‖ = 1

Definicion 6.3.8. Diremos que un conjunto X ⊆ Rn es acotado si existe un numero real

M > 0 tal que:

‖x‖ ≤M

para todo x ∈ X.

Observacion 6.3.5. Note que la cota anterior es uniforme, es decir: sup‖x‖ : x ∈

X≤M .

Ejemplo 6.3.17. El conjunto

U =

(x, y, z) ∈ R3 : x2 + (y − 1)2 + z2 ≤ 1 ∧ (x− 1)2 + y2 + z2 ≤ 1

es acotado pues si (x, y, z) ∈ U entonces, de la primera ecuacion se obtiene

x2 + z2 ≤ x2 + (y − 1)2 + z2 ≤ 1

y de la segunda

y2 ≤ (x− 1)2 + y2 + z2 ≤ 1

314

Page 317: Apuntes MAT-023 USM

Apuntes Mat023 (Segundo semestre 2014)

se sigue (x2 + z2

)+ y2 ≤ 2

ası

‖(x, y, z)‖ =√x2 + z2 + y2 ≤

√2

se sigue

sup ‖(x, y, z)‖(x,y,z)∈U

≤√

2

en otras palabras

U ⊆ B[(0, 0, 0) ,

√2]

U esta contenido en la bola cerrada de centro (0, 0, 0) y radio√

2.

Definicion 6.3.9 (Heine-Borel). Un conjunto K ⊆ Rn se dice compacto si es cerrado y

acotado.

Ejemplo 6.3.18. La bola cerrada B (a, ε) es un conjunto compacto sobre en Rn. En efecto,

como B (a, ε) es cerrado, basta verificar que es acotado. Note que para todo x ∈ B (x, ε),

se tiene que:

‖x‖ ≤ ‖x− a‖+ ‖a‖ < ε+ ‖a‖

Ejemplo 6.3.19. Sean L ⊆ Rn yK ⊆ Rm conjuntos compactos, respectivamente. Entonces,

L×K es compacto en Rn+m = Rn × Rm

Definicion 6.3.10. Un conjunto A ⊆ Rn se llama disconexo si existen dos conjuntos

abiertos U y V de Rn tales que:

1. U ∩ A 6= ∅ ∧ V ∩ A 6= ∅.

2. (U ∩ A) ∩ (V ∩ A) = ∅.

3. (U ∩ A) ∪ (V ∩ A) = A

Por otro lado, un conjunto A ⊆ Rn se dice conexo si no es disconexo.

Ejemplo 6.3.20. N y Q son disconexos.

Ejemplo 6.3.21. R es conexo.

Ejemplo 6.3.22. Todo intervalo real es conexo.

Teorema 6.3.4. Sean A ⊆ Rn un conjunto conexo y C ⊆ Rn tales que A ⊆ C ⊆ A,

entonces C es conexo.

315

Page 318: Apuntes MAT-023 USM

Apuntes Mat023 (Segundo semestre 2014)

Ejercicios del capıtulo

1. Muestre que un plano en R3 es cerrado y su interior es vacıo.

2. En los siguientes conjuntos, determine cuales son: Abiertos, Cerrados. Determine en

cada caso el interior, la clausura y frontera del conjunto.

a) (x, y) ∈ R2 / y = 2x b) x ∈ Rn : ‖x‖ ≤ 1c) (x, y, z) ∈ R3 / z < x2 + y2 d) (x, y, z) ∈ R3 / x = 0e) Una recta en R2 o R3 f) Un plano en R3

g) (x, y, z) ∈ R3 / x = y = z = (−1)n, n ∈ N h) (x, y, z) ∈ R3 / x < y

i) (x, y) ∈ R2 / x =1

n+ 1, n ∈ N j) x ∈ Rn : ‖x‖ = 1

3. Demuestre rigurosamente que:

a) (0, 1) no es un punto interior de A = (x, y) ∈ R2 / |x|+ |y| < 1.

b) (0, 0) es un punto interior de A.

c) (1, 0) es un punto de acumulacion de A.

d)

(1

2,1

2

)es un punto de frontera de A.

4. Probar que B (a; r) = x ∈ Rn : ‖x− a‖ < r es abierto en Rn.

5. Si A es un conjunto cerrado que contiene cada numero racional r ∈ [0, 1], probar que

[0, 1] ⊆ A.

6. Encuentre un conjunto en R2 que no sea Abierto ni Cerrado.

7. Sea R = [−1, 2]× [3, 4[

a) ¿Es Abierto?, ¿Es Cerrado?

b) Determine R.

8. Si K = [0, 1]× [0, 1] ∪ (2, 2)

a) ¿Es Abierto?

b) ¿Es cerrado?

9. Sea Ai : i ∈ I una coleccion de subconjuntos abiertos de Rn muestre que:

a) ∪i∈IAi es un conjunto abierto (la union arbitraria de abiertos es un abierto)

316

Page 319: Apuntes MAT-023 USM

Apuntes Mat023 (Segundo semestre 2014)

b) ∩i∈I′Ai es un conjunto abierto, donde I ′ ⊆ I es un conjunto finito (la interseccion

finita de abiertos es un abierto)

c) Muestre que la interseccion arbitraria de abiertos no es necesariamente un

abierto, para ello considere en Rn la coleccion de bolas abiertas B(θ; 1

n

)con

n ∈ N.

10. Encontrar A, A,A′ y ∂A si

A =

(x, y) ∈ R2 : x > 0, y > 0∪ (−1,−1)

11. Si T : Rn → Rm es una transformacion lineal y P ∈ Rn, r > 0 ¿En que consiste el

conjunto T (B (P ; r))?¿Es abierto?

12. Muestre que p ∈ A si y solo si existe una sucesion de puntos en A, digamos ann∈Ntal que an → p (esto es

lımn→∞

d (an, p) = 0

en el sentido de los limites de sucesiones en R)

13. Considere el subconjunto de R

U =

x ∈ R : x =

1

n+

1

mpara algun n,m ∈ N

encontrar U, U, U ′ y ∂U .

14. Muestre que si U y V son abiertos en R entonces U × V es abierto en R2.

15. Sean U, V subconjuntos de Rn probar:

a) U ⊂ V ⇒ U ⊂ V

b) U ∪ V = U ∪ V

16. Si U ⊆ Rn muestre que:

a) U y ∂U son disjuntos y U = U ∪ ∂Ub) Muestre que la frontera es vacıa si y solo si el conjunto es abierto y cerrado.

c) Muestre que U es abierto si y solo si ∂U = U − U

17. Si U es abierto ¿Es verdad que U =(U)

?

18. Muestre que todo abierto de R2 es una union numerable de cajas abiertas de la forma

]a, b[× ]c, d[ y tambien es una union numerable de bolas abiertas.

317

Page 320: Apuntes MAT-023 USM

Capıtulo 7 : Funciones de varias variables

Definiciones basicas

Definicion 7.1.1. Consideremos una funcion f : U ⊆ Rn → Rm. Diremos que:

1. f es una funcion real de varias variables si n ≥ 2 y m = 1.

2. f es una funcion vectorial de una variable real si n = 1 y m ≥ 2.

3. f es una funcion vectorial de varias variables si n ≥ 2 y m ≥ 2.

En general, las funciones reales de varias variables se denotan con letras minusculas,

como por ejemplo: f, g, h, etc. y las funciones vectoriales se anotan con letras mayusculas,

tales como: F,G,H, etc.

Ejemplo 7.1.1. Son funciones reales de varias variables las siguientes funciones:

1. f : R2 → R, (x, y) 7→ f (x, y) = x2 + 2y.

2. g : R3 → R, (x, y, z) 7→ g (x, y, z) = sin (xyz).

3. h : R3 → R, (x, y, z) 7→ h (x, y, z) = e−12(x2+y2+z2).

Ejemplo 7.1.2. Un ejemplo importante de funcion de varias variables corresponde a la

k−esima proyeccion sobre R de un vector x = (x1, x2, . . . , xn) ∈ Rn. Mas precisamente, se

define la k−esima proyeccion de x ∈ Rn como la funcion πk : Rn → R definida por:

πk (x) = πk (x1, x2, . . . , xn) = xk

Esta funcion, en particular, nos permitira representar funciones tales como:

f (x, y, z) =2x2 + y3

x2 + z2 + 1

como algebra de proyecciones. Es decir, podemos escribir f (x, y, z) en la forma:

f (x, y, z) =2π2

1 (x, y, z) + π32 (x, y, z)

π21 (x, y, z) + π2

3 (x, y, z) + 1

Ejemplo 7.1.3. Son funciones vectoriales las siguientes funciones:

1. F : R→ R2, t 7→ F (t) = (C1et, C2e

−t ), con C1, C2 ∈ R.

2. G : R→ R3, t 7→ G (t) = (α cos t, α sin t, βt), con α, β > 0.

3. H : R2 → R3, (x, y) 7→ H (x, y) = (ln (x2 + y2 + 1) , cos (x+ y) , x+ 3y).

318

Page 321: Apuntes MAT-023 USM

Apuntes Mat023 (Segundo semestre 2014)

Ejemplo 7.1.4. Notamos que, si en el ejemplo (3) anterior definimos:

f1 (x, y) = ln(x2 + y2 + 1

)f2 (x, y) = cos (x+ y)

f3 (x, y) = x+ 3y

entonces:

H (x, y) = (f1 (x, y) , f2 (x, y) , f3 (x, y))

donde

fi (x, y) = (πi H) (x, y)

Ejemplo 7.1.5. Sean fi : Ui ⊆ Rn → R funciones de varias variables con dominio Ui, con

i = 1, 2, . . . ,m. Suponga que:

U =m⋂i=1

Ui

Entonces, la transformacion F : U ⊆ Rn → Rm definida por:

F (x) = (f1 (x) , f2 (x) , . . . , fm (x)) , x = (x1, x2, . . . , xn) ∈ U

es una funcion vectorial de varias variables. Las funciones reales de varias variables fi se

llaman funciones componentes de F.

Definicion 7.1.2. Sea f : D ⊆ Rn → R, (x1, x2, . . . , xn)→ f (x1, x2, . . . , xn). Llamaremos

dominio maximo de f al conjunto:

Dom ( f) = x ∈ Rn : f (x) ∈ R

Ası mismo, el dominio maximo de F : D ⊆ Rn → Rm, x→ F (x) = (f1 (x) , f2 (x) , . . . , fm (x))

es dado por:

Dom (F) =m⋂i=1

Dom ( fi)

Ejemplo 7.1.6. Hallar el dominio maximo de:

f (x, y) =

√x2 + y2 − 1

ln (|x| − y)

Solucion. El dominio maximo en R2 para que esta expresion represente una funcion a

valores reales en tal conjunto es

Dom ( f) =

(x, y) ∈ R2 : x2 + y2 − 1 ≥ 0 ∧ |x| − y > 0 ∧ |x| − y 6= 1

319

Page 322: Apuntes MAT-023 USM

Apuntes Mat023 (Segundo semestre 2014)

Ejemplo 7.1.7. Si

F (x, y, z) =

(sin (xyz)

xyz, y cos

(1

x

))Determine el dominio maximo de F.

Solucion. El dominio maximo corresponde a

Dom (F) =

(x, y, z) ∈ R3 : xyz 6= 0

Definicion 7.1.3. El recorrido de F : U ⊆ Rn → Rm se define como:

Rec ( F) = y ∈ Rm : ∃x ∈ U, y = F (x)

Observacion 7.1.1. En general, para funciones de varias variables es difıcil calcular el

recorrido. Sin embargo, puede ser util tomar restricciones. Esto es imagenes directas de

conjuntos adecuados.

Ejemplo 7.1.8. El recorrido de f : R2 → R, (x, y)→ f (x, y) = xy2 + yx2 es todo R. En

efecto, si

T =

(x, y) ∈ R2 : x = y

= (x, x) : x ∈ R

Entonces, f∣∣T

= x · x2 + x · x2 = 2x3, se sigue que f (T ) = R luego R =f (T ) ⊆Rec(f) esto

implica rec (f) = R.

Ejemplo 7.1.9. El recorrido de f : R2 − (0, 0) → R, (x, y)→ f (x, y) = x3y2

x2+y2es R. En

efecto f (1, 0) = 0 luego 0 ∈Rec(f). Sea

T =

(x, y) ∈ R2 : y = x, x 6= 0

entonces

f (x, y)∣∣T

=x5

2x2=x3

2

se sigue

R−0 = f (T )

ası

R−0 = f (T ) ⊆ Rec (f) ⊆ R

y 0 ∈Rec(f) se sigue

Rec (f) = R

320

Page 323: Apuntes MAT-023 USM

Apuntes Mat023 (Segundo semestre 2014)

Graficos, conjuntos de nivel y trazas

Definicion 7.2.1. Sea f : U ⊆ Rn → R una funcion. Llamaremos grafico de f al conjunto

definido por:

Graf (f) =

(x, f (x)) ∈ Rn+1 : x ∈ U

Observacion 7.2.1. Si f : U ⊆ R2 → R es una funcion real de dos variables reales,

diremos que:

z = f (x, y) , (x, y) ∈ Ues una superficie en R3.

Observacion 7.2.2. Respecto de lo anterior, hay algunas superficies clasicas que es

conveniente reconocer. Tales superficies son conocidas como superficies cuadricas.

Definicion 7.2.2. Una superficie cuadrica es una ecuacion del tipo:

P (x, y, z) = 0

donde P (x, y, z) es un polinomio de segundo grado en tres variables. En particular, se

consideran la superficies cuadricas en su forma normal, es decir, considerando las ecuaciones:

Ax2 +By2 + Cz2 +D = 0

o bien:

Ax2 +By2 + Cz = 0

Observacion 7.2.3. Las superficies cuadricas mas importantes son las siguientes:

1. Esfera:

x2 + y2 + z2 = r2

321

Page 324: Apuntes MAT-023 USM

Apuntes Mat023 (Segundo semestre 2014)

2. Elipsoide:

x2

a2+ y2

b2+ z2

c2= 1, a, b, c 6= 0

3. Hiperboloide de una hoja:

x2

a2+ y2

b2− z2

c2= 1, a, b, c 6= 0

322

Page 325: Apuntes MAT-023 USM

Apuntes Mat023 (Segundo semestre 2014)

4. Hiperboloide de dos hojas:

−x2

a2+ y2

b2− z2

c2= 1, a, b, c 6= 0

5. Paraboloide:

x2

a2+ y2

b2= cz, a, b 6= 0 ∧ c > 0

323

Page 326: Apuntes MAT-023 USM

Apuntes Mat023 (Segundo semestre 2014)

6. Paraboloide hiperbolico:

x2

a2− y2

b2= cz, a, b 6= 0 ∧ c > 0

Definicion 7.2.3. Sea f : U ⊆ Rn → R y c ∈ R. Llamaremos conjunto de nivel c de f

al conjunto definido por:

Lc (f) = x ∈ U : f (x) = c ⊆ Rn

En particular, si n = 2 diremos que Lc (f) es la curva de nivel c de f . Si n = 3, diremos

que Lc (f) es la superficie de nivel c de f.

Observacion 7.2.4. Note que:

Lc (f) = f−1 (c)

Observacion 7.2.5. De los conjuntos anteriores, nos interesa el aspecto grafico para n = 2

y n = 3. La coleccion de los graficos superpuestos de un numero adecuado de curvas de

nivel o superficies de nivel para una funcion permite esbozar el grafico de la funcion dada.

Ejemplo 7.2.1. Las curvas de nivel c de la funcion f (x, y) = x2 + y2 son circunferencias

de centro en (0, 0) y radio√c para c > 0. Para c = 0 es solo un punto y para c < 0 es

conjunto vacıo.

Ejemplo 7.2.2. Las curvas de nivel c de la funcion f (x, y) =√x2 + y2 son circunferencias

de centro en (0, 0) y radio c para c > 0. Para c = 0 es solo un punto y para c < 0 es

conjunto vacıo.

324

Page 327: Apuntes MAT-023 USM

Apuntes Mat023 (Segundo semestre 2014)

Observacion 7.2.6. Se puede observar que para los ejemplos anteriores las curvas de nivel

son, basicamente, las mismas, es decir, cırculos concentricos desde el origen. En este caso,

las curvas de nivel no son suficiente para determinar el grafico de las funciones anteriores.

Debemos considerar la nocion de traza.

Definicion 7.2.4. Llamaremos traza de una superficie S : z = f (x, y) a la interseccion

de dicha superficie con alguno de los planos coordenados. Denotaremos las trazas de una

superficie S mediante los sımbolos Ty y Tx, si la interseccion se efectua con los planos xz e

yz, respectivamente.

Ejemplo 7.2.3. Por ejemplo, las trazas de las superficies S1 : z = x2 + y2 y S2 : z =√x2 + y2 estan dadas por:

(x, y, z) ∈ Ty ⇐⇒ (x, y, z) ∈ S1 ∩ (x, y, z) : y = 0 ⇐⇒ z = x2, y = 0

para S1, y para S2 por:

(x, y, z) ∈ Ty ⇐⇒ (x, y, z) ∈ S1 ∩ (x, y, z) : y = 0 ⇐⇒ z = |x| , y = 0

En particular, son algunas trazas las que nos permiten distinguir los graficos de las

superficies S1 y S2.

Ejemplo 7.2.4. Sea f : D ⊆ R2 → R definida por

f (x, y) =

√x2 + y2 − 1√4− x2 − y2

1. Determine el dominio D de f .

D =

(x, y) ∈ R2 : x2 + y2 − 1 ≥ 0 ∧ 4− x2 − y2 > 0

=

(x, y) ∈ R2 : x2 + y2 ≥ 1 ∧ 4 > x2 + y2

=

(x, y) ∈ R2 : 1 ≤ x2 + y2 < 4

2. Demostrar que(

32, 0)∈D,(1,√

3)∈ ∂D y (0, 0) 6∈ D.

Claramente B((

32, 0)

; 14

)⊂ D luego el punto es punto interior. Si r > 0 la bola

B((

1,√

3)

; r)

contiene puntos del conjunto y puntos del complemento y finalmente

B((0, 0) ; 1

2

)∩D = ∅.

3. Describir los conjuntos de nivel de f y graficar la funcion.

325

Page 328: Apuntes MAT-023 USM

Apuntes Mat023 (Segundo semestre 2014)

Los conjuntos de nivel de f son

Nc =

(x, y) ∈ R2 :

√x2 + y2 − 1√4− x2 − y2

= c

=

(x, y) ∈ R2 :

√x2 + y2 − 1 = c

√4− x2 − y2

=

(x, y) ∈ R2 : x2 + y2 − 1 = c2

(4− x2 − y2

)=

(x, y) ∈ R2 :

(1 + c2

)x2 +

(1 + c2

)y2 = 1 + 4c2

=

(x, y) ∈ R2 : x2 + y2 =

1 + 4c2

1 + c2

son circunferencias de radio

√1+4c2

1+c2y centro (0, 0). Por otro lado, si y = 0 se tiene

z =

√x2 − 1√4− x2

que tiene por grafica

ası la grafica es

326

Page 329: Apuntes MAT-023 USM

Apuntes Mat023 (Segundo semestre 2014)

Ejercicios del capıtulo

1. Encontrar los dominios de las siguientes funciones:

a) f : D ⊆ R2→ R (x, y)→√y − x2

x2 − y2

b) f : D ⊆ R2→ R (x, y)→√y − x2 ln (1− x2 + y2)

ln (x2 − y2)

c) f : D ⊆ R2→ R3 (x, y)→

(x

x+ y,

√x

x+ y2,

x2

|x| −√x2 + y2

)2. Esbozar curvas de nivel y graficas de las siguientes funciones:

a) f : R2→ R (x, y)→ x− y + 2

b) f : R2→ R (x, y)→ x2 + 9y2

c) f : R2→ R (x, y)→ max |x| , |y|

3. Describir el comportamiento de las curvas de nivel f (x, y) = c para c en R con:

a) f (x, y) = x2 + 4y2 + 5 b) f (x, y) = 4− x2 − y2

c) f (x, y) = |8− |x2 + y2 − 4||+ 1 d) f (x, y) = (x2 + y2) e−(x2+y2)

4. Considere la funcion f (x, y) = x2 + y2 y defina para θ ∈[0, π

2

[el conjunto

Sθ =

(x, y, z) ∈ R3 : y = (tan θ)x

ver la forma de la curva G (f)∩Sθ donde G (f) corresponde a la grafica de la funcion

f . Hacer lo mismo con la funcion f (x, y) =√x2 + y2 + ex

2+y2 .

5. Describir los conjuntos de nivel de las funciones:

a) f : R3→ R (x, y, z)→ −x2 − y2 − z2

b) f : R3→ R (x, y, z)→ 4x2 − y2 + 9z2

c) f : R3→ R (x, y, z)→ z − x2 − 4y2

6. Representar en R3 los conjuntos:

a) A =

(x, y, z) ∈ R3 : z2 +

y2

4= 1

b) B =

(x, y, z) ∈ R3 : x = y2

c) B =

(x, y, z) ∈ R3 : x2 + y2 = 1 ∧ x+ y + z = 0

327

Page 330: Apuntes MAT-023 USM

Apuntes Mat023 (Segundo semestre 2014)

7. Utilizando coordenadas polares describir las curvas de nivel de la funcion

f (x, y) =

2xy

x2 + y2si (x, y) 6= (0, 0)

0 si (x, y) = (0, 0)

328

Page 331: Apuntes MAT-023 USM

Capıtulo 8 : Lımites y continuidad

Definiciones

Definicion 8.1.1. Sean U ⊆ Rn, a ∈ U ′ y f : U → R una funcion. Diremos que el numero

real L es el lımite de f (x) cuando x tiende a a si:

∀ ε > 0,∃ δ > 0, 0 < ‖x− a‖Rn < δ =⇒ |f (x)− L| < ε (8.1)

Lo anterior se representa mediante el sımbolo:

lımx→a

f (x) = L

Observacion 8.1.1. En particular, si f : U ⊆ R2 → R y (a, b) ∈ U ′, la definicion de lımite

en (8.1) queda como:

∀ ε > 0,∃ δ > 0, 0 <

√(x− a)2 + (y − b)2 < δ =⇒ |f (x, y)− L| < ε

y en sımbolos:

lım(x,y)→(a,b)

f (x, y) = L

Usualmente, a los lımites del tipo anterior, se les conoce como lımites dobles.

Ejemplo 8.1.1. Demuestre que:

lım(x,y)→(2,−1)

(3x+ 2y) = 4

Solucion. Por demostrar que, dado cualquier ε > 0, existe δ > 0 tal que si:

0 <

√(x− 2)2 + (y + 1)2 < δ

implica que:

|3x+ 2y − 4| < ε

En efecto, note que:

|3x+ 2y − 4| = |3 (x− 2) + 2 (y + 1)|≤ 3 |x− 2|+ 2 |y + 1|< 3δ + 2δ = 5δ

Por tanto, dado ε > 0, existe δ = ε/5 tal que si 0 <√

(x− 2)2 + (y + 1)2 < δ, entonces

|3x+ 2y − 4| < ε. Por tanto, se concluye que lım(x,y)→(2,−1) (3x+ 2y) = 4.

329

Page 332: Apuntes MAT-023 USM

Apuntes Mat023 (Segundo semestre 2014)

Ejemplo 8.1.2. Demuestre que:

lım(x,y)→(0,0)

x2 cos (x2 + y2)√x2 + y2

= 0

Solucion. Note que si (x, y) 6= (0, 0)∣∣∣∣∣x2 cos (x2 + y2)√x2 + y2

− 0

∣∣∣∣∣ =|x|2√x2 + y2

∣∣cos(x2 + y2

)∣∣≤ |x| |x|√

x2 + y2

≤√x2 + y2

√x2 + y2√

x2 + y2

=√x2 + y2 = δ

por lo tanto, dado ε > 0 existe δ = ε tal que si 0 <√

(x− 0)2 + (y − 0)2 < δ entonces∣∣∣∣∣x2 cos (x2 + y2)√x2 + y2

− 0

∣∣∣∣∣ < ε.

Observacion 8.1.2. Es importante destacar que la nocion de lımite es un concepto que

puede tratarse de manera mas general que en el caso de funciones reales de varias variables.

En particular, el concepto de lımite se puede extender a funciones vectoriales considerando

la norma correspondiente al espacio de llegada. Mas precisamente, tenemos:

Definicion 8.1.2. Sean U ⊆ Rn, a ∈ U ′ y F : U → Rm una funcion vectorial. Diremos

que el vector L ∈ Rm es el lımite de F (x) cuando x tiende a a si:

∀ ε > 0,∃ δ > 0, 0 < ‖x− a‖Rn < δ =⇒ ‖F (x)− L‖Rm < ε

Sin embargo, para nuestro propositos de calculo, es util el siguiente teorema:

Teorema 8.1.1. Sean U ⊆ Rn, x ∈ U ′ y F : U → Rm una funcion vectorial tal que:

F (x) = (f1 (x) , f2 (x) , . . . , fm (x))

Entonces, el vector L = (L1, L2, . . . , Lm) ∈ Rm es el lımite de F (x) cuando x tiende a a,

si y solo si:

lımx→a

fi (x) = Li

para todo i = 1, 2, . . . ,m. En otras palabras, F (x) tiende al vector L cuando x tiende a

a, si y solo si, la convergencia se da en cada coordenada de F (x) a la correspondiente

coordenada de L.

330

Page 333: Apuntes MAT-023 USM

Apuntes Mat023 (Segundo semestre 2014)

La demostracion de este resultado se sigue de la desigualdad

|fi (x)− Li| ≤ ‖(f1 (x) , f2 (x) , . . . , fm (x))− (L1, L2, . . . , Lm)‖

≤√m

m∑i=1

|fi (x)− Li|

Observacion 8.1.3. En vista del resultado anterior, centraremos nuestra atencion en

funciones reales de varias variables.

Teorema 8.1.2. Sean F : U ⊆ Rn → Rm una funcion vectorial y a ∈ U ′. Suponga que:

lımx→a

F (x) = L ∧ lımx→a

F (x) = M

entonces, L = M.

Demostracion. Sea ε > 0. Por hipotesis, existen δ1, δ2 > 0 tales que:

0 < ‖x− a‖ < δ1 =⇒ ‖f (x)− L‖ < ε

2

y:

0 < ‖x− a‖ < δ2 =⇒ ‖f (x)−M‖ < ε

2

Considerando, δ = mın δ1, δ2, se tiene que:

‖L−M‖ = ‖(L− f (x)) + (f (x)−M)‖≤ ‖f (x)− L‖+ ‖f (x)−M‖<

ε

2+ε

2= ε

Lo anterior implica que L = M .

Este resultado nos entregara una tecnica para demostrar la no existencia de ciertos

lımites.

Ejemplo 8.1.3. Determine si acaso existe el siguiente lımite:

lım(x,y,z)→(0,0,0)

sin (xy) + z

x2 + y2 + |z|

Solucion. Anotemos f (x, y, z) = sin(xy)+zx2+y2+|z| y consideremos el conjunto:

T =

(x, y, z) ∈ R3 : x = y = 0 ∧ z 6= 0

Ahora bien:

f∣∣T

=z

|z|=

1 , z > 0

−1 , z < 0

Por unicidad del lımite, se concluye que el lımite lım(x,y,z)→(0,0,0)sin(xy)+zx2+y2+|z| no existe.

331

Page 334: Apuntes MAT-023 USM

Apuntes Mat023 (Segundo semestre 2014)

Ejemplo 8.1.4. Sea f : R2 r (0, 0) → R una funcion de dos variables definida por:

f (x, y) =xy

x2 + y2

Considere:

Tm =

(x, y) ∈ R2 : y = mx

con m 6= 0. Note que: (f∣∣Tm

)(x, y) = f (x,mx)

=mx2

x2 +m2x2

=m

1 +m2

Notamos que si (x, y) → (0, 0) a traves de Tm, es decir, si consideramos el lımite

lım(x,y)→(0,0)(x,y)∈Tm

f (x, y), se tiene que:

lım(x,y)→(0,0)(x,y)∈Tm

f (x, y) = lımx→0

f (x,mx)

= lımx→0

m

1 +m2

=m

1 +m2

y por tanto, se puede concluir que el lımite de f (x, y) no existe, pues depende directamente

de la pendiente de la recta de aproximacion al origen y = mx.

Ejemplo 8.1.5. Considere el lımite

lım(x,y)→(0,0)

x2y

x4 + y2

note que

lım(x,y)→(0,0)

y=0

x2y

x4 + y2= lım

x→0

x20

x4 + 02= lım

x→00 = 0

lım(x,y)→(0,0)

x=0

x2y

x4 + y2= lım

y→0

02y

04 + y2= lım

y→00 = 0

si tomamos otra recta de la forma y = mx (con m 6= 0) entonces

lım(x,y)→(0,0)y=mx

x2y

x4 + y2= lım

x→0

x2 (mx)

x4 + (mx)2

= lımx→0

mx

x2 +m2= 0

332

Page 335: Apuntes MAT-023 USM

Apuntes Mat023 (Segundo semestre 2014)

se sigue que por todas las rectas que contienen al origen el lımite es cero, note que estas

rectas cubren todos los puntos del plano pero esto NO implica que el lımite sea cero,

la razon de esto es que las rectas no son la unicas trayectorias por las cuales podemos

acercarnos al origen, en efecto, al tomar la trayectoria y = x2 se tiene

lım(x,y)→(0,0)

y=x2

x2y

x4 + y2= lım

x→0

x2x2

x4 + x4

= lımx→0

1

2

=1

2

se sigue que el lımite no existe.

Lo anterior se puede generalizar introduciendo la idea de camino en Rn. Consideremos:

Definicion 8.1.3. Un camino en Rn es una funcion ϕ : I → Rn, cuyo dominio es un

intervalo I ⊆ R. Es decir:

ϕ (t) = (ϕ1 (t) , ϕ2 (t) , . . . , ϕn (t)) , t ∈ I

y cada funcion componente ϕi : I ⊆ R→ R es continua en I.

Definicion 8.1.4. Sean U ⊆ Rn, a ∈ U ′ y un camino de la forma ϕ : [0, 1]→ U . Diremos

que ϕ (t) converge propiamente a a, cuando t→ 0+ si:

lımt→0+

ϕ (t) = a

y ϕ (t) 6= a, para todo t ∈ (0, 1]. Anotamos lo anterior mediante el sımbolo ϕ (t)p→ a.

Teorema 8.1.3. Sean L ∈ R, f : U ⊆ Rn → R y a ∈ U ′ tales que:

lımx→a

f (x) = L

entonces:

lımt→0+

f (ϕ (t)) = L

para todo camino ϕ : [0, 1]→ U tal que ϕ (t)p→ a.

se sigue de este teorema, que si por dos caminos distintos el limite es diferente o bien,

por un camino no existe entonces el limite general no existe.

Ejemplo 8.1.6. Muestre que el lımite:

lım(x,y)→(0,0)

x3 + y3

x− y

no existe

333

Page 336: Apuntes MAT-023 USM

Apuntes Mat023 (Segundo semestre 2014)

Solucion. Note que:

f (x, y) =x3 + y3

x− y=x3 − y3 + 2y3

x− y

=(x2 + xy + y2

)+

2y3

x− y

Ahora bien, suponga que:2y3

x− y= r

luego:

x = y +2

ry3

Defina:

ϕ (t) =

(t+

2

rt3, t

)entonces

lımt→0

ϕ (t) = (0, 0)

y

lımt→0

f (ϕ (t)) = lımt→0

(t2 + t

(t+

2

rt3)

+

(t+

2

rt3)2

+ r

)= r

luego tomando r = 1 y r = 2 obtenemos lımites distintos. El lımite no existe. La tecnica

empleada en este caso se conoce como la tecnica de los conjuntos de nivel.

Observacion 8.1.4. Otro procedimiento habitual para investigar acerca de la existencia

o inexistencia de un lımite es transformar mediante un cambio de variables adecuado el

espacio R2 completo. Es decir, considerando el cambio de coordenadas:x = r cos θ

y = r sin θ

la funcion f (x, y) = xyx2+y2

queda como:

f (x (r, θ) ; y (r, θ)) = f (r cos θ, r sin θ)

=r2 cos θ sin θ

r2(cos2 θ + sin2 θ

)= cos θ sin θ

Ahora bien, notando que si (x, y) → (0, 0) implica que r → 0, se observa que el lımite

anterior no existe, pues depende del angulo θ de entrada al origen (0, 0).

334

Page 337: Apuntes MAT-023 USM

Apuntes Mat023 (Segundo semestre 2014)

Ejemplo 8.1.7.

lım(x,y)→(0,0)

x3y2

(x2 + y2)2 = 0

Solucion. Usando coordenadas polares

x = r cos θ

y = r sin θ

se tiene

x3y2

(x2 + y2)2 =(r3 cos3 θ)

(r2 sin2 θ

)(r2)2

= r cos3 θ sin2 θ

se sigue

lım(x,y)→(0,0)

x3y2

(x2 + y2)2 = lımr→0

θ∈[0,2π[

(r cos3 θ sin2 θ

)= 0

Note tambien quex3y2

(x2 + y2)2 = r cos3 θ sin2 θ

implica ∣∣∣∣ x3y2

(x2 + y2)2

∣∣∣∣ =∣∣r cos3 θ sin2 θ

∣∣ ≤ r =√x2 + y2

luego podemos aplicar el teorema de acotamiento para demostrar que el lımite existe (ver

calculo de lımites).

Definicion 8.1.5. Sean f : U ⊆ R2 → R una funcion de dos variables y (a, b) ∈ U ′. Se

definen los lımites iterados de f como los lımites univariados:

lımx→a

(lımy→b

f (x, y)

)y

lımy→b

(lımx→a

f (x, y))

Teorema 8.1.4. Sean f : U ⊆ R2 → R una funcion y (a, b) ∈ U ′ tales que:

lım(x,y)→(a,b)

f (x, y) = L

Entonces, lımx→a (lımy→b f (x, y)) y lımy→b (lımx→a f (x, y)) existen, y ademas:

lımx→a

(lımy→b

f (x, y)

)= lım

y→b

(lımx→a

f (x, y))

= L

335

Page 338: Apuntes MAT-023 USM

Apuntes Mat023 (Segundo semestre 2014)

Ejemplo 8.1.8. Calcule los lımites iterados en (0, 0) de la funcion:

f (x, y) =xy

x2 + y2, (x, y) 6= (0, 0)

Solucion. Note que:

lımx→0

(lımy→0

f (x, y)

)= lım

x→00 = 0 = lım

y→0

(lımx→0

f (x, y))

Por tanto, ambos lımites iterados existen y tienen valor cero, sin embargo, sabemos que

f (x, y) no posee lımite en (0, 0).

Ejemplo 8.1.9. Determine la existencia del lımite:

lım(x,y)→(0,0)

sin (x+ y)

x+ 3y

Solucion. Usando lımites iterados

lımx→0

(lımy→0

sin (x+ y)

x+ 3y

)= lım

x→0

(sinx

x

)= 1

lımy→0

(lımx→0

sin (x+ y)

x+ 3y

)= lım

y→0

(sin y

3y

)=

1

3

como los lımites son distintos el lımite no existe.

Observacion 8.1.5. Este resultado se puede extender a funciones con mas variables.

Calculo de lımites

Observacion 8.2.1. En la seccion anterior, se establecieron procedimientos para indicar

que una funcion no posee lımite. Nuestro interes, ahora, se centra en aquellas funciones

que sı lo poseen. Por tanto, debemos entregar algunos elementos de calculo:

Algebra de lımites

Teorema 8.2.1. Sean f, g : U ⊆ Rn → R funciones tales que:

lımx→a

f (x) = L ∧ lımx→a

g (x) = M

para x ∈ U ′ entonces:

1. ∀α ∈ R, (αf (x)) = α lımx→a

f (x) = αL,

2. lımx→a

(f (x) + g (x)) = lımx→a

f (x) + lımx→a

g (x) = L+M

336

Page 339: Apuntes MAT-023 USM

Apuntes Mat023 (Segundo semestre 2014)

3. lımx→a

(f (x) g (x)) =(

lımx→a

f (x))(

lımx→a

g (x))

= LM

4. Si M 6= 0, lımx→a

(f (x) /g (x)) = lımx→a

f (x) / lımx→a

g (x) = L/M

Ejemplo 8.2.1. Calcule:

lım(x,y)→(0,0)

sin (3xy)

sinx sin y

Solucion. Usaremos el lımite fundamental de una variable lımu→0sinuu

= 1

lım(x,y)→(0,0)

sin (3xy)

sinx sin y= lım

(x,y)→(0,0)

sin(3xy)3xy

sinx sin y3xy

= lım(x,y)→(0,0)

sin(3xy)3xy

13

(sinxx

) (sin yy

)= 3

Ejemplo 8.2.2. Calcule:

lım(x,y)→(0,0)

1− cosxy

x2y sin y

Solucion. Recordemos que

lımu→0

1− cosu

u2=

1

2

luego

lım(x,y)→(0,0)

1− cosxy

x2y sin y= lım

(x,y)→(0,0)

1− cosxy

x2y2(

sin yy

)= lım

(x,y)→(0,0)

1−cosxy

(xy)2(sin yy

)=

1

2

Desigualdades y Teorema del Sandwich o teorema de acotamiento

Observacion 8.2.2. Una herramienta muy importante para el calculo de lımites el teo-

rema del sandwich o teorema del acotamiento. Este teorema requiere el uso adecuado de

desigualdades notables, las mas usuales son:

1. Sea x = (x1, x2, . . . , xn) ∈ Rn, entonces |xi| ≤ ‖x‖, para todo i = 1, 2, . . . , n.

2. a) (a+ b)2 ≤ 2 (a2 + b2)

337

Page 340: Apuntes MAT-023 USM

Apuntes Mat023 (Segundo semestre 2014)

b) 2ab ≤ a2 + b2

3. |sinx| ≤ 1, para todo x ∈ R.

4. |sinx| ≤ |x|, para todo x ∈ R.

5. a, b, c > 0 =⇒ ab+c≤ a

b

Teorema 8.2.2. Sean f, g, h : U ⊆ Rn → R y a ∈ U ′ tales que:

f (x) ≤ g (x) ≤ h (x) , ∀x ∈ U

Suponga que lımx→a f (x) = lımx→a h (x) = L, entonces:

lımx→a

g (x) = L

Ejemplo 8.2.3. Verificaremos que:

lım(x,y)→(0,0)

sin2 y

x2 + |y|= 0

Solucion. En efecto, basta notar que:∣∣∣∣ sin2 y

x2 + |y|

∣∣∣∣ ≤ |y|2

x2 + |y|≤ |y|

2

|y|= |y|

se sigue que

− |y| ≤ sin2 y

x2 + |y|≤ |y|

como lım(x,y)→(0,0) |y| = 0 se sigue que

lım(x,y)→(0,0)

sin2 y

x2 + |y|= 0

Ejemplo 8.2.4. Calcular

lım(x,y)→(0,0)

x sin3 y

|x|+ y2

Solucion. Note que ∣∣∣∣ x sin3 y

|x|+ y2

∣∣∣∣ =|x| |sin y|3

|x|+ y2≤ |x| |y|

3

|x|+ y2≤ |x| |y|

ası

− |x| |y| ≤ x sin3 y

|x|+ y2≤ |x| |y|

se sigue por el teorema de acotamiento que

lım(x,y)→(0,0)

x sin3 y

|x|+ y2= 0

338

Page 341: Apuntes MAT-023 USM

Apuntes Mat023 (Segundo semestre 2014)

Ejemplo 8.2.5. Demostrar que

lım(x,y)→(0,0)

x3y2

(x2 + y2)2 = 0

Solucion. Usando coordenadas polares

x3y2

(x2 + y2)2 = r cos3 θ sin2 θ

luego ∣∣∣∣ x3y2

(x2 + y2)2

∣∣∣∣ =∣∣r cos3 θ sin2 θ

∣∣ ≤ r =√x2 + y2

ası

−√x2 + y2 ≤ x3y2

(x2 + y2)2 ≤√x2 + y2

y como

lım(x,y)→(0,0)

√x2 + y2 = 0

se sigue por el teorema de acotamiento que

lım(x,y)→(0,0)

x3y2

(x2 + y2)2 = 0

Continuidad

Definicion 8.3.1. Sean U ⊆ Rn, a ∈ U ′ ∩ U y f : U → R una funcion real de varias

variables. Diremos que f es continua en a si:

∀ ε > 0, ∃ δ > 0, ‖x− a‖Rn < δ =⇒ |f (x)− f (a)| < ε (8.2)

Ademas, diremos que f es continua en U si f es continua en cada punto de U .

Observacion 8.3.1. Note que la definicion dada anteriormente y por la proposicion en

(8.2)), dice que f es continua en a si:

lımx→a

f (x) = f (a)

Definicion 8.3.2. Diremos que f : U ⊆ Rn → R es discontinua en a ∈ U ∩ U ′, si f no

es continua en a.

Ejemplo 8.3.1. Sea πk : Rn → R la k−esima proyeccion de x ∈ Rn. Entonces, πk es

continua en Rn. En efecto, para cada a ∈ Rn, tenemos que:

|πk (x)− πk (a)| = |xk − ak| ≤ ‖x− a‖

bastara tomar δ = ε.

339

Page 342: Apuntes MAT-023 USM

Apuntes Mat023 (Segundo semestre 2014)

Ejemplo 8.3.2. Demuestre que f : R2 → R definida por:

f (x, y) =

xy2

x4 + y2, (x, y) 6= (0, 0)

0 , (x, y) = (0, 0)

es continua en (0, 0).

Solucion. Basta notar que: ∣∣∣∣ xy2

x4 + y2

∣∣∣∣ ≤ |x| y2

y2= |x|

Ası, si (x, y)→ (0, 0), entonces |x| → 0. Por el Teorema del Sandwich se concluye que:

lım(x,y)→(0,0)

xy2

x4 + y2= 0

como f (0, 0) = 0, se tiene que f es continua en (0, 0).

Ejemplo 8.3.3. Sean U = (x, y) : |x| < y2 ⊆ R2 y f : U → R definida por:

f (x, y) =

xy

x2 + y2, (x, y) ∈ U

0 , (x, y) /∈ U

¿Es continua en (0, 0)?

Solucion. Note que si (x, y) ∈ U

|f (x, y)− 0| =

∣∣∣∣ xy

x2 + y2

∣∣∣∣ =|x| |y|x2 + y2

≤ |y|3

x2 + y2

≤ |y|3

|y|2= |y|

y si (x, y) 6∈ U entonces

|f (x, y)− 0| = 0 ≤ |y|

se sigue que, para todo (x, y) ∈ R2

|f (x, y)− 0| ≤ |y|

y por el teorema de acotamiento

lım(x,y)→(0,0)

f (x, y) = 0

La funcion es continua en (0, 0).

340

Page 343: Apuntes MAT-023 USM

Apuntes Mat023 (Segundo semestre 2014)

Definicion 8.3.3. Se dice que una funcion f : U ⊆ Rn → Rm es Lipschitziana en U si:

∃K > 0, ∀x,y ∈ U, ‖f (x)− f (y)‖ ≤ K ‖x− y‖

La constante K se llama constante de Lipschitz.

Toda funcion lipschitziana es continua en U . En efecto, supongamos que f : U ⊆ Rn →Rm es una funcion lipschitziana en U y que ε > 0, entonces:

‖f (x)− f (a)‖ ≤ K ‖x− a‖ < Kδ

Ası, tomando δ = εK

, se verifica que f es continua en a. Como a ∈ U es cualquiera, se

obtiene, finalmente, la continuidad de f en U .

Ejemplo 8.3.4. Toda transformacion lineal T : Rn → Rm es una funcion lipschitziana.

Algebra de funciones continuas

El siguiente teorema facilita la identificacion de funciones continuas:

Teorema 8.4.1 (Algebra de funciones continuas). Sean f, g : U ⊆ Rn → R funciones

continuas en a ∈ U ′ ∩ U y α ∈ R, entonces αf, f + g, f − g y fg son continuas en a. Si

g (a) 6= 0, entonces f/g es continua en a.

Ejemplo 8.4.1. Son funciones continuas en R3:

1. f (x, y, z) = 3xy2 + z2

2. g (x, y, z) = sinx+sin y+sin zx2+y2+z2+1

3. h (x, y, z) = ex2

sin (y) + z2

Teorema 8.4.2. Sean f : U ⊆ Rn → Rm y g : V ⊆ Rm → Rp tales que f (U) ⊆ V .

Suponga, ademas, que f es continua en a ∈ U ′ ∩ U y que g es continua en b = f (a) ∈ V ′,entonces:

g f : U ⊆ Rn → Rp

es continua en a.

Demostracion. Sea ε > 0. Como g es continua en b = f (a), existe η > 0 de modo que si

y ∈ V , entonces:

‖y − b‖ < η =⇒ ‖g (y)− g (b)‖ < ε

341

Page 344: Apuntes MAT-023 USM

Apuntes Mat023 (Segundo semestre 2014)

Ahora bien, para y = f (x), existe δ > 0 tal que:

‖x− a‖ < δ =⇒ ‖f (x)− f (a)‖ < η

Ası, para x ∈ U tal que:

‖x− a‖ < δ =⇒ f (x) ∈ V ∧ ‖f (x)− f (a)‖ < η =⇒ ‖g (f (x))− g (f (a))‖ < ε

Ejemplo 8.4.2. Una aplicacion inmediata del teorema anterior es el cambio de variables.

Ilustramos lo anterior con el siguiente ejemplo: sea f : R2 → R definida por:

f (x, y) =

sin(x2+y2)x2+y2

, si x2 + y2 6= 0

1 , si x2 + y2 = 0

Estudie la continuidad de f en (0, 0).

Solucion. Naturalmente, haciendo u = x2 + y2, tenemos que:

(x, y)→ (0, 0) =⇒ u→ 0

Ası:

lım(x,y)→(0,0)

sin (x2 + y2)

x2 + y2= lım

u→0

sinu

u= 1

Como f (0, 0) = 1, obtenemos que f es continua en (0, 0).

Ejemplo 8.4.3. Usando compuesta de continuas podemos garantizar la continuidad de:

1. f (x, y) =√x2 + sin2 y + 1

2. g (x, y, z) =

√|x|+y2+z2

z2+y2+x|y|

Continuidad de funciones vectoriales

Observacion 8.5.1. La continuidad para una funcion vectorial de varias variables F : U ⊆Rn → Rm en a ∈ U ∩ U ′ se define por:

∀ε > 0, ∃δ > 0, ‖x− a‖Rn < δ =⇒ ‖F (x)− F (a)‖Rm < ε

Observacion 8.5.2. Al igual que en los lımites para funciones vectoriales, tenemos que la

continuidad de F : U ⊆ Rn → Rm definida por x 7→ F (x) = (f1 (x) , f2 (x) , . . . , fm (x)) en

a ∈ U ′ ∩ U se obtiene a traves de la continuidad de las funciones componentes en x. Mas

precisamente:

342

Page 345: Apuntes MAT-023 USM

Apuntes Mat023 (Segundo semestre 2014)

Teorema 8.5.1. Sean F : U ⊆ Rn → Rm una funcion vectorial definida por:

F (x) = (f1 (x) , f2 (x) , . . . , fm (x))

y a ∈ U ∩U ′. Entonces, F es continua en a ∈ U ∩U ′, si y solo si, fi es continua en a, para

cada i = 1, 2, . . . ,m.

Ejemplo 8.5.1. La funcion F : R2 → R3 definida por:

F (x, y) =(x2y, sinx, 3x2 cos y

)es continua en el plano.

Ejemplo 8.5.2. Sea f : R2 → R la funcion definida por:

f (x, y) =

2x−y2+2√

1−x2−y2, si x2 + y2 < 1

0 , si x2 + y2 ≥ 1

Analizar, completamente, la continuidad de f (x, y).

Solucion. Consideremos los siguientes subconjuntos del plano:

A =

(x, y) ∈ R2 : x2 + y2 > 1

B =

(x, y) ∈ R2 : x2 + y2 = 1

C =

(x, y) ∈ R2 : x2 + y2 < 1

Tenemos los siguientes casos:

1. Si (x, y) ∈ A, entonces f (x, y) = 0. Luego, f es continua en A por ser una funcion

constante en A.

2. Si (x, y) ∈ C, entonces f (x, y) = 2x−y2+2√1−x2−y2

, con 1−x2− y2 > 0. Luego, f es continua

en C por algebra de funciones continuas y composicion de funciones continuas.

3. Si (a, b) ∈ B, entonces, f (x, y) no puede ser continua en (a, b) ∈ B si:

2a− b2 + 2 6= 0

pues, si:

(x, y)→ (a, b) =⇒

1− x2 − y2 → 0 ∧ 2x− y2 + 2→ k, con k 6= 0

con lo cual la expresion 2x−y2+2√1−x2−y2

se indefine. Por tanto, los puntos (a, b) ∈ B tales

que:

2a− b2 + 2 = 0

343

Page 346: Apuntes MAT-023 USM

Apuntes Mat023 (Segundo semestre 2014)

estan dados por el sistema: 2a− b2 + 2 = 0

a2 + b2 = 1

Es decir, (a, b) = (−1, 0). Por tanto, investigamos el lımite:

lım(x,y)→(−1,0)

2x− y2 + 2√1− x2 − y2

Note que:

2x− y2 + 2√1− x2 − y2

=1− x2 − y2 + x2 + 2x+ 1√

1− x2 − y2

=√

1− x2 − y2 +(x+ 1)2√1− x2 − y2

Por tanto, para que el lımite:

lım(x,y)→(−1,0)

2x− y2 + 2√1− x2 − y2

exista es necesario y suficiente que el lımite:

lım(x,y)→(−1,0)

(x+ 1)2√1− x2 − y2

exista. Sin embargo, note que:

lımx→−1−

lımy→0−

(x+ 1)2√1− x2 − y2

= lım

x→−1−

(x+ 1)2

√1− x2

= 0

y que ademas, al considerar la trayectoria:

ϕ =

(x, y) ∈ C : −(x4 + 4x3 + 7x2 + 4x

)= y2

vemos que:

lım(x,y)→(−1,0)

(x+ 1)2√1− x2 − y2

= lımx→−1

(x+ 1)2

√1− x2 + x4 + 4x3 + 7x2 + 4x

= 1

Por tanto:

lım(x,y)→(−1,0)

(x+ 1)2√1− x2 − y2

no existe. En este caso, el lımite:

lım(x,y)→(−1,0)

2x− y2 + 2√1− x2 − y2

no existe. Ası, f no es continua en (−1, 0). Por lo tanto, f es continua en:

R2 −

(x, y) ∈ R2 : x2 + y2 = 1.

344

Page 347: Apuntes MAT-023 USM

Apuntes Mat023 (Segundo semestre 2014)

Ejemplo 8.5.3. Determine el mayor subconjunto de R2 en el cual la funcion

f (x, y) =

x−yx3−y si x3 6= y

0 si x3 = y

es continua.

Solucion. Si (a, b) ∈ R2 es tal que a3 6= b entonces

lım(x,y)→(a,b)

f (x, y) = lım(x,y)→(a,b)

(x− yx3 − y

)=

a− ba3 − b

= f (a, b)

luego f es continua en (a, b). Si a3 = b entonces

lım(x,y)→(a,a3)

f (x, y) = lım(x,y)→(a,a3)

x− yx3 − y

≈ a− a3

a3 − a3

si a− a3 6= 0 este lımte no existe (es un numero distinto de cero sobre cero). Si a− a3 = 0

entonces tenemos una forma indetermnada 00. a−a3 = 0⇒ a = 1, 0,−1 de donde obtenemos

que los unicos puntos de la curva y = x3 donde f puede ser continua son (1, 1), (0, 0) y

(−1,−1). Vamos a analizar estos lımites

1. lım(x,y)→(1,1)

(x− yx3 − y

)Note queoir la trayectoria y = 1 se tiene

lımx→1

(x− 1

x3 − 1

)=

1

3

pero por la trayectoria x = 1 se tiene

lımy→1

(1− y1− y

)= 1

luego el lımite no existe. f no es continua en (1, 1)

2. lım(x,y)→(0,0)

(x− yx3 − y

)Note que por la trayectoria y = 0 se tiene

lımx→0

( xx3

)= @

ası el lımite no existe. f no es continua en (0, 0)

345

Page 348: Apuntes MAT-023 USM

Apuntes Mat023 (Segundo semestre 2014)

3. lım(x,y)→(−1,−1)

(x− yx3 − y

)usamos limites iterados

lımy→−1

lımx→−1

(x− yx3 − y

)= lım

y→−1

(−1− y−1− y

)= 1

y

lımx→−1

lımy→−1

(x− yx3 − y

)= lım

x→−1

(x+ 1

x3 + 1

)=

1

3

se sigue que el lımite no existe y la funcion no es continua en (−1,−1).

El mayor subconunto de R2 en el cual f es continua es

U =

(x, y) ∈ R2 : x3 6= y

Ejemplo 8.5.4. Determine el mayor subconjunto de R2 en el cual la funcion

f (x, y) =

sin(xy)y

si (x, y) ∈ R2 ∧ y 6= 0

0 si (x, y) ∈ R2 ∧ y = 0

es continua.

Solucion. Si (a, b) ∈ R2 es tal que b 6= 0 entonces f es continua en (a, b) por algebra de

continuas. Supongamos que (a, b) ∈ R2 es tal que b = 0 entonces

lım(x,y)→(a,0)

f (x, y) = lım(x,y)→(a,0)

sin (xy)

y

= lım(x,y)→(a,0)

x

(sin (xy)

(xy)

)= a

pero f (a, 0) = 0 luego la funcion es continua solo si a = 0 es decir en el punto (0, 0). El

mayor subconjunto de R2 en el cual f es continua es

U =

(x, y) ∈ R2 : y 6= 0∪ (0, 0)

346

Page 349: Apuntes MAT-023 USM

Apuntes Mat023 (Segundo semestre 2014)

Finalmente enunciamos un teorema que nos habla sobre el comportamiento de las

funciones continuas sobre los conjuntos compactos y conexos:

Teorema 8.5.2. Sea f : U ⊆ Rn → Rm una funcion continua en U , entonces:

1. Si A ⊆ U es conexo, entonces f (A) es conexo.

2. Si K ⊆ U es compacto, entonces f (K) es compacto.

Ejercicios del capıtulo

1. Determine el valor de α ∈ R de modo que el lımite:

lım(x,y)→(0,0)

x− x cosxy

y sin2 (3x)+

(x2 − xy)√

1− α2

x2 − y2

exista.

2. Sea f : R2 − (0, 0) → R la funcion definida por

f (x, y) =x2 sin

(1

x+y

)x2 + |x|+ 2

calcular lım(x,y)→(0,0)

f (x, y)

3. Considere la funcion definida por

f (x, y) =

x

x+ y2si x 6= −y2

0 si x = −y2

analizar la continuidad de f en R2.

4. Sean f y g funciones definidas en R2 − (0, 0) por

f (x, y) =sin(√

x2 + y2)

√x2 + y2 + x2 + y2

g (x, y) = α

(x3 − y3

|x|+ |y|

)que valor debe tener α para que

lım(x,y)→(0,0)

f (x, y) + g (x, y)

exista.

347

Page 350: Apuntes MAT-023 USM

Apuntes Mat023 (Segundo semestre 2014)

5. Estudiar la continuidad de la funcion

f (x, y) =

x3 + sin (y2) y ≤ x

x2 − xy + y3 + cos (y2) y > x

6. Estudiar la continuidad de la funcion

f (x, y) =

x2−y2x2+y2−1

x2 + y2 6= 1

0 x2 + y2 = 1

7. Determine si los siguientes lımites existen

1) lım(x,y)→(0,0)

|sinx sin y||xy|

2) lım(x,y)→(0,0)

x2y2

x2y2 + (y − x)2

3) lım(x,y)→(0,0)

xy sin (y3)

x4 + y44) lım

(x,y)→(0,0)

sin (xy4)

x2 + y8

5) lım(x,y)→(0,0)

y3√|x|

|x|+ y46) lım

(x,y)→(0,0)

x3 + y2

|x|+ |y|

7) lım(x,y)→(0,0)

sinx+ sin y

x+ y8) lım

(x,y)→(1,0)

cos(√|xy|

)− 1

y

9) lım(x,y)→(1,0)

(x− 1)4 y2

(x− 1)8 + y410) lım

(x,y)→(0,0)

x3y√x2 + y2

sin (xy)

11) lım(x,y)→(0,0)

x2y

x2 + y12) lım

(x,y)→(0,0)

2x2 − y2

x2 + 2y2

13) lım(x,y)→(0,0)

x4 + 2x2y2 + 2y4

(x2 + y2)2 14) lım(x,y)→(0,0)

x6 + y6

x4 + y4

348

Page 351: Apuntes MAT-023 USM

Capıtulo 9 : Diferenciacion en varias variables

En secciones anteriores hemos estudiado algunos metodos para dibujar la grafica de

funciones de varias variables. Utilizando solo esos metodos es muy difıcil obtener suficiente

informacion para captar las caracterısticas generales de una funcion con expresion algo

complicada. Sabemos del calculo de una variable que la derivada nos puede ayudar mucho

en esta tarea; por ejemplo, nos permite cuantificar los cambios en la grafica, localizar

maximos, mınimos, estudian concavidad, etc. La derivada tambien tiene otras muchas

aplicaciones como el estudiante habra descubierto en calculo de una variable. Una funcion

diferenciable de R2 en R deberıa ser una funcion con grafica suave, que no tenga dobleces

bruscos.

Para hacer precisas estas ideas necesitamos una definicion de lo que entenderemos por

diferenciable en el caso de funciones de varias variables. Comenzamos con una nocion de

derivada que descansa en nuestros conocimientos de una variable.

Derivadas parciales

Observacion 9.1.1. En lo que sigue denotaremos por ei, con i = 1, 2, . . . , n, a los n

vectores de la base canonica de Rn, mas precisamente, ei representa el vector de Rn que

tiene todas su componentes cero salvo la i-esima componente la cual es igual a uno.

Definicion 9.1.1. Sean f : U ⊆ Rn → R y a ∈U . Se define la derivada parcial de f en a

respecto a la i-esima variable como el lımite:

∂f

∂xi(a) = lım

t→0

f (a + tei)− f (a)

t

= lımt→0

f (a1, a2, . . . , ai + t, . . . , an)− f (a1, a2, . . . , an)

t

si este existe.

Observacion 9.1.2. Para f : U ⊆ Rn → R, las derivadas parciales se denotan por:∂f∂xi, fxi , Dif , etc.

Observacion 9.1.3. Si n = 2, f : U ⊆ R2 → R, (x, y) → f (x, y) y a = (a, b) ∈U ,

entonces:

∂f

∂x(a, b) = lım

t→0

f (a + te1)− f (a)

t

= lımt→0

f ((a, b) + t (1, 0))− f (a, b)

t

= lımt→0

f (a+ t, b)− f (a, b)

t

349

Page 352: Apuntes MAT-023 USM

Apuntes Mat023 (Segundo semestre 2014)

y

∂f

∂y(a, b) = lım

t→0

f (a + te2)− f (a)

t

= lımt→0

f ((a, b) + t (0, 1))− f (a, b)

t

= lımt→0

f (a, b+ t)− f (a, b)

t

note que corresponden a lımites de una variable, a saber, la variable t.

Observacion 9.1.4. En particular, si n = 2, 3 y 4, anotamos: ∂f∂x1

= ∂f∂x, ∂f∂x2

= ∂f∂y, ∂f∂x3

= ∂f∂z

y ∂f∂x4

= ∂f∂w

, segun corresponda respecto al numero de variables de f y el nombre y orden

que a estas asignemos.

Ejemplo 9.1.1. Sea f : R2 → R, (x, y)→ f (x, y) = x3y + 2x2y3 + 2x, calcular ∂f∂x

(−1, 1)

y ∂f∂y

(1, 2) si estas existen.

Solucion. Por definicion

∂f

∂x(−1, 1) = lım

t→0

f (−1 + t, 1)− f (−1, 1)

t

= lımt→0

((−1 + t)3 1 + 2 (−1 + t)2 13 + 2 (−1 + t)

)−((−1)3 1 + 2 (−1)2 13 + 2 (−1)

)t

= lımt→0

t (t2 − t+ 1)

t= lım

t→0

(t2 − t+ 1

)= 1

y

∂f

∂y(1, 2) = lım

t→0

f (1, 2 + t)− f (1, 2)

t

= lımt→0

(13 (2 + t) + 2 (12) (2 + t)3 + 2 (1)

)− (132 + 2 (12) (23) + 2)

t

= lımt→0

t (2t2 + 12t+ 25)

t= lım

t→0

(2t2 + 12t+ 25

)= 25

note que en estos lımites, una de las variables se deja fija y se calcula la derivada usual de

una variable respecto a la otra, en otras palabras

∂f

∂x(x, y) = lım

t→0

f (x+ t, y)− f (x, y)

t

= lımt→0

h (x+ t)− h (x)

t= h′ (x)

350

Page 353: Apuntes MAT-023 USM

Apuntes Mat023 (Segundo semestre 2014)

donde hemos pensado h (x) = f (x, y), al estar y fija esta es solo una funcion de la variable

x. En el ejemplo

∂f

∂x(x, y) =

∂x

(x3y + 2x2y3 + 2x

)= 3x2y + 4xy3 + 2

evaluando en (−1, 1)∂f

∂x(−1, 1) = 3 (−1)2 + 4 (−1) + 2 = 1

y

∂f

∂y(x, y) =

∂y

(x3y + 2x2y3 + 2x

)= x3 + 6x2y2

luego∂f

∂y(1, 2) = 13 + 6

(22)

= 25

Ejemplo 9.1.2. Si f (x, y, z) = 2xyz2 + sin (xy2) + 2x entonces

∂f

∂x(x, y, z) =

∂x

(2xyz2 + sin

(xy2)

+ 2x)

= 2yz2 + y2 cos(xy2)

+ 2

∂f

∂y(x, y, z) =

∂y

(2xyz2 + sin

(xy2)

+ 2x)

= 2xz2 + 2xy cos(xy2)

∂f

∂z(x, y, z) =

∂z

(2xyz2 + sin

(xy2)

+ 2x)

= 4xyz

Ejemplo 9.1.3. Verifique que:

∂u

∂x+∂u

∂y+∂u

∂z= 1

si:

u = x+x− yy − z

Solucion. Derivando

∂x

(x+

x− yy − z

)=

y − z + 1

y − z∂

∂y

(x+

x− yy − z

)= − x− z

(y − z)2

∂z

(x+

x− yy − z

)=

x− y(y − z)2

se sigue (y − z + 1

y − z

)+

(− x− z

(y − z)2

)+

(x− y

(y − z)2

)= 1

351

Page 354: Apuntes MAT-023 USM

Apuntes Mat023 (Segundo semestre 2014)

Ejemplo 9.1.4. Verifique que:

xzx + yzy = xy + z

si:

z = xy + xeyx

Solucion. Derivando

∂x

(xy + xe

yx

)=

1

x

(xe

1xy − ye

1xy + xy

)∂

∂y

(xy + xe

yx

)= x+ e

1xy

luego

xzx + yzy

= x

(1

x

(xe

1xy − ye

1xy + xy

))+ y

(x+ e

1xy)

= xeyx − ye

yx + xy + xy + ye

yx

= xeyx + 2xy

= xy + xy + xeyx

= xy + z

Ejemplo 9.1.5. Sea f : R2 → R la funcion definida por:

f (x, y) =

xy

x2+y2, si (x, y) 6= (0, 0)

0 , si (x, y) = (0, 0)

Calcule las derivadas parciales de f en (0, 0).

Solucion. Es importante recordar que f no es continua en (0, 0). Sin embargo, la conti-

nuidad no afecta la existencia de las derivadas parciales. En efecto:

∂f

∂x(0, 0) = lım

t→0

f (t, 0)− f (0, 0)

t

= lımt→0

0− 0

t= 0

Analogamente, se tiene que

∂f

∂y(0, 0) = lım

t→0

f (0, t)− f (0, 0)

t

= lımt→0

0t02+t2

− 0

t= lım

t→00

= 0

352

Page 355: Apuntes MAT-023 USM

Apuntes Mat023 (Segundo semestre 2014)

Ejemplo 9.1.6. Sea f : R2 → R la funcion definida por:

f (x, y) =

x2 , y ≤ x

xy , y > x

Calcule ∂f∂y

(1, 1).

Solucion. Calculamos ∂f∂y

(1, 1). Note que:

lımt→0+

f (1, 1 + t)− f (1, 1)

t= lım

t→0+

1 · (1 + t)− 1

t

= lımt→0+

t

t= 1

y, analogamente, se obtiene que:

f (1, 1 + t)− f (1, 1)

t→ 0, si t→ 0−

Por tanto, ∂f∂y

(1, 1), no existe.

en la figura se muestra el dominio y en dos colores distintos la parte y ≤ x e y > x, al

aproximarnos al punto (1, 1) por los puntos (1, 1 + t) se obtienen dos valores distintos de

lımite para t > 0 y t < 0.

Observacion 9.1.5. La sola existencia de las derivadas parciales no es un buen concepto

de ser derivable en el caso de funciones de varias variables. Por ejemplo, la funcion

f (x, y) =

1 si xy = 0

0 si xy 6= 0

353

Page 356: Apuntes MAT-023 USM

Apuntes Mat023 (Segundo semestre 2014)

no es continua en (0, 0) pero las derivadas parciales existen en ese punto, luego la existencia

de las derivadas parciales no implica continuidad, si la existencia de las derivadas parciales

fuera el concepto de “ser derivable” no podrıamos recuperar el teorema de una variable,

derivable en un punto implica continua en el punto. Veremos que el concepto de ser

derivable estudia las variaciones de la funcion en una forma mas global y no en una

direccion especıfica como las derivadas parciales.

Ejemplo 9.1.7. Considere la siguiente ecuacion en derivadas parciales:∂2u

∂t2=∂2u

∂x2, 0 < x < 1, t > 0

u(0, t) = u(1, t) = 0, t > 0(9.1)

Demuestre que la funcion:

u(x, t) =∞∑n=1

αn cos(nπt) + βn sin(nπt)

sin(nπx) (9.2)

es solucion del problema (9.1).

Indicacion: Asuma las hipotesis necesarias para la derivacion de la serie.

Suponga que la solucion u(x, t) dada por (9.2) satisface las ecuaciones:

u(x, 0) = sin(7πx), 0 < x < 1

∂u

∂t(x, 0) = x(1− x), 0 < x < 1

Calcular los coeficientes αn y βn de la solucion u(x, t).

Solucion. Sean 0 < x < 1 y t > 0. Asumiendo las condiciones necesarias para la

convergencia de la serie, tenemos que:

∂u

∂x=

∂x

∞∑n=1

(αn cos(nπt) + βn sin(nπt)

)sin(nπx)

=∞∑n=1

∂x

(αn cos(nπt) + βn sin(nπt)

)sin(nπx)

=∞∑n=1

(αn cos(nπt) + βn sin(nπt)

) ∂∂x

sin(nπx)

=∞∑n=1

nπ(αn cos(nπt) + βn sin(nπt)

)cos(nπx)

354

Page 357: Apuntes MAT-023 USM

Apuntes Mat023 (Segundo semestre 2014)

Ası:

∂2u

∂x2=

∂x

∞∑n=1

nπ(αn cos(nπt) + βn sin(nπt)

)cos(nπx)

(9.3)

= −∞∑n=1

n2π2(αn cos(nπt) + βn sin(nπt)

)sin(nπx)

Por otro lado:

∂u

∂t=

∂t

∞∑n=1

(αn cos(nπt) + βn sin(nπt)

)sin(nπx)

(9.4)

=∞∑n=1

∂t

(αn cos(nπt) + βn sin(nπt)

)sin(nπx)

=∞∑n=1

∂t

(αn cos(nπt) + βn sin(nπt)

)sin(nπx)

=∞∑n=1

(− nπαn sin(nπt) + nπβn cos(nπt)

)sin(nπx)

Ası:

∂2u

∂t2=

∂t

∞∑n=1

(− nπαn sin(nπt) + nπβn cos(nπt)

)sin(nπx)

(9.5)

= −∞∑n=1

(n2π2αn cos(nπt) + n2π2βn sin(nπt)

)sin(nπx)

= −∞∑n=1

n2π2(αn cos(nπt) + βn sin(nπt)

)sin(nπx)

Entonces, comparando la ultima ecuacion en (9.3) y en (9.5) y notando, ademas, que:

u(0, t) = u(1, t) = 0, t > 0

pues sin(nπ) = 0, se tiene que u(x, t) dada por la ecuacion (9.2) satisface el problema

dado por (9.1). Finalmente, debemos calcular los coeficientes αn y βn de la solucion u(x, t)

sabiendo que dicha solucion cumple con las siguientes condiciones iniciales:

u(x, 0) = sin(7πx), 0 < x < 1

∂u

∂t(x, 0) = x(1− x), 0 < x < 1

En efecto, si t = 0, entonces de (9.2) obtenemos:

u(x, 0) =∞∑n=1

αn sin(nπx) (9.6)

355

Page 358: Apuntes MAT-023 USM

Apuntes Mat023 (Segundo semestre 2014)

y de (9.4) obtenemos:

∂u

∂t(x, 0) =

∞∑n=1

nπβn sin(nπx) (9.7)

Entonces, como u(x, 0) = sin(7πx) es un elemento del sistema ortonormal , tenemos que:

αn =⟨

sin(7πx), sin(nπx)⟩

=

∫ 1

−1

sin(7πx) · sin(nπx)dx

= δ7n

=

0 , n 6= 7

1 , n = 7

Por otro lado, si en la ecuacion (9.7) escribimos Bn = nπβn, obtenemos:

x(1− x) =∞∑n=1

Bn sin(nπx), 0 < x < 1

y entonces los coeficientes Bn corresponden a los coeficientes de Fourier en la extension

impar de x(1− x), 0 < x < 1 . Es decir:

356

Page 359: Apuntes MAT-023 USM

Apuntes Mat023 (Segundo semestre 2014)

Bn =⟨x(1− x), sin(nπx)

⟩= 2

∫ 1

0

x(1− x) · sin(nπx) dx

= − 2

∫ 1

0

x(

cos(nπx))′dx+

2

∫ 1

0

x2(

cos(nπx))′dx

=2

[− x cos(nπx)

∣∣∣10

+

∫ 1

0

cos(nπx) dx+ x2 cos(nπx)∣∣∣10−

− 2

∫ 1

0

x cos(nπx) dx

]

=2

[− cos(nπ) + cos(nπ)− 2

∫ 1

0

x cos(nπx) dx

]

=4

n2π2

∫ 1

0

x(

sin(nπx))′dx

=4

n2π2

[x sin(nπx)

∣∣∣10−∫ 1

0

sin(nπx) dx

]=

4

n3π3

(cos(nπ)− 1

)=

0 , n es par

− 8

n3π3, n es impar

Por lo tanto:

βn = − 8

π4(2n− 1)4, ∀n ∈ N

Ası, en resumen, los coeficientes αn y βn son:

αn = δn7 ∧ βn = − 8

π4(2n− 1)4

para todo n ≥ 1. En particular, la solucion esta dada por:

u(x, t) = cos(7πt) sin(7πx)− 8

π4

∞∑n=1

sin(nπt) sin(nπx)

(2n− 1)4

357

Page 360: Apuntes MAT-023 USM

Apuntes Mat023 (Segundo semestre 2014)

Ejercicios de la seccion

1. Hallar ∂f∂x

y ∂f∂y

para las siguientes funciones

a) f (x, y) = xy2

b) f (x, y) = ex3+y2

c) f (x, y) = x ln (x2 + y2)

2. Si

f (x, y) =

x2y3

x2+y2si (x, y) 6= (0, 0)

0 si (x, y) = (0, 0)

determine el conjunto D =

(x, y) ∈ R2 : ∂f(x,y)∂x

existe

y estudiar si la funcion

G : D ⊆ R2 → R

(x, y) → G (x, y) =∂f (x, y)

∂x

es continua en todo D.

3. Si u (x, y) = arctan(x2−y2xy

)hallar ux, uy y verificar que

xux + yuy = 0

4. Si u (x, y, z) = x2y + y2z + z2x verificar que

∂u

∂x+∂u

∂y+∂u

∂z= (x+ y + z)2

5. Determine todas las funciones f : R3 → R, (x, y, z)→ f (x, y, z) que cumplen

∂f

∂x= ex+y + 3x2y + 4yz + y2 + 2

∂f

∂y= ex+y + 2yz3 + 2xy + 4xz + x3

∂f

∂z= 3y2z2 + 4xy

6. Si

f (x, y) =

|x|+

√y2 + |x|

x+ ysi x+ y > 0

x3 si x+ y ≤ 0 ∧ x2 + y2 ≤ 4

y2 + x si x+ y ≤ 0 ∧ x2 + y2 > 4

determine si ∂f∂x

(0, 0) y ∂f∂y

(0, 0) existen.

358

Page 361: Apuntes MAT-023 USM

Apuntes Mat023 (Segundo semestre 2014)

Interpretacion de la derivada parcial

En esta seccion veremos una interpretacion de las derivadas parciales y buscaremos la

ecuacion del plano tangente a la grafica de una funcion de dos variables.

Considere una funcion f : D ⊆ R2 → R, su grafica esta contenida en R3. Suponga que

estamos interesados en estudiar el comportamiento de la funcion en un punto (x0, y0) de

su dominio, como no contamos con tecnicas de varias variables, intentamos estudiar el

comportamiento descendiendo a una variable, para ello, vamos a intersectar la grafica con

dos planos x = x0 e y = y0.

La interseccion de la grafica con el plano x = x0 es una curva, la cual puede ser

interpretada como el grafico de la funcion de una variable

z = f (x0, y) = g (y)

en el plano x = x0 (note que si x = x0 y z = f (x, y) entonces z = f (x0, y)). En

este plano podemos aplicar las tecnicas de una variable para conocer el crecimiento y

decrecimiento, concavidad y otros, sin embargo, este analisis solo es un comportamiento de

la curva, no necesariamente un comportamiento global, por ejemplo un maximo sobre esta

curva puede no ser un maximo de la funcion en su dominio. La recta tangente a la grafica

de esta funcion de una variable es

z − g (y0) = g′ (y0) (y − y0)

esto es

z − f (x0, y0) =∂f

∂y(x0, y0) (y − y0)

pues

g′ (y0) = lımh→0

g (y0 + h)− g (y0)

h

= lımh→0

f (x0, y0 + h)− f (x0, y0)

h

=∂f

∂y(x0, y0)

note que la recta esta en el plano x = x0, se sigue que los puntos de ella cumplen

x = x0

y = y

z = f (x0, y0) +∂f

∂y(x0, y0) (y − y0)

359

Page 362: Apuntes MAT-023 USM

Apuntes Mat023 (Segundo semestre 2014)

o en ecuaciones parametricas x

y

z

=

x0

0

f (x0, y0)− y0∂f∂y

(x0, y0)

+ y

0

1∂f∂y

(x0, y0)

donde y ∈ R, el vector director es en este caso 0

1∂f∂y

(x0, y0)

De manera similar, la interseccion de la grafica con el plano y = y0 es una curva, la

cual puede ser interpretada como el grafico de la funcion de una variable

z = f (x, y0) = p (x)

sobre el plano y = y0 (note que si y = y0 y z = f (x, y) entonces z = f (x, y0)). La recta

tangente a la grafica de esta funcion de una variable es

z − p (x0) = p′ (x0) (x− x0)

esto es

z − f (x0, y0) =∂f

∂x(x0, y0) (x− x0)

pues

p′ (x0) = lımh→0

p (x0 + h)− p (x0)

h

= lımh→0

f (x0 + h, y0)− f (x0, y0)

h

=∂f

∂x(x0, y0)

se sigue que los puntos de esta recta cumplen

x = x

y = y0

z = f (x0, y0) +∂f

∂x(x0, y0) (x− x0)

o en ecuaciones parametricas x

y

z

=

0

y0

f (x0, y0)− x0∂f∂x

(x0, y0)

+ x

1

0∂f∂x

(x0, y0)

360

Page 363: Apuntes MAT-023 USM

Apuntes Mat023 (Segundo semestre 2014)

el vector director corresponde a 1

0∂f∂x

(x0, y0)

de estos calculos se sigue que en el punto (x0, y0, f (x0, y0)) de la grafica los vectores 1

0∂f∂x

(x0, y0)

y

0

1∂f∂y

(x0, y0)

son tangentes, se sigue que el vector 1

0∂f∂x

(x0, y0)

× 0

1∂f∂y

(x0, y0)

=

−∂f∂x

(x0, y0)

−∂f∂y

(x0, y0)

1

es perpendicular a la superficie, ası(

−∂f∂x

(x0, y0) ,−∂f∂y

(x0, y0) , 1

)· (x− x0, y − y0, z − f (x0, y0)) = 0

deberıa corresponder a un plano tangente a la superficie, el problema para definir esto

directamente como el plano tangente a la grafica es el de formalizar en que sentido queremos

decir que la funcion y el plano se parecen cerca del punto, el calculo realizado depende solo

de la existencia de las derivadas parciales pero por ejemplo en el caso de la funcion

f (x, y) =

1 si xy = 0

0 si xy 6= 0

los calculos nos llevarıan a que el plano tangente es z = 1 sin embargo los valores que toma

la funcion en todo entorno abierto de (0, 0) no se parecen a uno (tan cerca como queramos

existen valores cero).

Veamos un ejemplo concreto, consideremos la funcion f : R2 → R, (x, y)→ f (x, y) =

(x2 − 2xy) e−2x2−y4+2y y estudiemos la funcion en el punto (x0, y0) = (1, 0). La grafica de f

361

Page 364: Apuntes MAT-023 USM

Apuntes Mat023 (Segundo semestre 2014)

es

si cortamos con el plano y = 0 se obtiene la curva z = f (x, 0) = x2e−2x2

la recta tangente en x = 1 es

z − f (1, 0) =d

dx

(x2e−2x2

)∣∣∣∣x=1

(x− 1)

esto es

z − e−2 = 2xe−2x2 − 4x3e−2x2∣∣∣x=1

(x− 1)

z − e−2 =(2e−2 − 4e−2

)(x− 1)

362

Page 365: Apuntes MAT-023 USM

Apuntes Mat023 (Segundo semestre 2014)

es decir

x = x

y = 0

z = e−2 − 2e−2 (x− 1)

de manera similar, si cortamos con el plano x = 1 se obtiene la curva z = f (1, y) =

(1− 2y) e−2−y4+2y la recta tangente en y = 0 es

z − f (1, 0) =d

dy

((1− 2y) e−2−y4+2y

)∣∣∣∣y=0

(y − 0)

esto es

z − e−2 = −4ye−y4+2y−2

(−2y3 + y2 + 1

)∣∣∣y=0

y

z − e−2 = 0

es decir

x = 1

y = y

z = e−2

los directores de estas rectas son

1

0

−2e−2

y

0

1

0

se sigue que el normal corresponde

a 1

0

−2e−2

× 0

1

0

=

2e−2

0

1

363

Page 366: Apuntes MAT-023 USM

Apuntes Mat023 (Segundo semestre 2014)

y ası el plano tangente corresponde a(2e−2, 0, 1

)·(x− 1, y − 0, z − e−2

)= 0

es decir

2e−2 (x− 1) + z − e−2 = 0

o equivalentemente

z = e−2 − 2e−2 (x− 1)

en la figura, en blanco estan las curvas y las rectas tangentes, la interseccion de estas

rectas y curvas corresponde al punto (1, 0, f (1, 0)) y el plano que contiene a estas rectas

tangentes es el plano tangente, como se puede apreciar la propiedad de tangencia es local,

no significa que toque a la grafica en un unico punto.

Ejercicios de la seccion

1. Considere la funcion f : R2 → R, (x, y)→ f (x, y) = ex sin y + e2y cosx. Determine

las ecuaciones parametricas de las rectas tangentes a la grafica de f en el punto

(0, 0, 1) que son paralelas a los planos x = 0 e y = 0.

2. Determine una recta normal a la superficie

x2 +y2

4+ z2 = 1

364

Page 367: Apuntes MAT-023 USM

Apuntes Mat023 (Segundo semestre 2014)

en el punto(

1√2, 1, 1

2

).

3. Si f : R2 → R, (x, y) → f (x, y) = (x2 − 2xy) e−2x2−y4+2y estudiar los extremos

(maximos y mınimos) locales y globales de la funcion g : R→ R, t→ g (t) = f (t, 0).

Diferenciabilidad

Definicion 9.3.1. Sean U ⊆ Rn y f : U → R una funcion. Diremos que f es diferenciable

en a ∈U si existe una transformacion lineal T : Rn → R tal que:

lımh→0

|f (a + h)− f (a)− T (h)|‖h‖

= 0

Observacion 9.3.1. Si n = 2, por ejemplo, el lımite en la definicion anterior toma la

forma:

lım(h,k)→(0,0)

|f (a+ h, b+ k)− f (a, b)− T (h, k)|√h2 + k2

= 0

Recordemos, ademas, que si T : Rn → R es una transformacion lineal, entonces

existen constantes A1, A2, . . . , An ∈ R (respecto, por ejemplo, de las bases canonicas,

respectivamente) tales que:

[T ] =(A1 A2 · · · An

)∈M1×n (R)

El siguiente teorema indica la condicion necesaria que deben cumplir las constantes

A1, A2, . . . , An en caso de que una funcion f sea diferenciable en a:

Teorema 9.3.1. Sean U ⊆ Rn y f : U → R una funcion diferenciable en a ∈U , entonces:

Ai =∂f

∂xi(a)

para cada i = 1, 2, . . . , n.

Demostracion. Como f : U → R es diferenciable en a, existe una transformacion lineal

T : Rn → R tal que:

lımh→0

|f (a+ h)− f (a)− T · h|‖h‖

= 0

Note que haciendo:

h = t · eiel lımite anterior implica el siguiente lımite:

lımt→0

|f (a+ t · ei)− f (a)− T (t · ei)|‖t · ei‖

= 0 (9.8)

365

Page 368: Apuntes MAT-023 USM

Apuntes Mat023 (Segundo semestre 2014)

Por otro lado, suponga que la transformacion lineal T : Rn → R tiene representacion

matricial:

[T ]1C =(A1 A2 · · · An

)donde C es la base canonica de Rn y 1 representa la base canonica de R. Ası:

[T · (t · ei)]1 = [T ]1C [t · ei]C= t · Ai

Ası:

lımt→0

|f (a+ t · ei)− f (a)− T (t · ei)|‖t · ei‖

= lımt→0

|f (a+ t · ei)− f (a)− t · Ai||t|

= lımt→0

∣∣∣∣f (a+ t · ei)− f (a)

t− Ai

∣∣∣∣=

∣∣∣∣ ∂f∂xi (a)− Ai∣∣∣∣

y como el valor del lımite en (9.8) es 0 (por la diferenciabilidad de f en a) se tiene que:∣∣∣∣ ∂f∂xi (a)− Ai∣∣∣∣ = 0

Esto es:∂f

∂xi(a) = Ai

para cada i = 1, 2, . . . , n.

Observacion 9.3.2. Se sigue entonces que si f es diferenciable en x = a las derivadas

parciales deben existir en el punto.

Podemos entonces reformular la definicion como:

Definicion 9.3.2. Sean U ⊆ Rn y f : U → R una funcion. Diremos que f es diferenciable

en a ∈U si:

1. Para cada i = 1, 2, . . . , n, existen las derivadas parciales ∂f∂xi

(a).

2. Asumiendo, h = (h1, h2, . . . , hn) ∈ Rn, se tiene que:

lımh→0

∣∣∣f (a + h)− f (a)−∑n

i=1∂f∂xi

(a)hi

∣∣∣‖h‖

= 0

366

Page 369: Apuntes MAT-023 USM

Apuntes Mat023 (Segundo semestre 2014)

Observacion 9.3.3. Con el cambio de variables h = x− a el lımite

lımh→0

∣∣∣f (a + h)− f (a)−∑n

i=1∂f∂xi

(a)hi

∣∣∣‖h‖

= 0

se puede escribir en la forma

lımx→a

∣∣∣f (x)− f (a)−∑n

i=1∂f∂xi

(a) (xi − ai)∣∣∣

‖x− a‖= 0

Observacion 9.3.4. Se debe hacer notar que la suma:

n∑i=1

∂f

∂xi(a) · hi

se obtiene vıa el isomorfismo canonico que existe entre M1×1 (R) y el espacio vectorial real

unidimensional. En efecto sabemos que:

[T ]1C =(

∂f∂x1

(a) ∂f∂x2

(a) · · · ∂f∂xn

(a))

y como h = (h1, h2, . . . , hn), tenemos:

[T · h]1 = [T ]1C [h]C

=(

∂f∂x1

(a) ∂f∂x2

(a) · · · ∂f∂xn

(a))

h1

h2

...

hn

=

(n∑i=1

∂f

∂xi(a) · hi

)1×1

Ejemplo 9.3.1. Considere la funcion f : R2 → R definida por:x2y4

x2 + y2si (x, y) 6= (0, 0)

0 si (x, y) = (0, 0)

estudiar la diferenciabilidad de f en (0, 0).

Solucion. Notemos que

∂f

∂x(0, 0) = lım

h→0

f (h, 0)− f (0, 0)

h

= lımh→0

h204

h2+02− 0

h= lım

h→00

= 0

367

Page 370: Apuntes MAT-023 USM

Apuntes Mat023 (Segundo semestre 2014)

de manera similar

∂f

∂y(0, 0) = lım

h→0

f (0, h)− f (0, 0)

h

= lımh→0

02h4

02+h2− 0

h= lım

h→00

= 0

luego las derivadas parciales existen, notemos que

lım(x,y)→(0,0)

∣∣∣f (x, y)− f (0, 0)− ∂f∂x

(0, 0) (x− 0)− ∂f∂y

(0, 0) (y − 0)∣∣∣√

x2 + y2

= lım(x,y)→(0,0)

∣∣∣ x2y4x2+y2

∣∣∣√x2 + y2

= lım(x,y)→(0,0)

x2y4

(x2 + y2)3/2(por polares)

= 0

se sigue que f es diferenciable en (0, 0).

Ejemplo 9.3.2. Considere la funcion g : R3 → R definida por:

g (x, y, z) =

sin (xyz)

x2 + y2 + |z|, (x, y, z) 6= (0, 0, 0)

0 , (x, y, z) = (0, 0, 0)

Demuestre que g es diferenciable en el punto (0, 0, 0).

Solucion. Calculamos, primeramente, las derivadas parciales de f en (0, 0, 0). Esto es:

∂f

∂x(0, 0, 0) = lım

t→0

f (t, 0, 0)− f (0, 0, 0)

t

= lımt→0

0

t= 0

368

Page 371: Apuntes MAT-023 USM

Apuntes Mat023 (Segundo semestre 2014)

Analogamente, se tiene que∂f

∂y(0, 0, 0) =

∂f

∂z(0, 0, 0) = 0. Luego:

lım(h,k,l)→(0,0,0)

∣∣∣f (h, k, l)− f (0, 0, 0)− ∂f∂x

(0, 0, 0) · h+ ∂f∂y

(0, 0, 0) · k + ∂f∂z

(0, 0, 0) · l∣∣∣

√h2 + k2 + l2

=

= lım(h,k,l)→(0,0,0)

|f (h, k, l)|√h2 + k2 + l2

= lım(h,k,l)→(0,0,0)

|sin (hkl)|(h2 + k2 + |l|)

√h2 + k2 + l2

≤ lım(h,k,l)→(0,0,0)

|hkl||l|√h2 + k2 + l2

≤ lım(h,k,l)→(0,0,0)

|hkl||l| |k|

= lım(h,k,l)→(0,0,0)

|h|

= 0

Por tanto, f es diferenciable en (0, 0, 0).

Ejemplo 9.3.3. Consideremos f : R2 → R la funcion definida por:

f (x, y) =

sin(xy)x2+|y| , (x, y) 6= (0, 0)

0 , (x, y) = (0, 0)

Verifique que f no es diferenciable en (0, 0).

Solucion. Notamos, primeramente que:

∂f

∂x(0, 0) = lım

t→0

f (t, 0)− f (0, 0)

t

= lımt→0

1

0

t2 + 0

= 0

Analogamente se obtiene que:∂f

∂y(0, 0) = 0

Ahora bien, como:

lım(h,k)→(0,0)

∣∣∣f (h, k)− f (0, 0)− ∂f∂x

(0, 0) · h− ∂f∂y

(0, 0) · k∣∣∣

√h2 + k2

= lım(h,k)→(0,0)

|f (h, k)|√h2 + k2

= lım(h,k)→(0,0)

1√h2 + k2

|sin (hk)|h2 + |k|

369

Page 372: Apuntes MAT-023 USM

Apuntes Mat023 (Segundo semestre 2014)

Ahora bien, note que:

lımh→0

lımk→0

1√h2 + k2

|sin (hk)|h2 + |k|

= lım

h→00 = 0

y si consideramos la trayectoria ϕ : h = k, con h > 0 tenemos que:

lımh→0+

1√h2 + h2

|sin (h2)|h2 + h

= lımh→0+

1√2

|sin (h2)|h3 + h2

=1√2

lımh→0

|sin (h2)|h2

h2

h3 + h2

=1√2

Por tanto, f no es diferenciable en (0, 0).

Ejemplo 9.3.4. Toda transformacion lineal es diferenciable, la transformacion lineal que

mas se parece es la misma funcion.

Observacion 9.3.5. Supongamos que f : U ⊆ R2 → R es una funcion diferenciable en

(a, b) ∈U . Esto quiere decir que:

lım(h,k)→(0,0)

∣∣∣f (a+ h, b+ k)− f (a, b)− ∂f∂x

(a, b) h− ∂f∂y

(a, b) k∣∣∣

√h2 + k2

= 0 (9.9)

Ahora bien, haciendo el cambio de variables:x = a+ h

y = b+ k

tenemos que:

(h, k)→ (0, 0) ⇐⇒ (x, y)→ (a, b)

Por tanto, el lımite en (9.9) queda:

lım(x,y)→(a,b)

∣∣∣f (x, y)− f (a, b)− ∂f∂x

(a, b) · (x− a)− ∂f∂y

(a, b) · (y − b)∣∣∣√

(x− a)2 + (y − b)2= 0

Intuitivamente, para (x, y) suficientemente cercano a (a, b), tenemos que:∣∣∣f (x, y)− f (a, b)− ∂f∂x

(a, b) · (x− a)− ∂f∂y

(a, b) · (y − b)∣∣∣√

(x− a)2 + (y − b)2' 0

370

Page 373: Apuntes MAT-023 USM

Apuntes Mat023 (Segundo semestre 2014)

Ası:

f (x, y)− f (a, b)− ∂f

∂x(a, b) · (x− a)− ∂f

∂y(a, b) · (y − b) ' 0

y por tanto:

f (x, y) ' ∂f

∂x(a, b) · (x− a) +

∂f

∂y(a, b) · (y − b) + f (a, b)

Es decir, para (x, y) suficientemente cercano a (a, b), f (x, y) puede ser aproximada por el

plano:

z = f (a, b) +∂f

∂x(a, b) · (x− a) +

∂f

∂y(a, b) · (y − b)

el cual es tangente a la superficie:

S : z = f (x, y)

en una vecindad de (a, b). Ası, tenemos la siguiente definicion:

Definicion 9.3.3. Sea f : U ⊆ R2 → R una funcion diferenciable en (a, b). Llamaremos

plano tangente a la superficie z = f (x, y) en (a, b) al plano de ecuacion:

z − f (a, b) = (x− a)∂f

∂x(a, b) + (y − b) ∂f

∂y(a, b)

Ejemplo 9.3.5. Encontrar la ecuacion del plano tangente a la superficie S dada por la

grafica de la funcion:

z = (x+ y)2 − 2x

en el punto (1, 1, 0).

Solucion. Calculamos las derivadas parciales

∂z

∂x=

∂x

((x+ y)2 − 2x

)= 2x+ 2y − 2

⇒∂z

∂x(1, 1) = 2

y

∂z

∂y=

∂y

((x+ y)2 − 2x

)= 2x+ 2y

⇒∂z

∂y(1, 1) = 4

el plano tangente es

z = 0 + 2 (x− 1) + 4 (y − 1)

es decir

z = 2x+ 4y − 6

371

Page 374: Apuntes MAT-023 USM

Apuntes Mat023 (Segundo semestre 2014)

Ejemplo 9.3.6. Sean a, b, c numeros reales positivos. Verifique si el cono:

x2

a2+y2

b2=z2

c2

y la esfera:

x2 + y2 +

(z − b2 + c2

c

)2

=b2

c2

(b2 + c2

)son tangentes entre sı en los puntos (0, ±b, c).

Solucion. Notemos que los puntos entregados pertenecen a la interseccion

02

a2+

(±b)2

b2=c2

c2

y

02 + (±b)2 +

(c− b2 + c2

c

)2

=b2

c2

(b2 + c2

)Notemos que

x2

a2+y2

b2=

z2

c2

z = c

√x2

a2+y2

b2

(el punto (0, ±b, c) tiene tercera coordenada positiva, por eso consideramos solo la raız

positiva). El vector normal al plano tangente es(− ∂

∂x

(c

√x2

a2+y2

b2

)∣∣∣∣∣ , − ∂

∂y

(c

√x2

a2+y2

b2

)∣∣∣∣∣ , 1)

=

−cx

a2

√a2y2+b2x2

a2b2

∣∣∣∣∣∣(x,y)=(0,±b)

,−cy

b2

√a2y2+b2x2

a2b2

∣∣∣∣∣∣(x,y)=(0,±b)

, 1

=

(0,∓c

b, 1)

para la superficie

x2 + y2 +

(z − b2 + c2

c

)2

=b2

c2

(b2 + c2

)se tiene ∣∣∣∣z − b2 + c2

c

∣∣∣∣ =

√b2

c2(b2 + c2)− (x2 + y2)

luego

z − b2 + c2

c= ±

√b2

c2(b2 + c2)− (x2 + y2)

372

Page 375: Apuntes MAT-023 USM

Apuntes Mat023 (Segundo semestre 2014)

evaluando en (0,±b, c) se tiene

c− b2 + c2

c= ±

√b2

c2(b2 + c2)− b2

c− b2 + c2

c= ±b

2

c

se sigue que el signo correcto es −. Luego

z =b2 + c2

c−√b2

c2(b2 + c2)− (x2 + y2)

ası el normal corresponde a

(−zx,−zy, 1)|(0,±b)donde

zx =∂

∂x

(b2 + c2

c−√b2

c2(b2 + c2)− (x2 + y2)

)=

x√b4+b2c2−c2x2−c2y2

c2

zy =∂

∂y

(b2 + c2

c−√b2

c2(b2 + c2)− (x2 + y2)

)=

y√b4+b2c2−c2x2−c2y2

c2

evaluando

zx = 0

zy =±b√

b4+b2c2−c2(±b)2c2

=±bb2

c

= ±cb

ası

(−zx,−zy, 1)|(0,±b)=

(0,∓c

b, 1)

se sigue que tienen los mismos planos tangentes.

Teorema 9.3.2. Sean U ⊆ Rn y f : U → R una funcion diferenciable en a ∈U , entonces

f es continua en a.

373

Page 376: Apuntes MAT-023 USM

Apuntes Mat023 (Segundo semestre 2014)

Demostracion. Dado ε > 0, existe δ > 0 tal que:∣∣∣∣∣f (a+ h)− f (a)−n∑i=1

∂f

∂xi(a) · hi

∣∣∣∣∣ ≤ ε ‖h‖

para todo ‖h‖ < δ. Entonces, por la desigualdad triangular, tenemos que:

|f (a+ h)− f (a)| ≤ ε ‖h‖+

∣∣∣∣∣n∑i=1

∂f

∂xi(a) · hi

∣∣∣∣∣≤ ε ‖h‖+ ‖h‖

n∑i=1

∣∣∣∣ ∂f∂xi (a)

∣∣∣∣≤ (ε+ nA) ‖h‖

donde A = max∣∣∣ ∂f∂xi (a)

∣∣∣ : i = 1, 2, . . . , n

. Ahora bien, eligiendo δ = mınδ, ε

ε+nA

se

obtiene que:

‖h‖ < δ =⇒ |f (a+ h)− f (a)| < ε

Observacion 9.3.6. Como ya es sabido desde el calculo diferencial en una variable, el

teorema anterior dice que la continuidad es una condicion necesaria para la diferenciabilidad,

pero no es suficiente. En efecto, considere:

Ejemplo 9.3.7. Verifique que f : R2 → R definida por:

f (x, y) =

x |y|3/2

x2 + y2, si (x, y) 6= (0, 0)

0 , si (x, y) = (0, 0)

es continua, pero no diferenciable en (0, 0).

Solucion. La continuidad es inmediata. En efecto:

|x| |y|3/2

x2 + y2=

|xy|x2 + y2

√|y|

≤ 1

2

x2 + y2

x2 + y2

√|y| =

√|y|2

donde la ultima expresion converge a 0, cuando (x, y)→ (0, 0). Ahora bien, por el Teorema

del Sandwich, se obtiene que:

lım(x,y)→(0,0)

x |y|3/2

x2 + y2= 0 = f (0, 0)

374

Page 377: Apuntes MAT-023 USM

Apuntes Mat023 (Segundo semestre 2014)

Por tanto, f es continua en (0, 0). Por otro lado, es facil ver que:

∂f

∂x(0, 0) =

∂f

∂y(0, 0) = 0

Luego:

lım(h,k)→(0,0)

∣∣∣f (h, k)− f (0, 0)− ∂f∂x

(0, 0) h− ∂f∂y

(0, 0) k∣∣∣

√h2 + k2

= lım(h,k)→(0,0)

|f (h, k)|√h2 + k2

= lım(h,k)→(0,0)

|h | |k|3/2

(h2 + k2)3/2

Al tomar la recta de aproximacion h = k en el ultimo lımite se observa que no existe. Por

tanto, la funcion f no es diferenciable en el (0, 0).

Teorema 9.3.3 (Algebra de funciones diferenciables). Sean f, g : U ⊆ Rn → R funciones

diferenciables en a ∈U y α ∈ R una constante, entonces αf, f + g, f − g, fg son

diferenciables en a. Si, ademas, g (a) 6= 0, entonces f/g tambien es diferenciable en a.

Teorema 9.3.4. Sean f : U ⊆ Rn → R y a ∈U . Si las derivadas parciales ∂f

∂xiexisten en

una bola alrededor de a y son continuas en a entonces f es diferenciable en a.

Observacion 9.3.7. La condicion en el teorema anterior es solo suficiente, mas no necesaria

como puede verse con el siguiente ejemplo:

Ejemplo 9.3.8. Verificar que la funcion f : R2 → R definida por:

f (x, y) =

(x2 + y2) sin

(1√

x2 + y2

), si (x, y) 6= (0, 0)

0 , si (x, y) = (0, 0)

es diferenciable en todo punto de R2 pero las funciones derivadas parciales no son continuas

en R2.

Solucion. Del algebra de funciones diferenciables se obtiene que f es diferenciable en todo

punto distinto de (0, 0). Notemos que para (x, y) 6= (0, 0) se cumple

∂x

(x2 + y2

)sin

(1√

x2 + y2

)

= 2x sin

(1√

x2 + y2

)+(x2 + y2

)cos

(1√

x2 + y2

)(−1

2

(x2 + y2

)−3/22x

)

= 2x sin

(1√

x2 + y2

)− x√

x2 + y2cos

(1√

x2 + y2

)

375

Page 378: Apuntes MAT-023 USM

Apuntes Mat023 (Segundo semestre 2014)

y por simetrıa

∂y

(x2 + y2

)sin

(1√

x2 + y2

)

= 2y sin

(1√

x2 + y2

)− y√

x2 + y2cos

(1√

x2 + y2

)en (0, 0) calculamos las derivadas parciales

∂f

∂x(0, 0) = lım

h→0

f (h, 0)− f (0, 0)

h

= lımh→0

h2 sin(

1|h|

)h

= lımh→0

h sin

(1

|h|

)= 0

y por simetrıa ∂f∂y

(0, 0) = 0. La funcion derivada parcial esta definida por

∂f

∂x(x, y) =

2x sin

(1√x2+y2

)− x√

x2+y2cos

(1√x2+y2

)si (x, y) 6= (0, 0)

0 si (x, y) = (0, 0)

esta funcion no es continua en (0, 0), en efecto, considere los puntos

(xn, yn) =

(1

2nπ + π4

, 0

)entonces si n→ +∞, (xn, yn)→ (0, 0) pero

∂f

∂x(xn, yn) =

2

2nπ + π4

sin(

2nπ +π

4

)− cos

(2nπ +

π

4

)=

2

2nπ + π4

1√2− 1√

2

luego cuando n→ +∞ se sigue (xn, yn)→ (0, 0) pero ∂f∂x

(xn, yn)→ − 1√2, el lımite no es

cero (usando otra sucesion se muestra que en realidad el lımite no existe por ejemplo con

(xn, yn) =(

12nπ+π

3, 0)

da otro lımite). Se sigue que f no es de clase C1 sin embargo

lım(x,y)→(0,0)

|f (x, y)|√x2 + y2

= lım(x,y)→(0,0)

∣∣∣∣(x2 + y2) sin

(1√x2+y2

)∣∣∣∣√x2 + y2

= lım(x,y)→(0,0)

√x2 + y2

∣∣∣∣∣sin(

1√x2 + y2

)∣∣∣∣∣= 0

376

Page 379: Apuntes MAT-023 USM

Apuntes Mat023 (Segundo semestre 2014)

aquı hemos usado el acotamiento

0 ≤√x2 + y2

∣∣∣∣∣sin(

1√x2 + y2

)∣∣∣∣∣ ≤√x2 + y2

luego f es diferenciable en (0, 0) y en todos los otros puntos por algebra de funciones

diferenciables pero f no tiene derivadas parciales continuas en el origen.

Ejemplo 9.3.9. Considere la funcion f : R2 → R definida por

f (x, y) =

x+ y2 si x < y2

x+ y4 si x ≥ y2

1. Determine el mayor subconjunto de R2 en el cual f es continua.

Por algebra de continuas y la definicion de la funcion esta es continua en todos los

puntos (x0, y0) en los cuales x0 < y20 y x0 > y2

0. Vamos a analizar en los puntos de la

forma (a2, a) que son los puntos en el cual la definicion de la funcion cambia.

Notemos que

lım(x,y)→(a2,a)

f (x, y) =

a2 + a2 por la region x < y2

a2 + a4 por la region x ≥ y2

377

Page 380: Apuntes MAT-023 USM

Apuntes Mat023 (Segundo semestre 2014)

luego la funcion es continua si a2 + a2 = a2 + a4 esto es a = 1, 0,−1. Se sigue que el

mayor subconjunto de R2 en el cual f es continua es(x, y) ∈ R2 : x 6= y2

∪ (0, 0) , (1, 1) , (1,−1)

2. Estudiar la diferenciabilidad de f en (0, 0).

Primero determinamos las derivadas parciales en el origen

∂f

∂x(0, 0) = lım

h→0

f (h, 0)− f (0, 0)

h

= lımh→0

f (h, 0)

h

note que los puntos de la forma (h, 0) estan en las dos regiones de definicion de f

dependiendo del signo de h

lımh→0+

f (h, 0)

h= lım

h→0+

h+ 04

h= 1

lımh→0−

f (h, 0)

h= lım

h→0+

h+ 02

h= 1

se sigue que∂f

∂x(0, 0) = 1

de manera similar

∂f

∂y(0, 0) = lım

h→0

f (0, h)− f (0, 0)

h

= lımh→0

f (0, h)

h

los puntos de la forma (0, h) con h 6= 0 siempre pertenecen a la region x < y2 se sigue

lımh→0

f (0, h)

h= lım

h→0

h2

h= 0

ası ∂f∂y

(0, 0) = 0. Ahora estudiamos la diferenciabilidad

lım(x,y)→(0,0)

∣∣∣f (x, y)− f (0, 0)− ∂f∂x

(0, 0) (x− 0)− ∂f∂y

(0, 0) (y − 0)∣∣∣√

x2 + y2

= lım(x,y)→(0,0)

|f (x, y)− x|√x2 + y2

pero

|f (x, y)− x| =

y2 si x < y2

y4 si x ≥ y2

378

Page 381: Apuntes MAT-023 USM

Apuntes Mat023 (Segundo semestre 2014)

se sigue que

|f (x, y)− x|√x2 + y2

=

y2√x2+y2

si x < y2

y4√x2+y2

si x ≥ y2, (x, y) 6= (0, 0)

de donde obtenemos

lım(x,y)→(0,0)

|f (x, y)− x|√x2 + y2

= 0

luego f es diferenciable en (0, 0).

3. Si existe, determine el plano tangente a la grafica de f en (2, 0, f (2, 0)).

En un entorno de (2, 0) la funcion esta definida como x+ y4 luego es diferenciable y

el plano tangente corresponde a

z − f (2, 0) = fx (2, 0) (x− 2) + fy (2, 0) (y − 0)

⇔z − 2 = (x− 2) + 0

⇔z = x

Ejemplo 9.3.10. Determine (justificando) los valores de α ∈ R+ para los cuales las funcion

f (x, y) =

|x|α y2

(x2 + y2)αsi (x, y) 6= (0, 0)

0 si (x, y) = (0, 0)

1. Es continua en (0, 0).

La funcion es continua en (0, 0) si y solo si

lım(x,y)→(0,0)

f (x, y) = f (0, 0) = 0

esto es

lım(x,y)→(0,0)

|x|α y2

(x2 + y2)α= 0

usando polares

lımr→0θ libre

rαr2 |cos θ|α sin2 θ

r2α= lım

r→0θ libre

rα+2−2α |cos θ|α sin2 θ

= lımr→0θ libre

r2−α |cos θ|α sin2 θ

= 0

si y solo si 2− α > 0 esto es 0 < α < 2.

379

Page 382: Apuntes MAT-023 USM

Apuntes Mat023 (Segundo semestre 2014)

2. Es diferenciable en (0, 0).

Primero calculamos las derivadas parciales

∂f

∂x(0, 0) = lım

h→0

f (h, 0)− f (0, 0)

h

= lımh→0

f (h, 0)

h

= lımh→0

1

h

|h|α 02

(h2 + 02)α= 0

de manera similar

∂f

∂y(0, 0) = lım

h→0

f (0, h)− f (0, 0)

h

= lımh→0

f (0, h)

h

= lımh→0

1

h

|0|α h2

(02 + h2)α

= 0

luego, la funcion es diferenciable si y solo si

lım(x,y)→(0,0)

|f (x, y)− f (0, 0)− fx (0, 0) (x− 0)− fy (0, 0) (y − 0)|√x2 + y2

= 0

es decir

lım(x,y)→(0,0)

|f (x, y)|√x2 + y2

= lım(x,y)→(0,0)

|x|α y2

(x2 + y2)α√x2 + y2

= 0

usando polares

lımr→0θ libre

rαr2 |cos θ|α sin2 θ

(r2α) r= lım

r→0θ libre

rα+2−2α−1 |cos θ|α sin2 θ

= lımr→0θ libre

r1−α |cos θ|α sin2 θ

= 0

si y solo si 1− α > 0 esto es 0 < α < 1.

Ejemplo 9.3.11. Sean g : R2 → R dada por:

g(x, y) =

y3

x2 + y2si (x, y) 6= (0, 0)

0 si (x, y) = (0, 0)

y f : R2 → R definida por:

f(x, y) =

1 si (x, y) = (0, 0)

eg(x,y) si (x, y) 6= (0, 0),

380

Page 383: Apuntes MAT-023 USM

Apuntes Mat023 (Segundo semestre 2014)

1. ¿Es f(x, y) continua en (0,0)?

2. Determine∂f

∂x(0, 0) y

∂f

∂y(0, 0).

3. ¿Es f(x, y) diferenciable en (0, 0)?

Solucion. 1. Puesto que la funcion e(·) es continua, estudiar la continuidad de f en

el origen equivale a estudiar la continuidad de g en dicho punto. Para probar la

continuidad de g en el origen utilizamos acotamiento. Se tiene:

0 ≤ |g(x, y)− 0| = |y3|x2 + y2

= |y| y2

x2 + y2

≤ |y|x2 + y2

x2 + y2

= |y| → 0

cuando (x, y)→ (0, 0). Por lo tanto

lım(x,y)→(0,0)

f(x, y) = exp

(lım

(x,y)→(0,0)g(x, y)

)= e0 = 1

y la funcion f es continua en el origen.

2. Procedemos por definicion. Se tiene que:

∂f

∂x(0, 0) = lım

h→0

e0/h2 − 1

h= 0

Por otro lado:∂f

∂y(0, 0) = lım

k→0

ek − 1

k= lım

k→0ek = 1

3. Para verificar la diferenciabilidad de f debemos probar que el siguiente lımite:

lım(h,k)→(0,0))

|f(h, k)− (f(0, 0) + fx(0, 0)h+ fy(0, 0)k)|√h2 + k2

=

= lım(h,k)→(0,0))

∣∣∣exp(

k3

h2+k2

)− 1− k

∣∣∣√h2 + k2

sea cero. Sin embargo, este lımite no es nulo. En efecto, basta elegir la trayectoria

h = k (el puntaje se otorga por elegir una trayectoria apropiada) y calcular:

lımk→0

|ek/2 − 1− k|√2|k|

= lımk→0

∣∣∣∣ek/2 − 1− k√2k

∣∣∣∣ = lımk→0

∣∣∣∣∣ 12ek/2 − 1√

2

∣∣∣∣∣ =1

2√

26= 0

luego f no es diferenciable en el origen.

381

Page 384: Apuntes MAT-023 USM

Apuntes Mat023 (Segundo semestre 2014)

Ejercicios de la seccion

1. Hallar la ecuacion del plano tangente a la superficie z = x2 + y3 en el punto (3, 1, 10).

2. ¿Por que deben llamarse tangentes en (0, 0) las graficas de f (x, y) = x2 + y2 y

g (x, y) = −x2 − y2 + xy3?

3. Sea f : R2 → R definida por

f (x, y) =

x2y4

x2+y2si (x, y) 6= (0, 0)

0 si (x, y) = (0, 0)

Probar que f es de clase C1 (R2).

4. Sea p ≥ 1. Estudiar la diferenciabilidad en el origen de la funcion

‖·‖p : Rn → R

x→ ‖x‖p =

(n∑k=1

|xi|p)1/p

5. ¿Para que valores de α la funcion

fα (x, y) =

x3+xy2

|x|α+|y|α si (x, y) 6= (0, 0)

0 si (x, y) = (0, 0)

es diferenciable en (0, 0)?

6. Considere la superficie definida por

S =

(x, y, z) ∈ R3 : x sin

(x

y

)− z = 0

Si (a, b, c) ∈ S determine el plano tangente a la superficie en ese punto y mostrar que

pasa por el origen.

7. Utilizar la aproximacion mediante el plano tangente para estimar el valor de:

a) (0,99e0,1)10

b)√

(3,99)2 + (4,01)2 + (2,01)2

8. Sean α > 1 y f : Rn → R, x→f (x) una funcion tal que, para todo x,y ∈ Rn

|f (x)− f (y)| ≤ ‖x− y‖α

Muestre que f es diferenciable.

382

Page 385: Apuntes MAT-023 USM

Apuntes Mat023 (Segundo semestre 2014)

Derivadas de orden superior y funciones de clase Cn

Observacion 9.4.1. Sean U ⊆ Rn un conjunto abierto no vacıo y f : U → R una funcion

tal que las derivadas parciales ∂f∂xi

(x) existen para todo x ∈ U . Entonces, podemos definir

la funcion derivada parcial de f como la funcion ∂f∂xi

: U ⊆ Rn → R dada por x 7→ ∂f∂xi

(x).

Las derivadas parciales de orden superior son, entonces, derivadas parciales de la funcion∂f∂xi

: U ⊆ Rn → R. Mas precisamente:

Definicion 9.4.1. Sean U ⊆ Rn un conjunto abierto no vacıo y f : U → Rm una funcion

tal que las derivadas parciales ∂f∂xi

(x) existen para todo x ∈ U . Se define la derivada parcial

de segundo orden para f como:

∂2f

∂xj∂xi(x) =

∂xj

(∂f

∂xi

)(x)

Mas precisamente:

∂2f

∂xj∂xi(x) = lım

t→0

1

t

(∂f

∂xi(x + tej)−

∂f

∂xi(x)

)Ademas, si i 6= j la derivada ∂2f

∂xj∂xi(x) recibe el nombre de derivada parcial mixta. Si i = j,

anotamos ∂2f∂x2i

(x).

La derivada parcial de tercer orden para f se define como:

∂3f

∂xk∂xj∂xi(x) =

∂xk

(∂2f

∂xj∂xi

)(x)

considerando ∂2f∂xj∂xi

: U∗ ⊆ Rn → R, en donde U∗ corresponde al conjunto de puntos de

U para los cuales la segunda derivada parcial esta definida. En particular, si i = j = k,

anotamos entonces:∂3f

∂x3i

(x)

Finalmente, las derivadas parciales de orden superior se definen como las derivadas

parciales sucesivas de ∂f∂xi

: U ⊆ Rn → R.

Ejemplo 9.4.1. Hallar las derivadas parciales de segundo orden de la funcion:

f (x, y) = arctanx

y

Solucion.

∂f

∂x=

∂x

(arctan

x

y

)=

y

x2 + y2

∂f

∂y=

∂y

(arctan

x

y

)= − x

x2 + y2

383

Page 386: Apuntes MAT-023 USM

Apuntes Mat023 (Segundo semestre 2014)

y las de segundo orden

∂2f

∂x2=

∂x

(y

x2 + y2

)=

−2xy

(x2 + y2)2

∂2f

∂y∂x=

∂y

(y

x2 + y2

)=

x2 − y2

(x2 + y2)2

∂2f

∂x∂y=

∂x

(− x

x2 + y2

)=

x2 − y2

(x2 + y2)2

∂2f

∂y2=

∂y

(− x

x2 + y2

)=

2xy

(x2 + y2)2

notar que, en este caso,∂2f

∂x2+∂2f

∂y2= 0

y∂2f

∂y∂x=

∂2f

∂x∂y

Ejemplo 9.4.2. Hallar∂3z

∂x∂y2, si:

z = sinxy

Solucion.

zy =∂

∂y(sinxy) = x cosxy

luego

zyy =∂

∂y(x cosxy) = −x2 sinxy

y finalmente

zyyx =∂

∂x

(−x2 sinxy

)= −2x sinxy − x2y cosxy

Definicion 9.4.2. Sean U ⊆ Rn un conjunto abierto no vacıo. Diremos que la funcion

f : U → R es de clase Cn sobre U, o bien que f ∈ Cn (U) si todas las derivadas parciales

hasta el orden n−esimo son funciones continuas en U .

Observacion 9.4.2. A modo de resumen, tenemos que:

f ∈ C1 (U) =⇒ f es diferenciable en U =⇒ f es continua en U

Teorema 9.4.1 (Schwarz). Sean U ⊆ Rn un conjunto abierto no vacıo y f ∈ C2 (U),

entonces, para todo x ∈ U y para todo i 6= j

∂2f

∂xj∂xi(x) =

∂2f

∂xi∂xj(x)

384

Page 387: Apuntes MAT-023 USM

Apuntes Mat023 (Segundo semestre 2014)

Observacion 9.4.3. Del teorema anterior y bajo la hipotesis de clase Cn adecuada se

puede establecer igualdad de derivadas parciales mixtas de tercer orden y superior, note

por ejemplo que

∂3f

∂y∂x2=

∂2

∂y∂x

(∂f

∂x

)=

∂2

∂x∂y

(∂f

∂x

)=

∂x

(∂2f

∂y∂x

)=

∂x

(∂2f

∂x∂y

)=

∂3f

∂x2∂y

si f ∈ C3.

Observacion 9.4.4. La continuidad de las derivadas parciales de segundo orden es funda-

mental en el teorema anterior. Esto se puede ilustrar considerando el siguiente ejemplo:

Ejemplo 9.4.3. Sea f : R2 → R la funcion definida por:

f (x, y) =

xy(x2−y2x2+y2

)si (x, y) 6= (0, 0)

0 si (x, y) = (0, 0)

Verifique que:∂2f

∂y∂x(0, 0) 6= ∂2f

∂x∂y(0, 0)

Solucion. Note que

∂f

∂x(x, y) =

∂x

(xy

(x2 − y2

x2 + y2

))=

y (x4 + 4x2y2 − y4)

(x2 + y2)2

para (x, y) 6= (0, 0) y∂f

∂x(0, 0) = lım

h→0

f (h, 0)− f (0, 0)

h= 0

luego

∂f

∂x(x, y) =

y(x4+4x2y2−y4)

(x2+y2)2si (x, y) 6= (0, 0)

0 si (x, y) = (0, 0)

385

Page 388: Apuntes MAT-023 USM

Apuntes Mat023 (Segundo semestre 2014)

se sigue que

∂2f

∂y∂x(0, 0) = lım

h→0

1

h

(∂f

∂x(0, h)− ∂f

∂x(0, 0)

)= lım

h→0

1

h

(h (0− h4)

(02 + h2)2

)= −1

por otro lado

∂f

∂y(x, y) =

∂y

(xy

(x2 − y2

x2 + y2

))=−x (−x4 + 4x2y2 + y4)

(x2 + y2)2

y∂f

∂y(0, 0) = 0

se sigue

∂2f

∂x∂y(0, 0) = lım

h→0

1

h

(∂f

∂y(h, 0)− ∂f

∂y(0, 0)

)= lım

h→0

1

h

(−h (−h4)

(h2 + 02)2

)= 1

se sigue ∂2f∂y∂x

(0, 0) 6= ∂2f∂x∂y

(0, 0) en este caso la funcion no es de clase C2 (R2).

Ejemplo 9.4.4. Si u = ln (x2 + y2) verificar que

∂2u

∂x2+∂2u

∂y2= 0

Solucion. Derivando

∂u

∂x=

∂x

(ln(x2 + y2

))=

2x

x2 + y2

∂u

∂y=

∂y

(ln(x2 + y2

))=

2y

x2 + y2

y de segundo orden

∂2u

∂x2=

∂x

(2x

x2 + y2

)= −2

x2 − y2

(x2 + y2)2

∂2u

∂y2=

∂y

(2y

x2 + y2

)= 2

x2 − y2

(x2 + y2)2

ası∂2u

∂x2+∂2u

∂y2= −2

x2 − y2

(x2 + y2)2 + 2x2 − y2

(x2 + y2)2 = 0

386

Page 389: Apuntes MAT-023 USM

Apuntes Mat023 (Segundo semestre 2014)

Ejercicios de la seccion

1. Si u = arctan(xy/√

1 + x2 + y2)

probar que uxy = (1 + x2 + y2)−3/2

y

uxxyy =15xy

(1 + x2 + y2)7/2

2. Sean a, b, c constantes positivas. Si z = z (x, y) es tal que a2x2 + b2y2 = c2z2 entonces

muestre que

zxxzyy = (zxy)2

3. Si u = ln (x2 + y2 + z2) probar que

xuyz = yuzx = zuxy

4. Si f (x, y) = xyx+y

demuestre que

x2∂2f

∂x2+ 2xy

∂2f

∂x∂y+ y2∂

2f

∂y2= 0

Gradiente y matriz jacobiana

Definicion 9.5.1. Sean U ⊆ Rn y f : U → R una funcion tal que todas las derivadas

parciales ∂f∂xi

(a), i = 1, 2, . . . , n de f existen en a ∈U . Se define el gradiente de f en a

como el vector en Rn dado por:

∇f (a) =

(∂f

∂x1

(a) ,∂f

∂x2

(a) , . . . ,∂f

∂xn(a)

)Ejemplo 9.5.1. Sea f (x, y, z) = 3x2 + sin y2 + 2xz2. Calcule ∇f (x, y, z).

Solucion. En este caso

∇f (x, y, z) =

(∂f (x, y, z)

∂x,∂f (x, y, z)

∂y,∂f (x, y, z)

∂z

)es decir

∇f (x, y, z) =(2z2 + 6x, 2y cos y2, 4xz

)Ejemplo 9.5.2. Considere f (x, y, z) = 3x2 + 2xy − 4z3. Sean S la superficie dada por:

S : f (x, y, z) = 1

y Π el plano tangente a S en (1, 1, 1). Si u,v ∈ Π linealmente independientes, entonces

pruebe que existe λ ∈ R tal que:

u× v = λ · ∇f (1, 1, 1)

387

Page 390: Apuntes MAT-023 USM

Apuntes Mat023 (Segundo semestre 2014)

Solucion. Si u,v ∈ Π son linealmente independientes entonces u× v es normal al plano,

pero de las secciones anteriores sabemos que el normal a la grafica de una funcion z = h (x, y)

tiene la forma k (−hx,−hy, 1) donde k es una constante, la superficie f (x, y, z) = 1 la

podemos mirar como la grafica de

z =3

√3

4x2 +

1

2yx− 1

4

se sigue

(−zx,−zy, 1)

=

−2

33√

232x+ 1

2y

(3x2 + 2yx− 1)23

,−2x

3

√34x2 + 1

2yx− 1

4

9x2 + 6yx− 3, 1

evaluando en (1, 1)

(−zx,−zy, 1) =

−4

33

√3

4+

1

2− 1

4

32

+ 12

3 + 2− 1,−2

3

√34

+ 12− 1

4

9 + 6− 3, 1

=

(−2

3,−1

6, 1

)note que

∇f (x, y, x) =(6x+ 2y, 2x,−12z2

)luego

∇f (1, 1, 1) = (6 + 2, 2,−12)

= (8, 2,−12)

se sigue que existe λ ∈ R tal que

u× v =k

(−2

3,−1

6, 1

)= λ∇f (1, 1, 1)

Observacion 9.5.1. El isomorfismo canonico ϕ : Rn ' M1×n (R) vuelve a jugar un rol

importante en relacionar el vector gradiente ∇f (a) y la matriz de la diferencial Df (a).

Sabemos que la diferencial de una funcion f : Rn → R esta dada por la transformacion

lineal Df (a) : Rn → R tal que:

lımh→0

|f (a + h)− f (a)−Df (a) · h|‖h‖

= 0

pero ϕ−1 (Df (a)) = ∇f (a), por lo tanto, podemos escribir:

lımh→0

|f (a + h)− f (a)−∇f (a) · h|‖h‖

= 0

388

Page 391: Apuntes MAT-023 USM

Apuntes Mat023 (Segundo semestre 2014)

Observacion 9.5.2. Nuestro proposito, ahora, es tratar la diferenciabilidad para funciones

vectoriales de la forma f : U ⊆ Rn → Rm.. Teniendo en cuenta que la topologıa del espacio

euclidiano Rm se obtiene considerando la norma ‖·‖Rm , tenemos que:

Definicion 9.5.2. Sean U ⊆ Rn y f : U → Rm una funcion vectorial. Diremos que f es

diferenciable en a ∈U si existe una transformacion lineal T : Rn → Rm tal que:

lımh→0

‖f (a + h)− f (a)− T (h)‖‖h‖

= 0

Definicion 9.5.3. Sean U ⊆ Rn y f : U → Rm una funcion tal que todas las derivadas

parciales∂fi∂xj

existen en a. Llamaremos matriz jacobiana de f en a a la matriz de derivadas

parciales:

J f (a) =

(∂fi∂xj

(a)

)m×n

=

∂f1

∂x1

(a)∂f1

∂x2

(a) · · · ∂f1

∂xn(a)

∂f2

∂x1

(a)∂f2

∂x2

(a) · · · ∂f2

∂xn(a)

......

. . ....

∂fm∂x1

(a)∂fm∂x2

(a) · · · ∂fm∂xn

(a)

∈Mm×n (R)

Ejemplo 9.5.3. Sea f : R2 → R2 definida por f (x, y) = (2x2y + y, 2xy − y2). Luego, si

anotamos u = 2x2y + y y v = 2xy − y2, entonces la matriz jacobiana de f esta dada por:

Jf (x, y) =

∂u

∂x

∂u

∂y∂v

∂x

∂v

∂y

=

(4xy 2x2

2y 2x− 2y

)Teorema 9.5.1. Sean f : U ⊆ Rn → Rm una funcion definida por f (x) = (f1 (x) , f2 (x) , . . . , fm (x))

y a ∈U . f es diferenciable en a, si y solo si, fi : U → R es diferenciable en a, para cada

i = 1, 2, . . . ,m. Ademas:

Df (a) =

(∂fi∂xj

(a)

)m×n

= Jf (a)

Ejemplo 9.5.4. ¿Es diferenciable g (x, y, z) =(x+ sinx cos y2, x2y + 1,

√z2 + 1

)en R3?

389

Page 392: Apuntes MAT-023 USM

Apuntes Mat023 (Segundo semestre 2014)

Solucion. La respuesta es si, pues las funciones componentes son todas diferenciables en

R3.

Observacion 9.5.3. Por definicion, tenemos que si f : U ⊆ Rn → Rm es diferenciable en

a, entonces existe una transformacion lineal T : Rn → Rm tal que:

lımh→0

‖f (a + h)− f (a)− T (h)‖‖h‖

= 0

Ademas, note que para cada i = 1, 2, . . . ,m, se tiene que:

|fi (a + h)− fi (a)− Ti (h)| ≤ ‖f (a + h)− f (a)− T (h)‖

para todo h. Ası, si h = t · ej, tenemos que:

0 ≤∣∣∣∣fi (a + t · ej)− fi (a)

t− Ti (ej)

∣∣∣∣=

∣∣∣∣fi (a + t · ej)− fi (a)− Ti (tej)t

∣∣∣∣≤ ‖f (a + t · ej)− f (a)− T (tej)‖

‖tej‖→ 0

si acaso t→ 0. Lo anterior implica que:

[Ti]j =∂fi∂xj

(a)

en donde [Ti]j representa el j−esimo coeficiente de la matriz [Ti], respecto de las bases

canonicas. Todo lo anterior implica que en caso de que f sea diferenciable en a, la matriz

jacobiana Jf (a) es la matriz de la diferencial Df (a). Desde ahora en adelante, anotamos:

Df (a) = Jf (a)

La regla de la cadena

Teorema 9.6.1. Sean f : U ⊆ Rn → Rm, con U abierto y g : V ⊆ Rm → Rp, con

V abierto, tales que f (U) ⊆ V . Suponga que f es diferenciable en a ∈ U y que g es

diferenciable en b = f (a) ∈ V , entonces:

g f : U ⊆ Rn → Rp

es diferenciable en a. Ademas:

D (g f) (a) = Dg (f (a))Df (a)

= Dg (b)Df (a)

390

Page 393: Apuntes MAT-023 USM

Apuntes Mat023 (Segundo semestre 2014)

Observacion 9.6.1. Es importante destacar que Dg (b) ∈ Mp×m (R) y que Df (a) ∈Mm×n (R).

Observacion 9.6.2. Supongamos que z = g (f (x)) e y = f (x), con b = f (a), entonces

la regla de la cadena en terminos de las matrices jacobianas queda en la forma:

D (g f) (a)

= Dg (b) ·Df (a)

=

∂z1

∂y1

(b)∂z1

∂y2

(b) · · · ∂z1

∂ym(b)

∂z2

∂y1

(b)∂z2

∂y2

(b) · · · ∂z2

∂ym(b)

......

. . ....

∂zp∂y1

(b)∂zp∂y2

(b) · · · ∂zp∂ym

(b)

·

∂y1

∂x1

(a)∂y1

∂x2

(a) · · · ∂y1

∂xn(a)

∂y2

∂x1

(a)∂y2

∂x2

(a) · · · ∂y2

∂x2

(a)

......

. . ....

∂ym∂x1

(a)∂ym∂x2

(a) · · · ∂ym∂xn

(a)

=

(m∑k=1

∂zi∂yk

(b)∂yk∂xj

(a)

)i×j

∈Mp×n (R)

Por tanto, en notaciones clasicas, mirando las componentes del producto, la regla viene

dado por las formulas:

∂zi∂xj

=m∑k=1

∂zi∂yk

∂yk∂xj

=∂zi∂y1

∂y1

∂xj+∂zi∂y2

∂y2

∂xj+ · · ·+ ∂zi

∂ym

∂ym∂xj

para i = 1, 2, . . . , p y j = 1, 2, . . . , n.

Ejemplo 9.6.1. Sea f : R2 → R, (u, v)→ f (u, v) una funcion diferenciable. Se define la

funcion w : R2 → R por

w (x, y) = f(x2 − y2, y2 − x2

)calcular la Jacobiana de w y muestre que

y∂w

∂x+ x

∂w

∂y= 0

Solucion. Por la regla de la cadena

Jw =(wx wy

)=(fu fv

)( 2x −2y

−2x 2y

)=

(2xfu − 2xfv −2yfu + 2yfv

)391

Page 394: Apuntes MAT-023 USM

Apuntes Mat023 (Segundo semestre 2014)

ası

y∂w

∂x+ x

∂w

∂y

= y (2xfu − 2xfv) + x (−2yfu + 2yfv)

= 0

Ejemplo 9.6.2. Sean g : R3 → R y f : R3 → R3 tales que:

f (x, y, z) =(u (x, y, z) ; v (x, y, z) ; w (x, y, z)

)Definamos F : R3 → R mediante la ecuacion:

F (x, y, z) = (g f) (x, y, z)

Por la regla de la cadena:

DF (x, y, z) = Dg (u, v, w) ·Df (x, y, z)

=

(∂g

∂u

∂g

∂v

∂g

∂w

∂u

∂x

∂u

∂y

∂u

∂z∂v

∂x

∂v

∂y

∂v

∂z∂w

∂x

∂w

∂y

∂w

∂z

Ası, por ejemplo:

∂F

∂x=∂g

∂u

∂u

∂x+∂g

∂v

∂v

∂x+∂g

∂w

∂w

∂x

Ejemplo 9.6.3. Sea g : R3 → R y consideremos el cambio de variables a coordenadas

esfericas:

ϕ (r, θ, φ) :

x = r cos θ sinφ

y = r sin θ sinφ

z = r cosφ

Defina z (r, θ, φ) = g(x (r, θ, φ) , y (r, θ, φ) , z (r, θ, φ)

). Calcule ∂z

∂r, ∂z∂θ

y ∂z∂φ

.

Solucion.

∂z

∂r=

∂g

∂x

∂x

∂r+∂g

∂y

∂y

∂r+∂g

∂z

∂z

∂r

= (cos θ sinφ)∂g

∂x+ (sin θ sinφ)

∂g

∂y+ (cosφ)

∂g

∂z

∂z

∂θ=

∂g

∂x

∂x

∂θ+∂g

∂y

∂y

∂θ+∂g

∂z

∂z

∂θ

= (−r sin θ sinφ)∂g

∂x+ (r cos θ sinφ)

∂g

∂y+ (0)

∂g

∂z

= (−r sin θ sinφ)∂g

∂x+ (r cos θ sinφ)

∂g

∂y

392

Page 395: Apuntes MAT-023 USM

Apuntes Mat023 (Segundo semestre 2014)

∂z

∂φ=

∂g

∂x

∂x

∂φ+∂g

∂y

∂y

∂φ+∂g

∂z

∂z

∂φ

= (r cos θ cosφ)∂g

∂x+ (r sin θ cosφ)

∂g

∂y+ (−r sinφ)

∂g

∂z

Ejemplo 9.6.4. Considere:

w = x3yez

Suponga que: x = s2 − t2y = s2 + t2

z = s+ t

Pruebe que:∂w

∂s(1, 0) +

∂w

∂t(1, 0) = 10e

Solucion. Usando la regla de la cadena

∂w

∂s=

∂w

∂x

∂x

∂s+∂w

∂y

∂y

∂s+∂w

∂y

∂y

∂s∂w

∂t=

∂w

∂x

∂x

∂t+∂w

∂y

∂y

∂t+∂w

∂y

∂y

∂t

recordar que las derivadas de w quedan evaluadas en x = s2 − t2, y = s2 + t2, z = s+ t ası

∂w

∂s(s, t) =

∂w

∂x

(s2 − t2, s2 + t2, s+ t

) ∂x∂s

(s, t)

+∂w

∂y

(s2 − t2, s2 + t2, s+ t

) ∂y∂s

(s, t)

+∂w

∂y

(s2 − t2, s2 + t2, s+ t

) ∂y∂s

(s, t)

y

∂w

∂t(s, t) =

∂w

∂x

(s2 − t2, s2 + t2, s+ t

) ∂x∂t

(s, t)

+∂w

∂y

(s2 − t2, s2 + t2, s+ t

) ∂y∂t

(s, t)

+∂w

∂y

(s2 − t2, s2 + t2, s+ t

) ∂y∂t

(s, t)

evaluando en (1, 0) se obtiene

∂w

∂s(1, 0) =

∂w

∂x(1, 1, 1)

∂x

∂s(1, 0) +

∂w

∂y(1, 1, 1)

∂y

∂s(1, 0) +

∂w

∂z(1, 1, 1)

∂z

∂s(1, 0)

∂w

∂t(1, 0) =

∂w

∂x(1, 1, 1)

∂x

∂t(1, 0) +

∂w

∂y(1, 1, 1)

∂y

∂t(1, 0) +

∂w

∂z(1, 1, 1)

∂z

∂t(1, 0)

393

Page 396: Apuntes MAT-023 USM

Apuntes Mat023 (Segundo semestre 2014)

note que

∂w

∂x(x, y, z) =

(3x2yez

)∂w

∂y(x, y, z) =

(x3ez

)∂w

∂z(x, y, z) =

(x3yez

)evaluando

∇w (1, 1, 1) = (3e, e, e)

por otro lado

∂x

∂s(s, t) = 2s y

∂x

∂t(s, t) = −2t

∂y

∂s(s, t) = 2s y

∂y

∂t(s, t) = 2t

∂z

∂s(s, t) = 1 y

∂z

∂t(s, t) = 1

evaluando

∂x

∂s(1, 0) = 2 y

∂x

∂t(1, 0) = 0

∂y

∂s(1, 0) = 2 y

∂y

∂t(1, 0) = 0

∂z

∂s(1, 0) = 1 y

∂z

∂t(1, 0) = 1

se sigue

∂w

∂s(1, 0) =

∂w

∂x(1, 1, 1)

∂x

∂s(1, 0) +

∂w

∂y(1, 1, 1)

∂y

∂s(1, 0) +

∂w

∂z(1, 1, 1)

∂z

∂s(1, 0)

= (3e) (2) + (e) (2) + (e) (1)

= 9e

y

∂w

∂t(1, 0) =

∂w

∂x(1, 1, 1)

∂x

∂t(1, 0) +

∂w

∂y(1, 1, 1)

∂y

∂t(1, 0) +

∂w

∂z(1, 1, 1)

∂z

∂t(1, 0)

= (3e) (0) + (e) (0) + (e) (1)

= e

se sigue∂w

∂s(1, 0) +

∂w

∂t(1, 0) = 10e

394

Page 397: Apuntes MAT-023 USM

Apuntes Mat023 (Segundo semestre 2014)

Ejemplo 9.6.5. Sea w = f (r) una funcion diferenciable, definamos h (x, y, z) = f(√

x2 + y2 + z2)

.

Demuestre que: (dw

dr

)2

= ‖∇h‖2

Solucion. La derivada de w respecto a r es

dw

dr= f ′ (r)

luego (dw

dr

)2

= (f ′ (r))2

por otro lado, usando la regla de la cadena

hx = f ′ (r) rx = f ′ (r)x

r

hy = f ′ (r) ry = f ′ (r)y

r

hz = f ′ (r) rz = f ′ (r)z

r

se sigue

‖∇h‖2 =(f ′ (r)

x

r

)2

+(f ′ (r)

y

r

)2

+(f ′ (r)

z

r

)2

=

(f ′ (r)

r

)2 (x2 + y2 + z2

)=

(f ′ (r)

r

)2 (r2)

= (f ′ (r))2

como se querıa probar.

Ejemplo 9.6.6. Sean f : [0,+∞) → R una funcion diferenciable y u : R → R definida

por:

u = f(‖(x, y, z)‖2)

donde: x = ρ cosϕ cosψ

y = ρ cosϕ sinψ

z = ρ sinϕ

Demuestre que∂u

∂ϕ= 0.

395

Page 398: Apuntes MAT-023 USM

Apuntes Mat023 (Segundo semestre 2014)

Solucion. Usando la regla de la cadena, note que u (ρ, ϕ, ψ) = f (g (h (ρ, ϕ, ψ))) donde

p → f (p)

(x, y, z) → g (x, y, z) = x2 + y2 + z2

(ρ, ϕ, ψ) → h (ρ, ϕ, ψ) = (ρ cosϕ cosψ, ρ cosϕ sinψ, ρ sinϕ)

se sigue

∂u

∂ϕ= f ′

∂g

∂x

∂x

∂ϕ+ f ′

∂g

∂y

∂y

∂ϕ+ f ′

∂g

∂z

∂z

∂ϕ

= f ′(ρ2)

(2ρ cosϕ cosψ) (−ρ sinϕ cosψ)

+f ′(ρ2)

(2ρ cosϕ sinψ) (−ρ sinϕ sinψ)

+f ′(ρ2)

(2ρ sinϕ) (ρ cosϕ)

= 2ρf ′(ρ2) (− cosϕ sinϕ cos2 ψ − cosϕ sinϕ sin2 ψ + sinϕ cosϕ

)= 2ρf ′

(ρ2)

(0)

= 0

otra forma de verlo es

u = f(‖(x, y, z)‖2)

con x = ρ cosϕ cosψ

y = ρ cosϕ sinψ

z = ρ sinϕ

queda

u = f(ρ2)

luego la derivada es 0.

Ejemplo 9.6.7. Cambiar las variables x, y, z en la ecuacion

x∂u

∂x+ y

∂u

∂y+ z

∂u

∂z= nu

por ξ, η, ϕ de la forma

ξ =x

z, η =

y

zy ϕ = z

y probar que la ecuacion se transforma en

ϕ∂v

∂ϕ= nv

deducir de esto que u tiene la forma

u (x, y, z) = znF(xz,y

z

)para una funcion F arbitraria.

396

Page 399: Apuntes MAT-023 USM

Apuntes Mat023 (Segundo semestre 2014)

Solucion. Definimos

v (ξ (x, y, z) , η (x, y, z) , ϕ (x, y, z)) = u (x, y, z)

con el cambio de variable propuesto

∂u

∂x=

∂v

∂ξ

∂ξ

∂x+∂v

∂η

∂η

∂x+∂v

∂ϕ

∂ϕ

∂x∂u

∂y=

∂v

∂ξ

∂ξ

∂y+∂v

∂η

∂η

∂y+∂v

∂ϕ

∂ϕ

∂y∂u

∂z=

∂v

∂ξ

∂ξ

∂z+∂v

∂η

∂η

∂z+∂v

∂ϕ

∂ϕ

∂z

se sigue

∂u

∂x=

∂v

∂ξ

(1

z

)+∂v

∂η(0) +

∂v

∂ϕ(0)

∂u

∂y=

∂v

∂ξ(0) +

∂v

∂η

(1

z

)+∂v

∂ϕ(0)

∂u

∂z=

∂v

∂ξ

(− x

z2

)+∂v

∂η

(− y

z2

)+∂v

∂ϕ(1)

se sigue que

x∂u

∂x+ y

∂u

∂y+ z

∂u

∂z= nu

se transforma en

∂v

∂ξ

(xz

)+∂v

∂η

(yz

)+∂v

∂ξ

(−xz

)+∂v

∂η

(−yz

)+ z

∂v

∂ϕ= nv

es decir

ϕ∂v

∂ϕ= nv

esta ecuacion es de variables separadas, la resolvemos

ln |v| = n ln |ϕ|+ C

luego

v = Kϕn

note que la constante es respecto a la variable ϕ, luego puede depender de ξ, η, ponemos

v = K (ξ, η)ϕn

luego

u (x, y, z) = v (ξ (x, y, z) , η (x, y, z) , ϕ (x, y, z))

= K (ξ (x, y, z) , η (x, y, z))ϕ (x, y, z)n

= K(xz,y

z

)zn

397

Page 400: Apuntes MAT-023 USM

Apuntes Mat023 (Segundo semestre 2014)

Observacion 9.6.3. Muchas veces abusando de la notacion, en lugar de poner

v (ξ (x, y, z) , η (x, y, z) , ϕ (x, y, z)) = u (x, y, z)

simplemente se trabaja con u (ξ, η, ϕ) y se escribe

∂u

∂x=∂u

∂ξ

∂ξ

∂x+∂u

∂η

∂η

∂x+∂u

∂ϕ

∂ϕ

∂x

Observacion 9.6.4. Otra manera de enfrentar el problema anterior, cuando es posible

despejar en forma explıcita es la siguiente, note que

ξ =x

z, η =

y

zy ϕ = z

implica

x = ξϕ

y = ηϕ

z = ϕ

luego

P (ξ, η, ϕ) = u (x (ξ, η, ϕ) , y (ξ, η, ϕ) , z (ξ, η, ϕ))

luego

∂P

∂ϕ=

∂u

∂x

∂x

∂ϕ+∂u

∂y

∂y

∂ϕ+∂u

∂z

∂z

∂ϕ

=∂u

∂x(ξ) +

∂u

∂y(η) +

∂u

∂z

luego

ϕ∂P

∂ϕ=

∂u

∂x(ξϕ) +

∂u

∂y(ηϕ) +

∂u

∂zϕ

= x∂u

∂x+ y

∂u

∂y+ z

∂u

∂z= nu

= nP

de donde

P (ξ, η, ϕ) = ϕnK (ξ, η)

y ası

u (x (ξ, η, ϕ) , y (ξ, η, ϕ) , z (ξ, η, ϕ)) = ϕnK (ξ, η)

esto es

u (x, y, z) = znK

(x

y,z

y

)

398

Page 401: Apuntes MAT-023 USM

Apuntes Mat023 (Segundo semestre 2014)

Estudiaremos el comportamiento de la regla de la cadena en derivadas de orden superior

Ejemplo 9.6.8. Suponiendo z ∈ C2 (R2), reescriba la ecuacion diferencial:

∂2z

∂x2+ 2

∂2

∂x∂y− 3

∂2z

∂y2= 0

en nuevas variables u y v, definidas por: u = 3x− y y v = x+ y.

Solucion. Definamos

P (u (x, y) , v (x, y)) = z (x, y)

entonces

zx = Puux + Pvvx

esto es

zx = 3Pu + Pv

derivando nuevamente respecto a x

zxx = 3 (Pu)x + (Pv)x

= 3 ((Pu)u ux + (Pu)v vx) + ((Pv)u ux + (Pv)v vx)

= 3 ((Pu)u 3 + (Pu)v) + ((Pv)u 3 + (Pv)v)

= 9Puu + 6Puv + Pvv

y

zxy = (3Pu + Pv)y

= 3 (Pu)y + (Pv)y

= 3 (Puuuy + Puvvy) + (Pvuuy + Pvvvy)

= 3 (Puu (−1) + Puv) + (Pvu (−1) + Pvv)

= −3Puu + 2Puv + Pvv

con respecto a la variable y

zy = Puuy + Pvvy

= −Pu + Pv

derivando nuevamente

zyy = − (Puuuy + Puvvy) + (Pvuuy + Pvvvy)

= − (−Puu + Puv) + (−Pvu + Pvv)

= Puu − 2Puv + Pvv

399

Page 402: Apuntes MAT-023 USM

Apuntes Mat023 (Segundo semestre 2014)

se sigue que∂2z

∂x2+ 2

∂2

∂x∂y− 3

∂2z

∂y2= 0

queda reescrita como

(9Puu + 6Puv + Pvv) + 2 (−3Puu + 2Puv + Pvv)− 3 (Puu − 2Puv + Pvv) = 0

es decir

(9− 6− 3)Puu + (6 + 4 + 6)Puv + (1 + 2− 3)Pvv = 0

ası

Puv = 0

esta ecuacion implica

P (u, v) = F (u) +G (v)

donde F , G son funciones de clase C2 de una variable se sigue

z (x, y) = P (u (x, y) , v (x, y))

= F (u (x, y)) +G (v (x, y))

= F (3x− y) +G (x+ y)

como ejercicio, verificar que si z (x, y) = F (3x− y)+G (x+ y) entonces ∂2z∂x2

+2 ∂2

∂x∂y−3∂

2z∂y2

=

0.

Ejemplo 9.6.9. Las ecuaciones u = f (x, y) , x = X (s, t) e y = Y (s, t) definen u como

una funcion de s y t: u = F (s, t). Si las funciones son de clase C2 pruebe que:

∂2F

∂s2=∂f

∂x

∂2X

∂s2+∂2f

∂x2

(∂X

∂s

)2

+ 2∂X

∂x

∂Y

∂s

∂2f

∂x∂y+∂f

∂y

∂2Y

∂s2+∂2f

∂y2

(∂Y

∂s

)2

Solucion. Por la regla de la cadena

Fs = fxxs + fyys

Fss = (fx)s xs + fxxss + (fy)s ys + fyys

= (fxxxs + fxyys)xs + fxxss

+ (fyxxs + fyyys) ys + fyys

= fxx (xs)2 + 2fxyysxs + fyy (ys)

2 + fxxss + fyys

se sigue

∂2F

∂s2=∂2f

∂x2

(∂x

∂s

)2

+ 2∂x

∂s

∂y

∂s

∂2f

∂x∂y+∂2f

∂y2

(∂y

∂s

)2

+∂f

∂x

∂2x

∂s2+∂f

∂y

∂2y

∂s2

400

Page 403: Apuntes MAT-023 USM

Apuntes Mat023 (Segundo semestre 2014)

Ejemplo 9.6.10. Sea g : R→ R una funcion de clase C1 (R) muestre que f (x, y) = g(yx

)para x 6= 0 cumple

x∂f

∂x+ y

∂f

∂y= 0

Solucion. La funcion es diferenciable y por la regla de la cadena se tiene

fx =

(−yx2

)g′(yx

)fy =

(1

x

)g′(yx

)luego

xfx + yfy =

(−yx

)g′(yx

)+(yx

)g′(yx

)= 0

Ejemplo 9.6.11. Sea p : R2 → R, una funcion diferenciable tal que

x∂p

∂x+ y

∂p

∂y= 0

defina h (u, v) = p (v, uv) muestre que∂h

∂v= 0.

Solucion. La compuesta de funciones diferenciables es diferenciables luego

hv = pxxv + pyyv

= px + pyu

ası

vhv = vpx + pyuv

= xpx + ypy

= 0

se sigue

hv = 0

Ejemplo 9.6.12. Sea u = u(x, t) una funcion de clase C2. Transformar la ecuacion de las

vibraciones de una cuerda:∂2u

∂t2= c2 ∂

2u

∂x2, c > 0

a unas nuevas variables independientes α y β, en donde:

α = x− ct ∧ β = x+ ct

401

Page 404: Apuntes MAT-023 USM

Apuntes Mat023 (Segundo semestre 2014)

Solucion. Sea u = u(x, t) ∈ C2. Considere, ademas, las nuevas variables independientes α

y β dadas por las ecuaciones α = x− ct y β = x+ ct. Entonces, por la regla de la cadena,

tenemos que:

∂u

∂t=∂u

∂α

∂α

∂t+∂u

∂β

∂β

∂t

= c(∂u∂β− ∂u

∂α

)y tambien:

∂u

∂x=∂u

∂α+∂u

∂β

Ası, por la regla de la cadena nuevamente, tenemos que:

∂2u

∂t2=

∂t

c(∂u∂β− ∂u

∂α

)= c

∂t

(∂u∂β

)− ∂

∂t

(∂u∂α

)= c

∂2u

∂α ∂β

∂α

∂t+∂2u

∂β2

∂β

∂t

− c

∂2u

∂α2

∂α

∂t+

∂2u

∂β ∂α

∂β

∂t

= c

∂2u

∂α ∂β· (−c) +

∂2u

∂β2· c− c∂2u

∂α2· (−c) +

∂2u

∂β ∂α· c

= c2

∂2

∂β2− 2

∂2u

∂αβ+∂2u

∂α2

Analogamente:

∂2u

∂x2=

∂x

∂u

∂α+∂u

∂β

=

∂x

(∂u∂α

)+

∂x

(∂u∂β

)=∂2u

∂α2

∂α

∂x+

∂2u

∂β ∂α

∂β

∂x+

∂2u

∂α ∂β

∂α

∂x+∂2u

∂β2

∂β

∂x

=∂2u

∂α2+ 2

∂2u

∂α ∂β+∂2u

∂β2

en donde la igualdad ∂2u∂β ∂α

= ∂2u∂α∂β

se obtiene a traves del Teorema de Schwarz, pues u ∈ C2.

Reemplazando las derivadas anteriores en la ecuacion:

∂2u

∂t2= c2 ∂

2u

∂x2

tenemos:

c2

∂2u

∂β2− 2

∂2u

∂α∂β+∂2u

∂α2

= c2

∂2u

∂β2− 2

∂2u

∂α∂β+∂2u

∂α2

402

Page 405: Apuntes MAT-023 USM

Apuntes Mat023 (Segundo semestre 2014)

Simplificando la ecuacion anterior se obtiene, finalmente, que:

∂2u

∂α ∂β= 0

Ejemplo 9.6.13. Sea g : R → R una funcion derivable tal que g(0) = −1 y g′(0) = 2.

Considere la funcion f : R2 → R definida por:

f(x, y) = xy · g(

1x− 1

y

)1. Determine la ecuacion del plano tangente a la superficie z = f(x, y) en el punto(

1, 1, f(1, 1)).

2. Obtenga una funcion h(x, y) tal que:

x2 ∂z

∂x+ y2∂z

∂y= z h(x, y)

Solucion. Definimos a u = u(x, y) = 1x− 1

y. Puesto que u es diferenciable en (1, 1) y g(·)

es derivable, entonces z es diferenciable en (1, 1). Se tiene entonces que:

z(1, 1) = 1 · 1 · g(u(1, 1)) = g(0) = −1

Ademas, por la regla de la cadena:

zx = y · g(u) + xy · g′(u)ux

= yg(u)− y

xg′(u)

⇒ zx(1, 1) = −3

Y, de forma analoga:

zy = x · g(u) + xy · g′(u)uy

= xg(u) +x

yg′(u)

⇒ zy(1, 1) = 1

Con esto, la ecuacion del plano tangente es:

−3(x− 1) + (y − 1)− (z + 1) = 0

−3x+ y − z = −1

Hacemos:

x2zx = x2yg(u)− xyg′(u)

y2zy = xy2g(u) + xyg′(u)

403

Page 406: Apuntes MAT-023 USM

Apuntes Mat023 (Segundo semestre 2014)

Basta sumar y obtenemos:

x2zx + y2zy = (x+ y)xyg(u)

Pero z = xy g(u), de modo que:

x2zx + y2zy = (x+ y)z =⇒ f(x, y) = x+ y

Ejemplo 9.6.14. Sea f : R2 → R, (x, y)→ f (x, y) una funcion de clase C2.

1. Muestre que si g (r, θ) = f (r cos θ, r sin θ) entonces

∂2f

∂x2+∂2f

∂y2=∂2g

∂r2+

1

r2

∂2g

∂θ2+

1

r

∂g

∂r

Usando la regla de la cadena

gr = fxxr + fyyr

= fx cos θ + fy sin θ

grr = (fxxxr + fxyyr) cos θ + (fyxxr + fyyyr) sin θ

= (fxx cos θ + fxy sin θ) cos θ + (fyx cos θ + fyy sin θ) sin θ

= fxx cos2 θ + 2 sin θ cos θfxy + fyy sin2 θ

gθ = fxxθ + fyyθ

= −fxr sin θ + fyr cos θ

y ası

gθθ = −r ((fxxxθ + fxyyθ) sin θ + fx cos θ)

+r ((fyxxθ + fyyyθ) cos θ − fy sin θ)

= −r ((fxx (−r) sin θ + fxyr cos θ) sin θ + fx cos θ)

+r ((fyx (−r) sin θ + fyyr cos θ) cos θ − fy sin θ)

= r2fxx sin2 θ − 2r2fxy sin θ cos θ + r2fyy cos2 θ

−rfx cos θ − rfy sin θ

= r2(fxx sin2 θ − 2fxy sin θ cos θ + fyy cos2 θ

)−r (fx cos θ + fy sin θ)

404

Page 407: Apuntes MAT-023 USM

Apuntes Mat023 (Segundo semestre 2014)

entonces

∂2g

∂r2+

1

r2

∂2g

∂θ2+

1

r

∂g

∂r=

(fxx cos2 θ + 2 sin θ cos θfxy + fyy sin2 θ

)+(fxx sin2 θ − 2fxy sin θ cos θ + fyy cos2 θ

)− (fx cos θ + fy sin θ)

r

+1

r(fx cos θ + fy sin θ)

= fxx + fyy

como se querıa demostrar.

2. Determine una funcion f : (x, y) ∈ R2 : 1 ≤ x2 + y2 ≤ 9 → R tal que

∂2f

∂x2+∂2f

∂y2= 0 para 1 < x2 + y2 < 9

f (x, y) = 1 si x2 + y2 = 1

f (x, y) = 3 si x2 + y2 = 9

suponiendo que g (r, θ) = f (r cos θ, r sin θ) no depende del angulo, es decir, ∂g∂θ

= 0.

Pasando a polares

∂2f

∂x2+∂2f

∂y2= 0 para 1 < x2 + y2 < 9

⇔∂2g

∂r2+

1

r2

∂2g

∂θ2+

1

r

∂g

∂r= 0 para 1 < r < 3

f (x, y) = 1 si x2 + y2 = 1

⇔g (r, θ) = 1 si r = 1

yf (x, y) = 3 si x2 + y2 = 9

⇔g (r, θ) = 3 si r = 3

si no depende del angulo tenemos

d2g

dr2+

1

r

dg

dr= 0 para 1 < r < 3

g (1) = 1

g (3) = 3

esto tiene solucion

g (r) = A+B ln r

405

Page 408: Apuntes MAT-023 USM

Apuntes Mat023 (Segundo semestre 2014)

donde

1 = A

3 = A+B ln 3

que tiene solucion A = 1, B = 2ln 3

, ası

f (x, y) = 1 +2

ln 3ln√x2 + y2

Gradiente y planos tangentes

Observacion 9.7.1. Sea f : U ⊆ Rn → R una funcion diferenciable tal que ∇f (a) 6= 0.

Consideremos el conjunto de nivel:

S : Lc (f) = x ∈ U : f (x) = c

Suponga que λ : t 7→ λ (t) una curva diferenciable de R → Rn tal que Im(λ) ⊆ S. Note

que, en vista de las hipotesis para la curva λ, tenemos que:

f (λ (t)) = c, ∀t

entonces, por la regla de la cadena, tenemos que:

∇f (λ (t)) · λ′ (t) =n∑i=1

∂f

∂xi(λ (t))λ′i (t) = 0

en particular, si existe t0 ∈ R tal que λ (t0) = a, entonces, de la ecuacion anterior concluımos

que:

〈∇f (a) , λ′ (t0)〉 = 0

como λ es cualquier curva diferenciable tal que se tiene que:

∇f (a) ⊥ S

puesto que λ′ es tangente a S. Se tiene entonces:

Teorema 9.7.1. Sean U un conjunto abierto no vacıo, f : U ⊆ Rn → R una funcion

diferenciable y S : Lc (f) el conjunto de nivel de c para f . Suponga que que f (a) = c (es

decir, a ∈ S) y que ∇f (a) 6= 0 entonces:

∇f (a) ⊥a S

donde ⊥a significa que la ortogonalidad de ∇f (a) al conjunto de nivel S : Lc (f) se da en

el punto a.

406

Page 409: Apuntes MAT-023 USM

Apuntes Mat023 (Segundo semestre 2014)

Ejemplo 9.7.1. Hallar un vector normal unitario a la superficie definida por:

2xy3z + z lnx+ y sin y = 0

en (1, 2π, 0).

Solucion. El punto dado pertenece a la superficie 2xy3z + z lnx + y sin y = 0 la cual

corresponde al conjunto de nivel cero de la funcion f : D ⊆ R3 → R dada por

f (x, y, z) = 2xy3z + z lnx+ y sin y

Sabemos que el gradiente es normal a la superficie,

∇f (x, y, z) =

(1

xz + 2y3z, sin y + y cos y + 6xy2z, 2xy3 + lnx

)evaluando

∇f (1, 2π, 0) =(0, 2π, 2 (2π)3)

=(0, 2π, 16π3

)es un vector normal a la superficie en el punto dado.

Observacion 9.7.2. Recordemos que si u0 ∈ Rn la ecuacion del plano Π que pasa por u0

y que es normal n es dada por:

n · (x− u0) = 0

Por lo tanto, el plano tangente Π a una superficie S : f (x) = c en a, esta dado por:

Π : ∇f (a) · (x− a) = 0

pues ∇f (a) ⊥a S y a ∈ S

Ejemplo 9.7.2. Encontrar el plano tangente a la superficie:

x2 + y2 + z2 = 1

en (1, 0, 0).

Solucion. La superficie es el conjunto de nivel 1 de la funcion h (x, y, z) = x2 + y2 + z2 y

el punto (1, 0, 0) pertenece a la superficie, se sigue que el vector normal al plano tangente

es dado por

∇h (x, y, z) = (2x, 2y, 2z)

ası el plano tangente es

∇h (1, 0, 0) · (x− 1, y − 0, z − 0) = 0

407

Page 410: Apuntes MAT-023 USM

Apuntes Mat023 (Segundo semestre 2014)

es decir

(2, 0, 0) · (x− 1, y − 0, z − 0) = 0

luego

x = 1

es el plano tangente.

Ejemplo 9.7.3. Sea C la curva interseccion de la esfera x2 + y2 + z2 = R2 (donde R > 0)

y el plano x+ y + z = 0. Demuestre que si (x0, y0, z0) ∈ C entonces la recta tangente a la

curva en (x0, y0, z0) donde x0, y0, z0 son reales distintos tiene ecuacion

x− x0

y0 − z0

=y − y0

z0 − x0

=z − z0

x0 − y0

Solucion. Definamos

E =

(x, y, z) ∈ R3 : x2 + y2 + z2 = R2

=

(x, y, z) ∈ R3 : f (x, y, z) = 0

P =

(x, y, z) ∈ R3 : x+ y + z = 0

=

(x, y, z) ∈ R3 : h (x, y, z) = 0

donde f (x, y, z) = x2 + y2 + z2−R2 y h (x, y, z) = x+ y+ z entonces C = E ∩P . entonces

∇f (x0, y0, z0) ⊥ C en (x0, y0, z0)

∇h (x0, y0, z0) ⊥ C en (x0, y0, z0)

(el gradiente es perpendicular a los conjuntos de nivel) se sigue que el vector director de la

recta tangente a C en (x0, y0, z0) es paralelo a

∇f (x0, y0, z0)×∇h (x0, y0, z0)

pero

∇f (x0, y0, z0) = (2x0, 2y0, 2z0)

∇h (x0, y0, z0) = (1, 1, 1)

ası

∇f (x0, y0, z0)×∇h (x0, y0, z0)

= (2x0, 2y0, 2z0)× (1, 1, 1)

= (2y0 − 2z0, 2z0 − 2x0, 2x0 − 2y0)

ası la recta es

(x, y, z) = (x0, y0, z0) + t (2y0 − 2z0, 2z0 − 2x0, 2x0 − 2y0)

408

Page 411: Apuntes MAT-023 USM

Apuntes Mat023 (Segundo semestre 2014)

esto es

x = x0 + 2t (y0 − z0)

y = y0 + 2t (z0 − x0)

z = z0 + 2t (x0 − y0)

despejando t

x− x0

y0 − z0

= 2t

y − y0

z0 − x0

= 2t

z − z0

x0 − y0

= 2t

de donde obtenemosx− x0

y0 − z0

=y − y0

z0 − x0

=z − z0

x0 − y0

como se queria demostrar.

Observacion 9.7.3. En secciones anteriores encontramos la ecuacion del plano tangente

a la grafica de una funcion diferenciable z = f (x, y) en un punto (x0, y0, f (x0, y0)) el cual

tiene ecuacion

z = f (x0, y0) +∂f

∂x(x0, y0) (x− x0) +

∂f

∂y(x0, y0) (y − y0)

Note que la grafica corresponde al conjunto

S =

(x, y, z) ∈ R3 : z − f (x, y) = 0

el cual puede ser interpretado como el conjunto de nivel cero de F (x, y, z) = z − f (x, y),

luego en el punto

(x0, y0, f (x0, y0)) = (x0, y0, z0)

el plano tangente a S tiene ecuacion

∇F (x0, y0, z0) · (x− x0, y − y0, z − z0) = 0

pero

∂F

∂x(x0, y0, z0) = −∂f

∂x(x0, y0)

∂F

∂y(x0, y0, z0) = −∂f

∂y(x0, y0)

∂F

∂z(x0, y0, z0) = 1

409

Page 412: Apuntes MAT-023 USM

Apuntes Mat023 (Segundo semestre 2014)

ası el plano es(−∂f∂x

(x0, y0) ,−∂f∂y

(x0, y0) , 1

)· (x− x0, y − y0, z − z0) = 0

es decir

z = f (x0, y0) +∂f

∂x(x0, y0) (x− x0) +

∂f

∂y(x0, y0) (y − y0)

La propiedad del gradiente entonces nos permite obtener el resultado obtenido por otros

metodos.

Derivada direccional y direcciones de crecimiento maximo

Definicion 9.8.1. Sean a ∈U , f : U ⊆ Rn → R una funcion cualquiera y u ∈ Rn un

vector unitario en Rn (es decir, un vector tal que ‖u‖ = 1). Se define la derivada direccional

de f en a en la direccion de u como el lımite:

Du (a) =∂f

∂u(a) = lım

t→0

f (a + tu)− f (a)

t

si acaso existe.

Ejemplo 9.8.1. Calcular la derivada direccional de f : R2 → R, (x, y) → f (x, y) =

x3y + 2x2y3 en el punto a = (1, 2) en la direccion u =(

1√2, 1√

2

)Solucion. Por definicion

∂f

∂u(a) = lım

t→0

f (a + tu)− f (a)

t

= lımt→0

f(

(1, 2) + t(

1√2, 1√

2

))− f (1, 2)

t

= lımt→0

f(

1 + t√2, 2 + t√

2

)− f (1, 2)

t

= lımt→0

(1 + t√

2

)3 (2 + t√

2

)+ 2

(1 + t√

2

)2 (2 + t√

2

)3

− (2 + 24)

t

=63

2

√2 (Usando la regla de L’ Hopital)

Observacion 9.8.1. Note que los vectores canonicos ei, con i = 1, 2, . . . , n, de Rn son

vectores unitarios, luego las derivadas parciales son casos particulares de la derivada

direccional. Mas precisamente, las derivadas parciales son derivadas direccionales en la

direccion de los vectores de la base canonica de Rn.

410

Page 413: Apuntes MAT-023 USM

Apuntes Mat023 (Segundo semestre 2014)

Observacion 9.8.2. Se debe hacer notar, que ∂f∂u

(a) mide la razon de cambio de f en la

direccion de la recta vectorial:

x = a + tu, t ∈ R

ası como las derivadas direccionales miden la razon de cambio de f es la direccion de los

ejes coordenados. La interpretacion de estas derivadas es la misma.

Ejemplo 9.8.2. Sea f : R2 → R la funcion definida por:

f (x, y) =

|x| y√x2 + y2

, si x2 + y2 6= 0

0 , si x2 + y2 = 0

Verifique que f posee derivada direccional en (0, 0) en cualquier direccion y que no es

diferenciable en dicho punto.

Solucion. Sea u = (u, v) un vector unitario

∂f

∂u(0, 0) = lım

t→0

f (tu, tv)− f (0, 0)

t

= lımt→0

|tu| tv

t√

(tu)2 + (tv)2

= lımt→0

|t| |u| tvt |t|√u2 + v2

= lımt→0|u| v

= |u| v

luego las derivadas direccionales existen en toda direccion. Note que ∂f∂x

(0, 0) = ∂f∂y

(0, 0) = 0

para ello tomar las direcciones (u, v) = (1, 0) y (0, 1) respectivamente.

Para la diferenciabilidad

lım(x,y)→(0,0)

∣∣∣f (x, y)− f (0, 0)− ∂f∂x

(0, 0) (x− 0)− ∂f∂y

(0, 0) (y − 0)∣∣∣

‖(x, y)− (0, 0)‖

= lım(x,y)→(0,0)

|xy|√x2 + y2√x2 + y2

= lım(x,y)→(0,0)

|xy|x2 + y2

note que este lımite no existe, usando las trayectorias y = x e y = 0 obtenemos lımites

distintos. La funcion no es diferenciable.

411

Page 414: Apuntes MAT-023 USM

Apuntes Mat023 (Segundo semestre 2014)

Observacion 9.8.3. Del ejemplo anterior podemos afirmar que la existencia de todas las

derivadas direccionales en un punto no implica diferenciabilidad en el punto.

Teorema 9.8.1. Sea f : U ⊆ Rn → R una funcion diferenciable en a ∈U . Si u =

(u1, u2, . . . , un) es un vector unitario en Rn, entonces:

∂f

∂u(a) = 〈∇f (a) , u〉

=n∑i=1

∂f

∂xi(a)ui

Demostracion. Definamos g (t) = f (a + tu), esta funcion nes diferenciable en 0 por se

compuesta de diferenciables, ademas por la regla de la cadena

g′ (t) = ∇f (a + tu) · u

evaluando en t = 0 se obtiene

g′ (0) = ∇f (a) · u

por otro lado

g′ (0) = lımt→0

g (t)− g (0)

t− 0

= lımt→0

f (a + tu)− f (a)

t

=∂f

∂u(a)

ası∂f

∂u(a) = ∇f (a) · u

Ejemplo 9.8.3. Verifique que la derivada direccional de z = y2

xen cualquier punto de la

elipse:

2x2 + y2 = c2

en la direccion de la normal a la curva es nula.

Solucion. Note que la curva es el conjunto de nivel c2 de la funcion g (x, y) = 2x2 + y2

luego en el punto (x0, y0) de la elipse, la direccion normal es

n =∇g (x0, y0)

‖∇g (x0, y0)‖=

1√16x2

0 + 4y20

(4x0, 2y0)

=1√

4x20 + y2

0

(2x0, y0)

412

Page 415: Apuntes MAT-023 USM

Apuntes Mat023 (Segundo semestre 2014)

luego la derivada direccional es

∂z

∂n(x0, y0)

= ∇z (x0, y0) · ∇g (x0, y0)

‖∇g (x0, y0)‖

=1√

4x20 + y2

0

(−y

20

x20

,2y0

x0

)· (2x0, y0)

=1√

4x20 + y2

0

(−2y2

0

x0

+2y2

0

x0

)= 0

como se querıa demostrar.

Observacion 9.8.4. Suponiendo las condiciones del teorema anterior, utilizando las

propiedades euclidianas de Rn, obtenemos:

∂f

∂u(a) = 〈∇f (a) , u〉

= ‖∇f (a)‖ ‖u‖ cos θ

= ‖∇f (a)‖ cos θ

en donde θ = ∠ (∇f (a) ; u) ∈ [0, π]. Note que:

−‖∇f (a)‖ ≤ ∂f

∂u(a) ≤ ‖∇f (a)‖

Ası, si en particular ∠ (∇f (a) ; u) = 0 entonces

∂f

∂u(a) = ‖∇f (a)‖

Es decir, la maxima razon de cambio de f se obtiene en la direccion del gradiente ∇f (a) y

de manera similar, si ∠ (∇f (a) ; u) = π entonces

∂f

∂u(a) = −‖∇f (a)‖

maximo decrecimiento.

Teorema 9.8.2. Sean U ⊆ Rn abierto y f una funcion diferenciable en a ∈ U , entonces

la direccion de maximo crecimiento de f es ∇f(a)‖∇f(a)‖ . Ademas, obtenemos que:

‖∇f (a)‖ = max

∂f

∂u(a) : ‖u‖ = 1

De manera similar, la menor derivada direccional de f se obtiene en la direccion de

− ∇f(a)‖∇f(a)‖ y

−‖∇f (a)‖ = mın

∂f

∂u(a) : ‖u‖ = 1

413

Page 416: Apuntes MAT-023 USM

Apuntes Mat023 (Segundo semestre 2014)

Ejemplo 9.8.4. Un objeto que busca el calor esta localizado en el punto (2, 3) en un

plato de metal plano cuya temperatura en el punto (x, y) es T (x, y) = 30 − 8x2 − 2y2.

Determine el camino que sigue el objeto si este se mueve en forma continua en la direccion

de maximo incremento de la temperatura en cada punto. Ind. Describir el camino como

curva parametrica x = x (t), y = y (t).

Solucion. El objeto se mueve en la direccion del gradiente luego(dx

dt,dy

dt

)= k∇T = k (x,−4y)

se sigue

dx

dt= −16kx

dy

dt= −4ky

con (x (0) , y (0)) = (2, 3) de donde

dy

dx=

dydtdxdt

=−4ky

−16kx=

y

4x

y (2) = 3

resolvemos el P.V.I.

dy

dx=

y

4xy (2) = 3

la ecuacion tiene solucion y = C 4√x de donde obtenemos

3 = C4√

2⇒ C =34√

2

y ası

y = 3

(4

√x

2

)

Ejemplo 9.8.5. Sea f(x, y, z) = 3x2y + 2xy2 + z3. Calcule la derivada direccional de f en

(−1, 0, 1) en la direccion de un vector normal a la superficie:

sin(xy2) + xy2z2 = 2x− yz − 1

en el punto (0, 1,−1).

414

Page 417: Apuntes MAT-023 USM

Apuntes Mat023 (Segundo semestre 2014)

Solucion. Sea g(x, y, z) = sin(xy2) + xy2z2 − 2x + yz + 1 y considere la superficie S :

g(x, y, z) = 0. En primer lugar, note que P (0, 1,−1) ∈ S, pues g(0, 1,−1) = 0. Ademas,

sabemos que:

∇g(0, 1,−1) ⊥P S

en donde ⊥P denota perpendicularidad en P (0, 1,−1). Ası:

gx = y2 cos(xy2) + y2z2 − 2

gy = 2xy cos(xy2) + 2xyz2 + z

gz = 2xzy2 + y

y por tanto:

∇g(0, 1,−1) =(gx(0, 1,−1), gy(0, 1,−1), gz(0, 1,−1)

)= (0,−1, 1)

Por otro lado, como f es diferenciable en (−1, 0, 1), puesto que sus derivadas parciales:

fx = 6xy + 2y2

fy = 3x2 + 4xy

fz = 3z2

son funciones continuas en (−1, 0, 1), tenemos que:

∂f

∂u(−1, 0, 1) =

⟨∇f(−1, 0, 1), u

⟩en donde ∇f(−1, 0, 1) = (0, 3, 3). Ası, finalmente, un vector unitario en la direccion del

vector normal ∇g(0, 1,−1) se obtiene haciendo u = ∇g(0, 1,−1)/‖∇g(0, 1,−1)‖ y entonces:

∂f

∂u(−1, 0, 1) =

⟨∇f(−1, 0, 1), u

⟩=⟨

(0, 3, 3),(0, −√

2/2,√

2/2)⟩

= 0

Ejercicios del capıtulo

1. Calcule las derivadas parciales de las siguientes funciones:

a) f (x, y) = ln(

sin x+a√y

)415

Page 418: Apuntes MAT-023 USM

Apuntes Mat023 (Segundo semestre 2014)

b) f (x, y, z) = xyz

c) f (x, y) = 1

arctan( yx)

d) f (x, y, z) = (x+ z)x+y

e) f (x, y) = arcsin x√x2+y2

2. Suponga que g : R→ R es una funcion continua. Calcule las derivadas parciales de f

si:

a) f (x, y) =∫ x+y

ag (t) dt

b) f (x, y, z) =

∫ sin(x sin(y+z2))

xy

z g (t) dt

3. Calcular las derivadas parciales de las siguientes funciones:

a) f(x, y) = 13x3 − 3x2y + 3xy2 + y3

b) f(x, y) = ln (x+ 2√x2 + 3y2)

c) f(x, y) =x2 − 2xy

x+ y

d) f(x, y) = x4 − x3y + x2y2 − xy3 + y4

e) f(x, y) =x+ y

x− yf ) f(x, y) = ln(x2 + y2)

g) f(x, y) = xy + yx

h) f(x, y) = x cos

(x

y

)i) f(x, y) = arctan

(2

x+ y

)j ) f(x, y, z) = xy+x − yz−y + zx+y

k) f(x, y, z) = logx(x+ y + z)

4. Pruebe que si f(x, y) =2x

xyentonces fx(3, 1) + fy(3, 1) = 1

5. Pruebe que si f(x, y) = Ax4 + 28x2y2 + Cy4, entonces x∂f

∂x+ y

∂f

∂y= 4f(x, y)

6. Sea f(x, y) = ex+y. Pruebe que:∂m+nf(x, y)

∂xm∂yn= ex+y.

416

Page 419: Apuntes MAT-023 USM

Apuntes Mat023 (Segundo semestre 2014)

7. Sea z = f(x, y). Verifique que∂2z

∂x∂y=

∂2z

∂y∂x

a) z = x2 − 4xy + 3y2

b) z = ln(x+ y)

c) z = (x3 + y3)10

d) z = (x+ y) sec(xy)

e) z = x2 cos(y−2)

f ) z =4

πarctan(xy)

8. Verifique que f(x, y) = arctan(yx

)satisface la ecuacion

∂2f

∂x2+∂2f

∂y2= 0

9. Hallar∂2z

∂x∂y(x, y),

si z =√

2xy + y2, siendo 2xy + y2 > 0.

10. Muestre que u(x, t) = e−n2kt sin(nx) satisface la ecuacion

∂u

∂t= k

∂2u

∂x2

11. Sea

f(x) =

x2y

x4 + y2si (x, y =6= (0, 0)

0 si (x, y) = (0, 0)

Pruebe que existen todas las derivadas direccionales de f en (0,0) pero f no es

continua en el origen.

12. Sea f(x, y) =

xy3

x2 + y2si (x, y) 6= (0, 0)

0 si (x, y) = (0, 0)

a) ¿f es continua en (0,0)?

b) ¿Existen∂f

∂x(0, 0),

∂f

∂y(0, 0)?

13. Sea f(x, y) =

x4(y + 2)2

x2 + y2si (x, y) 6= (0,−2)

0 si (x, y) = (0,−2)

a) Analice la continuidad de f en (0,-2).

417

Page 420: Apuntes MAT-023 USM

Apuntes Mat023 (Segundo semestre 2014)

b) Calcule, si existe,∂f

∂x(0,−2)

14. Sea

f(x, y) =

(x2 + (y − 1)2)k sin

(1√

x2 + (y − 1)2

)si (x, y) 6= (0, 1)

0 si (x, y) = (0, 1)

Determine todos los valores de k ∈ R de modo que∂f

∂x(0, 1) y

∂f

∂y(0, 1) existan.

15. Sea

f(x, y) =

x2 arctan

(yx

)− y2 arctan

(x

y

)si xy 6= 0

0 si xy = 0

Calcule, si existen,∂f

∂x(0, 1) y

∂f

∂y(1, 0)

16. Verifique que la funcion f : R→ R2 definida por f (x) = (x2, x3) es diferenciable en

R. Calcule Df (x).

17. Sea f : R3 → R2 la funcion definida por:

f (x, y, z) =(xy2, yz2

)Demuestre, por definicion, que f es diferenciable en R3. Calcule Df (x, y, z)

18. Considere f : Rn → Rm. Suponga que existe M > 0 tal que:

‖f (x) ‖ < M ‖x‖2

para todo x ∈ Rn. Verifique que f es derivable en 0 y que ademas Df (0) = 0.

19. Sea f : R2 → R la funcion f (x, y) =√|xy|. ¿Es f diferenciable en (0, 0)?

20. Sea:

f (x, y) =

x2y2

x2 + y4, si (x, y) 6= (0, 0)

0 , si (x, y) = (0, 0)

a) Calcular ∂f∂x

en cada punto donde exista.

b) Decidir ∂f∂x

es continua en el punto (0, 0) y si acaso f es diferenciable en (0, 0).

418

Page 421: Apuntes MAT-023 USM

Apuntes Mat023 (Segundo semestre 2014)

21. Demuestre que las siguientes funciones son derivables y calcule su derivada:

a) f (x, y) =(ey, sinxy, x2 + 2y3

)b) g (x, y, z, w) =

(xyzw2, esinw, x2

)22. Calcule ∇f (x, y), en terminos de derivadas de g (x) y h (x) , para las funciones f (x, y)

siguientes:

a) f (x, y) = g (x)

b) f (x, y) = g (y)

c) f (x, y) = g (x+ y)2 + g (x)h(y) − 3g (x2 − 3y2)

23. Verifique lo siguiente:

a) Si z = xy + xey/x, entonces x ∂z∂x

+ y ∂z∂y

= xy + z.

b) Si u = (x− y) (y − z) (z − x), entonces ∂u∂x

+ ∂u∂y

= −∂u∂z

.

a) Se supone que ∂z∂y

= xx2+y2

. Calcule z = z (x, y).

b) Se supone que ∂u∂x

= x2+y2

xy que u (1, y) = sin y. Calcule u = u (x, y).

24. Sea z = logy x. Calcule ∂z∂y

.

25. Sea z = f(x+yx−y

), con f una funcion clase C1 (R). Verifique que:

x∂z

∂x+ y

∂z

∂y= 0

26. Sean f : R→ R y u (x, y) = xy f(x+yxy

). Hallar una funcion escalar g (x, y) tal que

se verifique la relacion:

x2∂u

∂x− y2∂u

∂y= g (x, y) u (x, y)

27. Calcule:d

dt(g f) (t)

∣∣∣t=2

si f (t) = (t, t2 − 4, et−2) y g : R3 → R es tal que:

∂g

∂x(2, 0, 1) = 4 ∧ ∂g

∂y(2, 0, 1) = 2 ∧ ∂g

∂z(2, 0, 1) = 2

419

Page 422: Apuntes MAT-023 USM

Apuntes Mat023 (Segundo semestre 2014)

28. Calcule Dh (x, y, z), si:

h (x, y, z) = f (u (x, y, z) , v (x, y) , w (y, z))

29. Se define ∂2u∂x∂y

= ∂∂x

(∂u∂y

). Considere z = u (x, y) eax+by tal que ∂2u

∂x∂y= 0. Hallar los

valores de las constantes a, b ∈ R de modo que:

∂2z

∂x∂y− ∂z

∂x− ∂z

∂y+ z = 0

30. Sea f : Rp → R una funcion. Se dice que f es homogenea de grado m si, para cada

x ∈ Rp y cualquier t ∈ R se tiene que:

f (t x) = tm f (x)

Demuestre que si f es homogenea de grado m y diferenciable, entonces:

〈Df (x) , x〉 = mf (x)

31. Calcule ∂z∂x

, si:

z = f (u)

con u = xy + yx.

32. Hallar u′ (x), si:

u = f (x, y, z)

donde y = ϕ (x) y ψ (x, y).

33. Sea f (x, y, z) = x sin(xy

)− z definida sobre:

A =

(x, y, z) ∈ R3 : y 6= 0

Considere la superficie:

S =

(x, y, z) ∈ R3 : f (x, y, z) = 0

Verifique que el plano tangente a S en cualquier punto de S pasa por el origen.

34. Encontrar la ecuacion del plano tangente a la superficie S dada por la grafica de la

funcion:

a) z = x2 + y2 en el punto (0, 0, 0).

b) z = (x+ y)2 − 2x en el punto (1, 1, 0).

420

Page 423: Apuntes MAT-023 USM

Apuntes Mat023 (Segundo semestre 2014)

c) f (x, y) =√x2 + y2 + (x2 + y2) en (1, 0, 2).

d) f (x, y, z) =√x2 + 2xy − y2 + 1 en el punto

(1, 1,−2,

√3).

35. Considere la superficie:

S : z2 = −3x2y + cos(

2xy +π

4

)a) ¿Se puede calcular el plano tangente a S en el punto

(0, 0,√

2)? Explique.

b) ¿Se puede calcular el plano tangente a S en el punto(

1,−π4,√

3π2

)? Explique.

36. Sea f (x, y) = x2 − xy + 2y2. Hallar la derivada direccional de f en el punto (1, 2) y

en direccion que forma con el eje x positivo un angulo de 60.

37. Sea f (x, y) = x3 − 2x2y + xy2 + 1. Hallar la derivada direccional de f en el punto

(1, 2) en la direccion que va desde este punto al punto (4, 6).

38. Determinar los valores de las constantes a, b, c ∈ R de modo que la derivada direccional

de:

f (x, y, z) = a xy2 + b yz + c z2x3

en el punto (1, 2,−1) tenga un valor maximo de 64 en una direccion paralela al eje−→X .

39. Sea f : R2 → R la funcion definida por:

f (x, y) =

x2yx3−y2 , si x3 − y2 6= 0

0 , si x3 − y2 = 0

Verifique que f tiene en (0, 0) derivada en cualquier direccion, pero que no es diferen-

ciable en dicho punto.

40. Calcule la derivada direccional de la funcion f en el punto P y en direccion del vector

v.

a) f(x, y) = 2x2 + 3xy + 4y2, P (2, 1), v = (1, 1).

b) f(x, y) = arctan

(x

y

), (−3, 3), v = (3, 4).

c) f(x, y, z) = x2 + 3xy + 4y2, (1, 1, 2), v = (2, 3, 0).

d) f(x, y, z) =

(x

y

)z, P (1, 1, 1), v = (2, 1,−1)

41. Calcule la derivada de f(x, y) = x4 +x3y2 +y en (1, 1) y en la direccion de la tangente

a la curva y = x4.

421

Page 424: Apuntes MAT-023 USM

Apuntes Mat023 (Segundo semestre 2014)

42. Calcule la derivada direccional de f(x, y) = xy + x2 + x4y, en (1, 1) y en la direccion

del vector que forma una angulo de 60 con el eje x.

43. Encuentre la maxima derivada direccional de f en P .

a) f(x, y) = 2x2 + 3xy + 4y2, en P (1, 1).

b) f(x, y, z) = e−(x+y+z), en P (5, 2, 3).

44. Dada la funcion f(x, y) se sabe que:

f ′(x0; v) = 2 si x0 = (1, 2) y v = (2, 2)

f ′(x0; v) = −2 si x0 = (1, 2) y v = (1,−1)

Calcule ∇f(1, 2) y f ′(x0; v) cuando x0 = (1, 2) y v = (4, 6)

45. Suponga que la temperatura en el punto (x, y, z) esta dada por T (x, y, z) = 3x2 +

2y2− 4z. Determine el valor de la razon de cambio de T en P (−1,−3, 2) en direccion

a Q(−4, 1,−2). ¿Cual es la taza de maxima variacion?.

46. Hallar los puntos (x, y) y las direcciones para las que la derivada direccional de

f(x, y) = 3x2 + y2 tiene el valor maximo si (x, y) esta en el cırculo x2 + y2 = 1.

47. Calcule la derivada direccional de f(x, y, z) = x2 + y2− z2 en (3, 4, 5) a lo largo de la

curva interseccion de las superficies 2x2 + 2y2 − z2 = 25 y x2 + y2 = z2.

48. Calcular la ecuacion del plano tangente a la superficie dada por:

a) f(x, y) = 3x2 + 8xy en el punto (x0, y0) = (1, 0).

b) f(x, y) =√x2 + y2 en el punto (x0, y0) = (1, 2)

c) f(x, y) =

∫ x2+y2

0

e−√tdt en el punto (x0, y0) = (1, 1)

d) f(x, y, z) = sen(xyz)− 1

2en el punto

(3,π

6,1

3

)49. Hallar una constante c tal que en todo punto de la interseccion de las dos esferas:

(x− c)2 + y2 + z2 = 3 x2 + (y − 1)2 + z2 = 1

los planos tangentes correspondientes sean perpendiculares el uno al otro.

50. Demuestre que la superficie x2 − 2yz + y3 = 4 es ortogonal a cualquiera de las

superficies de la familia x2 + 1 = (2 − 4a)y2 + az2 en el punto de interseccion

(1,−1, 2).

422

Page 425: Apuntes MAT-023 USM

Apuntes Mat023 (Segundo semestre 2014)

51. Determinar en que punto de la superficie z = 3xy − x3 − y3 el plano tangente es

horizontal (paralelo al plano z = 0)

52. Pruebe que si u = x2y + y2z + z2x, entonces∂u

∂x+∂u

∂y+∂u

∂z= (x+ y + z)2

53. Si w = x2 + y2 + z2 encuentre el diferencial total de w

54. Sea f(x, y) = xyx+y

Demuestre que : x2∂2f

∂x2+ 2xy

∂2f

∂x∂y+ y2∂

2f

∂y2= 0

55. Probar que el volumen formado por el plano tangente a la superficie xyz = m3, en

cualquier punto, y los planos coordenados es constante.

56. Hallar∂2z

∂x∂y(x, y),

si z =√

2xy + y2, siendo 2xy + y2 > 0.

57. Calcular la ecuacion del plano tangente a la superficie dada por

z = 3x2 + 8xy

en el punto (x, y) = (1, 0).

58. Calcular las derivadas de segundo orden:

a) f(x, y) = x3 + 3x2y + 6xy2 − y3

b) 2x4 − 3x2y2 + y4 = z

59. Sea f(x, y) = ϕ(x2 + y2), considere ϕ diferenciable, pruebe que:

y∂f

∂x− x∂f

∂y= 0

60. Demuestre que la funcion u(x, t) satisface la ecuacion∂2u

∂t2= a2∂

2u

∂x2para:

a) u(x, t) = ϕ(x− at) + ψ(x+ at) donde ϕ y ψ son funciones de clase C2.

b) u(x, t) = sen(akt) sen(kx) con k ∈ Z.

61. Se dice que una funcion es armonica si verifica que

∇2f =∂2f

∂x2+∂2f

∂y2= 0

423

Page 426: Apuntes MAT-023 USM

Apuntes Mat023 (Segundo semestre 2014)

Sea f : R2 → R una funcion de clase C2 armonica y sean

x = eu cos v y = eu sen v

Consideremos la funcion g : R2 → R definida como

g(u, v) = f(x(u, v) , y(u, v)).

a) Pruebe que (∂g

∂u

)2

+

(∂g

∂v

)2

= e2u

[(∂f

∂x

)2

+

(∂f

∂y

)2]

b) Pruebe que g es armonica. Es decir

∇2g =∂2g

∂u2+∂2g

∂v2= 0

62. Si x2 = y2 + f(x2 + z2), Calcular:

xy∂y

∂x+ yz

∂y

∂x

∂z

∂x− zx∂z

∂x

63. Sea f(x, y) = x2g(x2y). Pruebe que x∂f

∂x− 2y

∂f

∂y= 2f(x, y).

64. Sea f(x, y) = g(x, y) eax+by con∂2g

∂x∂y= 0. Determine los valores de las constantes a

y b para que∂2f

∂x∂y− ∂f

∂x− ∂f

∂y+ f = 0

65. Sea f : R2 → R la funcion definida por:

f (x, y) =

|x|y√x2+y2

, si x3 − y2 6= 0

0 , si x3 − y2 = 0

Verifique que f es continua en (0, 0), que posee derivada direccional en (0, 0) en

cualquier direccion que no es diferenciable en dicho punto.

66. Sea f : R2 → R la funcion definida por:

f (x, y) =

xy

, si y 6= 0

0 , si y = 0

a) Calcule ∂f∂x

(0, 0) y ∂f∂y

(0, 0).

424

Page 427: Apuntes MAT-023 USM

Apuntes Mat023 (Segundo semestre 2014)

b) Sea u = (α, β) tal que αβ 6= 0. ¿Existe ∂f∂u

(0, 0)?

67. Una mosca se encuentra volando, circularmente, en alguna cocina. La duena de casa,

ante la presencia del insecto, rocıa el aire con algun insecticida. La distribucion del

veneno en el aire se puede modelar mediante la superficie:

z = 10 + 6 cosx cos y + 3 cos 2x+ 4 cos 3y

Ante el peligro, la mosca establece (rapidamente) un sistema de referencia y determina

que se encuentra en el punto(π3, π

3, 6). ¿En que direccion deberıa arrancar la mosca

si quiere sobrevivir?

a) Encontrar la ecuacion del plano tangente a la superficie x = e2y−z en (1, 1, 2) y

un vector unitario normal a este plano.

b) Sea f (x, y, z) = z − ex sin y y sea a =(ln 3, 3π

2,−3

). Calcular el valor de c tal

que a ∈ S, donde:

(x, yz) ∈ S ⇐⇒ f (x, y, z) = c

Hallar, ademas, el plano tangente a esta superficie en a.

c) Sea f (x, y) = x√x2+y2

. Hallar el plano tangente a la superficie f (x, y) = z en el

punto(3,−4, 3

5

).

68. Una funcion u esta definida por una ecuacion de la forma:

u = xy f

(x+ y

xy

)Demostrar que u satisface una ecuacion en derivadas parciales de la forma:

x2∂u

∂x− y2∂u

∂y= uG (x, y)

Calcule, ademas, G (x, y).

69. Sean k una constante positiva y g (x, t) = 12

x√kt

. Considere:

f (x, t) =

∫ g(x,t)

0

e−u2

du

Demuestre que:

k∂2f

∂x2=∂f

∂t

70. Las ecuaciones:

u = f (x, y) ∧ x = X (s, t) ∧ y = Y (s, t)

definen u como una funcion de s y t, digamos: u = F (s, t).

425

Page 428: Apuntes MAT-023 USM

Apuntes Mat023 (Segundo semestre 2014)

a) Emplee una forma adecuada de la regla de la cadena para expresar las derivadas

parciales ∂F∂s

y ∂F∂t

en funcion de ∂f∂x, ∂f∂y, ∂X∂s, ∂X∂t, ∂Y∂s

y ∂Y∂t

.

b) Asumiendo que f ∈ C2, demuestre que:

∂2F

∂s2=∂f

∂x

∂2X

∂s2+∂2f

∂x2

(∂X

∂s

)2

+ 2∂X

∂s

∂Y

∂s

∂2f

∂x∂y+∂f

∂y

∂2Y

∂s2+∂2f

∂y2

(∂Y

∂s

)2

71. Resuelva el ejercicio anterior para los casos:

a) X (s, t) = s+ t e Y (s, t) = st.

b) X (s, t) = st e Y (s, t) = st.

c) X (s, t) = 12

(s− t) e Y (s, t) = 12

(s+ t).

72. La sustitucion x = es e y = et transforma f (x, y) en g (s, t) siendo g (s, t) = f (es, et).

Se sabe, ademas, que f satisface la ecuacion en derivadas parciales:

x2∂2f

∂x2+ y2∂

2f

∂y2+ x

∂f

∂x+ y

∂f

∂t= 0

Demuestre que g satisface otra ecuacion de la forma:

a∂2g

∂s2+ b

∂2g

∂t2+ c

∂g

∂s+ d

∂g

∂t= 0

para ciertas constantes a, b, c, d ∈ R. Calcule los valores de dichas constantes.

73. Escriba la ecuacion:

x∂z

∂x+ y

∂z

∂y= x2y

en las variables u y v mediante el cambio de variables:

u = x ∧ v =y

x

74. Represente la ecuacion:∂2

∂x2+ 2

∂2z

∂x∂y− 3

∂2z

∂y2= 0

en las variables u y v definidas por el cambio de variables:

u = 3x− y ∧ v = x+ y

75. Escriba la ecuacion:

y∂2z

∂x2+ (x+ y)

∂2z

∂x∂y+ x

∂2z

∂y2= 0

en terminos de u y v las cuales vienen dadas por el cambio de variables:

u = x2 − y2 ∧ v = y − x

426

Page 429: Apuntes MAT-023 USM

Capıtulo 10 : Maximos y mınimos

Extremos locales

Definicion 10.1.1. Sean U ⊆ Rn un conjunto abierto y f : U ⊆ Rn→ R una funcion.

Diremos que x0 ∈ U es un punto de:

1. Mınimo local, si existe un δ > 0 tal que ∀x ∈ B (x0, δ) ∩ U, f (x0) ≤ f (x)

2. Maximo local, si existe un δ > 0 tal que ∀x ∈ B (x0, δ) ∩ U, f (x0) ≥ f (x)

3. Extremo local si es maximo o mınimo local.

Cuando trabajamos con funciones diferenciables de una variable los extremos locales

ocurren en los puntos en los cuales f ′ (x) = 0, este resultado se extiende a funciones de

varias variables.

Suponga que x0 = (x01, x

02, . . . , x

0n) es un punto de maximo local de f : U ⊆ Rn → R

entonces existe un δ > 0 tal que

∀x ∈ B (x0, δ) ∩ U, f (x0) ≥ f (x)

es decir, ∀x ∈ U, ‖x− x0‖ < δ ⇒ f (x0) ≥ f (x). Definamos la funcion

g : I ⊆ R→ Rt → g (t) = f

(t, x0

2, x03, . . . , x

0n

)427

Page 430: Apuntes MAT-023 USM

Apuntes Mat023 (Segundo semestre 2014)

donde I es un intervalo abierto con x01 ∈ I entonces∥∥(t, x0

2, x03, . . . , x

0n

)−(x0

1, x02, . . . , x

0n

)∥∥ < δ ⇒ g(x0

1

)= f (x0) ≥ f

(t, x0

2, x03, . . . , x

0n

)= g (t)

pero ∥∥(t, x02, x

03, . . . , x

0n

)−(x0

1, x02, . . . , x

0n

)∥∥ < δ ⇔∣∣t− x0

1

∣∣ < δ

luego |t− x01| < δ ⇒ g (x0

1) ≥ g (t) de esta forma g tiene un maximo local en t = x01 se

sigue que como g es diferenciable

g′(x0

1

)= 0

pero

g′(x0

1

)= lım

h→0

g (x01 + h)− g (x0

1)

h

= lımh→0

f (x01 + h, x0

2, x03, . . . , x

0n)− f (x0

1, x02, x

03, . . . , x

0n)

h

= lımh→0

f (x0 + he1)− f (x0)

h=∂f (x0)

∂x1

se sigue∂f (x0)

∂x1

= 0

el mismo procedimiento se puede realizar para todas las variables.

Teorema 10.1.1. Sea f : U ⊆ Rn → R una funcion diferenciable, donde U es un conjunto

abierto. Si x0 ∈ U es un extremo local de f entonces ∇f (x0) = 0.

Definicion 10.1.2. Sea f : U ⊆ Rn → R una funcion diferenciable, donde U es un

conjunto abierto. Llamaremos puntos crıticos de f a todos aquellos puntos x0 ∈ U que

cumplen ∇f (x0) = 0. Si x0 es un punto crıtico que no es un extremo local entonces x0 se

dice punto de silla.

428

Page 431: Apuntes MAT-023 USM

Apuntes Mat023 (Segundo semestre 2014)

Ejemplo 10.1.1. Considere la funcion f :

R2 → R, (x, y) → f (x, y) = x2 + y2, esta

funcion es diferenciable en todo R2, sus puntos

crıticos son aquellos (x, y) tales que

∇f (x, y) = (0, 0)

pero(∂f

∂x(x, y) ,

∂f

∂y(x, y)

)= (2x, 2y) = (0, 0)

es decir, el unico punto crıtico es (0, 0). Por

la forma de la funcion sabemos que (0, 0) es

un punto de mınimo ya que

f (0, 0) = 0 ≤ x2 + y2 = f (x, y)

mas aun es un mınimo global (en el, la funcion

asume el menor valor que puede tomar en todo

su dominio)Ejemplo 10.1.2. Considere la funcion f : R2 →R, (x, y) → f (x, y) = −x2 − y2, esta funcion es

diferenciable en todo R2, sus puntos crıticos son

aquellos (x, y) tales que

∇f (x, y) = (0, 0)

pero(∂f

∂x(x, y) ,

∂f

∂y(x, y)

)= (−2x,−2y) = (0, 0)

es decir, el unico punto crıtico es (0, 0). Por la forma de la funcion sabemos que (0, 0)

es un punto de maximo ya que

f (0, 0) = 0 ≥ −x2 − y2 = f (x, y)

mas aun es un maximo global (en el, la funcion asume el mayor valor que puede tomar en

todo su dominio).

Ejemplo 10.1.3. Considere la funcion f : R2 → R, (x, y) → f (x, y) = x2 − y2, esta

funcion es diferenciable en todo R2, sus puntos crıticos son aquellos (x, y) tales que

∇f (x, y) = (0, 0)

429

Page 432: Apuntes MAT-023 USM

Apuntes Mat023 (Segundo semestre 2014)

pero (∂f

∂x(x, y) ,

∂f

∂y(x, y)

)= (2x,−2y) = (0, 0)

es decir, el unico punto crıtico es (0, 0). Note que, si x, y 6= 0 entonces

−y2 = f (0, y) < 0 = f (0, 0) < f (x, 0) = x2

de esta forma, arbitrariamente cerca de (0, 0) hay puntos en los cuales la funcion toma

valores mayores y menores. (0, 0) es un punto de silla.

Figura: La silla de montar

Claramente, no siempre es posible realizar este analisis para ver si los puntos son

maximos, mınimos o puntos de silla, necesitamos algun criterio que nos permita distinguir

entre estos tipos de puntos.

En calculo de una variable tenemos el siguiente criterio: Si x0 es un punto crıtico de f ,

entonces f ′′ (x0) > 0 implica que x0 es punto de mınimo local y si f ′′ (x0) < 0 de maximo

local. Si f ′′ (x0) = 0 no hay informacion. Este metodo requiere el signo de la segunda

derivada veamos como se puede generalizar a funciones de varias variables:

Proposicion 10.1.1. Sea n ∈ N y f : [a, b] → R una funcion tal que f ′, f ′′, . . . , f (n)

existen en [a, b] ademas f (n) es continua en [a, b] y diferenciable en ]a, b[ entonces existe

c ∈ ]a, b[ tal que

f (b) =n∑k=0

f (k) (a)

k!(b− a)k +

f (n+1) (c)

(n+ 1)!(b− a)n+1

430

Page 433: Apuntes MAT-023 USM

Apuntes Mat023 (Segundo semestre 2014)

Demostracion. La demostracion se basa en el teorema del valor medio, Para x ∈ [a, b]

defina

P (x) =n∑k=0

f (k) (a)

k!(x− a)k

y F : [a, b]→ R

x→ F (x) = f (x)− P (x)−(f (b)− P (b)

(b− a)n+1

)(x− a)n+1

la funcion esta bien definida ademas

F (a) = F (b) = 0

por el teorema de Rolle existe un c1 ∈ ]a, b[ tal que F ′ (c1) = 0 pero note que F ′ (a) = 0

entonces existe c2 ∈ ]a, c1[ tal que

F ′′ (c2) = 0

pero F ′′ (a) = 0... este proceso continua hasta la existencia de un c = cn+1 ∈ ]a, cn[ tal que

F (n+1) (c) = 0 como P (n+1) (x) ≡ 0 se sigue

f (n+1) (c) =

(f (b)− P (b)

(b− a)n+1

)(n+ 1)!

es decirf (n+1) (c)

(n+ 1)!(b− a)n+1 + P (b) = f (b)

Si f : [a, b]→ R es una funcion tal que f ′, f ′′ existen en [a, b] ademas f ′′ es continua en

[a, b] y diferenciable en ]a, b[ entonces existe c ∈ ]a, b[ tal que

f (b) =2∑

k=0

f (k) (a)

k!(b− a)k +

f (3) (c)

(n+ 1)!(b− a)3

es decir

f (b) = f (a) + f ′ (a) (b− a) +f ′′ (a)

2(b− a)2 +

f (3) (c)

3!(b− a)3

pongamos x0 y x0 + h en lugar de a, b entonces

f (x0 + h) = f (x0) + f ′ (x0)h+f ′′ (x0)

2h2 +

f (3)(cx0+hx0

)3!

h3

si x0 es un punto crıtico f ′ (x0) = 0 y ası

f (x0 + h)− f (x0) =f ′′ (x0)

2h2 +

f (3)(cx0+hx0

)3!

h3

= h2

(f ′′ (x0)

2+f (3)

(cx0+hx0

)3!

h

)

431

Page 434: Apuntes MAT-023 USM

Apuntes Mat023 (Segundo semestre 2014)

Suponga que f ′′ (x0) 6= 0, como

lımh→0

f (3)(cx0+hx0

)3!

h = 0

se sigue que existe un δ > 0 tal que 0 < |h| < δ ⇒∣∣∣∣∣f (3)(cx0+hx0

)3!

∣∣∣∣∣ < |f ′′ (x0)|4

luego para 0 < |h| < δ ⇒

f ′′ (x0)

2− |f

′′ (x0)|4

<f ′′ (x0)

2+f (3)

(cx0+hx0

)3!

h <f ′′ (x0)

2+|f ′′ (x0)|

4

de donde podemos concluir que para 0 < |h| < δ la cantidad f ′′(x0)2

+f (3)

(cx0+hx0

)3!

h tiene el

mismo signo que f ′′ (x0). Ası, si f ′′ (x0) > 0 entonces

f (x0 + h)− f (x0) = h2

(f ′′ (x0)

2+f (3)

(cx0+hx0

)3!

h

)> h2

luego, para 0 < |h| < δ se cumple f (x0 + h) > f (x0) es decir x0 es un mınimo. El mismo

argumento entrega que x0 es un maximo local cuando f ′′ (x0) < 0 (notar que el signo de h

no influye por estar al cuadrado).

Teorema 10.1.2 (Formula de Taylor de orden 2 en varias variables). Suponga que U ⊆ Rn

es un abierto y f : U → R tiene derivadas parciales continuas hasta el tercer orden entonces

f (x0 + h) = f (x0) +∇f (x0) h +1

2hTHf (x0) h +R2 (h,x0)

donde

lım‖h‖→0

R2 (h,x0)

‖h‖2 = 0

h =

h1

h2

...

hn

432

Page 435: Apuntes MAT-023 USM

Apuntes Mat023 (Segundo semestre 2014)

y

Hf (x0) =

∂2f

∂x21

(x0)∂2f

∂x1∂x2

(x0)∂2f

∂x1∂x3

(x0) · · · ∂2f

∂x1∂xn(x0)

∂2f

∂x2∂x1

(x0)∂2f

∂x22

(x0)∂2f

∂x2∂x3

(x0) · · · ∂2f

∂x2∂xn(x0)

∂2f

∂x3∂x1

(x0)∂2f

∂x3∂x2

(x0)∂2f

∂x23

(x0) · · · ∂2f

∂x3∂xn(x0)

......

.... . .

...

∂2f

∂xn∂x1

(x0)∂2f

∂xn∂x2

(x0)∂2f

∂xn∂x3

(x0) · · · ∂2f

∂x2n

(x0)

La demostracion es el teorema de Taylor de una variable y la regla de la cadena, en

efecto, si ponemos

g (t) = f (x0 + ht)

entonces

g (1) = f (x0 + h)

g (0) = f (x0)

y por la regla de la cadena g′ (t) = Df (x0 + ht)D (x0 + ht) es decir

g′ (t) = ∇f (x0 + ht) · h

=n∑i=1

∂f

∂xi(x0 + ht)hi

ası

g′ (0) =n∑i=1

∂f

∂xi(x0)hi = ∇f (x0) · h

ademas

g′′ (t) =d

dt

(n∑i=1

∂f

∂xi(x0 + ht)hi

)

=n∑i=1

hid

dt

(∂f

∂xi(x0 + ht)

)

433

Page 436: Apuntes MAT-023 USM

Apuntes Mat023 (Segundo semestre 2014)

usando la regla de la cadena

g′′ (t) =n∑i=1

hi∇(∂f

∂xi(x0 + ht)

)· h

=n∑i=1

hi

(n∑j=1

∂2f

∂xj∂xi(x0 + ht)hj

)

=n∑i=1

n∑j=1

∂2f

∂xj∂xi(x0 + ht)hjhi

=n∑

i,j=1

∂2f

∂xj∂xi(x0 + ht)hjhi

de manera similar

g′′′ (t) =n∑

i,j,k=1

∂2f

∂xk∂xj∂xi(x0 + ht)hjhihk

se sigue

g (1) = g (0) + g′ (0) 1 +g′′ (0)

2!12 +

g′′′ (c)

3!13

reemplazando

f (x0 + h)

= f (x0) +n∑i=1

∂f

∂xi(x0)hi +

1

2!

n∑i,j=1

∂2f

∂xj∂xi(x0)hjhi +

1

3!

n∑i,j,k=1

∂2f

∂xk∂xj∂xi

(x0 + hc1

0

)hjhihk

note que1

2!

n∑i,j=1

∂2f

∂xj∂xi(x0)hjhi =

1

2hTHf (x0) h

donde

h =

h1

h2

...

hn

434

Page 437: Apuntes MAT-023 USM

Apuntes Mat023 (Segundo semestre 2014)

y

Hf (x0) =

∂2f

∂x21

(x0)∂2f

∂x1∂x2

(x0)∂2f

∂x1∂x3

(x0) · · · ∂2f

∂x1∂xn(x0)

∂2f

∂x2∂x1

(x0)∂2f

∂x22

(x0)∂2f

∂x2∂x3

(x0) · · · ∂2f

∂x2∂xn(x0)

∂2f

∂x3∂x1

(x0)∂2f

∂x3∂x2

(x0)∂2f

∂x23

(x0) · · · ∂2f

∂x3∂xn(x0)

......

.... . .

...

∂2f

∂xn∂x1

(x0)∂2f

∂xn∂x2

(x0)∂2f

∂xn∂x3

(x0) · · · ∂2f

∂x2n

(x0)

Observacion 10.1.1. Bajo las hipotesis de continuidad del teorema se tiene

∂2f

∂xi∂xj=

∂2f

∂xj∂xi

por lo tanto la matriz Hf (x0) llamada Matriz Hessiana es una matriz simetrica.

Ahora bien, si x0 es punto crıtico entonces ∇f (x0) = 0 se sigue

n∑i=1

∂f

∂xi(x0)hi = 0

luego la expansion en Taylor quedara

f (x0 + h) = f (x0) +1

2hTHf (x0) h +R2 (h,x0)

y podemos generalizar el criterio de la segunda derivada de funciones reales.

Definicion 10.1.3. Sea A una matriz simetrica. Diremos que una forma cuadratica

CA : Rn → R, x → CA (x) = xTAx es definida positiva si para todo x ∈ Rn se cumple

CA (x) = xTAx ≥ 0 y CA (x) = 0 solo para x = 0. Diremos que es definida negativa si

para todo x ∈ Rn se cumple CA (x) = xTAx ≤ 0 y CA (x) = 0 solo para x = 0.

Recordemos que toda matriz simetrica A es diagonalizable ortogonalmente, se sigue

que existe una matriz Q invertible QQT = I tal que

A = QTDQ

435

Page 438: Apuntes MAT-023 USM

Apuntes Mat023 (Segundo semestre 2014)

donde D = diag (λ1, λ2, . . . , λn) es una matriz diagonal entonces

xTAx = (Qx)T D (Qx)

si ponemos

y = Qx =

y1

y2

...

yn

entonces

xTAx =n∑i=1

λiy2i

la forma xTAx es definida positiva (negativa) si y solo si todos los valores propios de A

son positivos (negativos).

Proposicion 10.1.2. Si la forma xTAx es definida positiva entonces existe c > 0 tal que

xTAx ≥ c ‖x‖2

En efecto, basta considerar c como el mınimo de los valores propios de la matriz A.

entonces, siguiendo el calculo anterior con

xTAx = (Qx)T D (Qx)

y = Qx =

y1

y2

...

yn

se tiene

xTAx =n∑i=1

λiy2i ≥ c

n∑i=1

y2i = c ‖y‖2

pero

‖y‖2 = ‖Qx‖2 = (Qx)T (Qx) = xTQTQx = xT Ix = ‖x‖2

dando el resultado deseado.

Teorema 10.1.3. Sean U ⊆ Rn abierto, f : U ⊆ Rn → R una funcion de clase C3 (U) y

x0 es un punto crıtico de f . Si f (h) = 12hTHf (x0) h es definida positiva entonces x0 es

un mınimo relativo, si es definida negativa es un maximo relativo.

436

Page 439: Apuntes MAT-023 USM

Apuntes Mat023 (Segundo semestre 2014)

Demostracion. Supongamos que es definida positiva, por la proposicion anterior existe un

c > 0 tal que1

2hTHf (x0) h ≥ c ‖h‖2

y de la expansion de Taylor tenemos

f (x0 + h) = f (x0) +∇f (x0) h +hTHf (x0) h

2+R2 (h,x0)

donde

lım‖h‖→0

R2 (h,x0)

‖h‖2 = 0

como x0 es punto crıtico ∇f (x0) · h = 0 y ası

f (x0 + h) = f (x0) +hTHf (x0) h

2+R2 (h,x0)

≥ f (x0) + c ‖h‖2 +R2 (h,x0)

como

lım‖h‖→0

R2 (h,x0)

‖h‖2 = 0

existe un δ > 0 tal que 0 < ‖h‖ < δ ⇒∣∣∣R2(h,x0)

‖h‖2

∣∣∣ < c2

se sigue para 0 < ‖h‖ < δ

f (x0 + h)− f (x0) ≥ ‖h‖2

(c+

R2 (h,x0)

‖h‖2

)≥ ‖h‖2

(c− c

2

)=c ‖h‖2

2> 0

ası 0 < ‖h‖ < δ implica

f (x0 + h) ≥ f (x0)

luego x0 es un punto de mınimo local ( es lo mismo quer decir, si x ∈ B (x0; δ) entonces

f (x) ≥ f (x0)). La demostracion de Maximo local es similar.

Teorema 10.1.4. Sean U ⊆ Rn abierto, f : U ⊆ Rn → R una funcion de clase C3 (U) y

x0 es un punto crıtico de f . Si Hf (x0) tiene todos sus valores propios no nulos y existen

dos con distinto signo entonces x0 es un punto silla.

Existen otros metodos para determinar si una matriz es definida positiva o negativa.

Sea

A =

a11 a12 . . . a1k . . . a1n

a21 a22 . . . a2k . . . a2n...

.... . .

......

...

ak1 ak2 . . . akk . . . akn...

... . . .... . . .

...

an1 an2 . . . ank . . . ann

437

Page 440: Apuntes MAT-023 USM

Apuntes Mat023 (Segundo semestre 2014)

Denotaremos por Ak a la submatriz

Ak =

a11 a12 . . . a1k

a21 a22 . . . a2k

......

. . ....

ak1 ak2 . . . akk

de esta forma

A1 = a11

A2 =

(a11 a12

a21 a22

)

A3 =

a11 a12 a13

a21 a22 a23

a31 a32 a32

...

An = A

si los determinantes de todas estas submatrices son positivos la matriz generara una

forma cuadratica definida positiva y el punto crıtico es un mınimo. Si los determinantes

se alternan en signo comenzando con el determinante de A1 negativo, A2 positivo, etc,

entonces la forma cuadratica sera definida negativa y el punto crıtico es un maximo local.

Si los determinantes son no nulos y no cumplen con los ordenes de signos anteriores, el

punto critico es punto silla.

Observacion 10.1.2 (Importante). Los criterios anteriores no entregan informacion si

existen valores propios nulos o los subdeterminates son cero.

Ejemplo 10.1.4. Analizar los extremos locales de la funcion f : R2 → R, (x, y) →f (x, y) = ln (x2 + y2 + 1)

Solucion. Los extremos locales estan en los puntos crıticos. Los puntos crıticos de f son

aquellos puntos (x, y) tales que

∇f (x, y) = (0, 0)

pero

∂f

∂x(x, y) =

2x

x2 + y2 + 1∂f

∂y(x, y) =

2y

x2 + y2 + 1

438

Page 441: Apuntes MAT-023 USM

Apuntes Mat023 (Segundo semestre 2014)

se sigue que el unico punto crıtico es (x, y) = (0, 0). Veamos si es un maximo, mınimo local

o un punto de silla. Calculemos la Hessiana, para ello necesitamos las derivadas de segundo

orden:

∂2f

∂x2(x, y) =

2 (x2 + y2 + 1)− 2x (2x)

(x2 + y2 + 1)2 =2y2 − 2x2 + 2

(x2 + y2 + 1)2

∂2f

∂x∂y(x, y) =

−4xy

(x2 + y2 + 1)2

∂2f

∂y2(x, y) =

2 (x2 + y2 + 1)− 2y (2y)

(x2 + y2 + 1)2 =2x2 − 2y2 + 2

(x2 + y2 + 1)2

Hf (x, y) =

2y2 − 2x2 + 2

(x2 + y2 + 1)2

−4xy

(x2 + y2 + 1)2

−4xy

(x2 + y2 + 1)2

2x2 − 2y2 + 2

(x2 + y2 + 1)2

entonces

Hf (0, 0) =

(2 0

0 2

)aplicando el criterio de los subdeterminantes vemos que |A1| = 2 > 0 y

∣∣∣∣ 2 0

0 2

∣∣∣∣ = 4 > 0

luego el extremo local es un mınimo.

Ejemplo 10.1.5. Analizar los extremos de f (x, y) = x3y + xy5 + xy.

Solucion. Buscamos los puntos crıticos

∇f (x, y) = (0, 0)

es decir

∂f

∂x(x, y) = 3x2y + y5 + y = y

(3x2 + y4 + 1

)= 0

∂f

∂y(x, y) = x3 + 5xy4 + x = x

(x3 + 5y4 + 1

)= 0

luego el unico punto crıtico es (x, y) = (0, 0). La Hessiana en este caso es

Hf (x, y) =

6xy 3x2 + 5y4 + 1

3x2 + 5y4 + 1 20xy3

luego

Hf (0, 0) =

0 1

1 0

439

Page 442: Apuntes MAT-023 USM

Apuntes Mat023 (Segundo semestre 2014)

entonces no funciona el metodo de los subdeterminantes pues el primero da cero. Note que

los valores propios de esta matriz son 1 y −1 por lo tanto es un punto de silla. Esto se

puede ver de la funcion misma pues

f (x, y) = xy(x2 + y4 + 1

)cerca del (0, 0) hay puntos donde la funcion es negativa y positiva luego es un punto de

silla.

Ejemplo 10.1.6. Clasificar los puntos crıticos de f : R3 → R, (x, y, z) → f (x, y, z) =

x2z + y2z + 23z3 − 4x− 4y − 10z

Solucion. La funcion esta definida en un abierto y es de clase C∞, buscamos los puntos

crıticos

∇f (x, y, z) =(2xz − 4, 2yz − 4, x2 + y2 + 2z2 − 10

)= (0, 0, 0)

esto es

2xz − 4 = 0

2yz − 4 = 0

x2 + y2 + 2z2 − 10 = 0

de las dos primeras ecuaciones se sigue que x, y, z 6= 0 y

x =2

z, y =

2

z

reemplazado en la tercera8

z2+ 2z2 − 10 = 0

ası

z = ±1,±2

tenemos 4 puntos crıticos

(2, 2, 1) , (−2,−2,−1) , (1, 1, 2) , (−2,−2,−1)

los clasificaremos usando el criterio de la hessiana y subdeterminantes

Hf (x, y, z) =

2z 0 2x

0 2z 2y

2x 2y 4z

entonces

∆1 = 2z

∆2 = (2z)2

∆3 = −8z(x2 + y2 − 2z2

)440

Page 443: Apuntes MAT-023 USM

Apuntes Mat023 (Segundo semestre 2014)

y asıPunto/Determinante ∆1 ∆2 ∆3 Tipo de punto

P1 = (2, 2, 1) + + - Punto silla

P2 = (−2,−2,−1) - + + Punto silla

P3 = (1, 1, 2) + + + Punto de mınimo local

P4 = (−2,−2,−1) - + - Punto de maximo local

notemos que

f (0, 0, z) =2

3z3 − 10z

luego

lımz→+∞

f (0, 0, z) = +∞

lımz→−∞

f (0, 0, z) = −∞

luego la funcion no tiene maximo ni mınimo en todo R3.

Ejemplo 10.1.7. Muestre que(π2, π

2, π

2

)es un punto de maximo de la funcion f (x, y, z) =

sinx+ sin y + sin z − sin (x+ y + z)

Solucion. Notemos que(π2, π

2, π

2

)es un punto crıtico pues

∇f (x, y, z)

= ∇ (sinx+ sin y + sin z − sin (x+ y + z))

= (cosx− cos (x+ y + z) , cos y − cos (x+ y + z) , cos z − cos (x+ y + z))

ası

∇f(π

2,π

2,π

2

)=

(cos

π

2− cos

(π +

π

2

), cos

π

2− cos

(π +

π

2

), cos

π

2− cos

(π +

π

2

))= (0, 0, 0)

la Hessiana es

Hf (x, y, z)

=

sin (x+ y + z)− sinx sin (x+ y + z) sin (x+ y + z)

sin (x+ y + z) sin (x+ y + z)− sin y sin (x+ y + z)

sin (x+ y + z) sin (x+ y + z) sin (x+ y + z)− sin z

441

Page 444: Apuntes MAT-023 USM

Apuntes Mat023 (Segundo semestre 2014)

ası

Hf(π

2,π

2,π

2

)

=

sin(π + π

2

)− sin

(π2

)sin(π + π

2

)sin(π + π

2

)sin(π + π

2

)sin(π + π

2

)− sin

(π2

)sin(π + π

2

)sin(π + π

2

)sin(π + π

2

)sin(π + π

2

)− sin

(π2

)

=

−2 −1 −1

−1 −2 −1

−1 −1 −2

los subdeterminantes son

∆1 = −2

∆2 = 3

∆3 = −4

luego el punto crıtico es un punto de maximo local.

Ejemplo 10.1.8. Sea f : R3 → R una funcion de clase C2 tal que:

∇f(x, y, z) =(2x− y, 3y2 − x, 2z − 2

)Verifique que el punto P

(112, 1

6, 1)

es un punto crıtico de f y clasifique el punto crıtico P .

Solucion. El punto P(

112, 1

6, 1)

es un punto crıtico de f si anula el gradiente ∇f . En

efecto:

∇f(

112, 1

6, 1)

=(2 · 1

12− 1

6, 3 · 1

36− 1

12, 2− 2

)= (0, 0, 0)

Ahora bien, como ∇f(x, y, z) =(2x− y, 3y2 − x, 2z − 2

), tenemos que:

fx = 2x− yfy = 3y2 − xfz = 2z − 2

442

Page 445: Apuntes MAT-023 USM

Apuntes Mat023 (Segundo semestre 2014)

Luego, fxx = 2, fxy = −1, fxz = 0, fyy = 6y, fyz = 0 y fzz = 2 . Ademas, como f es de

clase C2 sobre R3 , la matriz hessiana esta bien definida y vale:

Hf(x, y, z) =

2 −1 0

−1 6y 0

0 0 2

Ası:

Hf(

112, 1

6, 1)

=

2 −1 0

−1 1 0

0 0 2

y como:

∆1 = 2 > 0, ∆2 =

∣∣∣∣ 2 −1

−1 1

∣∣∣∣ = 1 > 0, ∆3 = 2 ·∣∣∣∣ 2 −1

−1 1

∣∣∣∣ = 2 > 0

se concluye que Hf(

112, 1

6, 1)

es definida positiva, y por el criterio del hessiano, f tiene un

mınimo local en P(

112, 1

6, 1)

.

Maximos y mınimos en compactos y/o con restricciones

El siguiente teorema generaliza el teorema de una variable que afirma que toda funcion

continua f : [a, b]→ R alcanza su maximo y mınimo absolutos, es decir, existen x0, x1 ∈[a, b] tales que f (x0) ≤ f (x) ≤ f (x1) para todo x ∈ [a, b].

Teorema 10.2.1 (De Weiertrass). Sean K ⊆ Rn un conjunto cerrado y acotado y f :

K → R una funcion continua entonces existen puntos x0 y x1 en K tales que

∀x ∈ K, f (x0) ≤ f (x) ≤ f (x1)

esto es f alcanza su maximo y mınimos absolutos en K.

Para utilizar tal teorema es conveniente la siguiente proposicion.

Proposicion 10.2.1. La imagen inversa de un conjunto abierto por una funcion continua

es un conjunto abierto. La imagen inversa de un cerrado por una funcion continua es un

conjunto cerrado.

Por ejemplo, si consideramos la funcion continua f (x, y) = y − x2 entonces

f−1 (]0,+∞[) = f−1 (u ∈ R : u > 0)=

(x, y) ∈ R2 : f (x, y) ∈ ]0,+∞[

=

(x, y) ∈ R2 : y − x2 > 0

=

(x, y) ∈ R2 : y > x2

443

Page 446: Apuntes MAT-023 USM

Apuntes Mat023 (Segundo semestre 2014)

es un conjunto abierto.

Si consideramos h (x, y) = x2 + y2 entonces h es continua y

h−1 ([1, 2]) =

(x, y) ∈ R2 : h (x, y) ∈ [1, 2]

=

(x, y) ∈ R2 : 1 ≤ x2 + y2 ≤ 2

es la imagen inversa de un cerrado por una funcion continua, entonces es un conjunto

cerrado.

Ejemplo 10.2.1. Hallar los valores extremos de f (x, y) = x2 + y2 − x− y + 1 en el disco

D = (x, y) ∈ R2 : x2 + y2 ≤ 1

Solucion. La funcion f (x, y) = x2 + y2 − x − y + 1 es continua. Note que el disco lo

podemos ver como

D = h−1 (]−∞, 1])

donde h (x, y) = x2 + y2 entonces D es cerrado y esta contenido en la bola B ((0, 0) ; 2)

luego es acotado. Por el teorema de los extremos absolutos se tiene que f (x, y) alcanza su

maximo y mınimos absolutos en D, es decir, existen puntos (u0, v0) y (u1, v1) en D tales

que

∀ (x, y) ∈ D, f (u0, v0) ≤ f (x, y) ≤ f (u1, v1)

Note que tales extremos pueden estar en la bola abierta B = (x, y) ∈ R2 : x2 + y2 < 1 o

en la circunferencia unitaria

S1 =

(x, y) ∈ R2 : x2 + y2 = 1

Primero busquemos los puntos crıticos en B.

∇f (x, y) =

(∂f

∂x(x, y) ,

∂f

∂x(x, y)

)= (0, 0)

= (2x− 1, 2y − 1) = (0, 0)

el unico punto crıtico en B es (x, y) =(

12, 1

2

). El conjunto S1 lo podemos parametrizar por

c : [0, 2π]→ S1 tal que t→ c (t) = (cos t, sin t), de esta forma la funcion f en el conjunto

S1 es

f (cos t, sin t) = 2− cos t− sin t

para t ∈ [0, 2π] como es una funcion de una variable buscamos sus valores extremos con el

calculo diferencial y el teorema de los extremos en R.

f ′ = 0⇔ sin t− cos t = 0

444

Page 447: Apuntes MAT-023 USM

Apuntes Mat023 (Segundo semestre 2014)

tiene las soluciones t = π4

y t = π4

+ π en el intervalo ]0, 2π[ luego se tienen los puntos

crıticos(1

2,1

2

),

(1√2,

1√2

),

(− 1√

2,− 1√

2

)y (1, 0) = (cos 0, sin 0) = (cos 2π, sin 2π)

evaluando en la funcion obtenemos

f

(1

2,1

2

)=

(1

2

)2

+

(1

2

)2

− 1

2− 1

2+ 1 =

1

2

f

(1√2,

1√2

)=

1

2+

1

2− 1√

2− 1√

2+ 1 = 2−

√2

f

(− 1√

2,− 1√

2

)=

1

2+

1

2+

1√2

+1√2

+ 1 = 2 +√

2

f (1, 0) = 1− 1 + 1 = 1

de donde(− 1√

2,− 1√

2

)es el punto de maximo y

(12, 1

2

)es el punto de mınimo en el disco.

Nota: La funcion es f (x, y) =(x− 1

2

)2+(y − 1

2

)2+ 1

2el valor mınimo se alcanza en(

12, 1

2

)y el maximo en el disco sera en el punto del disco que encuentre mas lejos de

(12, 1

2

)este es

(− 1√

2,− 1√

2

).

Extremos restringidos Multiplicadores de Lagrange

Es comun en problemas querer maximizar o minimizar alguna funcion sujeta a res-

tricciones dadas. Por ejemplo; En economıa suponga que se quiere vender dos tipos de

productos I y II. Sean x e y la cantidad de productos I y II respectivamente. Representemos

por f (x, y) la ganancia obtenida cuando se vende una cantidad x de I y y de II, nuestra

produccion esta limitada o controlada por nuestro capital, de esta forma tenemos una

restriccion del tipo g (x, y) = c. El problema podrıa plantearse como

max f (x, y)(x,y)∈R2,g(x,y)=c

Supongamos que x0 es un punto extremo del problema

Optimizar f (x)x∈Rn,g(x)=c

donde f : U ⊆ Rn → R y optimizar es maximo o mınimo. Sea c : R→ Rn donde

g (c (t)) = c

445

Page 448: Apuntes MAT-023 USM

Apuntes Mat023 (Segundo semestre 2014)

y c (0) = x0 entonces por la regla de la cadena

∇g (c (t)) · c′ (t) = 0

se sigue que

∇g (x0) · c′ (0) = 0

como x0 es extremo de la f se sigue que f (c (t)) tiene un maximo o mınimo en t = 0 luego

∇f (c (t)) · c′ (t) = D (f c) (t)

∇f (x0) · c′ (0) = 0

de esta forma ∇f (x0) y ∇g (x0) deben ser paralelos es decir ∃λ ∈ R tal que

∇f (x0) = λ∇g (x0)

el numero λ se llama multiplicador de Lagrange y la funcion de n+ 1 variables

L (x, λ) = f (x)− λ (g (x)− c)

es llamado Lagrangiano del problema. Podemos entonces formular el siguiente teorema:

Teorema 10.3.1. Sean f, g : U ⊆ Rn → R funciones de clase C1. Sea x0 ∈ U un punto

tal que g (x0) = c y ∇g (x0) 6= 0. Si

S = x ∈ U : g (x) = c

y f |S tiene un extremo local en x0 entonces existe λ0 ∈ R tal que

∇f (x0) = λ0∇g (x0)

Observacion 10.3.1. Es lo mismo que buscar los puntos crıticos del Lagrangiano L (x, λ) =

f (x)− λ (g (x)− c)

Ejemplo 10.3.1. Sea S ⊆ R2 la recta que pasa por (−1, 0) inclinada en un angulo de 45

y sea f : R2 → R, (x, y)→ x2 + y2. Hallar los extremos de f |S.

Solucion. La pendiente es tan (45) = 1 entonces

y − 0 = x− 1

se sigue que la restriccion es

g (x, y) = y − x+ 1 = 0

446

Page 449: Apuntes MAT-023 USM

Apuntes Mat023 (Segundo semestre 2014)

buscamos los puntos crıticos del Lagrangiano

L (x, y, λ) = x2 + y2 − λ (y − x+ 1)

ası

∂L

∂x(x, y, λ) = 2x+ λ = 0

∂L

∂y(x, y, λ) = 2y − λ = 0

∂L

∂λ(x, y, λ) = y − x+ 1 = 0

Note que de la primera y segunda se obtiene

λ = 2y = −2x⇔ y = −x =λ

2

reemplazando en la tercera se obtiene

y = −1

2⇒ x =

1

2

geometricamente podemos deducir que este punto es un mınimo (x0, y0) =(

12,−1

2

). Note

que el punto es un extremo de f |S pero no de f .

Ejemplo 10.3.2. Maximizar la funcion f (x, y, z) = x+ z sujeta a la restriccion x2 + y2 +

z2 = 1.

Solucion. El conjunto S = (x, y, z) ∈ R3 : x2 + y2 + z2 = 1 es cerrado y acotado y f es

una funcion continua en todo R2 de donde obtenemos que f |S debe tener un maximo y un

mınimo absolutos. Los buscamos con el metodo de Lagrange. Pongamos el Lagrangiano

L (x, y, z, λ) = x+ z − λ(x2 + y2 + z2 − 1

)y debemos buscar sus puntos crıticos

∂L

∂x(x, y, z, λ) = 1− 2λx = 0

∂L

∂y(x, y, z, λ) = −2λy = 0

∂L

∂z(x, y, z, λ) = 1− 2λz = 0

∂L

∂λ(x, y, z, λ) = x2 + y2 + z2 − 1 = 0

note que λ = 0 o y = 0. λ = 0 no puede ser (sistema inconsistente) entonces y = 0 se sigue

de la primera y tercera ecuaciones

x =1

z =1

447

Page 450: Apuntes MAT-023 USM

Apuntes Mat023 (Segundo semestre 2014)

ası x = z y reemplazando en la ultima

2x2 = 1⇒ x = ± 1√2

ası obtenemos dos puntos crıticos(1√2, 0,

1√2

)y

(− 1√

2, 0,− 1√

2

)reemplazando se tiene que el primero es un maximo y el segundo un mınimo.

Ejemplo 10.3.3. Hallar el mayor volumen que pueda tener una caja rectangular con tapa

sujeta a la restriccion de que el area de la superficie sea 10 m2.

Solucion. Sean x, y, z los lados de la caja entonces el volumen es V (x, y, z) = xyz, vemos

que esta funcion esta restringida a [0, 10]× [0, 10]× [0, 10] luego debe alcanzar el maximo

y mınimo absolutos. El area sera

2 (xy + xz + zy) = 10

ası el Lagrangiano sera

L (x, y, z, λ) = xyz − λ (xy + xz + zy − 5)

buscamos los puntos crıticos

∂L

∂x(x, y, z, λ) = yz − λ (y + z) = 0

∂L

∂y(x, y, z, λ) = xz − λ (x+ z) = 0

∂L

∂z(x, y, z, λ) = xy − λ (x+ y) = 0

∂L

∂λ(x, y, z, λ) = xy + xz + zy − 5 = 0

note que x 6= 0 pues si fuese cero zy = 5 pero λz = 0 se sigue λ = 0 y ası de la primera

yz = 0 contradiccion, lo mismo para y, z.

como son cantidades positivas tambien las sumas seran distintas de ceroyz

y + z=

xz

x+ z=

xy

x+ y

esto implica x = y = z sustituyendo en la ultima ecuacion nos da

3x2 = 5

de donde obtenemos el punto crıtico(√5

3,

√5

3,

√5

3

)

el volumen maximo es V =(√

53

)3

.

448

Page 451: Apuntes MAT-023 USM

Apuntes Mat023 (Segundo semestre 2014)

Ejemplo 10.3.4. Encuentre los extremos absolutos de la funcion f (x, y) = 2x2+y2−2y+1

sujeto a la restriccion x2 + y2 ≤ 4.

Solucion. La funcion es continua y el conjunto compacto, por el teorema de Weierstrass

se alcanza el maximo y mınimo absolutos en la region. Buscamos los puntos crıticos al

interior y en el borde usamos multiplicadores de Lagrange

∇f (x, y) = (4x, 2y − 2) = (0, 0)

⇔(x, y) = (0, 1)

en el borde los extremos deben cumplir

(4x, 2y − 2) = λ (2x, 2y)

x2 + y2 = 4

es decir

4x = 2λx

2y − 2 = 2λy

x2 + y2 = 4

de la primera

4x− 2λx = 2x (2− λ) = 0⇒ x = 0 ∨ λ = 2

si x = 0 entonces y = ±2 tenemos los puntos crıticos (0,±2). Si λ = 2 entonces

2y − 2 = 4y

y = −1 de donde

x2 + 1 = 4

x = −√

3,√

3 tenemos los puntos crıticos(±√

3,−1)

evaluando la funcion en (0, 1) , (0,±2)

y(±√

3,−1)

se obtiene

f (0, 1) = 2(02)

+ 12 − 2 (1) + 1 = 0

f(±√

3,−1)

= 2(±√

3)2

+ (−1)2 − 2 (−1) + 1 = 10

f (0, 2) = 2 (0)2 + (2− 1)2 = 1

f (0,−2) = 2 (0)2 + (−2− 1)2 = 9

se sigue que (0, 1) es el punto de mınimo global y(±√

3,−1)

puntos de maximo global.

449

Page 452: Apuntes MAT-023 USM

Apuntes Mat023 (Segundo semestre 2014)

Ejemplo 10.3.5. Una pieza larga de lamina galvanizada con ancho w se ha de doblar en

forma simetrica con tres lados rectos para hacer un canal de agua lluvia. En la figura se

muestra una seccion transversal. Determine las dimensiones que permiten el maximo flujo

de agua.

Solucion. Vamos a maximizar el area de la seccion transversal. Con los datos de la figura

A (x, θ) = 2

(1

2(x cos θ) (x sin θ)

)+ (w − 2x) (x sin θ)

= x2 sin θ cos θ + (w − 2x) (x sin θ)

=x2

2sin (2θ) + wx sin θ − 2x2 sin θ

450

Page 453: Apuntes MAT-023 USM

Apuntes Mat023 (Segundo semestre 2014)

donde (x, θ) ∈[0, w

2

]×[0, π

2

]buscamos los puntos crıticos al interior

Aθ = x2 cos (2θ) + wx cos θ − 2x2 cos θ

Ax = x sin (2θ) + w sin θ − 4x sin θ

despejando x en las dos

x2 cos (2θ) + wx cos θ − 2x2 cos θ = 0

x = −w(

cos θ

cos 2θ − 2 cos θ

)

x sin (2θ) + w sin θ − 4x sin θ = 0

x = −w(

sin θ

sin 2θ − 4 sin θ

)de donde obtenemos

−w(

cos θ

cos 2θ − 2 cos θ

)= −w

(sin θ

sin 2θ − 4 sin θ

)⇒

(cos θ) (sin 2θ − 4 sin θ) = (sin θ) (cos 2θ − 2 cos θ)

ası

2 cos2 θ sin θ − 4 cos θ sin θ = cos2 θ sin θ − 2 cos θ sin θ − sin3 θ

⇒2 cos2 θ − 4 cos θ = cos2 θ − 2 cos θ − sin2 θ

luego

2 cos2 θ − 4 cos θ −(cos2 θ − 2 cos θ − sin2 θ

)= 0

⇔1− 2 cos θ = 0

se sigue

θ =π

3y ası

x = −w

(sin(π3

)sin(

2π3

)− 4 sin

(π3

))=

w

3

451

Page 454: Apuntes MAT-023 USM

Apuntes Mat023 (Segundo semestre 2014)

miramos la Hessiana

HA (x, θ) =

(sin (2θ)− 4 sin θ 2x cos (2θ) + w cos θ − 4x cos θ

2x cos (2θ) + w cos θ − 4x cos θ −2x2 sin (2θ)− wx sin θ + 2x2 sin θ

)evaluando en el punto

(w3, π

3

)se tiene

HA(w

3,π

3

)=

(−3

2

√3 −1

2w

−12w −1

6

√3w2

)ası

∆1 = −3

2

√3 < 0

∆2 =1

2w2 > 0

es un maximo local. Note que

A(w

3,π

3

)=

(w/3)2

2sin

(2π

3

)+ w

(w3

)sin(π

3

)− 2

(w3

)2

sin(π

3

)=

1

12

√3w2

La funcion en los bordes de la caja es

A (0, θ) = 0

A(w

2, θ)

=(w/2)2

2sin (2θ) + w

(w2

)sin θ − 2 (w/2)2 sin θ =

1

8w2 sin 2θ ≤ 1

8w2

A (x, 0) = 0

A(x,π

2

)= wx− 2x2 ≤ 1

8w2

se sigue que(w3, π

3

)es punto de maximo global y el maximo es 1

12

√3w2

x2

2sin (2θ) + 3x sin θ − 2x2 sin θ

452

Page 455: Apuntes MAT-023 USM

Apuntes Mat023 (Segundo semestre 2014)

Este resultado tiene una extension a mas restricciones:

Si tenemos las restricciones una restriccion S dada por los puntos que satisfacen

g1 (x1, x2, . . . , xn) = c1

g2 (x1, x2, . . . , xn) = c2

...

gk (x1, x2, . . . , xn) = ck

y f restringida a S tiene un extremo en x0 ∈ S entonces, si∇g1 (x0) ,∇g2 (x0) , · · · ,∇gk (x0)

son l.i. existiran λ1, λ2, . . . , λk ∈ R tales que

∇f (x0) = λ1∇g1 (x0) + λ2∇g2 (x0) + · · ·+ λk∇gk (x0)

Ejemplo 10.3.6. Determine los extremos absolutos de f (x, y, z) = x + 2y + z sujeto a

las restricciones x2 + y2 = 1 y y + z = 1.

Solucion. f (x, y, z) = x+ 2y + z es una funcion continua. El conjunto definido por las

restricciones

S =

(x, y, z) ∈ R3 : x2 + y2 = 1 ∧ y + z = 1

=

(x, y, z) ∈ R3 : x2 + y2 = 1∩

(x, y, z) ∈ R3 : y + z = 1

note que el primer conjunto restringe a x, y al conjunto [−1, 1] y eso restringe a z en el

segundo conjunto. Es un conjunto acotado e interseccion de dos cerrados. Luego es cerrado

y acotado la funcion alcanza un maximo y mınimo absolutos en S.

453

Page 456: Apuntes MAT-023 USM

Apuntes Mat023 (Segundo semestre 2014)

El Lagrangiano sera

L (x, y, z, λ, µ) = (x+ 2y + z)− λ(x2 + y2 − 1

)− µ (y + z − 1)

buscamos los puntos crıticos

∂L

∂x= 1− 2λx = 0

∂L

∂y= 2− 2λy − µ = 0

∂L

∂z= 1− µ = 0

∂L

∂λ= x2 + y2 − 1 = 0

∂L

∂µ= y + z − 1 = 0

resolvemos µ = 1 reemplazando en la segunda obtenemos

1 = 2λy

de donde λ ni y pueden ser cero. De la primera obtenemos

x =1

2λ= y

de donde obtenemos en la cuarta

x = y = ± 1√2

de donde podemos obtener z en la 5. Ası(1√2,

1√2, 1− 1√

2

)y

(− 1√

2,− 1√

2, 1 +

1√2

)evaluando en la funcion f (x, y, z) = x+ 2y + z se tiene

f

(1√2,

1√2, 1− 1√

2

)=

1√2

+2√2

+ 1− 1√2

=2√2

+ 1

f

(− 1√

2,− 1√

2, 1 +

1√2

)= − 1√

2− 2√

2+ 1 +

1√2

= 1− 2√2

el primer es maximo y el segundo un mınimo.

454

Page 457: Apuntes MAT-023 USM

Apuntes Mat023 (Segundo semestre 2014)

Ejemplo 10.3.7. Sean

A =

(x, y, z, w) ∈ R4 : x+ y − z − 2w = 1

y

B =

(x, y, z, w) ∈ R4 : x− y + z + w = 2

determine el punto de A ∩B que se encuentra a menor distancia al cuadrado del origen.

Solucion. Vamos a resolver este problema de dos formas

mın (x2 + y2 + z2 + w2)

x+ y − z − 2w = 1

x− y + z + w = 2

usamos Lagrange con dos restricciones

(2x, 2y, 2z, 2w) = λ (1, 1,−1,−2) + µ (1,−1, 1, 1)

x+ y − z − 2w = 1

x− y + z + w = 2

esto es

2x = λ+ µ

2y = λ− µ2z = −λ+ µ

2w = µ− 2λ

x+ y − z − 2w = 1

x− y + z + w = 2

sistema que tiene solucion x = 2719, y = − 7

19, z = 7

19, w = − 3

19, λ = 20

19, µ = 34

19de donde

obtenemos que (27

19,− 7

19,

7

19,− 3

19

)es el punto de mınimo por la geometrıa del problema y por ser el unico punto crıtico o ver

siguiente seccion. La distancia cuadrado mınima es(27

19

)2

+

(− 7

19

)2

+

(7

19

)2

+

(− 3

19

)2

=44

19

Otra forma

x+ y − z − 2w = 1

x− y + z + w = 2

455

Page 458: Apuntes MAT-023 USM

Apuntes Mat023 (Segundo semestre 2014)

implica

x =1

2w +

3

2

y =3

2w + z − 1

2

se sigue que buscamos el mınimo de(1

2w +

3

2

)2

+

(3

2w + z − 1

2

)2

+ z2 + w2

calculamos su gradiente para buscar puntos criticos(∂

∂z

((1

2w +

3

2

)2

+

(3

2w + z − 1

2

)2

+ z2 + w2

),∂

∂w

((1

2w +

3

2

)2

+

(3

2w + z − 1

2

)2

+ z2 + w2

))= (0, 0)

esto es

3w + 4z − 1 = 0

7w + 3z = 0

se sigue

w = − 3

19

z =7

19

se sigue que es un punto de mınimo pues

H =

(4 3

3 7

)tiene determinante 19 y la primera entrada es 4. El valor mınimo es(

1

2

(− 3

19

)+

3

2

)2

+

(3

2

(− 3

19

)+

(7

19

)− 1

2

)2

+

(7

19

)2

+

(− 3

19

)2

=44

19

Ejemplo 10.3.8. Hallar y clasificar los extremos de la funcion f(x, y, z) = 2x2+y2+z2−xyen la region R ⊆ R3 definida por las ecuaciones:

x2 + y2 + z2 ≤ 1 ∧ −1/2 ≤ x ≤ 1/2 ∧ y ≥ −1/2

456

Page 459: Apuntes MAT-023 USM

Apuntes Mat023 (Segundo semestre 2014)

Solucion. Analizamos los extremos de f(x, y, z) = 2x2 + y2 + z2 − xy, primeramente, en

todo R3 a traves del criterio de la segunda derivada. Es decir, resolvemos el problema

sin restricciones y detectamos los puntos que pertenece al conjunto R. Ası, buscamos los

puntos (x, y, z) tales que:

∇f(x, y, z) = (4x− y, 2y − x, 2z) = (0, 0, 0)

Luego, el unico punto candidato a extremo es P 0(0, 0, 0) ∈ R y para obtener su clasificacion

se procede mediante la matriz hessiana, que viene dada por:

Hf(x, y, z) =

4 −1 0

−1 2 0

0 0 2

Note que la matriz Hf(x, y, z) es definida positiva en el punto P 0, pues:

∆1 = 4 > 0, ∆2 =

∣∣∣∣ 4 −1

−1 2

∣∣∣∣ = 7 > 0, ∆3 = 2 ·∣∣∣∣ 4 −1

−1 2

∣∣∣∣ = 14 > 0

Por tanto, la funcion f tiene en P 0 mınimo local y ademas, f(P0) = 0. Por otro lado,

analizamos f restringida a la funcion g(x, y, z) = x2 + y2 + z2 − 1 mediante el metodo de

los multiplicadores de Lagrange. Ası, el lagrangeano del problema esta definido como:

L(x, y, z, λ) = f(x, y, z)− λg(x, y, z)

y los posibles extremos cumplen que ∇L(x, y, z, λ) = (0, 0, 0, 0). Entonces, tenemos el

sistema:

4x− y − 2λx = 0

2y − x− 2λy = 0

2z − 2λz = 0

x2 + y2 + z2 = 1

Note que de la tercera ecuacion tenemos que las soluciones de 2z − 2λz = 0 son: z = 0 o

λ = 1. Luego, analizamos cada caso por separado:

Si λ = 1 se tiene el sistema:

4x− y − 2x = 0

2y − x− 2y = 0

el cual permite obtener como puntos crıticos los puntos P1 = (0, 0, 1) y P2 = (0, 0,−1), de

donde se sigue que f(P1

)= 1 y f

(P2

)= 1. Ahora bien, como la region R es compacta1 se

tiene que los puntos P1 y P2 son maximos.

1i.e. R es cerrado y acotado en R3.

457

Page 460: Apuntes MAT-023 USM

Apuntes Mat023 (Segundo semestre 2014)

Para el caso z = 0, tenemos que x2 + y2 = 1. Luego el sistema a resolver es:

4x− y − 2λx = 0

2y − x− 2λy = 0

el cual es un sistema lineal de la forma:(4− 2λ −1

−1 2− 2λ

)(x

y

)=

(0

0

)el cual tiene como solucion nula x = y = 0 si el determinante de la matriz es no nulo, esto

se tiene para los λ tal que 4λ2 − 12λ+ 7 = 0; es decir, λ1 =3 +√

2

2y λ2 =

3−√

2

2(Este

caso ya fue analizado). Resta analizar el caso en que el determinante sea cero, es decir,

tenemos dos rectas paralelas las cuales son:

y =(1±√

2)x

x =(− 1±

√2)y

Luego, solo debemos analizar las rectas l1 : y =(1 +√

2)x y l2 : y =

(1−√

2)x y ocupar

la restriccion x2 + y2 = 1. Para la recta l1 los puntos crıticos son:(± 1√

4+2√

2, ± 1+

√2√

4+2√

2, 0)

mientras que para la recta l2 los puntos crıticos son:(± 1√

4−2√

2, ± 1+

√2√

4−2√

2, 0)

. Luego,

usando las restricciones que definen R y las aproximaciones se descartan todos los puntos

a excepcion de:

P3 =(

1√4+2√

2, 1+

√2√

4+2√

2, 0)

= (0,3827 , 0,9239 , 0)

y en el cual se tiene f(P3

)= 0,7929, el cual es un maximo local dada la compacidad de R.

Criterio de la segunda derivada para extremos condicionados

En esta seccion presentamos un teorema que nos permite clasificar el punto obtenido

con los multiplicadores de Lagrange:

Teorema 10.3.2 (Hessiana orlada). Sean f, g : U ⊆ Rn → R funciones de clase C2.

Sean v0 ∈ U, g (v0) = c, ∇g (v0) 6= 0 y existe un λ0 ∈ R tal que ∇f (v0) = λ0g (v0). Si

458

Page 461: Apuntes MAT-023 USM

Apuntes Mat023 (Segundo semestre 2014)

L (λ,x) = f (x)− λ (g (x)− c) y

H = HL (v0) =

∣∣∣∣∣∣∣∣∣∣∣∣

0 − ∂g∂x1

− ∂g∂x2

· · · − ∂g∂xn

− ∂g∂x1

∂2L∂x21

∂2L∂x1∂x2

· · · ∂2L∂x1∂xn

− ∂g∂x2

∂2L∂x1∂x2

∂2L∂x22

· · · ∂2L∂x2∂xn

......

.... . .

...

− ∂g∂xn

∂2L∂x1∂xn

∂2L∂x2∂xn

· · · ∂2L∂x2n

∣∣∣∣∣∣∣∣∣∣∣∣entonces:

1. Si

H3 =

∣∣∣∣∣∣∣0 − ∂g

∂x1− ∂g∂x2

− ∂g∂x1

∂2L∂x21

∂2L∂x1∂x2

− ∂g∂x2

∂2L∂x1∂x2

∂2L∂x22

∣∣∣∣∣∣∣ < 0, H4 =

∣∣∣∣∣∣∣∣∣∣0 − ∂g

∂x1− ∂g∂x2

− ∂g∂x3

− ∂g∂x1

∂2L∂x21

∂2L∂x1∂x2

∂2L∂x1∂x3

− ∂g∂x2

∂2L∂x1∂x2

∂2L∂x22

∂2L∂x2∂x3

− ∂g∂x3

∂2L∂x1∂x3

∂2L∂x2∂x3

∂2L∂x23

∣∣∣∣∣∣∣∣∣∣< 0, . . .

entonces v0 es mınimo local de f sujeto a g (x) = c.

2. Si los determinantes se alternan en la forma H3 > 0, H4 < 0, H5 > 0 etc, entonces

v0 es maximo local de f sujeto a g (x) = c

3. Si los determinantes anteriores son distintos de cero pero siguen los patrones anteriores

entonces el punto es de silla.

Ejemplo 10.3.9. Estudiar los extremos de la funcion f (x, y) = cos2 x+ cos2 y sujeta a la

restriccion x− y = π4.

Solucion. la funcion de Lagrange es F (λ, x, y) = cos2 x+ cos2 y − λ(x− y − π

4

)se sigue

∂F

∂λ= −

(x− y − π

4

)∂F

∂x= −2 cosx sinx− λ

∂F

∂y= −2 cos y sin y + λ

luego

sin (2x) = −λsin (2y) = λ

x− y =π

4

459

Page 462: Apuntes MAT-023 USM

Apuntes Mat023 (Segundo semestre 2014)

se sigue

sin(

2(y +

π

4

))= − sin (2y)

cos 2y = − sin (2y)

ası

cos 2y + sin 2y = 0

⇔sin(

2y +π

4

)= 0

⇒y =

k

2π − π

8con k ∈ Z

se sigue (k

2π +

π

8,k

2π − π

8

)con k ∈ Z

son los puntos crıticos. Calculamos la Hessiana orlada∣∣∣∣∣∣∣∂2F∂λ2

∂F∂x∂λ

∂F∂y∂λ

∂F∂x∂λ

∂2F∂x2

∂F∂x∂y

∂F∂y∂λ

∂F∂x∂y

∂2F∂y2

∣∣∣∣∣∣∣ =

∣∣∣∣∣∣0 −1 1

−1 −2 cos 2x 0

1 0 −2 cos 2y

∣∣∣∣∣∣pero

2 cos 2x =√

2 (−1)k

2 cos 2y =√

2 (−1)k

se sigue ∣∣∣∣∣∣∣∂2F∂λ2

∂F∂x∂λ

∂F∂y∂λ

∂F∂x∂λ

∂2F∂x2

∂F∂x∂y

∂F∂y∂λ

∂F∂x∂y

∂2F∂y2

∣∣∣∣∣∣∣ =

∣∣∣∣∣∣∣0 −1 1

−1√

2 (−1)k+1 0

1 0√

2 (−1)k+1

∣∣∣∣∣∣∣ = 2 (−1)k√

2

se sigue que si k es impar

∣∣∣∣∣∣∣∂2F∂λ2

∂F∂x∂λ

∂F∂y∂λ

∂F∂x∂λ

∂2F∂x2

∂F∂x∂y

∂F∂y∂λ

∂F∂x∂y

∂2F∂y2

∣∣∣∣∣∣∣ < 0 y el punto es mınimo y si k es par es

460

Page 463: Apuntes MAT-023 USM

Apuntes Mat023 (Segundo semestre 2014)

un maximo. Note que

f (x, y) = cos2 x+ cos2 y

f

(k

2π +

π

8,k

2π − π

8

)= cos2

(k

2π +

π

8

)+ cos2

(k

2π − π

8

)=

1 + cos(kπ + π

4

)2

+1 + cos

(kπ − π

4

)2

= 1 +1

2

(cos(kπ +

π

4

)+ cos

(kπ − π

4

))= 1 +

1

2

(√2 (−1)k

)= 1 +

√2 (−1)k

2

para k impar

f

(k

2π +

π

8,k

2π − π

8

)= 1−

√2

2

para k par

f

(k

2π +

π

8,k

2π − π

8

)= 1 +

√2

2

Ejemplo 10.3.10. Hallar las dimensiones del mayor paralelepıpedo rectangular de aristas

paralelas a los ejes coordenados que puede ser inscrito en el elipsoide de ecuacion:(x3

)2

+(y

4

)2

+(z

5

)2

= 1

461

Page 464: Apuntes MAT-023 USM

Apuntes Mat023 (Segundo semestre 2014)

Solucion. El volumen del paralelepıpedo es

V (x, y, z) = (2x) (2y) (2z) = 8xyz

pero (x, y, z) debe estar sobre el elipsoide(x3

)2

+(y

4

)2

+(z

5

)2

= 1

y sus coordenadas son positivas o cero. Tenemos que resolver el problema

maxr(x,y,z)=0x,y,z≥0

V (x, y, z)

donde V (x, y, z) = 8xyz y r (x, y, z) =(x3

)2+(y4

)2+(z5

)2 − 1. Este problema tiene

solucion pues el elipsoide es un conjunto cerrado y acotado ademas la funcion es continua.

Determinaremos el valor maximo utilizando los multiplicadores de Lagrange

∇V (x, y, z) = λ∇r (x, y, z)

r (x, y, z) = 0

esto es

(8yz, 8xz, 8xy) = λ

(2x

32,2y

42,2z

52

)(x

3

)2

+(y

4

)2

+(z

5

)2

= 1

el sistema a resolver es

8yz =2λx

32

8xz =2λy

42

8xy =2λz

52(x3

)2

+(y

4

)2

+(z

5

)2

= 1

multiplicando las primeras tres ecuaciones por x, y, z respectivamente se obtiene

4xyz =λx2

32

4xyz =λy2

42

4xyz =λz2

52

462

Page 465: Apuntes MAT-023 USM

Apuntes Mat023 (Segundo semestre 2014)

sumando las tres ecuaciones

12xyz =λx2

32+λy2

42+λz2

52

= λ

((x3

)2

+(y

4

)2

+(z

5

)2)

= λ

asıλ

3= 4xyz =

λx2

32=λy2

42=λz2

52

si λ 6= 0 entonces

x2 =32

3⇒ x =

3√3

y2 =42

3⇒ y =

4√3

z2 =52

3⇒ z =

5√3

si λ = 0 entonces xyz = 0 y

yz = 0

xz = 0

xy = 0(x3

)2

+(y

4

)2

+(z

5

)2

= 1

esto nos da los puntos (3, 0, 0) , (0, 0, 5) , (0, 4, 0). Estos 3 puntos son puntos de mınimo

V = 0 y

V

(3√3,

4√3,

5√3

)= 8

(3√3

)(4√3

)(5√3

)=

160

3

√3

es el valor maximo.

Ejercicios del capıtulo

1. Determine los valores de a, b ∈ R para los cuales la funcion f (x, y) = ax2 + by2 tiene

un maximo, mınimo o punto silla en (0, 0).

2. Sea C ∈ R. Muestre que si f : U ⊆ Rn → R tiene un maximo (mınimo) local en

x = x0 entonces la funcion F : U → R definida por F (x) = f (x) + C tambien tiene

un maximo (mınimo) local en x = x0.

463

Page 466: Apuntes MAT-023 USM

Apuntes Mat023 (Segundo semestre 2014)

3. Determinar los valores de a, b ∈ R para los cuales la funcion f : R2 → R definida por

f (x, y) = x3 + 3ax+ y3 + by + 1

no tiene extremos locales.

4. Para cada una de las siguientes funciones determinar los puntos crıticos (o puntos

estacionarios), clasificarlos en maximos, mınimos y puntos silla:

a) z = x2 + (y − 1)2 i) z = x2 − xy + y2 − 2x+ y

b) z = x2 − (y − 2)2 j) z = sin (x) sin (y) sin (x+ y)

c) z = 1− x2 − y2 k) z = (5x+ 7y − 25) e−(x2+xy+y2)

d) z = (x− y − 1)2 l) z = (x2 + y2) e−(x2+y2)

e) z = 2x2 − xy − 3y2 − 3x+ 7y m) z = xy2 (x+ 2y − 1)

f) z = x (x2 + y2 − a2) n) z = x3 − x2y − x2 + y2

g) z = x3 + y3 − 3 (x+ y) + 1 o) z = (x+ y) (x2 + y2 − 6)

h) z = xy + (x− y)3 p) z = (x− y)3 + x4 + y4

5. Sea f (x, y) = 3x4−4x2y+y2. Demostrar que sobre toda recta de la forma y = mx la

funcion tiene un mınimo en (0, 0) pero no es mınimo en ningun entorno bidimensional

del origen. (Estudiar los puntos donde f (x, y) > 0 y f (x, y) < 0).

6. Considere la funcion z = f (x, y) que en los alrededores del punto (1, 1, 1) esta definida

implıcitamente por

z3 + 3x2y − y3z + y2 − 3x− 1 = 0

obtener la expansion de Taylor para z en (1, 1).

7. Sea f : R4 − (0, 0, 0, 0) → R dada por

f (x, y, z, u) = x+y

x+z

y+u

z+

1

u

tiene un punto crıtico en (1, 1, 1, 1) y clasificarlo.

8. Sea g : R→ R una funcion diferenciable que no se anula. Analizar los extremos de

f (x, y) =

∫ (x−1)2

0

g (t) dt+

∫ (y−1)2

0

g (t) dt

si g (0) > 0.

9. Determine los puntos crıticos de la funcion

f (x, y) = 2x4 + y4 − 4x2 − 2y2

y clasificarlos (obs.: Son 9 puntos).

464

Page 467: Apuntes MAT-023 USM

Apuntes Mat023 (Segundo semestre 2014)

11.* Considere la funcion z = f (x, y) definida implıcitamente por la expresion

F (x, y, z) = x3 + y3 + z3 − 3x− 3y + z + 4 = 0

obtener sus puntos crıticos y clasificarlos.

10. Determine los valores de las constantes a, b ∈ R para los cuales la integral∫ 1

0

(a+ bx− f (x))2 dx

sea mınimo (suponer f es continua en [0, 1]), estudiar los casos f (x) = x2 y f (x) =

x3 + x.

11. Sea f (x, y) = Ax2 + 2Bxy + Cy2 + 2Dx+ 2Ey + F donde A > 0 y B2 < AC.

a) Demostrar que f tiene un mınimo.

b) Demostrar que en el punto de mınimo (x1, y1) se cumple

f (x1, y1) = Dx1 + Ey1 + F

c) Demostrar que

f (x1, y1) =1

AC −B2

∣∣∣∣∣∣A B D

B C E

D E F

∣∣∣∣∣∣12. Metodo de los mınimos cuadrados: Dados n numeros distintos x1, x2, . . . , xn y

otros n numeros (no necesariamente distintos) es en general imposible encontrar una

recta f (x) = ax+ b que pase por todos los puntos (xi, yi) esto es f (xi) = yi∀i, no

obstante se puede encontrar los valores de a y b que hacen que el error cuadratico

total

E (a, b) =n∑i=1

(f (xi)− yi)2

sea mınimo. Encontrar tales valores!!!

13. Comprobar que f (x, y, z) = x4 + y4 + z4 − 4xyz tiene un punto crıtico en (1, 1, 1)

y clasificarlo como maximo, mınimo o punto silla. Expandir esta funcion en Taylor

entorno al punto (1, 1, 1) para visualizar directamente la forma cuadratica asociada a

la Hessiana Hf (1, 1, 1).

14. Estudiar los extremos relativos de la funcion f (x, y, z) = xyz (1− x− y − z)

465

Page 468: Apuntes MAT-023 USM

Apuntes Mat023 (Segundo semestre 2014)

15. Sea A ∈Mn×n (R), B ∈M1×n (R) y C ∈ R, se define F : Rn → R

F (X) = XTAX +BX + C

donde X ∈ Mn×1 (R) (el vector se mira como columna). Estudiar las condiciones

bajo las cuales esta funcion tiene maximos, mınimos o puntos silla.

16. Considere la funcion f : R3 → R dada por f (x, y, z) = sinx + sin y + sin z −sin (x+ y + z) determine la expansion de Taylor de f en

(π2, π

2, π

2

)´, muestre ademas

que el punto es un mınimo.

17. Considere la funcion z = f (x, y) que en los alrededores del punto (1, 1, 1) esta definida

implıcitamente por

z3 + 3x2y − y3z + y2 − 3x− 1 = 0

obtener la expansion de Taylor para z en (1, 1).

18. La Hessiana de cierta funcion f : D ⊆ R3 → R es

Hf (x, y, z) =

x2 y xz

y y2 yx

xz yx z2

¿En que subconjunto de R3 debe estar el punto critico (x, y, z) para que sea maximo,

mınimo o punto silla? ¿Cuando esta Hessiana no entrega informacion?.

19. Determine los extremos de la funcion f (x, y) = 2x− y con la restriccion g (x, y) =

3x2 + 2y2 − 33/2 = 0

20. Determine los extremos de la funcion f (x, y) = x2 + 8y si (x, y) se encuentra sobre

la elipse x2 + 4y2 = 5.

21. Utilizar el metodo de los multiplicadores para determinar los semiejes de la elipse

5x2 − 6xy + 5y2 − 32 = 0.

22. Usando multiplicadores de Lagrange, determinar la menor distancia entre las rectas

x+ 4

2=

y − 4

−1=z + 1

−2x+ 5

4=

y − 5

−3= z − 5− 5

23. Demostrar que el paralelepıpedo de mayor volumen que se puede inscribir en una

esfera es un cubo.

466

Page 469: Apuntes MAT-023 USM

Apuntes Mat023 (Segundo semestre 2014)

24. Hallar los puntos de la curva x2 + 4y2 − 4 = 0 que se encuentren mas cercanos y mas

lejanos a los puntos de la curva

x2 + y2 + 4x+ 2y − 20 = 0

25. Hallar los extremos absolutos de la funcion f (x, y) = x3

3− 3

2x2 + 2x+ y2 − 2y + 1 en

la region K = (x, y) : x ≥ 0, y ≥ 0, x+ y ≤ 1.

26. Hallar los extremos absolutos de la funcion f (x, y, z) = (x− 1)2 + (y − 1)2 + (z − 1)2

en la region

K =

(x, y, z) ∈ R3 : x2 + y2 + z2 ≤ 12

28.* Suponga que queremos determinar los extremos de la funcion f (x, y, z) sujeta a la

restriccion g (x, y, z) = 0 y utilizando el teorema de Lagrange obtenemos el punto

crıtico p0 = (x0, y0.z0), utilizando el teorema de la funcion implıcita demostrar que

el problema se puede reducir a estudiar los extremos de una funcion de 2 variables

(con el mismo punto crıtico), calcular la Hessiana de esta funcion de 2 variables y

expresarla en terminos de f y g para obtener un criterio que permita determinar si

el punto crıtico entregado por Lagrange es maximo, mınimo o silla (Esto es llamado

criterio de la hessiana reducida).

27. La fabrica de gelatinas BOB esta interesada en minimizar sus costos de produccion.

Los ingredientes utilizados en la elaboracion de su gelatina son tres: Grenetina, azucar

y frutas. La formula de produccion dice que el numero de gelatinas que se puede

producir es

g (x, y, z) = xαyβzγ

donde x es la cantidad de grenetina, y es la cantidad de azucar y z es la cantidad

de frutas (todas las cantidades en kilogramos) y las constantes α, β, γ son secretas.

Suponga que el precio por kilogramo de los ingredientes es: La grenetina $10,000,

azucar $250 y fruta $500. Si se quiere producir 10,000 gelatinas, ¿Que combinacion

de ingredientes minimiza los costos c (x, y, z) = 10,000x+ 250y + 500z? (la respuesta

puede quedar en terminos de α, β, γ).

467

Page 470: Apuntes MAT-023 USM

Capıtulo 11 : Funciones implıcitas e inversas

El teorema de la funcion implıcita

En los cursos anteriores, frecuentemente nos hemos encontrado con relaciones de la

forma

F (x, y) = 0

donde F : Ω ⊆ R2 → R es una funcion de clase C1 (Ω), por ejemplo, una elipse

x2

a2+y2

b2= 1⇔ F (x, y) = 0

donde

F (x, y) =x2

a2+y2

b2− 1

o la ecuacion cuadratica general

Ax2 +Bxy + Cy2 +Dx+ Ey + F = 0

Estas ecuaciones generalmente definen una curva en el plano que localmente (esto es, en

intervalos alrededor de un punto dado) puede ser descrita como la grafica de una funcion

y = φ (x) o de una funcion x = ψ (y).

Por ejemplo considere la funcion F : R2 → R, (x, y) → F (x, y) = x2 + y2 − 1, esta

funcion es clase C∞ (R2) y

F (x, y) = 0⇔ x2 + y2 = 1

Todos los puntos (x, y) con y > 0 que satisfacen

x2 + y2 = 1

corresponden a puntos de la grafica de la funcion

ψ (x) =√

1− x2 para x ∈ [−1, 1]

−1 −0,5 0,5 1

−0,5

0,5

1

y =√

1− x2

468

Page 471: Apuntes MAT-023 USM

Apuntes Mat023 (Segundo semestre 2014)

esto es, si (x, y) ∈ Graf (ψ), entonces F (x, y) = 0. En efecto

(x, y) ∈ Graf (ψ) ⇔ (x, y) = (x, ψ (x)) para algun x ∈ [−1, 1]

⇔ (x, y) =(x,√

1− x2)

para algun x ∈ [−1, 1]

luego (x, y) ∈ Graf (ψ) implica

F (x, y) = F(x,√

1− x2)

= x2 +(√

1− x2)2

− 1

= 0

lo que demuestra que todos los puntos de la grafica de ψ cumplen la relacion F (x, y) = 0

(estan sobre la circunferencia unitaria). De manera similar, los puntos (x, y) con y < 0 que

satisfacen F (x, y) = 0 corresponden a puntos de la grafica de la funcion

y = ϕ (x) = −√

1− x2 para x ∈ [−1, 1]

sin embargo, cerca de los puntos (1, 0) y (−1, 0) no podemos interpretar, los puntos de

la curva como puntos del grafico de una funcion de y = f (x), en este caso los puntos

corresponden a la grafica de

x = τ (y) =√

1− y2 para y ∈ [−1, 1] ∧ x = η (y) = −√

1− y2 para y ∈ [−1, 1]

respectivamente. En resumen, en cada punto (x0, y0) ∈ R2 que cumple F (x0, y0) = 0

existe una funcion ψ : ]x0 − ε, x0 + ε[ → R tal que ψ (x0) = y0 y que cumple para cada

x ∈ ]x0 − ε, x0 + ε[, F (x, ψ (x)) = 0 o bien existe una funcion ϕ : ]y0 − ε, y0 + ε[→ R tal

que ϕ (y0) = x0 y

F (ϕ (y) , y) = 0

para cada y ∈ ]y0 − ε, y0 + ε[.

469

Page 472: Apuntes MAT-023 USM

Apuntes Mat023 (Segundo semestre 2014)

x0x0−ε x0+ε

y0

y=ψ(x)

Si tenemos la funcion G : R2 → R, (x, y)→ G (x, y) = x7 + y7 + xy − 1 y la ecuacion

G (x, y) = 0

x7 + y7 + xy = 1

el problema no es tan sencillo. ¿Cerca de que puntos (x, y) podemos encontrar una funcion

y = ψ (x) cuya grafica coincida con la curva dada? es decir, los puntos del grafico de

y = ψ (x) esten sobre la curva

x7 + ψ7 (x) + xψ (x) = 1

supuesto que hemos resuelto el problema de la existencia de tal funcion, ¿Que propiedades

de G hereda la funcion y = ψ (x)? ¿es diferenciable? si es ası ¿Cuanto vale su derivada?.

Supongamos que la funcion que encontramos y = ψ (x) es derivable, entonces tenemos

G (x, ψ (x)) = 0

por la regla de la cadena

∂G

∂x(x, ψ (x))

∂ (x)

∂x+∂G

∂y(x, ψ (x))ψ′ (x) = 0

470

Page 473: Apuntes MAT-023 USM

Apuntes Mat023 (Segundo semestre 2014)

de esto se tiene

ψ′ (x) =−∂G

∂x(x, ψ (x))

∂G∂y

(x, ψ (x))

siempre que ∂G∂y

(x, ψ (x)) 6= 0.

Definicion 11.1.1. Sea F : U ⊆ Rn × Rm → Rm una funcion. Diremos que la funcion

ϕ : V ⊆ Rn → Rm, x→ ϕ (x) esta definida implıcitamente por la ecuacion

F (x,y) = 0

si ∀x ∈ V se cumple

F (x, ϕ (x)) = 0

Ejemplo 11.1.1. La funcion ϕ : R− → R definida por

ϕ (x) =3

√√1

4x6 − 8x3 +

1

2x3 +

2x

3

√√14x6 − 8x3 + 1

2x3

esta definida implıcitamente por la ecuacion x3 + 6xy = y3. En la figura se muestra la curva

x3 + 6xy = y3 y en verde la parte de ella descrita por ϕ. En este caso, puede resultar mas

simple trabajar directamente con la ecuacion que define la funcion en lugar de trabajar

con la expresion explıcita de la funcion.

471

Page 474: Apuntes MAT-023 USM

Apuntes Mat023 (Segundo semestre 2014)

El teorema de la funcion implıcita nos entrega condiciones sobre la funcion que define

la ecuacion, para garantizar todas estas propiedades

Teorema 11.1.1. Sean U abierto de Rn+1 y F : U → R es una funcion de clase Cp (U).

Denotemos por (x, z) los puntos de Rn+1 donde x ∈ Rn y z ∈ R. Supongamos que (x0, z0)

satisface

F (x0, z0) = 0 y∂F

∂z(x0, z0) 6= 0

entonces existe una bola abierta U que contiene a x0 ∈ Rn, una vecindad V de z0 ∈ Ry una unica funcion z = g (x) definida para x ∈ U y con recorrido en V que satisface

z0 = g (x0),

F (x, g (x)) = 0 para todo x ∈U

ademas z = g (x) es de clase Cp (U) y

Dg (x) = −DxF (x, g (x))∂F∂z

(x, g (x))

donde DxF es la derivada respeto a x es decir

DxF =

(∂F

∂x1

, . . . ,∂F

∂xn

)en terminos de componentes

∂g

∂xi(x) =

− ∂F∂xi

(x, g (x))∂F∂z

(x, g (x))

para i = 1, 2, . . . , n.

Ejemplo 11.1.2. Sea G : R3 → R una funcion de clase C1 (R3). Considere la superficie

G (x, y, z) = 0

suponga ademas que (x0, y0, z0) es un punto sobre la superficie, es decir

G (x0, y0, z0) = 0

si∂G

∂z(x0, y0, z0) 6= 0

por el teorema de la funcion implıcita existe un conjunto abierto U que contiene (x0, y0) y

una funcion z = z (x, y) definida en U tal que z0 = z (x0, y0) ademas

G (x, y, z (x, y)) = 0 para (x, y) ∈ U

472

Page 475: Apuntes MAT-023 USM

Apuntes Mat023 (Segundo semestre 2014)

el plano tangente a la grafica de z = z (x, y) en el punto (x0, y0, z0) es

z − z0 =∂z

∂x(x0, y0) (x− x0) +

∂z

∂y(x0, y0) (y − y0)

pero note que∂z

∂x(x0, y0) =

−∂G∂x

(x0.y0, z0)∂G(x0,y0,z0)

∂z

y

∂z

∂y(x0, y0) =

−∂G∂y

(x0.y0, z0)

∂G(x0,y0,z0)∂z

reemplazando

z − z0 =−∂G

∂x(x0.y0, z0)

∂G(x0,y0,z0)∂z

(x− x0) +−∂G

∂y(x0.y0, z0)

∂G(x0,y0,z0)∂z

(y − y0)

ası

∂G

∂x(x0.y0, z0) (x− x0) +

∂G

∂y(x0.y0, z0) (y − y0) +

∂G (x0, y0, z0)

∂z(z − z0) = 0

que es

∇G (x0, y0, z0) ((x, y, z)− (x0.y0, z0)) = 0

que es la ecuacion del plano tangente a una superficie que obtuvimos por otros medios (el

gradiente es perpendicular a los conjuntos de nivel)

Ejemplo 11.1.3. Suponga que F : R3 → R es una funcion de clase C1 (R3) que cumple(∂F∂x

) (∂F∂y

) (∂F∂z

)6= 0 muestre que las funciones z = z (x, y), x = x (y, z) y y = y (x, z)

definidas implıcitamente por

F (x, y, z) = 0

cumplen∂z

∂x

∂x

∂y

∂y

∂z= −1

Solucion. Como ∂F∂x6= 0 se sigue que la ecuacion define a x como funcion de y y z, es

decir, x = x (y, z) ademas

∂x

∂y= −

∂F∂y

∂F∂x

de manera similar (usando(∂F∂x

) (∂F∂y

) (∂F∂z

)6= 0) se tiene y = y (x, z) y z = z (x, y) donde

∂z

∂x= −

∂F∂x∂F∂z

y∂y

∂z= −

∂F∂z∂F∂y

473

Page 476: Apuntes MAT-023 USM

Apuntes Mat023 (Segundo semestre 2014)

entonces

∂z

∂x

∂x

∂y

∂y

∂z=

(−

∂F∂x∂F∂z

)(−

∂F∂y

∂F∂x

)(−

∂F∂z∂F∂y

)= −1

Ejemplo 11.1.4. ¿Cerca de cuales puntos es posible representar la superficie

x3 + 3y2 + 8xz2 − 3z3y = 1

como grafica de una funcion diferenciable z = k (x, y)?

Solucion. Son aquellos puntos para los cuales la derivada parcial respecto a la variable z

sea no nula, es decir

16xz − 9z2y 6= 0

z (16x− 9zy) 6= 0

Observacion 11.1.1. Tambien se puede intentar despejar las otras variables, formular el

teorema en tales casos.

Observacion 11.1.2. De la ecuacion

x3 + 3y2 + 8xz2 − 3z3y = 1

tambien se puede obtener una expresion para las derivadas parciales (derivando la ecuacion

respecto a x)

3x2 + 8z2 + 16xz∂z

∂x− 9z2y

∂z

∂x= 0

ası∂z

∂x=−3x2 − 8z2

16xz − 9z2y

Ejemplo 11.1.5. Sea f : R2 → R, (u, v)→ f (u, v) una funcion de clase C1 (R2). Supon-

gamos que ∂f∂v6= 0 en todo R2. Muestre que la ecuacion

f(x2 − y2, y2 − z2

)= 0

define implıcitamente una funcion z = z (x, y) para z 6= 0 y que la expresion

E (x, y, z) = yz∂z

∂x+ zx

∂z

∂y

no depende de f .

474

Page 477: Apuntes MAT-023 USM

Apuntes Mat023 (Segundo semestre 2014)

Solucion. Definamos G (x, y, z) = f (x2 − y2, y2 − z2) entonces G es compuesta de funcio-

nes C1 y por tanto es C1, ademas

∂G

∂z=

∂f

∂u

∂u

∂z+∂f

∂v

∂v

∂z

=∂f

∂v(−2z) 6= 0

por el teorema de la funcion implıcita existe una unica funcion z = z (x, y) de clase C1 tal

que

∂z

∂x=−∂G

∂x∂G∂z

= −fuux + fvvxfv (−2z)

= −fu2x+ fv0

fv (−2z)=

(fufv

)(xz

)similarmente

∂z

∂y=−∂G

∂y

∂G∂z

= −fuuy + fvvyfv (−2z)

= −fu (−2y) + fv (2y)

fv (−2z)=−fuy + fvy

fvz

se sigue

yz∂z

∂x+ zx

∂z

∂y= yz

((fufv

)(xz

))+ zx

(−fuy + fvy

fvz

)= xy

luego

yz∂z

∂x+ zx

∂z

∂y= xy

no depende de f .

Estudiaremos ahora el caso mas general en el cual F : RN × Rm → Rm. Supongamos

que tenemos un sistema de ecuaciones lineales

2u+ 3v + x− y = 0

u− v + 2x+ 3y = 0

sabemos que este sistema tiene infinitas soluciones 4 variables y dos ecuaciones (es cuestion

de rangos), las infinitas soluciones son porque podemos dejar variables en funcion de otras,

notemos lo siguiente

2u+ 3v = b1 = y − xu− v = b2 = −2x− 3y

475

Page 478: Apuntes MAT-023 USM

Apuntes Mat023 (Segundo semestre 2014)

para resolver el sistema

2u+ 3v = b1

u− v = b2

podemos utilizar Cramer y se obtiene

u =

∣∣∣∣ b1 3

b2 −1

∣∣∣∣∣∣∣∣ 2 3

1 −1

∣∣∣∣ =

∣∣∣∣ y − x 3

−2x− 3y −1

∣∣∣∣∣∣∣∣ 2 3

1 −1

∣∣∣∣ = −7

5x− 8

5y

y

v =

∣∣∣∣ 2 b1

1 b2

∣∣∣∣∣∣∣∣ 2 3

1 −1

∣∣∣∣ =

∣∣∣∣ 2 y − x1 −2x− 3y

∣∣∣∣∣∣∣∣ 2 3

1 −1

∣∣∣∣ =3

5x+

7

5y

luego las ecuaciones

2u+ 3v + x− y = 0

u− v + 2x+ 3y = 0

permiten despejar a u y v como funciones de x e y,

u (x, y) = −7

5x− 8

5y

v (x, y) =3

5x+

7

5y

Ahora suponga que tenemos ecuaciones no lineales

xu+ yvu2 = 2

xu3 + y2v4 = 2

en este caso se nos hace mucho mas difıcil despejar u, v en funcion de x e y. Estas ecuaciones

las podemos reinterpretar como

F (x, y, u, v) =(xu+ yvu2 − 2, xu3 + y2v4 − 2

)= (F1, F2) = (0, 0)

note que las ecuaciones se satisfacen en el punto (1, 1, 1, 1). Sabemos que una buena

aproximacion de F (x, y, u, v) cerca de (1, 1, 1, 1) es dada por

F (1, 1, 1, 1) +DF (1, 1, 1, 1) (x− 1, y − 1, u− 1, v − 1)T

476

Page 479: Apuntes MAT-023 USM

Apuntes Mat023 (Segundo semestre 2014)

donde

DF (x, y, u, v) =

(∂F1

∂x∂F1

∂y∂F1

∂u∂F1

∂v∂F2

∂x∂F2

∂y∂F2

∂u∂F2

∂v

)

=

(u vu2 x+ 2yvu yu2

u3 2yv4 3xu2 4v3y2

)evaluando en el punto

DF (1, 1, 1, 1) =

(1 1 3 1

1 2 3 4

)luego

F (x, y, u, v) ≈(

1 1 3 1

1 2 3 4

)(x− 1, y − 1, u− 1, v − 1)T

≈(

3u+ v + x+ y − 6

3u+ 4v + x+ 2y − 10

)luego la ecuacion

F (x, y, u, v) = (0, 0)

debe ser similar a

3u+ v + x+ y = 6

3u+ 4v + x+ 2y = 10

en el cual podemos despejar u y v en funcion de x e y, para poder hacer esto necesitamos∣∣∣∣ 3 1

3 4

∣∣∣∣ =∣∣D(u,v)F (1, 1, 1, 1)

∣∣=

∣∣∣∣ ∂F1

∂u(1, 1, 1, 1) ∂F1

∂v(1, 1, 1, 1)

∂F2

∂u(1, 1, 1, 1) ∂F2

∂v(1, 1, 1, 1)

∣∣∣∣6= 0

(permite usar Cramer para despejar las variables) Despejando

u =

∣∣∣∣ 6− x− y 1

10− x− 2y 4

∣∣∣∣∣∣∣∣ 3 1

3 4

∣∣∣∣ =14

9− 2

9y − 1

3x

y

v =

∣∣∣∣ 3 6− x− y3 10− x− 2y

∣∣∣∣∣∣∣∣ 3 1

3 4

∣∣∣∣ =4

3− 1

3y

477

Page 480: Apuntes MAT-023 USM

Apuntes Mat023 (Segundo semestre 2014)

de donde deberıa cumplirse

ux = −1

3, uy = −2

9

vx = 0, vy = −1

3

(note que deberıamos poner

u (x, y) ≈ 14

9− 2

9y − 1

3x

v (x, y) ≈ 4

3− 1

3y

(es una aproximacion de primer orden de estas funciones) cerca del punto (1, 1, 1, 1)). Estas

ideas se resumen en el siguiente:

Teorema 11.1.2 (De la funcion implıcita). Sean Ω abierto de RN , Λ abierto de Rm,

F : Ω × Λ → Rm una funcion de clase Cp (Ω× Λ). Pongamos (x,y) para los puntos en

RN × Rm. Suponga que (x0,y0) es un punto tal que

F (x0,y0) = 0

y que DyF (x0,y0) es invertible, entonces existe un abierto U ⊆ RN , una funcion y = g (x)

de U a Rm de clase Cp (U) tal que

F (x, g (x)) = 0 para todo x ∈ U

y0 = g (x0) ademas

Dg (x) = −Dy (x,y)−1Dx (x,y)

Ejemplo 11.1.6. Mostrar que cerca del punto (x, y, u, v) = (1, 1, 1, 1) podemos resolver

xu+ yvu2 = 2

xu3 + y2v4 = 2

de manera unica para u y v como funciones de x y y. Calcular(∂u∂x

)(1, 1)

Solucion. Definamos

F : R2 × R2 → R2

((x, y) , (u, v)) →(xu+ yvu2 − 2, xu3 + y2v4 − 2

)entonces

F ((1, 1) , (1, 1)) = (0, 0)

478

Page 481: Apuntes MAT-023 USM

Apuntes Mat023 (Segundo semestre 2014)

calculemos D(u,v)F (x, y, u, v)

D(u,v)F (x, y, u, v) =

∂(xu+yvu2−2)∂u

∂(xu+yvu2−2)∂v

∂(xu3+y2v4−2)∂u

∂(xu3+y2v4−2)∂v

=

(x+ 2yuv yu2

3xu2 4y2v3

)en (1, 1, 1, 1) es

D(u,v)F (1, 1, 1, 1) =

(3 1

3 4

)el determinante es 12 − 3 = 9 6= 0 se sigue que es invertible y ası por el teorema de la

funcion implıcita podemos despejar de manera unica para u y v como funciones de x y y.

Ademas g (x, y) = (u (x, y) , v (x, y)) tiene por derivada

Dg (x, y) = −(x+ 2yuv yu2

3xu2 4y2v3

)−1

D(x,y) (x, y, u, v)

donde

D(x,y) (x, y, u, v) =

∂(xu+yvu2−2)∂x

∂(xu+yvu2−2)∂y

∂(xu3+y2v4−2)∂x

∂(xu3+y2v4−2)∂y

=

(u vu2

u3 2yv4

)entonces

Dg (x, y) = −(x+ 2yuv yu2

3xu2 4y2v3

)−1(u vu2

u3 2yv4

)en el punto en cuestion

Dg (1, 1) = −(

3 1

3 4

)−1(1 1

1 2

)=

(−1

3−2

9

0 −13

)de donde obtenemos (

∂u∂x

(1, 1) ∂u∂y

(1, 1)∂v∂x

(1, 1) ∂v∂y

(1, 1)

)=

(−1

3−2

9

0 −13

)se sigue

∂u

∂x(1, 1) = −1

3

479

Page 482: Apuntes MAT-023 USM

Apuntes Mat023 (Segundo semestre 2014)

Ejemplo 11.1.7. Discutir sobre la resolubilidad del sistema

3x+ 2y + z2 + u+ v2 = 0

4x+ 3y + z + u2 + v + w + 2 = 0

x+ z + w + u2 + 2 = 0

para u, v, w en terminos de x, y, z en el punto x = y = z = 0, u = v = 0 y w = −2. Si es

posible, obtener la ecuacion del plano tangente a

w (x, y, z) = 0

en (0, 0, 0).

Solucion. Definamos F : R6 → R3 dada por

F (x, y, z, u, v, w)

=(3x+ 2y + z2 + u+ v2, 4x+ 3y + z + u2 + v + w + 2, x+ z + w + u2 + 2

)el sistema pude verse como

F (x, y, z, u, v, w) = (0, 0, 0)

notamos que

D(u,v,w)F (0, 0, 0) =

1 2v 0

2u 1 1

2u 0 1

∣∣∣∣∣∣(0,0,0)

=

1 0 0

0 1 1

0 0 1

que tiene determinante 1, por el teorema de la funcion implıcita se sigue que podemos

despejar (u, v, w) en terminos de (x, y, z) ademas∂u∂x

∂u∂y

∂u∂z

∂v∂x

∂v∂y

∂v∂z

∂w∂x

∂w∂y

∂w∂z

∣∣∣∣∣∣∣(0,0,0)

=

1 0 0

0 1 1

0 0 1

−1 3 2 0

4 3 1

1 0 1

=

3 2 0

3 3 0

1 0 1

de esto obtenemos

∇W = (1, 0, 1)

y ası el plano es

(1, 0, 1) (x, y, z) = 0

x+ z = 0

480

Page 483: Apuntes MAT-023 USM

Apuntes Mat023 (Segundo semestre 2014)

Ejercicios de la seccion

1. Sea F : R2 → R, (u, v)→ F (u, v) una funcion de clase C∞ que satisface

F (0, 0) = 0

∂F

∂u(0, 0) +

∂F

∂v(0,0) 6= 0

Muestre que la ecuacion

F(x3 − yz, y3 − xz

)= 0

define una funcion z = z (x, y) de clase C∞ en un entorno del punto (x, y) = (1, 1)

que satisface z (1, 1) = 1 y la ecuacion

(xz + 3y3

) ∂z∂x

+(yz + 3x3

) ∂z∂y

= 9x2y2 − z2

2. Probar que cerca del punto (x0, y0, z0, u0, v0) =(1, 1, 0, π

2, 0)

se puede resolver el

sistema

x2 − y cos (uv) + z2 = 0

x2 + y2 − sin (uv) + 2z2 − 2 = 0

xy − sinu cos v + z = 0

de manera unica para x, y, z como funciones de u y v. Calcular ∂x∂v

(π2, 0)

3. Sea f : R3 → R definida por f (x, y, z) = g(x+ z

y, y + z

x

)donde g : R2 → R y

∂f∂z

(x, y, z) 6= 0

a) Verificar que la ecuacion f (x, y, z) = 0 define implıcitamente a z como una

funcion de clase C1 de las variables x e y.

b) Comprobar que

x∂z

∂x+ y

∂z

∂y= z − xy

4. Mostrar que el sistema

x2 − y2 + u2 + 2v2 = 5

x2 + y2 − u2 − v2 = −4

define implıcitamente a u = u (x, y), v = v (x, y) con u (0, 1) = 2 y v (0, 1) = −1.

Encontrar las diferenciales Du (0,1) y Dv (0, 1) ademas de la derivada ∂2v∂x2

(0, 1).

481

Page 484: Apuntes MAT-023 USM

Apuntes Mat023 (Segundo semestre 2014)

5. Suponga que F,G : R3 → R son dos funciones de clase C1 (R3) y x0 = (x0, y0, z0) ∈ R3

es un punto en el cual F (x0) = 0, G (x0) = 0, esto es un punto de la interseccion de

las superficies

S1 : F (x, y, z) = 0

S2 : G (x, y, z) = 0

Determine una condicion que permita asegurar que es posible despejar x = x (y),

z = z (y) como funciones de clase C1 en un entorno de y0 luego la interseccion se

puede parametrizar como

x = x (y)

y = y

z = z (y)

para y en un entorno de y0 (el problema nos dice cuando podemos parametrizar con

un parametro la interseccion de dos superficies)

482

Page 485: Apuntes MAT-023 USM

Apuntes Mat023 (Segundo semestre 2014)

El teorema de la funcion inversa

Si f : R→ R es una funcion de clase C1 (R) y x0 es un punto al que f ′ (x0) 6= 0 entonces,

por la continuidad de la derivada, podemos garantizar la existencia de un intervalo abierto

I tal que f ′ (x) para todo x ∈ I tiene el mismo signo de f ′ (x0), entonces la funcion f

es estrictamente creciente o estrictamente decreciente en I (dependiendo si f ′ (x0) > 0 o

f ′ (x0) < 0).

f ′f

x0

f ′(x0)

En la grafica podemos observar en verde la grafica de una funcion y en azul la de su

funcion derivada, en el punto x0 la derivada es no nula (estrictamente positiva), existe un

intervalo (marcado en rojo) en el cual la funcion derivada es positiva, note que en todo ese

intervalo la funcion f es estrictamente creciente.

Si restringimos nuestra funcion f al intervalo I entonces f es inyectiva (toda funcion

estrictamente creciente o estrictamente decreciente es inyectiva pues envıa numeros distintos

del dominio en imagenes distintas), si ademas restringimos el recorrido a f(I) entonces es

sobreyectiva y podemos definir una inversa (no de la funcion completa, se trata de una

inversa local.

483

Page 486: Apuntes MAT-023 USM

Apuntes Mat023 (Segundo semestre 2014) f ′

f

f−1

note que solo podemos garantizar la existencia, obtener una expresion para ella puede ser

muy complicado. ¿Como podemos trabajar entonces con tal funcion?, ¿Cuales propiedades

de f conserva su inversa?.

El teorema de la funcion inversa de una variable asegura que la inversa es una funcion

de la misma clase (en terminos de derivadas) ademas(f f−1

)(x) = x

entoncesd

dx

((f f−1

)(x))

= 1

por la regla de la cadenadf

dx

(f−1 (x)

) df−1

dx(x) = 1

entoncesdf−1

dx(x) =

1

f ′ (f−1 (x))

si queremos calcular la derivada de segundo orden

d2f−1

dx2(x) =

d

dx

1

f ′ (f−1 (x))

= −(f ′(f−1 (x)

))−2f ′′(f−1 (x)

)( 1

f ′ (f−1 (x))

)=−f ′′ (f−1 (x))

(f ′ (f−1 (x)))3

Si T : Rn → Rn es una transformacion lineal, sabemos que es invertible si y solo si su

matriz asociada (por ejemplo respecto a las bases canonicas) es invertible, sabemos que la

matriz asociada respecto a las bases canonicas corresponde a la matriz Jacobiana de la T .

JT (x0) = [T ]CC

484

Page 487: Apuntes MAT-023 USM

Apuntes Mat023 (Segundo semestre 2014)

luego, si det (JT (x0)) 6= 0 la transformacion lineal es invertible (derivada no nula implica

invertible), es razonable esperar que si una funcion se parece a una transformacion lineal

localmente entonces podamos obtener informacion de ella a traves de la transformacion

lineal a la cual se parece, eso es justamente lo que afirma el teorema de la funcion inversa

de varias variables.

Observacion 11.2.1. Supongamos que U, V son abiertos y F : U ⊆ Rn → V ⊆ Rm

(diferenciable) tiene inversa (diferenciable) entonces n = m. En efecto, si G : V → U es su

inversa entonces

F G : V → V

x → (F G) (x) = x

y

G F : U → U

x → (G F ) (x) = x

luego

D (F G) = Im

D (G F ) = In

ası

DF (G (x))DG (x) = Im

DG (F (v))DF (v) = In

si ponemos x =F (v) entonces G (x) = v entonces

DF (G (x))DG (x) = Im

DG (x)DF (G (x)) = In

pongamos A = DF (G (x)) y B = DG (x) entonces

AB = In

BA = Im

definamos las transformaciones lineales

TA : Rn → Rm

x → TA (x) = Ax

485

Page 488: Apuntes MAT-023 USM

Apuntes Mat023 (Segundo semestre 2014)

y

TB : Rm → Rn

x → TB (x) = Bx

entonces TA TB es IRm y TB TA es IRn de la primera se obtiene que TB es inyectiva

TB (x) = TB (y) ⇒ TA (TB (x)) = TA (TB (y))

⇒ x = y

y de la segunda que TB es sobreyectiva (si y ∈ Rn entonces TA (y) = v ∈ Rmentonces

TB (v) = TB (TA (y)) = y

de donde Im(TB) = Rn) se sigue por el teorema de las dimensiones que

Dim (Rn) = Dim (Rm)

ası n = m.

Teorema 11.2.1 (De la funcion inversa). Sean U un abierto de Rn, F : U ⊆ Rn → Rn

una funcion de clase Cp (U) y x0 ∈ U un punto en el cual det (DF (x0)) 6= 0 entonces

existe un abierto U1 ⊆ U tal que x0 ∈ U1 y F : U1 → V = F (U1) es invertible, su inversa

F−1 : V → U1 es de clase Cp (V ), ademas

DF−1 (F (x)) = (DF (x))−1 para x ∈ U1

en particular, si F (x0) = y0 entonces

DF−1 (y0) = (DF (x0))−1

Observacion 11.2.2. Si x = (x1, x2, . . . , xn) y F : D ⊆ Rn → Rn, x → F (x) =

(F1 (x) , F2 (x) , . . . , Fn (x)) entonces det (DF (x0)) se representa por el sımbolo

∂ (F1, F2, . . . , Fn)

∂ (x1, x2, . . . , xn)

∣∣∣∣x=x0

y es llamado el Jacobiano (es el determinate de la matriz Jacobiana).

486

Page 489: Apuntes MAT-023 USM

Apuntes Mat023 (Segundo semestre 2014)

Ejemplo 11.2.1. Muestre que la funcion

F : R2 → R2

(x, y) → F (x, y) = (u (x, y) , v (x, y)) = (ex cos y, ex sin y)

no tiene una inversa global (definida en R2), sin embargo, cerca de todo punto es posible

definir una inversa local. Calcular

DF−1(√

2,√

2)

si F(ln 2, π

4

)=(√

2,√

2)

y obtener una expresion para

DF−1 (u, v)

si (u, v) 6= 0.

Solucion. Notemos

F (0, 2π) = F (0, 0) = (1, 0)

luego F no es inyectiva y por tanto no tiene inversa global.

Notemos que F ∈ C∞ (R2) ademas

DF (x, y) =

∂F1

∂x∂F1

∂y

∂F2

∂x∂F2

∂y

=

∂u∂x

∂u∂y

∂v∂x

∂v∂y

=

(ex cos y −ex sin y

ex sin y ex cos y

)y

∂ (u, v)

∂ (x, y)= det

(ex cos y −ex sin y

ex sin y ex cos y

)= e2x 6= 0

ası, por el teorema de la funcion inversa, F es invertible cerca de cada punto de R2 (inversa

local) ademas

DF−1 (F (x, y)) = (DF (x, y))−1

=

(ex cos y −ex sin y

ex sin y ex cos y

)−1

de esto se obtiene

DF−1(F(

ln 2,π

4

))=

eln 2 cos

π

4−eln 2 sin

π

4

eln 2 sinπ

4eln 2 cos

π

4

−1

=

14

√2 1

4

√2

−14

√2 1

4

√2

487

Page 490: Apuntes MAT-023 USM

Apuntes Mat023 (Segundo semestre 2014)

esto es

DF−1(√

2,√

2)

=

14

√2 1

4

√2

−14

√2 1

4

√2

si F (x, y) = (u, v) entonces F (u, v) = (x, y) donde x = x (u, v) e y = y (u, v) ademas ∂x

∂u

(√2,√

2)

∂x∂v

(√2,√

2)

∂y∂u

(√2,√

2)

∂y∂v

(√2,√

2) =

14

√2 1

4

√2

−14

√2 1

4

√2

ası por ejemplo

∂x

∂u

(√2,√

2)

=1

4

√2

en general

DF−1 (u, v) = DF−1 (F (x, y))

=

(ex cos y −ex sin y

ex sin y ex cos y

)−1

=

(u −vv u

)−1

=

u

u2 + v2

v

u2 + v2

− v

u2 + v2

u

u2 + v2

Observacion 11.2.3. Note que el teorema de la funcion inversa afirma DF−1 (y0) =

(DF (x0))−1 luego∂ (u, v)

∂ (x, y)=

1∂(x,y)∂(u,v)

cuidando donde estan evaluadas las funciones.

Ejemplo 11.2.2. Sea f : R2 → R2 la funcion definida por

f (u, v) =(u2 + u2v + 10v, u+ v3

)1. Mostrar que tiene inversa cerca del punto (1, 1)

488

Page 491: Apuntes MAT-023 USM

Apuntes Mat023 (Segundo semestre 2014)

Solucion. Notemos que la funcion es de clase C∞ (R2) y

Df (1, 1) =

(∂F1

∂u∂F1

∂v∂F2

∂u∂F2

∂v

)

=

∂(u2+u2v+10v)∂u

∂(u2+u2v+10v)∂v

∂(u+v3)∂u

∂(u+v3)∂v

=

(2u (v + 1) u2 + 10

1 3v2

)∣∣∣∣(1,1)

=

(4 11

1 3

)luego

det

(4 11

1 3

)= 1 6= 0

y ası, por el teorema de la funcion inversa, la funcion es localmente invertible, ademas

f (1, 1) = (12, 2)

2. Calcular la derivada de su inversa en el punto (12, 2).

Solucion. Como f (1, 1) = (12, 2) se sigue

Df−1 (12, 2) = Df−1 (f (1, 1)) =

(4 11

1 3

)−1

=

(3 −11

−1 4

)

Ejemplo 11.2.3. Sea u : R2 → R, (x, y)→ u (x, y) una funcion que satisface

2xy∂u

∂x−(x2 − y2

) ∂u∂y

= 0

1. Utilizar el teorema de la funcion inversa para determinar condiciones sobre la funcion

ζ de modo que

F : D ⊆ R2 → R2

F (x, y) = (µ (x, y) , ζ (x, y))

=

(x

x2 + y2, ζ (x, y)

)sea invertible.

489

Page 492: Apuntes MAT-023 USM

Apuntes Mat023 (Segundo semestre 2014)

Solucion. Por el teorema de la funcion inversa, si∣∣∣∣∣ ∂µ∂x

∂µ∂y

∂ζ∂x

∂ζ∂y

∣∣∣∣∣ =

∣∣∣∣∣ − x2−y2(x2+y2)2

− 2xy

(x2+y2)2

∂ζ∂x

∂ζ∂y

∣∣∣∣∣=

∂ζ

∂y

(− x2 − y2

(x2 + y2)2

)+∂ζ

∂x

(2xy

(x2 + y2)2

)6= 0

2. Escribir la ecuacion

2xy∂u

∂x−(x2 − y2

) ∂u∂y

= 0

en las nuevas variables (µ, ζ) mediante regla de la cadena, resolver esa nueva ecuacion

y concluir que u debe tener la forma

u (x, y) = f

(x

x2 + y2

)donde f ∈ C1 (R) es una funcion arbitraria.

Solucion.

∂u

∂x=

∂u

∂µ

∂µ

∂x+∂u

∂ζ

∂ζ

∂x∂u

∂y=

∂u

∂µ

∂µ

∂y+∂u

∂ζ

∂ζ

∂y

de donde

∂u

∂x=

∂u

∂µ

(− x2 − y2

(x2 + y2)2

)+∂u

∂ζ

∂ζ

∂x

∂u

∂y=

∂u

∂µ

(− 2xy

(x2 + y2)2

)+∂u

∂ζ

∂ζ

∂y

se sigue

0 = 2xy∂u

∂x−(x2 − y2

) ∂u∂y

= 2xy

(∂u

∂µ

(− x2 − y2

(x2 + y2)2

)+∂u

∂ζ

∂ζ

∂x

)−(x2 − y2

)(∂u∂µ

(− 2xy

(x2 + y2)2

)+∂u

∂ζ

∂ζ

∂y

)= 2xy

∂u

∂ζ

∂ζ

∂x−(x2 − y2

) ∂u∂ζ

∂ζ

∂y

=∂u

∂ζ

(2xy

∂ζ

∂x−(x2 − y2

) ∂ζ∂y

)490

Page 493: Apuntes MAT-023 USM

Apuntes Mat023 (Segundo semestre 2014)

pero (2xy

∂ζ

∂x−(x2 − y2

) ∂ζ∂y

)6= 0

entonces∂u

∂ζ= 0

se sigue

u (µ, ζ) = f (µ)

ası

u (x, y) = f

(x

x2 + y2

)donde f es una funcion arbitraria.

Ejercicios de la seccion

1. Sea f : R→ R una funcion de clase C1 y sea

u = f (x)

v = −y + xf (x)

si f ′ (x0) 6= 0 probar que la funcion T (x, y) = (u, v) es invertible cerca de (x0, y0) y

que la inversa tiene la forma

x = f−1 (u)

y = −v + uf−1 (u)

encontrar DT−1.

2. Considere la funcion F (x, y) =(

(x− y)2 , x2

y

)para y 6= 0.

a) Probar que F admite inversa local en una vecindad de (−1, 1)

b) Sea F−1 : V ⊆ R2 → R2, (x, y) = (g (u, v) , h (u, v)) la inversa local de F ,

calcular la razon de cambio de h en (4, 1) en la direccion del vector (2,−1).

3. Sea F (x, y) = (f1 (x, y) , f2 (x, y)) = (x cos y, sin (x− y)). Mostrar que F tiene inversa

local en una vecindad del punto(π2, π

2

)y obtener la Jacobiana de la inversa en (0, 0).

4. Definimos x : R2 → R, (r, θ) → x (r, θ) = r cos θ y y : R2 → R, (r, θ) → y (r, θ) =

r sin θ.

491

Page 494: Apuntes MAT-023 USM

Apuntes Mat023 (Segundo semestre 2014)

a) Demostrar que∂ (x, y)

∂ (r, θ)

∣∣∣∣(r0,θ0)

= r0

b) ¿cuando se puede formar una funcion inversa suave de F : R2 → R2, (r, θ)→(x (r, θ) , y (r, θ))? Comprobarlo directamente y con el teorema de la funcion

inversa.

5. Definamos F : R3 → R3, (ρ, φ, θ) → F (ρ, φ, θ) = (x (ρ, φ, θ) , y (ρ, φ, θ) , z (ρ, φ, θ))

donde

x (ρ, φ, θ) = ρ sinφ cos θ

y (ρ, φ, θ) = ρ sinφ sin θ

z (ρ, φ, θ) = ρ cosφ

a) Muestre que∂ (x, y, z)

∂ (ρ, φ, θ)= ρ2 sinφ

b) ¿Cuando se puede despejar (ρ, φ, θ) en terminos de (x, y, z)?

Ejercicios del capıtulo

1. Si u1, u2 son las raıces de la ecuacion cuadratica en λ

x2 + y2

λ− 2+

z2

λ− 1= 1

y u3 es arctan (y/x) probar que

∂ (u1, u2, u3)

∂ (x, y, z)= − 4z

u1 − u2

2. La ecuacion x3 + y3 = 6xy define cerca del punto(2 3√

2, 2 3√

4)

a y como funcion de x.

Muestre que x = 2 3√

2 es un punto crıtico de tal funcion y clasificarlo en maximo,

mınimo o punto silla.

3. La ecuacion

f(yx,z

x

)= 0

define a z como funcion de x, y, z = g (x, y). Muestre que

x∂g

∂x+ y

∂g

∂y= g

492

Page 495: Apuntes MAT-023 USM

Apuntes Mat023 (Segundo semestre 2014)

4. Si ϕ, φ : D ⊆ R3 → R son funciones de clase C1 tales que

∂ (ϕ, φ)

∂ (y, z)6= 0

entonces las ecuaciones ϕ (x, y, z) = 0, φ (x, y, z) = 0 determinan y, z como funciones

de x ydydx

=∂(ϕ,φ)∂(z,x)∂(ϕ,φ)∂(y,z)

dzdx

=∂(ϕ,φ)∂(x.y)∂(ϕ,φ)∂(y,z)

5. Si z1 = er cos θ, z2 = er sin θ y x = r cos θ, y = r sin θ probar que

∂ (z1, z2)

∂ (x, y)=e2r

r

6. Suponga que x, y, z, r, θ, φ estan relacionados por

f (x, y, z) = 0

y

x = r sin θ cosφ y = r sin θ sinφ z = r cos θ

probar que

sin θ

(∂z

∂x

)(∂r

∂θsin θ + r cos θ

)+∂r

∂φsinφ

− sin θ cosφ

(∂r

∂θcos θ − r sin θ

)= 0

7. Sea a > 0. Si y4 + 3a2y2 − 5a2xy + 2a2x2 = 0, probar que

lımx→0

dy

dx= 0 o lım

x→0

dy

dx=

2

3

8. Si u, v son funciones de las variables x, y por medio de las ecuaciones f (x, y, u) = 0

y g (u, v, x) = 0 probar que

∂v

∂x=

fxgu − fugxfugv

∂v

∂y=

fygufugv

9. La ecuacion ∂f∂z

(x, z) = u define a z como funcion de x y u. Si ponemos z = g (x, u)

y se define ϕ (x, u) = f (x, g (x, u)) probar que

∂ϕ

∂u= u

∂z

∂u∂ϕ

∂x=

∂f

∂x+ u

∂z

∂x

493

Page 496: Apuntes MAT-023 USM

Apuntes Mat023 (Segundo semestre 2014)

10. Suponiendo que u, v son funciones de x, y las cuales definen a u, v en terminos de

x, y y satisfacen las ecuaciones

∂u

∂x+∂v

∂y+ u

∂u

∂y= 0

∂v

∂x+∂u

∂y+ u

∂v

∂y= 0

Probar que∂2x

∂u2− ∂2x

∂v2=∂x

∂v

11. Si u1x1 = u2x2 = u3x3 = u4x4 = (u1u2u3u4)1/2 entonces

∂ (u1, u2, u3, u4)

∂ (x1, x2, x3, x4)= −1

12. Determinar el Jacobiano de la transformacion

x = a (ρ cos θ sinφ)α

y = b (ρ sin θ sinφ)β

z = c (ρ cosφ)γ

donde a, b, c, α, β.γ son constantes positivas y determinar cuando es posible invertir

tal transformacion. Ind.: Compuestas de funciones.

494

Page 497: Apuntes MAT-023 USM

Parte III

Evaluaciones de anos anteriores

495

Page 498: Apuntes MAT-023 USM

Capıtulo 12 : Controles

Control 1

1. Dada la funcion f : D ⊆ R2 → R definida por f (x, y) =√

x−yx+y

a) Determine y grafique el dominio de f .

b) ¿El dominio de f es un conjunto abierto?

c) Determine la frontera del dominio de f y sus puntos de acumulacion.

d) Determine las curvas de nivel f (x, y) = c y grafique para c = 0, c = 1, c = 12

y

c = 2.

e) Analice que sucede cuando c crece indefinidamente.

2. Sea T : R3 [x]→ R2 [x] definida por

T [p (x)] = p′′ (x) +

∫ 1

0

p (x) dx

a) Pruebe que T es una transformacion lineal.

b) Sean

B1 =

1, x− 1, (x− 1)2 , (x− 1)3B2 = 1, x, x (x− 1)

bases de R3 [x] y R2 [x] respectivamente. Determine [T ]B2B1 y use esta matriz para

obtener el nucleo de T .

Control 2

1. Sea T : R4 → R3 una transformacion lineal definida por

T (x, y, z, w) = (x− y + z + w, x+ 2z − w, x+ y + 3z − 3w)

Encuentre una base y la dimension de Ker(T ) e Im(T ).

2. Describa explıcitamente una transformacion lineal T : R3 → R2 tal que

Im (T ) = 〈(1, 0,−1) , (1, 2, 2)〉

496

Page 499: Apuntes MAT-023 USM

Apuntes Mat023 (Segundo semestre 2014)

Control 3

1. Considere la funcion T : R2 [x]→ R3 definida por T (p (x)) = (p (0) , p (1) , p (2)). Sean

B = 1, x, x2 y C = (1, 0, 0) , (0, 1, 0) , (0, 0, 1) bases de R2 [x] y R3 respectivamente:

a) Pruebe que T es una transformacion lineal.

b) Determine Ker(T ) y [T ]CB.

c) ¿T es un isomorfismo?.

2. Considere la funcion f : D ⊆ R2 → R, definida por f (x, y) =√

sin (x2 + y2).

a) Encuentre el dominio de f y dibujarlo.

b) Determine si el dominio es abierto o cerrado.

Control 4

1. Sea

f (x, y) =

x+ y si |x|+ |y| ≥ 2

1 si |x|+ |y| < 2

¿Existe lım(x,y)→(1,1)

f (x, y)? Justificar.

2. Demuestre o de un contraejemplo de la propiedad: Para todo A,B ⊆ Rn

∂ (A ∪B) ⊆ ∂A ∪ ∂B

Control 5

1. Considere la funcion f : (x, y) ∈ R2 : x 6= 0 → R, definida por

f (x, y) =(x2 + y2

)arctan

(yx

)¿es posible definir f en (0, 1) de forma que f sea continua en tal punto?

2. Sea f : R2 → R definida por

f (x, y) =

xyx2+y

si x2 6= −y

0 si x2 = −y

Hallar ∂f∂x

(x, y) y ∂f∂x

(x, y) en todos los puntos que estas existan y determinar si son

continuas en (0, 0).

497

Page 500: Apuntes MAT-023 USM

Apuntes Mat023 (Segundo semestre 2014)

Control 6

1. Considere el sistema

x = u+ v + w

y = u2 + v2 + w2

z = u3 + v3 + w3

Calcule ∂v∂y

(2, 6, 8) y ∂w∂x

(2, 6, 8) donde (x, y, z) = (2, 6, 8) es la imagen de (u, v, w) =

(1, 2,−1).

2. Considere el sistema

u3 + xv2 + y = 0

v3 + yv + u2 = 0

a) Pruebe que es posible despejar x = x (u, v), y = y (u, v) en vecindades de los

puntos (x, y) = (1,−1) y (u, v) = (0, 1).

b) Hallar ∂u∂x

(1,−1) y ∂v∂x

(1,−1).

Control 7

1. Un servicio de entrega de paquetes requiere que las dimensiones de una caja rectan-

gular sean tales que la longitud mas el doble del ancho mas el doble de la altura

no rebase 108 cms. ¿Cual es el volumen de la caja mas grande que podra enviar la

companıa?

2. Considerar el sistema

xy2 + xzu+ yv2 = 3

u3yz + 2xv − u2v2 = 2

¿Es posible despejar u = u (x, y, z) y v = v (x, y, z) en vecindades de U de (x, y, z) y

V de (u, v) = (1, 1). Calcular ∂v∂y

(1, 1, 1).

3. Resuelva la ecuaciondy

dx=

1− xy2

2x2y

haciendo la sustitucion v = y/xn para algun n.

4. Muestre que la ecuaciondy

dx=y

x+ xmynf

(yx

)498

Page 501: Apuntes MAT-023 USM

Apuntes Mat023 (Segundo semestre 2014)

se transforma en una ecuacion de variables separables usando el cambio de variables

y = vx donde v = v (x). Use lo anterior para resolver la ecuacion

dy

dx=y

x+

sec2(yx

)y2

Control 8

1. Encuentre la solucion general de la ecuacion

d4y

dx4− d3y

dx3+

d2y

dx2− 3

dy

dx− 6y = 0

2. Un estanque contiene 50 litros de agua pura. Al estanque entra salmuera, que contiene

C gramos de sal por litro a razon de 1,5 litros por minuto. La mezcla bien revuelta,

sale a razon de 1 litro por minuto. Si despues de 30 minutos la concentracion de sal

en el estanque es de 30 gramos por litro. Hallar el valor de C.

Control 9

1. Hallar dzdx

si se cumple x3 + y3 + z3 = 0 y x2 + y2 + z2 = 1

2. La matriz

A =

(1 −1

1 2

)representa el jacobiano de una funcion f de clase C2 de R2 en R2 en el punto (1, 1).

Suponga que f (1, 1) = (−1, 0) y que f (x, y) = (u, v). Pruebe que f es localmente

invertible en (1, 1) y encuentre ∂y∂u

(−1, 0).

Control 10

1. Muestre que la ecuacion diferencial

2x4yy′ + y4 = 4x6

se reduce a una ecuacion homogenea mediante el cambio de variables y = zn para

cierto n ∈ R. Determine el valor de n y resuelva la ecuacion.

2. Resuelva la ecuacion x3yy′ + 2x2y2 − 1 = 0 usando el cambio de variables u = x2y.

499

Page 502: Apuntes MAT-023 USM

Apuntes Mat023 (Segundo semestre 2014)

Control 11

1. Usando la transformada de Laplace resuelva el problema de valor inicial

ty′′ − ty′ + y = 2(et − 1

)con y (0) = 0, y′ (0) = −1.

2. Obtenga la transformada de Laplace inversa de

s2 − ss3 + s2 + 9s+ 9

Control 12

1. Resuelva el problema de valores iniciales

dy

dx− ex−y + ex = 0

y (0) = 1

Ayuda. Use el cambio de variables u = ey.

2. Resuelva la ecuacion y2dx = (x3 − xy) dy usando un factor integrante de la forma

xnym.

Control 13

Sea α ∈ R− Z.

1. Encontrar el desarrollo en serie de Fourier de la funcion f : ]−π, π[→ R

f (x) = cos (αx)

2. Verificar que

cot (απ) =1

π

[1

α−∞∑n=1

n2 − α2

]

Control 14

1. La recta normal, en cada punto (x, y) de la curva dada, pasa por el punto (2, 0). Si

la curva pasa por (2, 3) encuentre su ecuacion. Justificar.

2. Resolver la ecuacion

(2x+ 1)2 y′′ − 2 (2x+ 1) y′ − 12y = 6x

500

Page 503: Apuntes MAT-023 USM

Apuntes Mat023 (Segundo semestre 2014)

Control 15

1. Obtenga la solucion de las ecuaciones

a) (6xy2 − 3x2) dx = − (6x2y + 3y2 − 7) dy

b) xy′ − y = yln y−lnx

2. En la schoperıa de “Don Ramon” se vende cerveza artesanal a 4 oC. Dado que se

esta en epoca de verano la temperatura interior de la schoperıa es de 30 oC. El senor

Ramon es muy exigente con la calidad de su producto y pide a los meseros no servir

Schops si es que estos estan a mas de 7 oC. Sabiendo que a los 60 segundos la cerveza

llega a los 5 oC y que ademas un mesero demora 2 minutos en transportar la cerveza

desde su fuente hasta la mesa ¿podran los meseros cumplir con las exigencias de Don

Ramon? (justifique matematicamente)

Control 16

1. Calcular

L[

sin t

t

](s)

2. Resolver

ty′′ + 2y′ + ty = 0

con y (0) = 1 e y (π) = 0.

3. Encontrar el desarrollo en serie de Fourier para

f (x) =

0 −π < x < 0

π − x 0 < x < π

y usando su resultado calcular∞∑n=1

1

(2n− 1)2

Control 17

1. Si ϕ es una funcion de una variable y z = yϕ (x2 − y2) probar que

1

x

∂z

∂x+

1

y

∂z

∂y=

z

y2

501

Page 504: Apuntes MAT-023 USM

Apuntes Mat023 (Segundo semestre 2014)

Control 18

1. Sea T : R3 [x]→ R3 [x] definida por

T(a0 + a1x+ a2x

2 + a3x3)

= a0 + a1 (x+ 1) + a2 (x+ 1)2 + a3 (x+ 1)3

a) Demuestre que T es una transformacion lineal.

b) Si B = 1, x, x2, x3 encuentre [T ]BB

c) Determine dim (T ). ¿Es T inyectiva? Justifique.

Control 19

1. Resolver la ecuacion

x2y′′ − 3xy′ + 3y = 2x4ex

Control 20

1. Resuelva la ecuacion integro diferencial

y′ (t) +

∫ t

0

y (u) du = f (t)

donde

f (t) =

1 0 ≤ t < 1

0 t > 1

con la condicion inicial y (0) = 0.

Control 21

1. Para x > 0 resuelva el P.V.I.

x2y′′ − 2xy′ + 2y = x4

con y (1) = 0, y′ (0) = 1.

Control 22

1. Considere las superficies en R3 dadas por las ecuaciones

y = f (x)

y

z2 + 2xz + y = 0

Determine la funcion f si se sabe que ambas superficies tienen el mismo plano

tangente en todo punto donde intersectan.

502

Page 505: Apuntes MAT-023 USM

Apuntes Mat023 (Segundo semestre 2014)

2. Dado el sistema

u = f (x)

v = g (x, y)

w = h (x, y, z)

para el caul se cumple ∂(f,g,h)∂(x,y,z)

6= 0:

a) Demuestre que localmente se pueden despejar las variables y, u, z como funciones

de (x, v, w) y que∂u

∂w=∂y

∂w= 0

b) Pruebe que∂z

∂x= − 1

gyhz

(∂ (h, g)

∂ (x, y)

)

Control 23

1. Calcule, si existe, el siguiente lımite

lım(x,y)→(0,0)

y√x sin (y3)

x4 + y4

2. Sea

f (x, y) =

x−yx3−y si y − x3 6= 0

1 si y − x3 = 0

Encuentre el dominio de continuidad de f .

Control 24

1. Sea f (x, y) = yne−x2/(4y). Hallar un valor de n tal que f cumpla la ecuacion

x2∂f

∂y=

∂x

(x2∂f

∂x

)2. Demostrar que el tetraedro acotado por los planos coordenados y cada plano tangente

a la superficie xyz = a3 es de volumen constante.

503

Page 506: Apuntes MAT-023 USM

Apuntes Mat023 (Segundo semestre 2014)

Control 25

1. Encuentre todas las soluciones de la ecuacion

dy

dx=

3

2x2 (y − 1)3

2. Resuelva el problema de valor inicial

y′ +5

9xy = 3x3 + x

con y (−1) = 4.

Control 26

1. Se esta celebrando una fiesta en una habitacion que contiene 1800 pies cubicos de

aire libre de monoxido de carbono. En el instante t = 0 varias personas comienzan

a fumar. El humo que contiene 6 por ciento de monoxido de carbono, se introduce

en la habitacion a razon de 0.15 pies cubicos por minuto, y la mezcla, removida por

ventilacion, sale a ese mismo ritmo por una ventana entreabierta. ¿Cuando deberıa

abandonar una persona prudente esa fiesta, si el nivel de monoxido de carbono

comienza a ser peligroso a partir de una concentracion de 0.00018?

Control 27

1. Un edificio tiene dos pisos. El primer piso esta sujeto al suelo rıgidamente y el segundo

es una masa m que pesa 16 toneladas. La estructura elastica del edificio se comporta

como un resorte que resiste a los desplazamientos horizontales del segundo piso;

requiere una fuerza horizontal de 5 toneladas para que el segundo piso se desplace

una distancia de 1 pie. Suponga que un temblor de tierra hace que el piso oscile

horizontalmente con una amplitud A0 y con una frecuencia ω, resultando una fuerza

externa F (t) = mA0ω2 sin (ωt) sobre el segundo piso.

a) ¿Cual es la frecuencia de las oscilaciones del segundo piso?

b) Si el suelo sufre una oscilacion cada 2.25 segundos con una amplitud de 3 pulgadas

¿cual es la amplitud de las oscilaciones forzadas resultantes del segundo piso?

Control 28

1. Resuelva el problema de valores iniciales

ty′′ − ty − y = 0

con y (0) = 0, y′ (0) = 3.

504

Page 507: Apuntes MAT-023 USM

Apuntes Mat023 (Segundo semestre 2014)

2. Resuelva el problema

y′′ − 4y′ + 4y =

t si 0 ≤ t < 3

t+ 2 si t ≥ 3

con y (0) = y′ (0) = 0.

Control 29

1. Resolver las ecuaciones

a) xy2y′ + y3 = x cosx

b) dydx

= − 3x2y+y2

2x3+3xycon y (1) = −2

Control 30

1. Sea T : R2 [x]→ R2 definida por T (p (x)) = (p (0) , p (1))

a) Pruebe que T es una transformacion lineal.

b) Determine una base para Ker(T ) e Im(T ). ¿Cuales son sus dimensiones?

c) Sea B = 1 + x, 2 + x2, 4 + x+ x2 base de R2 [x] y sea C la base canonica de

R2. encuentre [T ]CB.

Control 31

1. Resolver la ecuacion

y′ =1

16x2y2 − y + 4x (x+ 4)

Sabiendo que una solucion particular es de la forma axb.

2. Determine la funcion M (x, y) mas general de modo que la ecuacion diferencial

M (x, y) dx+

(xexy + 2xy +

1

x

)dy = 0

sea exacta y resuelva la ecuacion para una de tales M .

Control 32

1. Determine los maximos y mınimos globales de

f (x, y, z) = 2x2 + y2 + z2 − xy

sujeto a x2

2+ y2

4+ z2

8≤ 1.

505

Page 508: Apuntes MAT-023 USM

Apuntes Mat023 (Segundo semestre 2014)

Control 33

1. Use la transformada de Laplace para resolver el sistema

x′ = 4x− 2y

y′ = 5x+ 2y

con x (0) = 2 y y (0) = −2.

Control 34

1. Suponga que un tanque cilındrico recto con radio de la base 12

metro y altura 4 metros

tiene inicialmente 2 litros de agua pura. Una solucion de salmuera se bombea hacia

el tanque a una rapidez de(1 + 1

1+t

)litros por minuto, la concentracion de sal en el

flujo de entrada es de 12

kilogramo por litro. La solucion en el tanque es homogenea y

se extrae a 11+t

litros por minuto. Determinar la cantidad de sal en el tanque cuando

este se llena.

2. Sean u = (1, 0, 1), v = (1, 0,−1) y w = u× v vectores en R3 y C la base canonica de

R3:

a) Si T : R3 → R3 es una transformacion lineal definida por T (u) = w, T (v) = v

y T (w) = u, determine [T ]CC y [T T ]CC

b) Determine explıcitamente T−1 o argumente que no esta definida.

c) Si B = u,v,w determine [T ]BB

d) Encontrar una matriz A tal que A−1 [T ]BB A = [T ]CC

Control 35

1. ean a, b ∈ R, a 6= 0. Considere la ecuacion diferencial autonoma

dx

dt= a ((x− 1) (x− 4)− b)

I) Determine los valores y/o condiciones sobre a y b de modo que la funcion

x (t) ≡ 5 sea una solucion de equilibrio y ademas un atractor.

II) Para los valores y/o condiciones obtenidos en la parte anterior, bosquejar el

diagrama de fases.

506

Page 509: Apuntes MAT-023 USM

Apuntes Mat023 (Segundo semestre 2014)

III) Para los valores y/o condiciones obtenidos en la parte I), analizar el comporta-

miento de la solucion definida por el P.V.I.

dx

dt= a ((x− 1) (x− 4)− b)

x (0) = 1

esto es: Intervalos de crecimiento, decrecimiento y concavidad, limites a ±∞ si

estos tienen sentido (examinar el intervalo maximal de definicion).

Obs.: No se entregaran puntos por resolver explıcitamente y luego realizar el

analisis.

Control 36

1. Considere el sistema de ecuaciones

d

dtX = AX + B

donde X =

(x (t)

y (t)

), A =

(2 4

−1 −3

)y B =

(et

e−2t

)a) Resolver el sistema homogeneo d

dtX = AX, bosquejar el diagrama de fases y

clasificar la solucion de equilibrio. Si

(x (0)

y (0)

)=

(3

2

)determine la ecuacion

de la recta a la cual tiende la curva solucion cuando t→ +∞.

b) Resolver el sistema no homogeneo ddt

X = AX + B

2. Resolver el problema

d2y

dt2= t+ 2

∫ t

0

e−(t−u)y (u) du

y (0) = 0

y′ (0) = 0

Control 37

1. Considere la funcion

f (x, y) =

xαy

x2 + y2si (x, y) 6= (0, 0)

0 si (x, y) = (0, 0)

507

Page 510: Apuntes MAT-023 USM

Apuntes Mat023 (Segundo semestre 2014)

a) Determine los valores de α ∈ R+ para los cuales f es diferenciable en (0, 0)

b) Determine (para todo α ∈ R+) el plano tangente a la grafica en el punto(1, 1, 1

2

).

2. Sea f : D ⊆ R2 → R definida por

f (x, y) =

√x2 + y2 − 1√4− x2 − y2

a) Determine el dominio D de f .

b) Demostrar que(

32, 0)∈D,(1,√

3)∈ ∂D y (0, 0) 6∈ D.

c) Describir los conjuntos de nivel de f y graficar la funcion.

508

Page 511: Apuntes MAT-023 USM

Capıtulo 13 : Certamenes

Certamen 1

1. Sea f : R2 → R diferenciable tal que f (1, 1) = 1 y ∇f (1, 1) = (−2, 3). Sea

z = g (x, y) = f (x2y, y2f (x, y)). Encuentre la ecuacion del plano tangente a la

superficie z = g (x, y) en el punto (1, 1, 1).

2. Considere la funcion

f (x, y) =

1x

sin (x2 + x2y2) si x 6= 0

0 si x = 0

a) Pruebe que f es continua en R2

b) Encuentre fx (0, 0) y fy (0, 0)

c) Determine si f es diferenciable en (0, 0).

3. El sistema

x3 + uy2 + v = 0

y3 + yv + x2 = 0

define a u y v como funciones de x e y, en vecindades de los puntos (x, y) = (0, 1) y

(u, v) = (1,−1). Sea w = (1, 1), determine la derivada direccional de v = v (x, y) en

el punto (1, 0) en la direccion w.

4. El material para el fondo de una caja rectangular cuesta el triple por metro cuadrado

que el material para los lados y la tapa. Determine la maxima capacidad que tal caja

puede tener si la cantidad total de dinero disponible para el material es $12000 y el

material para el fondo cuesta $600 el metro cuadrado.

509

Page 512: Apuntes MAT-023 USM

Apuntes Mat023 (Segundo semestre 2014)

Certamen 2

1. Considere la aplicacion lineal T : R3 → R3 tal que T (1, 1, 0) = (2, 4,−2), T (1, 0, 1) =

(0, 2,−2) y T (0, 1, 1) = (2, 2, 2):

a) ¿Es T invertible?. Justifique.

b) Pruebe que existe una base B de R3 tal que [T ]BB es una matriz diagonal.

2. Sea g : R→ R una funcion continua y defina f : R2 → R por

f (x, y) =

∫ y

x

g (t) dt

a) Pruebe que f es diferenciable

b) Pruebe que el plano tangente a la superficie z = f (x, y) en el punto (a, b) pasa

por el origen si y solo si

bg (b)− ag (a) =

∫ b

a

g (t) dt

3. Encuentre el maximo de lnx+ln y+3 ln z sobre la porcion de esfera x2 +y2 +z2 = 5r2

donde x > 0, y > 0, z > 0. Use el resultado para probar que para numeros reales

positivos a, b, c tenemos

abc3 ≤ 27

(a+ b+ c

5

)5

4. Sea U = (x, y, z) ∈ R3 : xy 6= 0. Definamos f : U ⊆ R3 → R por

f (x, y, z) = g

(x+

z

y, y +

z

x

)donde g : R2 → R es una funcion de clase C1. Suponga que para cada

(x, y, z) ∈ U ,∂f

∂z(x, y, z) 6= 0

a) Considere un punto (x0, y0, z0) ∈ U tal que f (x0, y0, z0) = 0. Argumente que

f (x, y, z) = 0 define implıcitamente a z como funcion de las variables x e y, es

decir, z = z (x, y) en una vecindad de (x0, y0) tal que z0 = z (x0, y0).

b) Demuestre que la funcion z de la parte anterior satisface la ecuacion

x∂z

∂x+ y

∂z

∂y= z − xy

510

Page 513: Apuntes MAT-023 USM

Apuntes Mat023 (Segundo semestre 2014)

Certamen 3

1. Resuelva la siguiente ecuacion

y′ = e2xy2 − 2y − 9e−2x

con y (0) = 4 sabiendo que tiene una solucion particular de la forma y (x) = aekx.

2. Un tanque contiene inicialmente 60 gal. de agua pura. Entra al tanque, a una tasa de

2 gal./min., salmuera que contiene 1 lb. de sal por galon, y la solucion (perfectamente

mezclada) sale de el a razon de 3 gal. por minuto; el tanque se vacıa despues de una

hora exactamente.

a) Encuentre la cantidad de sal que hay en el tanque despues de t minutos.

b) ¿Cual es la cantidad maxima de sal que llega a tener el tanque?.

3. Resuelva la ecuacion

x(1− x2

)2y′′ −

(1− x2

)2y′ + x3y = 0

usando el cambio de variables t = −12

ln (1− x2)

4. Encuentre la solucion general de la ecuacion(x2 − 2x

)y′′ + 2 (1− x) y′ + 2y = 6

(x2 − 2x

)2

sabiendo que y = x− 1 es una solucion de la ecuacion homogenea asociada.

511

Page 514: Apuntes MAT-023 USM

Apuntes Mat023 (Segundo semestre 2014)

Certamen 4

1. Resuelva las ecuaciones

a) x3 dydx

+ x2y = 2y−4/3

b) (x2 + y2 + 1) dx− (xy + y) dy = 0

2. Suponga que un individuo infectado se introduce en una poblacion de tamano N,

todos los individuos de la cual son susceptibles a la enfermedad . Si suponemos que

la tasa de infeccion es proporcional al producto del numero de infectados y el de

susceptibles presentes, ¿cual sera el numero de infecciones en el tiempo t?.

3. Para x > 0, considere la ecuacion

xy′′ +(x2 − 1

)y′ + x3y = e−x

2/4

a) Use el cambio de variables t = x2/2 para encontrar la solucion general de la

homogenea.

b) Resuelva la no homogenea usando variacion de parametros.

512

Page 515: Apuntes MAT-023 USM

Apuntes Mat023 (Segundo semestre 2014)

Certamen 5

1. a) Pruebe que el cambio de variables u = ax + by + c transforma la ecuacion

diferencial y′ = f (ax+ by + c) en una ecuacion diferencial autonoma, donde

a, b y c son constantes reales.

b) Use el resultado anterior para transformar la ecuacion

dy

dx= (y − 2x)− (y − 2x)2 + 2 (ec-1)

en una ecuacion autonoma.

c) Haga la linea de fase de la ecuacion autonoma obtenida en el item b).

d) En el plano xy haga un esbozo de las soluciones de la ecuacion (ec-1).

e) Obtenga la solucion general de (ec-1).

2. Determine una solucion de la forma y (x) = xn para la ecuacion(x2 − 1

)y′′ (x)− 2xy′ (x) + 2y (x) = 0, x > 1

donde n es algun entero positivo.

Usando el resultado anterior, hallar la solucion del problema de valores iniciales(x2 − 1) y′′ (x)− 2xy′ (x) + 2y (x) = (x2 − 1)

2x > 1

y (2) = 2, y′ (2) = 10

3. Considere el sistema lineal de ecuaciones diferenciales

dx

dt= y

dy

dt= −4x− 2ay

con a ∈ R:

a) Para los diferentes valores del parametro real a, haga un retrato de fase del

sistema de ecuaciones indicando las direcciones en las cuales las curvas son

recorridas.

b) Obtenga la solucion general del sistema.

4. Supongamos que la tasa de cambio del precio x de un bien, crece en el tiempo a una

razon constante c como resultado de la inflacion constante, al mismo tiempo cae en

forma proporcional a la diferencia entre la oferta y en el tiempo t y alguna oferta de

equilibrio y0, es decir, dxdt

= c− α (y − y0). Tambien asumimos que la tasa de cambio

de la oferta es proporcional a la diferencia entre el precio y algun precio de equilibrio

x0, es decir, dydt

= β (x− x0). (α y β)son constantes de proporcionalidad positivas.

Asumiendo que en t = 0,x = x0 y y = y0

513

Page 516: Apuntes MAT-023 USM

Apuntes Mat023 (Segundo semestre 2014)

a) Muestre que el precio y la oferta oscilan alrededor de x0 y y0 + c/α respectiva-

mente.

b) Si en el tiempo t = T0 el precio es maximo ¿en que tiempo la oferta es maxima?

514

Page 517: Apuntes MAT-023 USM

Apuntes Mat023 (Segundo semestre 2014)

Certamen 6

1. Se ha determinado experimentalmente que la variacion de peso de un tipo de pez

sigue la leydp

dt= αe−

β3tp2/3 − βp

donde p = p (t) representa el peso, α y β son constantes positivas. Si p (0) = p0 > 0

determine el peso maximo del pez.

a) Hallar una ecuacion diferencial lineal con solucion general

c1et cos (2t) + c2e

t sin (2t) + t5

b) Considere la ecuacion y′′ + q (x) y = 0 donde q es continua en todo R y dos

soluciones y1, y2 las cuales satisfacen y1 (0) = 1, y′1 (0) = 0 y y2 (0) = 3, y′2 (0) = 1.

Si W (x) es el Wronskiano de y1 e y2 en x demuestre que W (x) = 1 para todo

x donde las soluciones esten definidas.

2. Muestre que ϕ (x) = ex es una solucion de la ecuacion xd2ydx2− (x+ 2) dy

dx+ 2y = 0 y

determinar la solucion general de

xd2y

dx2− (x+ 2)

dy

dx+ 2y = x3

3. Sea C la base canonica de R3. Sean T, L : R3→ R3 transformaciones lineales tales

que T (1, 1, 1) = (1,−3, 3), T (1, 1, 0) = (2,−3, 2), T (1, 0, 0) = (−1,−1, 2) y

[L]CC =

−2 −3 −2

1 1 1

4 6 5

a) Determine T explıcitamente.

b) Determine [T L]CC

c) ¿Que relacion existe entre T y L?

d) Determinar Ker(T ) e Im(L).

515

Page 518: Apuntes MAT-023 USM

Apuntes Mat023 (Segundo semestre 2014)

Certamen 7

1. a) Calcular la serie de Fourier de la funcion

f (x) =

1 si 0 < x < π

−1 si −π < x < 0

y usando la serie muestre queπ

4= 1− 1

3+

1

5− 1

7+ · · ·

b) Usando la serie de Fourier anterior, determine la serie de Fourier de

g (x) =

a si 0 < x < π

b si −π < x < 0

para a, b ∈ R (No se entregaran puntos por calcular esta serie con las formulas

de los coeficientes de Fourier).

2. Resolver el P.V.I.d2y

dt2+ 9y =

t si t < 1

0 si t > 1

con y (0) = y′ (0) = 0.

3. Resolver el sistema ∫ t

0

et−ux (u) du+

∫ t

0

(t− u) y (u) du = t2∫ t

0

x (u) du+

∫ t

0

(t− u)2 y (u) du = t3

4. Sea α ∈ R−1, 2. Considere el sistema de ecuaciones

d

dt

(x

y

)=

(α α− 2

1− α α

)(x

y

)a) Clasificar la solucion de equilibrio (silla, atractor, repulsor, etc.) para los distintos

valores de α.

b) Para α = 32

determine la solucion general del sistema y bosquejar el diagrama

de fases.

516

Page 519: Apuntes MAT-023 USM

Apuntes Mat023 (Segundo semestre 2014)

Certamen 8

1. Hallar y clasificar los puntos crıticos de f : R2 → R definida por

f (x, y) = x3 + y3 + 9x2 − 3y2 + 15x− 9y

¿Posee f extremos globales?.

2. Considere la funcion

f (x, y) =

x2y

|x|+ y2si (x, y) 6= 0

0 si (x, y) = 0

a) Determine los puntos del dominio de f en los cuales la funcion es continua.

b) ¿Es f diferenciable en (0, 0)?

c) Si a es un vector unitario de R2 y∂f

∂a(1, 1) es la derivada direccional de f en

(1, 1) en la direccion a, determine el valor de

maxa∈R2, ‖a‖=1

∂f

∂a(1, 1)

3. Determine las funciones f : R→ R de clase C1 (R) tales que las superficies

S1 =

(x, y, z) ∈ R3 : y = f (x)

S2 =

(x, y, z) ∈ R3 : z2 + 2xz + y = 0

son tangentes en los puntos de interseccion.

4. Suponga que la funcion de clase C2 (R), f : R2 → R, (x, y) → f (x, y) satisface la

ecuacion en derivadas parciales

∂2f

∂x2− 2

∂2f

∂x∂y− 3

∂2f

∂y2= 0

y defina g : R2 → R por

g (u, v) = f (αu+ βv, δu+ γv)

donde α, β, δ y γ son constantes. Determine valores enteros no nulos de α, β, δ y γ

para los cuales g cumple∂2g

∂u∂v= 0

517

Page 520: Apuntes MAT-023 USM

Apuntes Mat023 (Segundo semestre 2014)

Certamen 9

1. Resolver la ecuacion diferencial

td2y

dt2+ (1− 2t)

dy

dt− 2y = 0

con las condiciones iniciales y (0) = 1, y′ (0) = 2.

2. En el paraboloide 4x2 + y2 + 4z2− 16x− 6y− 8z+ 25 = 0 hallar el punto mas cercano

y mas alejado del plano 2x+ 2y + z = 0.

3. Muestre que existen funciones u = u (x, y), v = v (x, y) definidas en un abierto U que

contiene a (1, 1) tales que u (1, 1) = 0, v (1, 1) = 1,

ux3 + xy2 + u sinu = 1

y

v3 − 2xv + y3 = 0

ademas estudiar si la funcion F : U ⊆ R2 → R2, (x, y)→ F (x, y) = (u (x, y) , v (x, y))

es localmente invertible cerca de (1, 1) y en caso afirmativo calcular DF−1 (0, 1).

4. Sea f : R2 → R, (x, y)→ f (x, y) una funcion de clase C2.

a) Muestre que si g (r, θ) = f (r cos θ, r sin θ) entonces

∂2f

∂x2+∂2f

∂y2=∂2g

∂r2+

1

r2

∂2g

∂θ2+

1

r

∂g

∂r

b) Determine una funcion f : (x, y) ∈ R2 : 1 ≤ x2 + y2 ≤ 9 → R tal que

∂2f

∂x2+∂2f

∂y2= 0 para 1 < x2 + y2 < 9

f (x, y) = 1 si x2 + y2 = 1

f (x, y) = 3 si x2 + y2 = 9

suponiendo que g (r, θ) = f (r cos θ, r sin θ) no depende del angulo, es decir,∂g∂θ

= 0.

518

Page 521: Apuntes MAT-023 USM

Apuntes Mat023 (Segundo semestre 2014)

Certamen 10

1. Determine todas la funciones de clase C1 y positivas f : R→ R+, que cumplan

f (0) = 1 y tales que en todo intervalo [a, b] el area bajo la grafica de la funcion y

sobre el eje x sea igual a la longitud de arco de la grafica.

2. Determine la solucion general de la ecuacion

d3y

dt3− 6

d2y

dt2+ 11

dy

dt− 6y = et + t

3. Si se sabe que una solucion de

d2y

dx2+ f (x)

dy

dx+ g (x) y = 0

es ϕ (x) = x y f (x) = − 1x

(x+ 2), determine g (x) y la solucion general de

d2y

dx2+ f (x)

dy

dx+ g (x) y = xex

4. Construir una transformacion lineal T : R4 → R4 tal que

ker (T ) = (x, y, z, w) : x+ y + z = 0 ∧ x− 2y + w = 0

e

Im (T ) = (x, y, z, w) : x− y − z = 0 ∧ x+ 2w + y − z = 0

y determine [T ]BC donde

B = (1, 0, 0, 0) ; (1, 1, 0, 0) ; (1, 1, 1, 0) ; (1, 1, 1, 1)

y C es la base canonica de R4.

519

Page 522: Apuntes MAT-023 USM

Apuntes Mat023 (Segundo semestre 2014)

Certamen 11

1. Determine La serie de Fourier de ϕ : [−2, 2]→ R

x→ ϕ (x) =

0 si x ∈ [−2, 0]

x si x ∈ [0, 2]

¿a que converge la serie de Fourier de ϕ al evaluarla en x = 6?

2. Resolver el P.V.I.

d2y

dt2− y =

0 si t < 4

t2 si t > 4

donde y (0) = y (0) = 0.

3. Determine las funciones de orden exponencial ϕ, φ : R→ R que cumplen ϕ (0) = 0 y∫ t

0

ϕ (t− u)φ (u) du = t2∫ t

0

et−uϕ′ (u) du = t

a) Analizar para los distintos valores de µ en R la naturaleza de las soluciones de

equilibrio del sistema

d

dt

(x

y

)=

(µ 1

−1 µ

)(x

y

)y bosquejar en cada caso el diagrama de fases.

b) Resolver el P.V.I.

d

dt

(x

y

)=

(2 1

−1 2

)(x

y

)x (0) = y (0) = 1

520

Page 523: Apuntes MAT-023 USM

Apuntes Mat023 (Segundo semestre 2014)

Certamen 12

1. Considere el siguiente modelo de dinamica poblacional:

dy

dt= r

(1− y

k

)y − Ey donde 0 < E < r, k 6= 0

a) Hallar y clasificar las soluciones de equilibrio.

b) Si y (0) = k2

(1− E

r

)bosquejar la solucion y determinar lımt→+∞ y (t).

c) ¿Existen valores de los parametros E, k, r tal que dos puntos crıticos consecutivos

sean atractores?

2. Resolver el problema de valores iniciales

xy′′ (x) + (2x− 1) y′ (x)− 2y (x) = x2e−3x, x > 0

donde y (1/2) = y′ (1/2) = 0, si se sabe que y (x) = eαx es una solucion de la

homogenea asociada para un α adecuado.

3. Considere el problema

u′′ (t) + a2u (t) = 2 sin(ωt+

π

4

)u (0) = u′ (0) = 0

donde a es una constante conocida.

a) Si a 6= ω resolver la ecuacion diferencial con las condiciones dadas.

b) ¿Para que valores de ω ocurre el fenomeno de la resonancia? Justifique su

respuesta.

4. Considere el sistema de ecuaciones

dx

dt= −5x+ 4y

dy

dt= −4x+ 5y

a) Elaborar el retrato de fases del sistema y clasificar los puntos de equilibrio.

b) Bosquejar en el plano de fase la curva solucion con condiciones iniciales x (0) = 2,

y (0) = 3 ¿Existe algun instante t en el cual x (t) = y (t)?

c) Bosquejar en el plano de fase la curva solucion con condiciones iniciales x (0) = 4,

y (0) = 2 ¿Existe algun instante t en el cual x (t) = y (t)?

521

Page 524: Apuntes MAT-023 USM

Apuntes Mat023 (Segundo semestre 2014)

Certamen 13

1. Sea f : R3 → R diferenciable y tal que

∇f(0, 2, 1) = (1,−1,−2) y f(0, 2, 1) = 4

Considerar ademas la funcion g(u, v) = f(u− v2 , 3u− v , 3u2 − 2v) . Encuentre la

ecuacion del plano tangente a la superficie z = g(u, v) en el punto (1, 1, 4) .

2. La ecuacion x2 + y3 + z4 + u5 = 2 define a u como funcion de x , y y z y la

ecuacion x+ y2 + z3 = 1 define a z como funcion de x e y . Entonces, se puede

considerar u como funcion de x e y . Entonces:

Hallardu

d−→v(1 , −1) , donde −→v esta en la direccion (1 , 1).

3. Sean F : R3 → R3 y G : R2 → R3 dadas por F (x , y , z) = (u , v , w) determinado

por el sistema

u = x− yv = x2 − y2 + z

w = x− z

y G(x , y) = (x− 2y + 1 , 2x− y , x2 − y2)

a) Determinar los puntos de R3 en los que F es localmente invertible.

b) Determinardy

dw(0, 1, 0)

c) Calcular el diferencial de F−1 G en (1, 1)

4. Considerar la elipse que se obtiene al interceptar el cilindro x2 + y2 = 1 y el plano

x+ y + z = 0 . Encontrar la longitud del semieje mayor y del semieje menor.

522

Page 525: Apuntes MAT-023 USM

Apuntes Mat023 (Segundo semestre 2014)

Certamen 14

1. Resolver

y′′ − 2y′ + y − 2

∫ t

0

y(u)du = 5 ; t ≥ 0

con y(0) = 0 ; y′(0) = 0 .

2. Resolver el sistema

x′(t)

y′(t)

z′(t)

=

1 2 3

0 2 3

0 0 2

· x(t)

y(t)

z(t)

3. Considerar la funcion

f(x) = eλx ; −π < x < π

y f(x+ 2π) = f(x) . Pruebe o refute que:

π

sinh(λπ)=

1

λ+ 2

∞∑n=1

(−1)nλ

λ2 + n2

523

Page 526: Apuntes MAT-023 USM

Apuntes Mat023 (Segundo semestre 2014)

Certamen 15

1. Encuentre la solucion general de la ecuacion de Euler no homogenea:

x2y′′ + 2xy′ − 2y = 10 cos (lnx)

2. Una masa que pesa 8 lbs. estira 4 pies un resorte. Al principio esta masa parte

del reposo a 2 pies abajo de la posicion de equilibrio y el movimiento ocurre en un

medio que presenta una fuerza de amortiguamiento igual a la mitad de la velocidad

instantanea. Deducir la ecuacion del movimiento si se aplica una fuerza externa igual

a f(t) = t cos(2t)

3. Para x > 0 y haciendo el cambio x = 1t

encuentre la solucion general de la ecuacion

4x4y′′ + 8x3y′ + y = tg

(1

2x

)4. Considere la ecuacion diferencial

y′′ + 4xy +(6 + 4x2

)y = x2e−x

2

.

a) Pruebe que si y(x) = u(x)e−x2

es solucion, entonces u satisface una ecuacion

diferencial lineal no homogenea con coeficientes constantes.

b) Use el resultado de la parte anterior para determinar la solucion y(x) con

y(0) = 1 , y′(0) = 0

524

Page 527: Apuntes MAT-023 USM

Apuntes Mat023 (Segundo semestre 2014)

certamen 16

1. La Ley de Malthus supone que la tasa de crecimiento de una poblacion p , es

directamente proporcional al tamano de la poblacion en cada instante.

a) Escribir la ecuacion diferencial que representa esta relacion y verificar que

p(t0) = p0 , entonces p(t) = peα(t−t0) , para alguna constante α > 0

b) Sabiendo que la poblacion de la tierra aumento, en promedio, el 2 % anual desde

1960 (α = 0,02) y que en 1965 se estimaba en 3.340 millones de personas.

Calcular mediante este modelo en cuanto tiempo la poblacion se duplico (respecto

de 1960).

c) Este modelo tiene una correccion propuesta por Verhulst en 1837, que asume

que al crecer mucho la poblacion y tener que competir por el alimento y espacio,

el crecimiento se ve afectado por la falta de recursos. Este modelo esta dado por

la ecuacion logıstica

p′ = αp− βp2

con α, β constantes reales positivas. Resuelva esta ecuacion.

d) Calcular para ambos modelos el lımite cuando t tiene a infinito y explicar su

resultado.

2. Determine la distancia maxima y mınima del origen (0, 0, 0) a los puntos de la curva

definida por la interseccion de las superficies:

z = x2 + y2 ; x+ 2y + z = 10

3. Considere la transformacion F : R3 → R3 dada por

F (x, y, z) = (u, v, w), donde u = xy2 v = x+ 3y w = z − x.

Se verifica que F (4, 1, 2) = F (1, 2,−1) = (4, 7,−2).

a) Pruebe que en torno a los puntos (4, 1, 2) y (1, 2,−1) existen inversas locales

(x, y, z) = G(u, v, w) ; (x, y, z) = H(u, v, w)

que satisfacen G(4, 7,−2) = (4, 1, 2) y H(4, 7,−2) = (1, 2,−1) respectivamente.

b) Calcular∂x

∂ven el punto (4, 7,−2) para G o H (ESCOGER SOLO UNA DE

ELLAS).

525

Page 528: Apuntes MAT-023 USM

Apuntes Mat023 (Segundo semestre 2014)

Certamen 17

1. Resuelva la ecuacion diferencial

x3y′′′ + 4x2y′′ + xy′ − y = x2 lnx

para x > 0.

2. Considerar la funcion

f (t) =

0 si 0 ≤ t < 1

1 si t ≥ 1

Resuelva el sistema

9x′ − 32y′ − 32y = f (t)

−2x′ +

∫ t

0

x (u) du+ 8y′ + 8y = 0

con x (0) = 32 e y (0) = 9.

3. Use el cambio y = z1+x2

para resolver la ecuacion

(1 + x2

)y′′ + 4xy′ + 2

(3 + 2x2

)y =

2

cos (2x)+

2

sin (2x)+ 2x

526

Page 529: Apuntes MAT-023 USM

Apuntes Mat023 (Segundo semestre 2014)

Certamen 18

1. Sea T : R3 → R2 una transformacion lineal. Sean B = (1, 1, 1) , (1, 1, 0) , (1, 0, 0) y

U = (1, 1) , (1,−1) bases de R3 y R2 respectivamente.

Considere

A = [T ]UB =

(1 0 −1

−1 1 2

)Determine:

a) [T (1, 1, 0)]U

b) T (3, 2, 1)

c) ¿T es inyectiva?

a) Si w = f(y−xxy, z−yyz

). Probar que

x2∂w

∂x+ y2∂w

∂y+ z2∂w

∂z= 0

b) Sea f : R2 → R definida por

f (x, y) =

x2 si x2 ≥ y2

y si x2 < y2

determine el maximo dominio de continuidad de f .

2. Pruebe que el maximo valor de x2y2z2 bajo la condicion x2 + y2 + z2 = R2 es(R2

3

)3

.

Deduzca de esto que

3√x2y2z2 ≤ x2 + y2 + z2

3

3. Un canaleta cuya seccion transversal tiene forma de trapecio, con angulos en la base

iguales, se va a construir doblando bandas iguales a lo largo de ambos lados de una

larga pieza de metal, de 12 pulgadas de ancho. Encuentre los angulos de la base y las

dimensiones de los lados que producen la maxima capacidad de acarreo.

527

Page 530: Apuntes MAT-023 USM

Apuntes Mat023 (Segundo semestre 2014)

Certamen 19

1. Dada la funcion definida por

f (x, y) =

(y−2)2 sin(xy)x2+y2−4y+4

si (x, y) 6= (0, 2)

0 si (x, y) = (0, 2)

¿Es f una funcion continua en R2?

2. El volumen de un elipsoide de semiejes a, b, c es 4π3abc. Hallar el elipsoide con centro

(0, 0, 0) de volumen mınimo que pasa por (2,−3, 5).

3. Considere las ecuaciones

uv − 3x+ 2y = 0

u4 − v4 = x2 − y2

Verifique que ellas definen funciones u = u (x, y), v = v (x, y) en torno al punto

(u, v, x, y) = (1, 1, 1, 1), ademas, determine la ecuacion del plano tangente a la

superficie u = u (x, y) en (1, 1, 1).

4. Sea f : R2 → R de clase C2

a) Pruebe que si f es homogenea de grado p, es decir, f (tx, ty) = tpf (x, y) entonces

x2∂2f

∂x2+ 2xy

∂2f

∂x∂y+ y2∂

2f

∂y2= p (p− 1) f

Ind.: g (t) = f (tx, ty) derivar dos veces respecto a t.

b) Probar que si (x, y) · ∇f (x, y) = pf (x, y) para todo (x, y) entonces p es ho-

mogenea de grado p.

Ind.: Defina g (t) = f (tx, ty)− tpf (x, y) y calcule la derivada.

528

Page 531: Apuntes MAT-023 USM

Apuntes Mat023 (Segundo semestre 2014)

Certamen 20

1. Sea

f (x, y) =

xy3

x2+y2si (x, y) 6= (0, 0)

0 si (x, y) = (0, 0)

a) ¿Es f continua en (0, 0)?

b) Hallar fx (x, y) y fy (x, y).

2. (Plano tangente y regla de la cadena)

a) Dada la superficie S : x2 + 2y2 + 3z2 = 21 hallar la(s) ecuacion(es) del (de los)

plano(s) tangente(s) a S que es (son) paralelos(s) al plano x+ 4y + 6z = 0.

b) Si z = f (x, y) es de clase C1 y x = r cos θ e y = r sin θ, probar que

sin θ∂z

∂r+

cos θ

r

∂z

∂θ=∂z

∂y

3. (Maximos y mınimos)

a) Encuentre los maximos y/o mınimos de la funcion

f (x, y, z) = x2 + y2 + z2 + xy + yz + xz + x− 2y

b) Determine los angulos α, β, γ de un triangulo de modo que el producto de sus

senos sea maximo.

529

Page 532: Apuntes MAT-023 USM

Apuntes Mat023 (Segundo semestre 2014)

Certamen 21

1. Considere la funcion f definida como sigue:

f (x, y) =

2yx3

x2+y2si (x, y) 6= (0, 0)

0 si (x, y) = (0, 0)

a) Determine si la funcion f (x, y) es continua en todo R2.

b) Determine si la funcion f (x, y) es diferenciable en todo R2.

c) Determine el valor de fxy (0, 0) y fyx (0, 0).

2. Sean x la cantidad de sillas e y la cantidad de mesas producidas por un fabricante.

Si las funciones f (x, y) = 256− 3x− y y g (x, y) = 222 + x− 5y corresponden a los

precios unitarios de venta de los productos respectivamente, hallar las cantidades

de sillas y mesas de modo de obtener maximas utilidades sabiendo que el costo de

produccion total es C (x, y) = x2 + xy + y2.

3. Dada la ecuacion

sin (yz) + sin (xz) + sin (xy) = 0

a) Encuentre las condiciones para que z este definida implıcitamente como funcion

de las variables x e y cerca de (x, y, z) = (1, 0, π).

b) Encontrar el plano tangente a la grafica de z = g (x, y) en (x, y, z) = (1, 0, π).

4. Dada la funcion z = f (2x+ 3g (y))

a) Encuentre las condiciones para que la funcion z = f (2x+ 3g (y)) sea dos veces

diferenciable en R2.

b) Bajo los supuestos encontrados en la parte anterior, determine el valor de k de

modo que

k =

(∂z

∂x

∂2z

∂x∂y− ∂z

∂y

∂2z

∂x2

)

530

Page 533: Apuntes MAT-023 USM

Apuntes Mat023 (Segundo semestre 2014)

Certamen 22

1. Sea T : R3 → R3 la transformacion lineal definida por

T (x, y, z) = (x+ z, y + 3z, x+ y + αz)

con α ∈ R:

a) Determine el valor de la constante α para que dim Ker(T ) = 1 y en este caso

Calcule Ker (T ).

b) Para el valor anterior de α calcule Im (T ).

2. Sean p (x) y q (x) dos funciones continuas. Verificar que la sustitucion y = ez con

z = z (x) transforma la ecuacion diferencial

y′ + p (x) y = q (x) y ln y

en una ecuacion lineal de primer orden. Usando lo anterior, resolver la ecuacion

xy′ = 2x2y + (x+ 1) y ln y

3. Hallar la solucion general de una ecuacion diferencial lineal a coeficientes constantes

homogenea, cuya ecuacion caracterıstica es:

λ5 − 2λ4 + 6λ3 − 9λ2 + 8λ− 4 = 0

sabiendo que y = ex/2 cos(√

32x)

es una solucion de dicha ecuacion.

4. Hallar la solucion general de la ecuacion diferencial

d2y

dx2+ tan (x)

dy

dx+ cos2 (x) y = 0

utilizando para ello el cambio de variables t = sinx.

531

Page 534: Apuntes MAT-023 USM

Apuntes Mat023 (Segundo semestre 2014)

Certamen 23

1. (40 pts.) Sea f : R2 → R la funcion definida por:

f (x, y) =

−2y2 , (x, y) ∈ A

0 , (x, y) = (0, 0)(x2 + y2

)x2 + |y|

sin (x+ y) , (x, y) ∈ AC ∧ (x, y) 6= (0, 0)

en donde:

A =

(x, y) ∈ R2 : y > 0 ∧ −x ≤ y ≤ x

a) Calcule∂f

∂x(0, 0) y

∂f

∂y(0, 0).

b) ¿Es diferenciable f en (0, 0)?

2. Sean f : R3 → R una funcion tal que ∇f(1, 1,−2) = (−1, 1, 0) y h : R2 → R otra

funcion definida por h(s, t) = 2s2 + st:

(a) Encuentre un vector unitario normal a la curva de nivel

(s, t) : h(s, t) = 2

en

el punto (1, 0).

(b) Considere ω : R2 → R definida por

ω(s, t) = f(3x+ 2xy + z2, x+ y2, x− 2z2

)en donde: x(s, t) = 2st, y(s, t) = s2 − 2t y z(s, t) = s + t. Calcule

∇ω(1, 0).

(c) Encuentre la derivada direccional de ω en el punto (1, 0) en la direccion dada por

el vector calculado en la parte (a).

3. Sean f : R2 → R una funcion definida por:

f (x, y) = x− x2 − y2

y el conjunto U = (x, y) : x2 + y2 ≤ 1:

a) Mediante el criterio del hessiano, determine los extremos de f en U = (x, y) : x2 + y2 < 1,en caso de existencia.

b) Mediante el metodo de los multiplicadores de Lagrange, calcule los extremos de f

en ∂U = (x, y) : x2 + y2 = 1, si acaso existen.

532

Page 535: Apuntes MAT-023 USM

Apuntes Mat023 (Segundo semestre 2014)

Certamen 24

1. Sea x = x (t) una funcion que satisface el sistema de ecuaciones

x′′ = y − zy′′ = x′ + z′

z′′ = − (1 + x+ y)

donde x (0) = x′ (0) = 0, y (0) = −1, y′ (0) = 1 y z (0) = 0, z′ (0) = 1. Calcule, de ser

posible, el valor de x (π).

2. Sea f : R→ R la funcion periodica definida por

f (x) = e−x para − π < x < π

y f (x+ 2π) = f (x). Calcule la serie de Fourier de f .

3. Sea f : R2 → R una funcion de clase C2 (R2). Considere el cambio de variables

x = u+ v

y = uv2

y la funcion g : R2 → R definida por g (u, v) = f (x (u, v) , y (u, v)). Calcule el valor

de∂2g

∂u2(1, 1) +

∂2g

∂v2(1, 1)

sabiendo que ∂2f∂x2

(2, 1) = ∂2f∂y∂x

(2, 1) = ∂2f∂y2

(2, 1) = 1 y ∂f∂y

(2, 1) = 2.

4. Se desea construir un tanque que consistir a de un cilindro circular recto de altura h

y radio r, una tapa superior semiesferica de radio r y finalmente una tapa inferior

plana del mismo radio. Suponga que los costos de construccion de la tapa semiesferica

son de $20 por [m2]; que los costos de construccion de la cara lateral cilındrica son

de $8 por [m2] y de $5 por [m2] para la base circular plana.

a) Hallar el valor de h y r de modo que el costo de construccion sea mınimo

asumiendo que el volumen debe ser de 200π [m3].

b) ¿A cuanto es igual la relacion h : r?.

533

Page 536: Apuntes MAT-023 USM

Apuntes Mat023 (Segundo semestre 2014)

Certamen 25

1. Resuelva usando la transformada de Laplace el siguiente problema de valores iniciales

ty′′ − 2y′ + ty = 0 con y (0) = 1, y′ (0) = 0

2. Sea f : [0, π[→ R la funcion definida por:

f (t) =

2tπ

si 0 ≤ t < π2

2(π−t)π

si π2≤ t < π

Desarrollar f (t) en una serie de Fourier en terminos del seno.

3. Considere la ecuacion diferencial de primer orden

dx

dt= (x− 1) (x− a)x

√x2 + x+ 1

en donde a es un parametro real. Determine condiciones sobre a de modo que la

solucion de equilibrio x = 0 sea un punto atractor.

534

Page 537: Apuntes MAT-023 USM

Apuntes Mat023 (Segundo semestre 2014)

Certamen 26

1. Sea T : R2 [x]→ R2 la funcion definida por

T (p (x)) =

(p′ (1) , 6

∫ 1

0

p (x) dx

)a) Pruebe que T es una transformacion lineal.

b) Hallar el nucleo de T y una base para la imagen de T .

c) Sean B = 1, 1 + x, x+ x2 y D = (1,−1) , (0, 1) bases ordenadas de R2 [x] y

R2 respectivamente. Calcular [T ]DB .

2. Hallar la solucion general de la ecuacion

x2 (1− x) y′′ + 2x (2− x) y′ + 2 (1 + x) = x2

sabiendo que la homogenea asociada tiene una solucion de la forma y = x−2.

3. La ecuacion homogenea

y(4) − 4y′′′ + 11y′′ + 8y′ − 26y = 0

tiene una solucion de la forma y = e2x cos 3x. Resolver la ecuacion

y(4) − 4y′′′ + 11y′′ + 8y′ − 26y = x

4. Se suministra bacterias como alimento a una poblacion de protozoos a una tasa

constante de 1[

grmin

]. Se ha observado que las bacterias son consumidas a una tasa

de cuatro veces el cuadrado de su cantidad c (t). Hallar c (t) en funcion de c (0) = c0.

535

Page 538: Apuntes MAT-023 USM

Apuntes Mat023 (Segundo semestre 2014)

Certamen 27

1. Sea x > 1. Resuelva la ecuacion diferencial:

(1− x) y′′ + xy′ − y = (1− x)2 coshx

sabiendo que una solucion de la ecuacion homogenea asociada es y = ex.

2. Laplace:

a) Calcule Lt sen t

.

b) Si Lf (t)

= X (s), calcule f (t), sabiendo para ello que X (s) satisface la

ecuacion: (s2 + 1

)X (s) =

2s (1− e−2πs)

s2 + 1

3. Considere el siguiente sistema de ecuaciones: x′ + 2x+ 6

∫ t

0

y (u) du = −2

x′ + y′ + y = 0

en donde x (0) = −5 e y (0) = 6. Calcule el valor de x (ln 2).

4. Sea f : R2 → R la funcion definida por:

f (x, y) =

xy2

x2 + y2, si y > x2

x2y

x2 + y2, si y ≤ x2 ∧ (x, y) 6= (0, 0)

0 , si (x, y) = (0, 0)

Determine todos los puntos de R2 para los cuales la funcion f es continua.

536

Page 539: Apuntes MAT-023 USM

Apuntes Mat023 (Segundo semestre 2014)

Certamen 28

1. Considere la funcion f : R2 → R definida por:

f (x, y) =

xy sinx

x2 + y2, (x, y) 6= (0, 0)

0 , (x, y) = (0, 0)

a) Determine si f es continua en (0, 0).

b) Determine todas las derivadas direccionales de f en (0, 0).

c) Determine si f es diferenciable en (0, 0).

2. Sea z = z (x, y) una funcion de clase C2. Escriba la ecuacion:

∂2z

∂x2+ 2

∂2z

∂x∂y− 3

∂2z

∂y2= 0

en las variables u y v definidas por las ecuaciones:

u = 3x− y ∧ v = x+ y

3. Determine los valores de las constantes a, b y c en R de modo que la derivada

direccional de:

f (x, y, z) = acy2 + byz + cz2x3

en el punto (1, 2,−1) tenga un valor maximo de 64 en la direccion del eje X positivo.

4. Hallar las dimensiones del paralelepıpedo rectangular de mayor volumen con aristas

paralelas a los ejes coordenados que puede ser inscrito en el elipsoide de ecuacion:(x3

)2

+(y

4

)2

+(z

5

)2

= 1

537

Page 540: Apuntes MAT-023 USM

Apuntes Mat023 (Segundo semestre 2014)

Certamen 29

1. Resuelva la ecuacion diferencial:

(1 + x)2 y′′ − 3 (1 + x) y′ + 4y = (1 + x)3

utilizando para ello el cambio de variables et = 1 + x.

2. Resuelva la ecuacion diferencial:

x′′ + tx′ − x = 0, x (0) = 0, x′ (0) = 1

3. Sea z ∈ C2. Simplificar al maximo la ecuacion:

y∂2z

∂x2+ (x+ y)

∂2z

∂x∂y+ x

∂2z

∂y2= 0

considerando para ello el cambio de variables dado por:

u = y2 − x2 ∧ v = y − x

4. Sean f : R3 → R una funcion diferenciable tal que:

∇f (0, 2, 1) = (1,−1,−2) ∧ f (0, 2, 1) = 4

y g la funcion definida por g (u, v) = f (u− v2, 3u− v, 3u2 − 2v). Hallar la ecuacion

del plano tangente a la superficie z = g (u, v) en el punto (1, 1, 4) .

5. Determine el maximo y el mınimo absolutos de la funcion:

z = sinx+ sin y + sin (x+ y)

en la region:

0 ≤ x <π

2∧ 0 ≤ y ≤ π

2

538

Page 541: Apuntes MAT-023 USM

Apuntes Mat023 (Segundo semestre 2014)

Certamen 30

1. Sea g (x) =

∫ x

1

et

tdt. Hallar todos los valores de la constante a tales que la funcion

f definida por:

f (x) =1

xea g(x)

satisfaga la ecuacion diferencial lineal:

x2y′′ +(3x− x2

)y′ +

(1− x− e2x

)y = 0

Utilizar la informacion anterior para determinar la solucion general de la ecuacion en

el intervalo (0,+∞).

2. Utilizando la Transformada de Laplace, resuelva la ecuacion diferencial:

xy′′ − 2y′ + xy = 0

sabiendo que y (0) = 1 e y′ (0) = 0.

3. Considere la funcion f : R2 → R definida por:

f (x, y) =

x|y|3/2x2+y2

, (x, y) 6= (0, 0)

0 , (x, y) = (0, 0)

a) Determine si f es continua en (x, y) = (0, 0)

b) Determine si f es diferenciable en (x, y) = (0, 0)

4. Sea f (x, y) = ln (x2 + 2xy + 1) +

∫ x

0

sin2 t dt

a) Calcule ∇f (π, 1)

b) Calcular −→v ∈ R2 tal que ‖−→v ‖ = 1 y que∂f

∂−→v(π, 1) sea maxima.

5. Determine el maximo y el mınimo absoluto de la funcion:

z = x3 + y3 − 3xy

en la region:

0 ≤ x ≤ 2 ∧ −1 ≤ y ≤ 2

539

Page 542: Apuntes MAT-023 USM

Apuntes Mat023 (Segundo semestre 2014)

Certamen 31

1. Hallar la solucion general de la ecuacion:

xy′′ − 2 (x+ 1) y′ + (x+ 2) y = x3e2x

para x > 0, bajo el supuesto que la ecuacion homogenea tiene una solucion de la

forma y = emx

2. Resuelva la ecuacion diferencial:

2 (1 + x)2 y′′ − 6 (1 + x) y′ + 8y = (x+ 1)3

3. Considere la funcion f : R2 → R definida por:

f (x, y) =

x|y|3/2x2+y2

, (x, y) 6= (0, 0)

0 , (x, y) = (0, 0)

a) Determine si f es continua en (x, y) = (0, 0)

b) Determine si f es diferenciable en (x, y) = (0, 0)

4. Sean f, g : R→ R funciones de clase C2. Considere z : U ⊆ R2 → R definida por:

z (x, y) = x f(yx

)+ g

(yx

)Calcule el valor de: x2 ∂

2z

∂x2+ 2xy

∂2z

∂x∂y+ y2∂

2z

∂y2

5. Determine el maximo y el mınimo absoluto de la funcion:

z = x3 + y3 − 3xy

en la region:

0 ≤ x ≤ 2 ∧ −1 ≤ y ≤ 2

540

Page 543: Apuntes MAT-023 USM

Capıtulo : Bibliografıa

[1] Kreyszig, E. Advanced Engineering Mathematics, 9th Ed., John Wiley & Sons Inc.,

Singapore, 2006.

[2] Piskunov, N. Calculo Diferencial e Integral, Editorial Limusa S.A. de C.V., Mexico,

2007.

[3] Osses, A. Ecuaciones Diferenciales Ordinarias, CMM, Departamento de Ingenierıa

Matematica, U. de Chile, Santiago, 2010.

[4] Derrick, W & Grossman, S Ecuaciones diferenciales con aplicaciones, Fondo Educativo

Interamericano, Mexico, 1984.

[5] Apostol, T. Calculus: volumen 1, Editorial Reverte, Barcelona, 1967.

[6] Fernandez, C. & Rebolledo, R. Ecuaciones diferenciales ordinarias, Ediciones Univer-

sidad Catolica de Chile, Santiago, 1995.

[7] Hsu, H. Analisis de Fourier, Addison-Wesley Iberoamericana, EE.UU., 1987.

[8] Rocha, J.M. & Villa, G. Calculo infinitesimal de varias variables: vol. 1, I.P.N, Mexico,

2003.

[9] Taylor,H & Wade, T. Calculo diferencial e integral, Limusa–Wiley, Mexico, 1972.

[10] Gavilan, E. Dossier de problemas resueltos, Departamento de Matematicas, U. de

Concepcion, Concepcion.

[11] Martınez, C. Calculo real y vectorial en varias variables, Instituto de Matematicas, P.

U. Catolica de Valparaıso, Valparaıso, 2000.

[12] Jerrold E. Marsden & Anthony J. Tromba. Calculo vectorial, 5th Ed. Pearson. Estados

Unidos 2010.

541

Page 544: Apuntes MAT-023 USM

Parte I

Ecuaciones diferenciales ordinarias

5

sysadmin
Rectángulo
sysadmin
Rectángulo
sysadmin
Rectángulo